100% found this document useful (1 vote)
4K views

USMLE Step I Comprehensive Review

This document contains a quizlet study guide with 64 multiple choice questions related to USMLE Step 1 exam preparation. It provides the questions and possible answer choices in a printable format to allow for offline study. The document encourages students to study online at quizlet.com for additional learning tools.

Uploaded by

Jay KJ
Copyright
© © All Rights Reserved
Available Formats
Download as PDF, TXT or read online on Scribd
100% found this document useful (1 vote)
4K views

USMLE Step I Comprehensive Review

This document contains a quizlet study guide with 64 multiple choice questions related to USMLE Step 1 exam preparation. It provides the questions and possible answer choices in a printable format to allow for offline study. The document encourages students to study online at quizlet.com for additional learning tools.

Uploaded by

Jay KJ
Copyright
© © All Rights Reserved
Available Formats
Download as PDF, TXT or read online on Scribd
You are on page 1/ 222

quizlet.

com

https://quizlet.com/18530/print

USMLE Step I Comprehensive review


Step 2: Open the file
Open PDF

Step 3: Print it!


Study online at quizlet.com/_eaq

1.

<2

What is the lecithin:sphingomyelin ratio in respiratory


distress syndrome of newborns?

2.

>3.5 g/day of protein, along with generalized


edema, hypoalbuminemia, and
hyperlipidemia

How many grams of protein must be excreted in 24 hours


to produce the diagnosis of nephrotic syndrome?

3.

>5 mm

What is the size of a positive PPD test for the following?


(Reactive) Posttransplantation patient taking
immunosuppressive agents

4.

>5 mm

What is the size of a positive PPD test for the following?


(Reactive) Patient with AIDS

5.

>10 mm

What is the size of a positive PPD test for the following?


(Reactive) IV drug abuser

6.

>10 mm

What is the size of a positive PPD test for the following?


(Reactive) Recent immigrant from India

7.

>15 mm

What is the size of a positive PPD test for the following?


(Reactive) Healthy suburban male without any medical
illnesses

8.

0.2

How many milliliters of O2 per milliliter of blood?

9.

0.21; it is a fancy way of saying 21% of the


air is O2.

What is the FiO2 of room air?

10. 1:1. Remember, the flow through the


pulmonary circuit and the systemic circuit are
equal.

What is the ratio of pulmonary to systemic blood flow?

11.

What are the three posttranscriptional modifications?

1. 7-methyl guanine cap on the 5' end 2.


Addition of the poly(A) tail to the 3' end 3.
Removal of introns

12. 1. ADP 2. PG 3. TXA2

What are the three platelet aggregating factors?

13. 1. Autoimmune hypersplenism 2. Trauma 3.


Anemia 4. Spherocytosis 5. Sickle cell
anemia

What are the five conditions associated with normochromic


normocytic anemia with a normal MCV and an elevated
reticulocyte count?

14. 1. Benefits of the procedure 2. Purpose of the procedure 3. Risks of the


procedure 4. The nature of the procedure (what you are doing) 5. The
alternative to this procedure and its availability (Don't forget the last one; this is
where physicians get in trouble.)

What are the five pieces


of information
considered necessary
for fully informed
consent?

15. 1. Big 2. Bilateral 3. Berry aneurysm

What are the three Bs of


adult polycystic kidneys?

16. 1. Bunyavirus 2. Orthomyxovirus 3. Arenavirus 4. Reovirus Remember BOAR

What are the four


segmented RNA
viruses?

17. 1. Burkitt's lymphoma 2. Nasopharyngeal carcinoma 3. Thymic carcinoma

What three carcinomas


are associated with
EBV?

18. 1. C5a 2. Leukotriene B4 3. IL-8 4. Bacterial peptides

What are the four


chemotactic agents?

19. 1. Ca2+ : calmodulin ratio 2. Epinephrine 3. Glucagon

What three substances


stimulate
glycogenolysis?

20. 1. Caput medusae 2. Esophageal varices 3. Ascites 4. Splenomegaly 5.


Hemorrhoids

What are the five


components of portal
HTN?

21. 1. Cefamandole 2. Cefoperazone 3. Ceftriaxone

What three
cephalosporins are
eliminated via biliary
mechanisms?

22. 1. Cefamandole 2. Cefoperazone 3. Moxalactam

What three
cephalosporins can
produce disulfiram-like
reactions?

23. 1. Cefotetan 2. Cefoxitin 3. Ceftizoxime

What three
cephalosporins have
good penetration against
Bacteroides fragilis?

24. 1. CHF 2. Vena caval obstruction or constriction 3. Hepatic cirrhosis 4. Renal


artery stenosis

What four conditions


result in secondary
hyperaldosteronism?

25. 1. Clavulanic acid 2. Sulbactam 3. Tazobactam

What are the three Betalactamase inhibitors?

26. 1. Cloxacillin 2. Oxacillin 3. Nafcillin 4. Dicloxacillin 5. Methicillin (CONDM)

What are the five


penicillinase-resistant
penicillins?

27. 1. CNS 2. Renal tubules 3. Beta Islet cells of the pancreas 4. RBCs 5. GI
mucosa

What are the five tissues in


which glucose uptake is
insulin independent?

28. 1. Competition for carrier with similar chemical substances 2. Chemical


specificity needed for transportation 3. Zero-order saturation kinetics
(Transportation is maximal when all transporters are saturated.) 4. Rate of
transportation faster than if by simple diffusion

What are the four


characteristics of all proteinmediated transportation?

29. 1. Concentration (greater concentration gradient, greater diffusion rate) 2.


Surface area (greater surface area, greater diffusion rate) 3. Solubility
(greater solubility, greater diffusion rate) 4. Membrane thickness (thicker the
membrane, slower the diffusion rate) Molecular weight is clinically
unimportant

What four factors affect


diffusion rate?

30. 1. Conjunctivitis 2. Nongonococcal urethritis 3. Peripheral arthritis Can't see,


can't wee, can't kick with your knee

What is the triad of Reiter


syndrome?

31. 1. Cortisol (stimulates PEPCK) 2. Glucagon 3. GTP

What three substrates


control the enzyme PEPCK
for the conversion of
oxaloacetate (OAA) to
pyruvate in the cytoplasm?

32. 1. Cough 2. Coryza 3. Conjunctivitis

What are the three C's of


measles?

33. 1. Covalent bonding between the hapten and carrier 2. B-cell exposure to
hapten twice 3. T-cell exposure to carrier twice

What are the three rules


governing a secondary
immune response?

34. 1. Cytosine 2. Uracil (only in RNA) 3. Thymidine

What three bases are


pyrimidines?

35. 1. Cytotrophoblast 2. Syncytiotrophoblast 3. Extraembryonic mesoderm

What three embryonic cell


layers form the chorion?

36. 1 day prior to ovulation

How many days prior to ovulation


does LH surge occur in the
menstrual cycle?

37. 1. Decrease alpha-1 activity 2. Increase Beta-2 activity 3. Increase


ACh levels

Name the three methods of


vasodilation via the sympathetic
nervous system.

38. 1. Decrease concentrations of Ag levels 2. Administer IgG in high


concentrations 3. Inhibit B cells with Ag bound to IgG (complexes)
4. Turn off the original T or B cell with anti-Ab

What are the four ways to downregulate the immune system?

39. 1. Decrease the radius of the vessel 2. Increase the length of the
vessel 3. Increase the viscosity 4. Decrease the number of parallel
channels

What are the four ways to increase


TPR?

40. 1. Diarrhea 2. Dermatitis 3. Dementia 4. Death

What are the four Ds of niacin


deficiency?

41. 1. Dilation of the afferent arteriole 2. Constriction of the efferent


arteriole 3. Inhibition of reabsorption of sodium and water in the
collecting ducts

What are the three mechanisms of


action for atrial natriuretic peptide's
diuretic and natriuretic affects?

42. 1. Dorsal scapular 2. Suprascapular 3. Long thoracic 4. Nerve to


subclavius

What four branches of the brachial


plexus arise prior to the first rib?

43. 1. Epinephrine synthesis 2. Phosphatidyl choline 3. Creatine 4.


Methylation of cytosine 5. N-methyl cap of mRNA

What five pathways use SAM (Sadenosylmethionine) as the methyl


donor?

44. 1. Exercise 2. Emergencies (stress) 3. Exposure to cold (The three


Es)

Which three factors cause the


release of epinephrine from the
adrenal medulla?

45. 1. Fat 2. Forty 3. Female 4. Familial 5. Fertile

What are the five F's associated with


gallstones?

46. 1. Fibrillary protein 2. Amyloid protein 3. Glycosaminoglycans

What are the three components of


amyloid?

47. 1. Flow independent of BP 2. Flow proportional to local metabolism


3. Flow independent of nervous reflexes

What are the three characteristics of


autoregulation?

48. 1. Fructokinase 2. Glucokinase 3. Glycerol kinase 4. PEPCK 5. Pyruvate


carboxylase 6. Galactokinase 7. Fructose-1, 6-bisphosphatase 8. Glucose-6phosphatase

What are the eight liverspecific enzymes?

49. 1. Fructose-6-phosphate 2. Glucose-6-phosphate 3. Glyceraldehyde-3phosphate

At what three sites can the


HMP shunt enter into
glycolysis?

50. 1. Glucagon 2. NADH 3. Acetyl CoA 4. ATP

What four substances


increase the rate of
gluconeogenesis?

51. 1. Glucose 2. Amino acid (arginine) 3. Gastrin inhibitory peptide (GIP) 4.


Glucagon 5. -Agonists 6. ACh

What are the six


substances that promote
the secretion of insulin?

52. 1. Glucose from liver glycogen 2. Glucose from gluconeogenesis 3. Body


protein 4. Body fat

What is the order of fuel


use in a prolonged fast?

53. 1. Glycine 2. Aspartate 3. Glutamine

What three AAs are used


to synthesize the purine
ring?

54. 1. H2O2 2. Superoxide 3. Hydroxyl radical 4. Myeloperoxidase 5.


Hypochlorous acid

What five main oxidizing


reactions are used to kill
ingested organisms?

55. 1. HPV 2. EBV 3. Hepatitis B 4. Kaposi sarcoma

What are the four DNA


oncogenic viruses?

56. 1. IgA receptor 2. Transport of IgA across epithelial barriers 3. Protection of


IgA from degradative proteases

What are the three major


functions of secretory IgA?

57. 1. Iliocostalis 2. Longissimus 3. Spinalis ("I Love Science" muscles)

What three muscles


constitute the erector
spinae?

58. 1. Incompetent patient (determined by the courts) 2. Therapeutic privilege (in


the best interest of the patient when he or she is unable to answer) 3. Waiver
signed by the patient 4. Emergency

What are the four


exceptions to
requirements for informed
consent?

59. 1. Increase compliance 2. Decrease surface tension 3. Decrease probability of


pulmonary edema formation

What are the three


functions of surfactant?

60. 1. Increased amino acid uptake by muscles 2. Decreased protein breakdown


3. Increased protein synthesis

What are the three


effects of insulin on
protein metabolism?

61. 1. Increased glucose uptake by fat cells 2. Increased triglyceride uptake by fat
cells 3. Increased conversion of CHOs into fat 4. Decreased lipolysis in fat
tissue 5. Decreased ketone body formation

What are the five effects


of insulin on fat
metabolism?

62. 1. Increased solubility 2. Increased concentration gradient 3. Decreased


thickness of the membrane

What three factors


increase simple
diffusion?

63. 1. Increased synthesis and secretion of oxytocin 2. Increased release of PIF by


the hypothalamus 3. Inhibition of GnRH (suppressing FSH/LH) 4. Milk
secretion

What are the four effects


of suckling on the
mother?

64. 1. Increased tube radius 2. Increased velocity 3. Decreased viscosity 4.


Increased number of branches 5. Narrowing of an orifice

What five factors promote


turbulent flow?

65. 1. Inhibin 2. Estradiol (E2) 3. Androgen-binding protein 4. Meiosis inhibiting


factor (in fetal tissue) 5. Antimullerian hormone

What are the five


hormones produced by
Sertoli cells?

66. 1. Intracellular receptors 2. Membrane receptors 3. Enzymes 4. Intracellular


effectors

What are the four types of


signaling mechanisms?

67. 1. Laminin 2. Heparan sulfate (heparitin sulfate) 3. Fibronectin 4. Type IV


collagen

What are the four


components of the
basement membrane?

68. 1. Large sample size 2. Large effect size 3. Type I error is greater

In biostatistics, what are


the three criteria required
to increase power?

69. 1. Lateral pectoral 2. Lateral head of the median 3.


Musculocutaneus

What are the three branches of the lateral


cord of the brachial plexus?

70. 1. Less than 3 cm 2. Clean base 3. Level with the surrounding


mucosa

What three criteria allow you to


differentiate an ulcer from an erosion or
carcinoma?

71. 1. Liver 2. Kidney 3. Pituitary gland (via 5'-deiodinase


enzyme)

At which three sites in the body is T4


converted to T3?

72. 1. Liver 2. Muscle 3. Adipose tissue

What are the three tissues where


triacylglycerols are produced?

73. 1. Lowers surface tension, so it decreases recoil and


increases compliance 2. Reduces capillary filtration 3.
Promotes stability in small alveoli by lowering surface tension

What are the three main functions of


surfactant?

74. 1. Low pressure in the afferent renal arteriole 2. Low sodium


sensed by the macula densa 3. Increased -1-sympathetic
stimulation of the JG cells

What are the three stimuli that result in the


reninangiotensin-aldosterone secretion?

75. 1. Malate dehydrogenase 2. Isocitrate dehydrogenase 3. Ketoglutarate dehydrogenase

What three steps of the TCA cycle


generate NADH?

76. 1. Maltose 2. Maltotetrose 3. -Limit dextrans (-1, 6 binding)

What are the three end products of


amylase digestion?

77. 1. Masseter 2. Temporalis 3. Medial pterygoid 4. Lateral


pterygoid

What are the four muscles of mastication?

78. 1. Nails 2. Hair 3. Sweat glands (both apocrine and


sebaceous)

What are the three epidermal derivatives?

79. 1. Neisseria meningitidis 2. Haemophilus influenzae 3.


Streptococcus pneumoniae

What three bacteria are positive to


quellung reactive test?

80. 1. Notify police. 2. Try to detain the person making the threat.
3. Notify the threatened victim.

What three actions should take place


when one person threatens the life of
another? (Hint: think of the Tarasoff
decision.)

81. 1. One cell type 2. One Ab type 3. Random selection of


hypervariable regions, and only cells with bound Ag undergo
clonal expansion

What are the three rules of clonal


selection?

82. 1. Petechiae 2. Hyperactive mental status 3.


Occurrence within 24 to 48 hours of the initial insult
(e.g., long bone fracture)

What is the triad of fat embolism?

83. 1. Phosphorylation of mannose (lysosomes only) 2.


Removal of mannose residues 3. Formation of
glycosylate proteins 4. Phosphorylation of sulfate amino
acids

What are the four posttranslational modifications


done by the Golgi apparatus?

84. 1. Picornavirus 2. Calicivirus 3. Reovirus (Remember


PCR)

What are the three naked RNA viruses?

85. 1. Pinpoint pupils 2. Decreased respiratory rate 3. Coma

What are the three signs of morphine overdose?

86. 1. Plicae circularis (3 times) 2. Villi (30 times) 3.


Microvilli (600 times)

What three structures increase the surface area


of the GI tract?

87. 1. Posterior auricular vein 2. Posterior division of the


retromandibular vein

What two vessels come together to form the


external jugular vein?

88. 1. Propranolol 2. Timolol 3. Carteolol

What three -blockers are used in the treatment


of glaucoma?

89. 1. Protest 2. Despair 3. Detachment

What are the three stages that children aged 7


months to 5 years go through when they are
separated from a primary caregiver for a long
time?

90. 1. Provide antibacterial action 2. Lubricate 3. Begin


CHO digestion 4. Begin fat digestion

What are the four functions of saliva?

91. 1. Recognize self from nonself 2. Amplify via cell


division or complementation 3. Control the level of the
response 4. Remove foreign material

What are the four major functions of the


acquired immune system?

92. 1. Sartorius 2. Gracilis 3. Semitendinous

What three muscles constitute the pes


anserinus?

93. 1. Short attention span 2. Impulsivity 3. Hyperactivity

What are the three characteristics of ADHD?

94. 1. Sideroblastic anemias (i.e., porphyrin and heme


synthesis disorders) 2. Thalassemia 3. Iron deficiency 4.
Lead poisoning

What are the four reasons for hypochromic


microcytic anemia with a low MCV?

95. 1. Skin 2. Liver 3. Kidney

What three organs participate in production of


vitamin D?

96.

1. Skin color 2. Heart rate 3. Reflexes 4. Muscle tone 5. Respiratory rate APGAR,
Appearance, Pulse, Grimace, Activity, Respiration

What five things are


checked in the
APGAR test?

97.

1. Sleep paralysis 2. Hypnagogic hallucinations (while falling asleep) 3. Sleep


attacks with excessive daytime sleepiness 4. Cataplexy (pathognomonic)
Narcolepsy is a disorder of REM sleep, with REM occurring within 10 minutes of
sleep.

Name the four


components of the
narcoleptic tetrad.

98.

1. Steroids 2. Alcohol 3. Scuba diving 4. Sickle cell anemia

What are the four


most common
causes of femoral
head necrosis?

99.

1. Streptomycin 2. Gentamicin 3. Tobramycin

Which three
aminoglycosides
have vestibular
toxicity?

100. 1. Sustained release 2. IV drip 3. Phenytoin 4. Alcohol 5. Aspirin toxicity

Name five causes


for zero-order
metabolism.

101. 1. Sweat glands 2. Arterioles 3. Veins

In what three areas


of the body are
sympathetics the
predominant tone?

102. 1. Temporal 2. Zygomatic 3. Buccal 4. Mandibular 5. Cervical (Two Zebras Bit My


Clavicle.)

What are the five


terminal branches
of the facial nerve?

103. 1. The child poses an imminent danger to self or others. 2. The child is unable to
self-care daily at the appropriate developmental level. 3. The parents or guardians
have no control over the child or will not promise to ensure the child's safety even
though they refuse hospitalization.

What three
circumstances allow
a child to be
committed to
institutional care?

104. 1. The lesser omentum (consisting of the hepatoduodenal and hepatogastric


ligaments) 2. Falciform ligament 3. Coronary ligament of the liver 4. Triangular
ligament of the liver Liver is ventral; all other ligaments are dorsal mesentery
derivatives.

Name the four


ventral mesentery
derivatives.

105. 1. Tight junctions 2. Capillaries that lack fenestration 3. Very selective


pinocytosis by the capillaries

What are the three


reasons for the
effectiveness of the
blood-brain barrier?

106. 1. Tinea coli 2. Haustra 3. Epiploic appendages

What are the three


anatomic characteristics
that differentiate the large
bowel from the small
bowel and the rectum?

107. 1. TTP 2. Lipoic acid 3. Coenzyme A from pantothenate 4. NAD(H) (from


niacin or tryptophan) 5. FADH2 (from riboflavin)

What five cofactors and


coenzymes are required
by pyruvate
dehydrogenase?

108. 1. VSD 2. ASD 3. PDA

What are the three left-toright shunts?

109. 1. What did the patient want? 2. What would the patient say? 3. What is in the
patient's best interests?

What are the three


surrogate criteria?

110.

1. -Ketoglutarate dehydrogenase 2. Pyruvate dehydrogenase 3.


Transketolase Thiamine pyrophosphate (TPP) functions as a coenzyme vital
to tissue respiration. It is required for the oxidative decarboxylation of
pyruvate to form acetyl-coenzyme A, providing entry of oxidizable substrate
into the Krebs cycle for the generation of energy

With what three enzymes


is thiamin pyrophosphate
(TPP) associated?

111.

2.0 or greater

What lecithin:
sphingomyelin ratio
indicates lung maturity?

112.

2, 4-Dinitrophenol

What attaches to protons


and allows them to enter
into the mitochondria
without going through the
ATP-generating system?

113.

2 acetyl CoA per glucose, producing 12 ATPs per


acetyl CoA, resulting in a total of 24 ATPs produced
from glucose (via acetyl CoA) enter the TCA cycle

How many acetyl CoAs per glucose enter into the


TCA cycle?

114.

2 ATPs, because RBCs use only anaerobic


metabolism.

How many ATPs per glucose are generated from


glycolysis in RBCs?

115.

2 ATPs by oxidative phosphorylation

How many ATPs are produced from cytoplasmic


NADH oxidation using the glycerol phosphate
shuttle?

116.

2 ATPs, via the amino acyl tRNA synthase enzyme

How many high-energy bonds are used to


activate an AA?

117.

2 days prior to ovulation

How many days prior to ovulation does estradiol


peak in the menstrual cycle?

118.

2 NADPHs per acetyl CoA

How many NADPHs are used per addition of


acetyl CoA into a fatty acid chain?

119.

2% of population, 2 cm long, 2 feet from ileocecal


valve, 2 years old, and 2% of carcinoid tumors

What are the rules of 2 for Meckel diverticulum?

120. 2 to 4 hours

How long is the transit time through the small


intestine?

121. 3'-5' direction, synthesizing RNA in the 5'-3' direction

What direction does RNA polymerase move


along the template strand of DNA during
transcription?

122. 3 ATPs by oxidative phosphorylation

How many ATPs are produced from cytoplasmic


NADH oxidation using the malate shuttle?

123. 3'end of the codon (third position) on mRNA and 5'


end of the anticodon (first position) on tRNA.

Where on the codon and anticodon does the


wobble hypothesis take place?

124. 3' end. Phosphate (PO4) is at the 5' end.

Is the hydroxyl (-OH) end of DNA and RNA at the


3' or the 5' end?

125. 3 months of symptoms in 2 consecutive years

How many months in how many years must a


person cough with copious sputum production for
the diagnosis of chronic bronchitis to be made?

126. 3-Phosphoglycerate kinase and pyruvate kinase; this


produces two ATPs per enzyme (total four ATPs)

What two glycolytic enzymes catalyze the


substrate-level phosphorylations?

127. 3 to 4 days

How long is the transit time through the large


intestine?

128. 3 years old

By what age should children be able to draw


the following figures? Circle

129. 4.5 years old (Alphabetic order except with a diamond


last: circle, cross, rectangle, square, triangle)

By what age should children be able to draw


the following figures? Rectangle

130. 4.7 L, 5% of body weight

What is the amount in liters and percent body


weight for the following compartments?
Vascular fluid

131. 4 years old

By what age should children be able to draw


the following figures? Cross

132. 5' deiodinase

What enzyme is inhibited by propylthiouracil


(PTU)?

133. 5HT1D

Of what serotonin receptor is sumatriptan an


agonist?

134. 5-hydroxyindoleacetic acid (5-HIAA)

What urinary metabolite is increased in


patients with carcinoid syndrome?

135. 5 years old

By what age should children be able to draw


the following figures? Square

136. 6 ATPs (remember 2 pyruvates per glucose are


produced, and 2 NADHs result from production of acetyl
CoA, so 6 ATPs)

How many ATPs per glucose are produced by


pyruvate dehydrogenase?

137. 6 weeks 10 weeks

When does the primitive gut herniate out of


the embryo? When does it go back into the
embryo?

138. 6 years old

By what age should children be able to draw


the following figures? Triangle

139. 7 to 14 days

What is a plasma cell's life expectancy?

140. 7 years old

By what age should children be


able to draw the following
figures? Diamond

141. 8 to 25 hours

How long after ovulation does


fertilization occur?

142. 8 weeks (response to a face)

At what age does a child develop


Exogenous smile?

143. 9.3 L, 15% of body weight

What is the amount in liters and


percent body weight for the
following compartments?
Interstitial fluid

144. 9 kcal/g from fat metabolism; 4 kcal/g from both CHO and protein
metabolism

How many kilocalories per gram


are produced from the
degradation of fat? CHO?
Protein?

145. 10% are bilateral, 10% malignant, and 10% familial, 10% in children,
and 10% outside the adrenal gland

What are the rules of 10


regarding pheochromocytoma?

146. 11-Beta-Hydroxylase deficiency results in excess production of 11deoxycorticosterone, a weak mineralocorticoid. It increases BP, Na+,
and ECF volume along with production of adrenal androgens.

What adrenal enzyme deficiency


results in hypertension,
hypernatremia, and virilization?

147. 12 ATPs per acetyl CoA that enter the tricarboxylic acid (TCA) cycle
(Krebs cycle)

How many ATPs are generated


per acetyl coenzyme A (CoA)?

148. 12 days after ovulation

If no fertilization occurs, how


many days after ovulation does
the corpus luteum begin to
degenerate?

149. 12 to 16 weeks (in response to mother's face)

At what age does a child develop


Preferential smile?

150. 14 days

What is the bone marrow


maturation time for a phagocytic
cell?

151. 14 days in most women (Remember, the luteal


phase is always constant.)

How many days before the first day of menstrual


bleeding is ovulation?

152. 14 L, 33% of body weight

What is the amount in liters and percent body weight


for the following compartments? ECF

153. 16S subunit

What part of the 30S ribosome binds to the ShineDalgarno sequence?

154. 17-alpha-Hydroxylase deficiency

What adrenal enzyme deficiency results in


hypertension, hypernatremia, increased ECF volume,
and decreased adrenal androgen production?

155. 18 years old (except if emancipated)

What is the legal age to be deemed competent to make


decisions?

156. 20:1T4T3. There is an increase in the


production of T3 when iodine becomes deficient.

What is the ratio of T4:T3 secretion from the thyroid


gland?

157. 21--Hydroxylase deficiency leads to


hypotension, hyponatremia, and virilization.

What adrenal enzyme deficiency can be summed up


as a mineralocorticoid deficiency, glucocorticoid
deficiency, and an excess of adrenal androgens?

158. 28 L, 40% of body weight

What is the amount in liters and percent body weight


for the following compartments? ICF

159. 31 pairs

How many pairs of spinal nerves exit from the spinal


cord?

160. 38 ATPs if aerobic, 2 ATPs if anaerobic (36


ATPs[malate shuttle] + 4 ATPs[Glycolysis] - 2
ATPs[phosphorylate glucose] = 38 ATPs)

How many ATPs per glucose are generated in


glycolysis?

161. 42 L, 67% of body weight

What is the amount in liters and percent body weight


for the following compartments? Total body water

162. 45XO

What is the karyotype in Turner syndrome?

163. 50%. The uncles and older siblings are the most likely
perpetrators, although stepfathers also have a high rate.

What percent of sexual abuse cases are


committed by family members?

164. 50%, with 50% of them having the child

What percent of unwed mothers are


teenagers?

165. 60%

What is the PO2 of aortic blood in fetal


circulation?

166. 60S and 40S. The large subunits (60S) are made in the
nucleolus and the small subunits (40S) are made in the
nucleus.

What ribosomal subunit sizes do eukaryotic


cells have?

167. 61 codons code for AAs and 3 codons (UAA, UGA, UAG)
code for the termination of translation.

How many codons code for AAs? How


many for termination of translation?

168. 70S ribosomes in prokaryotes and 80S ribosomes in


eukaryotes

What is the size of the prokaryotic


ribosome?

169. 80S ribosomes (because they are eukaryotic)

What size ribosomes do fungi have?

170. 90% as HCO3-, 5% as carbamino compounds, and 5% as


dissolved CO2

What percentage of CO2 is carried in the


plasma as HCO3- ?

171. 90% of benign myxomas arise within the left atrium near
the fossa ovalis.

What is the most common cardiac tumor?


In what chamber does it most commonly
arise?

172. 95% in the kidneys; that's why adequate Na+ and fluid
intake is essential.

Where is lithium metabolized and excreted?

173. 100%; the percentage of blood flow through the pulmonary


and systemic circulations are equal.

What percentage of cardiac output flows


through the pulmonary circuit?

174. 100%, with about 20% remaining positive after 1 year

What percentage of children born to HIVpositive mothers will test positive for HIV at
birth?

175. 150 mcg/day is the minimal daily intake needed. Most


people ingest 500 mcg/day.

How much dietary iodine is necessary to


maintain normal thyroid hormone
secretion?

176. 0157:H7

What is the most common serotype of


enterohemorrhagic Escherichia coli?

177. 300 mOsm/L

What are the following changes seen in the


luminal fluid by the time it leaves the PCT of the
nephron? Osmolarity

178. About 80% of stage 4 sleep is recovered,


approximately half of REM is recovered, and only
one-third of total sleep is ever made up.

When attempting to make up sleep, what stage of


sleep is recovered?

179. Absolutely not! Answer any question you are asked.

If a patient asks you a question and you do not


know the answer, do you tell a white lie or simply
not respond?

180. Absolutely not. Diabetics who are given a -blocker


for an acute chest syndrome have significantly better
outcomes than those who do not.

Is DM a contraindication to use of a -blocker in a


patient who is having an MI?

181. Absolute refractory period is due to voltage


inactivation of sodium channels.

Name the period described by the following


statement: no matter how strong a stimulus is, no
further action potentials can be stimulated.

182. Abs to HBsAg

Abs to what hepatitis B Ag provide immunity?

183. Acamprosate (the number of glutamate receptors


increases with chronic alcohol abuse)

What medication is used to help alcoholics avoid


relapse by decreasing glutamate receptor activity?

184. Acanthosis nigricans

What is the term for thickened, hyperpigmented


skin in the axillae, groin, and skin folds associated
with malignancies, obesity, and DM?

185. Acarbose

What oral hypoglycemic agent inhibits glucosidase in the brush border of the small
intestine?

186. Accessory nucleus

What is the only CN nucleus found in the cervical


spinal cord?

187. Accidents

What is the MCC of death in children aged 1 to 14


in the United States?

188. Accommodation

What term describes the reflex that increases


the curvature of the lens, allowing near vision?

189. Accuracy (think of it as all the trues, because they are


the ones correctly identified)

What is the term for the total percentage of


correct answers selected on a screening test?

190. Acebutolol and Pindolol

What two -blockers decrease serum lipids?

191. ACE inhibitors (-pril)

Which antihypertensive class is used to help


stop the progression of microalbuminuria?

192. ACE inhibitors (the-pril agents)

Which class of antihypertensives produces


angioedema, hyperkalemia, and a dry cough?

193. Acetaldehyde dehydrogenase

What enzyme is inhibited by disulfiram?

194. Acetaldehyde dehydrogenase. When this enzyme is


blocked, acetaldehyde builds up, and its presence in
excess results in nausea and hypotension.

What enzyme is inhibited by disulfiram?

195. Acetazolamide

What is the only class of diuretics to retain Clused in the short-term treatment of glaucoma
and of acute mountain sickness?

196. Acetoacetate, acetone, and -hydroxybutyrate

Name the three ketone bodies.

197. Acetylcholinesterase breaks ACh into acetate and


choline (which gets resorbed by the presynaptic nerve
terminal).

What enzyme found in a cholinergic synapse


breaks down ACh? What are the byproducts?

198. Acetyl CoA and propionyl CoA (in odd chain fatty acids)

Name the end product or products: Fatty acid


oxidation

199. Acetyl CoA carboxylase

What is the rate-limiting step of the following?


Fatty acid synthesis

200. Acetyl CoA carboxylase

What enzyme catalyzes the rate-limiting step


in fatty acid synthesis?

201. Acetyl CoA, FADH2, and NADH

At the end of each round of -oxidation, what


is released?

202. Acetyl CoA (pyruvate to acetyl CoA)

What is the main inhibitor of pyruvate dehydrogenase?

203. ACh

What are the primary neurotransmitters at the following sites?


All motor neurons, postganglionic parasympathetic neurons

204. ACh

What are the primary neurotransmitters at the following sites?


Autonomic preganglionic neurons

205. Achalasia

What occurs when the lower esophageal sphincter fails to relax


during swallowing due to abnormalities of the enteric nervous
plexus?

206. Achalasia. (Think Chagas disease if it


presents in a person from Central or
South America.)

What disorder is defined by inability of the lower esophageal


sphincter to relax with swallowing and a bird beak barium
swallow result?

207. ACh, histamine, and gastrin

What three substances stimulate parietal cells?

208. Achondroplasia

What AD disorder due to a mutation in fibroblast growth factor


receptor 3 results in normal-size vertebral column and skull and
short, thick extremities?

209. Achondroplasia

What is the most common form of inherited dwarfism?

210. ACh receptor

To what host cell receptor does the rabies virus attach?

211.

Name the neurotransmitter at the neuromuscular junctions for


all of the voluntary muscles in the body.

ACh; think about the ANS.

212. Acid is needed for the activation of


pepsin and therefore needed for
protein digestion.

What pH (acidotic or alkalotic) is needed for pepsinogen to


pepsin conversion?

213. ACL

What structure of the knee is described thus? Prevents


anterior displacement and has lateral-to-medial attachment on
the tibia

214. Acral-lentiginous melanoma

What is the most common one? Melanoma in dark-skinned


persons

215. ACTH

What hormone excess produces adrenal hyperplasia?

216. Acting out

Outburst to cover up true feelings (emotion is covered, not


redirected).

217. Actinomyces israelii

What gram-positive anaerobic rod with branching


filaments is a component of the normal flora of the
mouth and female genital tract and is responsible
for draining abscesses with sulfur granules in the
exudates?

218. Action potential

What type of potential is characterized as being an


all-or-none response, propagated and not
summated?

219. Actually, yes. You should encourage your patient to


try other forms of medicine as long as they are not
contraindicated with the patient's preexisting illness.
You must be able to accept the health beliefs of your
patients, even if you don't agree.

Should you refer a patient to a form of folk


medicine even if you don't believe in it?

220. Acute CO poisoning

With what is cherry red intoxication associated?

221. Acute cystitis

WBCs in the urine

222. Acute glomerulonephritis

RBC casts in the urine

223. Acute pyelonephritis

WBC casts in the urine

224. Acute rejection

Based on the following information, is the renal


transplantation rejection acute, chronic, or
hyperacute? Weeks to months after
transplantation; abrupt onset of oliguria and
azotemia; seen as neutrophilic vasculitis and
interstitial lymphocytes

225. Acute toxic or viral nephrosis

Renal epithelial casts in the urine

226. Acute tubular necrosis

Name the most common type or cause. Acute


renal failure in the United States

227. Adaptive branch. The adaptive branch of the


immune system has a slow initiation with rapid
responses thereafter.

What branch of the immune system is acquired in


response to an Ag?

228. Addition (They are mutually


exclusive.)

If one event precludes another event, their probabilities are combined by


what method?

229. Adenine and guanine

Name the two purine bases found in both DNA and RNA.

230. Adenocarcinoma

What is the most common one? Primary lung cancer

231. Adenocarcinoma (30% to


35%)

What is the most common one? Primary malignant tumor of the lungs

232. Adenocarcinoma (because


of Barrett esophagus)

What is the most common one? Esophageal cancer in the United States

233. Adenocarcinoma of the


cervix

What is the most common one? Primary malignant tumor of the female
genital tract in the United States

234. Adenosine deaminase

What enzyme of the purine salvage pathway is deficient in the following?


SCID

235. Adenosine deaminase


(ADA) deficiency

What is the first human disease successfully treated with gene therapy?

236. Adenosine (it is an excellent


agent to use in emergencies
because its half-life is 30
seconds)

What antiarrhythmic agent is the DOC for treating and diagnosing


paroxysmal supraventricular tachycardia (PSVT) and AV nodal
arrhythmias?

237. Adenovirus

Name the DNA virus: Linear dsDNA; naked; replicates in the nucleus

238. Adenovirus

What is the MCC of viral conjunctivitis?

239. Adhesions and hernias

Name the most common type or cause. Intestinal obstructions in adults

240. ADH is secreted in


response to increased
plasma osmolarity and
decreased blood volume.

ADH is secreted in response to what two stimuli?

241. Adjustment disorder. It is a


diagnosis of exclusion (used
if no other choice).

What axis I disorder is characterized by a clinically significant syndrome


that affects social, occupational, and/or academic achievement; occurs
less than 3 months after a stressor; and abates less than 6 months after
the stressor is removed?

242. Adrenal medulla

What is the only organ in the body supplied by


preganglionic sympathetic fibers?

243. Adrenarche

What is the term to describe the increased rate of


secretion of adrenal androgens at the onset of
puberty?

244. Adulthood

What stage of male development is characterized by


the following LH and testosterone levels? LH
secretion drives testosterone production, with both
levels paralleling each other.

245. Adult polycystic kidney disease

What AD renal disorder is associated with mutations of


the PKD 1 gene on chromosome 16 and berry
aneurysms in the circle of Willis and presents in the
fifth decade with abdominal masses, flank pain,
hematuria, HTN, and renal insufficiency?

246. Adult tapeworms develop in the definitive host,


whereas cysticerci or larvae develop in the
intermediate host.

Where do adult tapeworms develop, in the


intermediate or definitive host?

247. Adult T-cell leukemia

What leukemia is associated with four-leaf-clover


lymphocytes on peripheral blood smear?

248. Aedes mosquito

What is the vector for yellow fever?

249. Aedes (the same for yellow fever)

What mosquito is the vector for dengue fever?

250. Afferent arteriole

Are the JG cells of the nephron a part of the afferent or


efferent arteriole?

251. Afferent pupillary defect (CN II lesion); in an


efferent pupillary defect (CN III), B/L constrict
when light is shined in the unaffected eye and
consentual pupil constriction occurs when light is
shined in the affected eye.

Is an afferent or efferent pupillary defect described as


B/L pupillary constriction when light is shined in the
unaffected eye and B/L paradoxical dilation when light
is shined in the affected eye?

252. Affinity

What type of binding occurs with one Fab or one


idiotype of IgG?

253. Affinity decreases; they are inversely


proportional.

What happens to affinity if you increase Km?

254. The affinity of the drug for its


receptor

What is the most important determinant of drug potency?

255. Affinity (one of each)

What is the term for the strength of the association between Ag


and an Ab?

256. Afterload

What is the term for the load a muscle is trying to move during
stimulation?

257. Afterload

What is the term for the force the ventricular muscle must
generate to expel the blood into the aorta?

258. Afterload

What equals the total tension on a muscle minus the preload?

259. After the eclipse period

Regarding the viral growth curve, is the internal virus present


before or after the eclipse period?

260. After the latent period

Regarding the viral growth curve, is the external virus present


before or after the latent period?

261. Ag-Ab complexes. The alternative


pathway protects without use of Abs;
the pathogen is the stimulus.

What is the stimulus for the classical pathway activation?

262. Aged adult

What stage of male development is characterized by the following


LH and testosterone levels? Decreased testosterone production
is accompanied by an increase in LH production.

263. Agents that decrease ACh function


reduce tremors and rigidity, not
bradykinesias.

Do agents used in the treatment of Parkinson's disease that


decrease ACh function have little effect on reducing tremors and
rigidity?

264. Agglutination test

What immunologic test checks for a reaction between Abs and a


particular Ag? (hint: ABO typing)

265. Agoraphobia. It also means having a


sense of humiliation or
hopelessness.

What phobia is described as the fear of open spaces?

266. Ag processing; it is needed for class I


molecules. Class II molecules have an
invariant chain that protects them from
breakdown.

What is the term for processing an APC's pinocytosed


material by fusing with a lysosomal granule and cleaving the
Ag into peptide fragments?

267. Agranulocytosis; approximately 2%


develop this side effect.

What potentially lethal side effect of clozapine should be


monitored with frequent blood drawing?

268. Air flows into the respiratory system.

What result occurs because of the negative alveolar


pressure generated during inspiration?

269. Akathisia

What antipsychotic movement disorder can occur at any


time and is characterized by a subjective sense of
discomfort that brings on restlessness, pacing, sitting down,
and getting up?

270. ALA dehydrogenase and ferrochelatase

What two enzymes of heme synthesis are inhibited by lead?

271. Alanine and glutamine

As what AAs do muscles send nitrogen to the liver?

272. Alanine cycle

In what cycle does glucose go to the muscle, where it is


converted to pyruvate and then into alanine before being
taken back to the liver?

273. Al-Anon

What is the name of the program that deals with


codependency and enabling behaviors for family members
of alcohol abusers?

274. Albendazole

What drug blocks glucose uptake, leading to decreased


formation of ATP and resulting in immobilization of the
parasite?

275. Albinism

What disorder is due to a deficiency in tyrosinase?

276. Albinism. Tyrosine is converted to melanin


by the enzyme tyrosinase.

What disease is produced by a deficiency in the enzyme


tyrosinase?

277. Albumin

What protein carries free fatty acids to the liver?

278. Alcohol abuse

Name the most common type or cause. Reversible HTN in


the United States

279. Alcohol abuse

Name the most common cause. Chronic pancreatitis

280. Alcohol and its related problems cost the


country approximately $100 billion a year.

What is the costliest health care problem in the United


States?

281. Alcohol consumption

Name the most common type or cause. Cirrhosis in the


United States

282. Alcoholic cirrhosis

Name the most common type or cause. Liver


transplantation in adults

283. Alcoholics Anonymous

What is the name of the 12-step program believed to be the


most successful for the treatment of alcohol abuse?

284. Alcoholism

Name the most common type or cause. Mallory-Weiss


syndrome

285. Alcohol. Nearly 10% of adults have a


problem with alcohol.

What is the most abused drug across all age groups?

286. Aldehyde dehydrogenase

What enzyme is blocked by disulfiram?

287. Aldolase B deficiencies are treated by


eliminating fructose from the diet.

What fructose metabolism enzyme is deficient in patients


with vomiting, apathy, diarrhea, jaundice, proximal renal
tubular acidosis, hypoglycemia, and hyperuricemia?

288. Aldosterone Remember, from the outer


cortex to the inner layer, Salt, Sugar, Sex.
The adrenal cortex gets sweeter as you go
deeper.

What is the major hormone produced in the following areas


of the adrenal cortex? Zona glomerulosa

289. Alendronate

What is the drug of choice for steroid-induced osteoporosis?

290. Alkaline phosphatase

What enzyme is associated with osteoblastic activity?

291. ALL

What leukemia affects a 4-year-old child with 3 months of


fever, fatigue, generalized lymphadenopathy, CNS
involvement, hepatosplenomegaly, bleeding, and platelet
count below 100, 000?

292. ALL

What is the most common one? Malignancy in children

293. All are lactose fermenters except Yersinia and


Proteus.

Which of the following Enterobacteriaceae family


membersYersinia, Klebsiella, Enterobacter,
Escherichia, Proteus, and Citrobacter do not ferment
lactose?

294. Allelic exclusion. It is to ensure that one B cell


produces only one Ab.

What is the name of the process that ensures that


each B cell produces only one heavy-chain variable
domain and one light chain?

295. ALL (nearly 15-20 times the normal risk)

What hematological malignancy is particularly likely to


affect patients with Down syndrome?

296. Allograft

Name the type of graft described by these transplants:


From one person to the next (the same species)

297. Allopurinol

What is the drug of choice for treating a patient with


hyperuricemia due to overproduction of uric acid?

298. Allotypes

What are the genetic variants of a molecule within


members of the same species?

299. alpha2-Receptors

What presynaptic receptor does NE use to terminate


further neurotransmitter release?

300. alpha-Cells; glucagon has stimulatory effects on


-cells and inhibitory effects on -cells.

What pancreatic islet cell secretes glucagons?

301. alpha-Motor neurons innervate extrafusal


muscle fibers (a motor unit), whereas gammamotor neurons innervate intrafusal muscle
fibers.

Do alpha-or gamma-motor neurons innervate


extrafusal muscle fibers?

302. Alteplase

Which thrombolytic agent, activated in the presence of


fibrin, is manufactured by recombinant DNA process?

303. Alternative hypothesis (what is left after the null


has been defined)

What is the name of the hypothesis you are trying to


prove?

304. Although both pathways are associated with disinhibition,


the indirect basal ganglia pathway is associated with a
decreased level of cortical excitation.

Does the direct or indirect basal ganglia


pathway result in a decreased level of
cortical excitation?

305. Altruism

Name these mature defense mechanisms:


Helping others without expecting any return

306. Alveolar dead space

What is the term for ventilation of


unperfused alveoli?

307. Alveolar macrophages

Name the macrophages by location: Lungs

308. Alveolar macrophages (dust cells)

In the alveoli, what cell type is part of the


mononuclear phagocytic system?

309. The alveoli at the apex are almost completely inflated prior
to inflation, and although they are large, they receive low
levels of alveolar ventilation.

Why is the apex of the lung hypoventilated


when a person is standing upright?

310. The alveoli at the base are small and very compliant, so
there is a large change in their size and volume and
therefore a high level of alveolar ventilation.

Why is the base of the lung hyperventilated


when a person is standing upright?

311.

Which of the following is not part of the


conducting portion of the respiratory system:
trachea, bronchi, alveoli, or larynx?

Alveoli; they are part of the respiratory portion.

312. Alzheimer disease

Senile plaques

313. Alzheimer disease

Neurofibrillary tangles

314. Alzheimer disease

Name the most common cause. Dementia


in persons aged 60 to 90 years

315. Alzheimer's (dementia of Alzheimer's type, DAT).


(Remember, Alzheimer's constitutes 65% of dementias
seen in patients 65 years old.)

What is the most common form of


dementia?

316. Amanita phalloides

What mushroom poisoning is associated


with fulminant hepatitis with extensive liver
necrosis?

317. Amantadine

What antiviral agent blocks the attachment and penetration of


influenza A virus?

318. Amantadine

What antiviral agent is used in the treatment of drug-induced


Parkinson's disease?

319. Amiloride

What is the treatment for nephrogenic diabetes insipidus induced


by lithium?

320. Amiloride

Which sodium channel blocker is used to treat lithium-induced


diabetes insipidus?

321. Aminoacyl-tRNA synthetase, which


uses 2 ATPs for this reaction.

What enzyme catalyzes the covalent bonding of the AA's carboxyl


group to the 3' end of tRNA?

322. Aminoglycosides

Which drugs bind to the 30S ribosomal subunit and interfere with
the initiation complex, causing a misreading of mRNA?

323. Aminoglycosides

Name the antimicrobial agent whose major side effect is listed.


CN VIII damage (vestibulotoxic)

324. Aminophylline

What phosphodiesterase inhibitor is used IV in the management of


status asthmaticus ?

325. AML-M3

Auer rods

326. Amnesia. (The person is aware of


the memory loss.)

What term describes the inability to recall personal information,


commonly associated with trauma?

327. Amphotericin B

What is the DOC for severe infections with Sporothrix, Mucor,


Histoplasma, Cryptococcus, Candida, and Aspergillus?

328. Ampicillin and amoxicillin

What are the two broad-spectrum penicillins?

329. Ampulla

What is the region of the fallopian tube where fertilization most


commonly occurs?

330. Ampulla of the fallopian tube

Where is the most frequent site of ectopic pregnancy?

331. Amygdala; it helps imprint an


emotional response in memory.

What area of the limbic system is responsible for attaching


emotional significance to a stimulus?

332. Amylase hydrolyzes alpha-1, 4-glucoside


linkages, forming alpha-limit dextrins, maltotriose,
and maltose.

What linkage of complex CHOs does pancreatic


amylase hydrolyze? What three complexes are
formed?

333. Amyl nitrate, sodium nitrate, or sodium thiosulfate

Name the antidote. Cyanide

334. Amyloidosis

Name the most common type or cause. Restrictive


cardiomyopathy

335. Amyotrophic lateral sclerosis (Lou Gehrig's


disease) is a LMN lesion at the level of the lesion
and UMN lesion below the level of the lesion.

What disease is described by bilateral flaccid


weakness of the upper limbs (LMN) and bilateral
spastic weakness of the lower limbs (UMN)
beginning at the cervical level of the spinal cord and
progressing up or down the cord?

336. Anatomical dead space, which ends at the level of


the terminal bronchioles.

What is the term for any region of the respiratory


system that is incapable of gas exchange?

337. Androgen-binding protein

What hormone, produced by the Sertoli cells, is


responsible for keeping testosterone levels in the
seminiferous tubules nearly 50 times that of the
serum?

338. Androgens are 19-carbon steroids.

How many carbons do androgens have?

339. Anemia

What toxicities are caused by the following agents?


Occupational nitrous oxide exposure

340. Anemia of chronic disease

Name the hypochromic microcytic anemia based on


the following laboratory values. Decreased iron,
TIBC, and percent saturation; increased ferritin

341. Anemic patients have a depressed O2 content


because of the reduced concentration of Hgb in
the blood. As for polycythemic patients, their O2
content is increased because of the excess Hgb
concentrations.

Why is O2 content depressed in anemic patients?

342. Angiosarcoma of the liver

Name the cancer associated with the following chemical agents. (Some
may have more than one answer.) Vinyl chloride

343. Anhedonia

What is the term to describe the inability to feel any pleasant emotions?

344. Ankylosing spondylitis

What seronegative spondyloarthropathy is seen in HLA-B27-positive


young men, involves the sacroiliac joints, has no subcutaneous nodules,
and has a bamboo spine appearance on radiograph?

345. Anopheles mosquito

What mosquito is responsible for the transmission of malaria?

346. Anopheles mosquito

What vector is associated with malaria?

347. Anorgasmia. (The overall


prevalence is 30%.)

What is the term for an inhibited female orgasm?

348. Anterior compartment (it's


the blood supply to the
posterior compartment)

In what compartment of the thigh is the profundus femoris artery found?

349. Anterior compartment of the


leg, deep peroneal nerve

Name the compartment of the lower extremity and the nerve based on its
movements. Dorsiflex the foot, extend the toes, and invert the foot

350. Anterior compartment of the


thigh, femoral nerve

Name the compartment of the lower extremity and the nerve based on its
movements. Flex the hip and extend the knee

351. Anterior hypothalamic zone;


lesions here result in
hyperthermia.

What area of the hypothalamus is responsible for recognizing an increase


in body temperature and mediates the response to dissipate heat?

352. Anterior limb of the internal


capsule is supplied by the
anterior cerebral artery.

What limb of the internal capsule is not supplied by the middle cerebral
artery?

353. Anterior nuclear group (Papez circuit


of the limbic system)

Name the thalamic nucleus based on its input and output. Input
from the mammillary bodies via the mammillothalamic tract and
the cingulated gyrus; output to the cingulated gyrus via the
anterior limb of the internal capsule

354. The anterior nucleus of the thalamus

What thalamic relay nucleus do the mammillary bodies project


to?

355. Anterior spinal artery

What branch off the vertebral artery supplies The ventrolateral


two-thirds of the cervical spinal cord and the ventrolateral part of
the medulla?

356. Anterior tibial artery

What artery turns into the dorsalis pedis when it crosses the
extensor retinaculum?

357. Anterior wall of the first part of the


duodenum

What is the most common one? Location of a duodenal ulcer

358. Anticipation

Name these mature defense mechanisms: Preparing for an


upcoming event

359. Anticipation, associated with fragile X


syndrome; Huntington's disease is
also associated with a decrease in
onset of age.

What is the term for the number of trinucleotide repeats


increasing with successive generations and correlating with
increased severity of disease?

360. Antidigoxin Fab fragments

Name the antidote. Digoxin

361. Antigenic determinant (epitope)

What is the term to describe the limited portion of an Ag that is


recognized by an Ab?

362. Antigenic determinant (epitope).


(Idiotypes bind to epitopes.)

What is the limited portion of a large Ag that will actually be


recognized and bound to an Ab and that contains approximately
five to six amino acids or four to five hexose units?

363. Anti-HAV IgG Abs are associated with


immunization or a prior infection. AntiHAV IgM is associated with acute or
recent infection.

Is an anti-HAV IgG Ab associated with immunization or recent


infection?

364. Antisocial

Name the cluster B personality disorder: Criminal behavior; lacking


friends, reckless, and unable to conform to social norms

365. Antisocial personality

What personality disorder affects 75% of the prison population?

366. Antithyroglobulin and


antimicrosomal Abs

What two Abs are used to diagnose Hashimoto's thyroiditis?

367. Anxiety disorders; for men it


is substance abuse.

Name the most common type. Psychiatric disorder in women of all ages

368. The aorta

What large-diameter vessel has the smallest cross-sectional area in


systemic circulation?

369. Aorta, azygos vein, and


thoracic duct Remember: 1
at T8, 2 at T10, and 3 at
T12

What structure or structures cross the diaphragm at T12 level?

370. The aorta has the greatest


velocity and the capillaries
have the slowest velocity.

What vessels in the systemic circulation have the greatest and slowest
velocity?

371. Aortic insufficiency

Name the valve abnormality based on the following criteria: Diastolic


murmur; increased preload, stroke volume, and aortic pulse pressure;
decreased coronary blood flow; no incisura; and peripheral vasodilation

372. Aortic stenosis

Name the valve abnormality based on the following criteria: Systolic


murmur, increased preload and afterload, decreased aortic pulse pressure
and coronary blood flow

373. Apneustic center (deep


breathing place)

What respiratory center in the caudal pons is the center for rhythm
promoting prolonged inspirations?

374. apo A-1

What apoprotein on HDL activates lecithin-cholesterol acyltransferase


(LCAT)?

375. apo B-48

What apoprotein is required for the release of chylomicrons from the


epithelial cells into the lymphatics?

376. APocrinE glands (APES is my memory aid) Axilla, Areola, and Anus all
begin with A. APES are hairy (associated with hair follicles). They smell
(odor production), and if confronted by an APE, your Adrenergic nervous
system would be firing (innervation).

What sweat gland type is


associated with odor
production and hair follicles
and is found in the axilla?

377. Apoptosis (Remember, there is a lack of inflammatory response.)

What is the term for


programmed cell death?

378. Appendix (second is terminal ileum)

What is the most common


one? Site for carcinoid
tumors

379. Arboviruses (bunyavirus, flavivirus, and togavirus)

What is the term given to


arthropod-borne viruses?

380. Arch of the aorta

Name the correct artery.


The left recurrent laryngeal
nerve passes around it.

381. Area 4

What Brodmann area is


associated with Primary
motor cortex?

382. Area 6

What Brodmann area is


associated with Premotor
cortex?

383. Area 8

What Brodmann area is


associated with Frontal
eye fields?

384. Area 17

What Brodmann area is


associated with Primary
visual cortex?

385. Area 22 and occasionally 39


and 40

What Brodmann area is associated with Wernicke's area?

386. Areas 1, 2, and 3

What Brodmann area is associated with Primary somatosensory


cortex?

387. Areas 5 and 7

What Brodmann area is associated with Somatosensory association


cortex?

388. Areas 18 and 19

What Brodmann area is associated with Visual association cortex?

389. Areas 41 and 42

What Brodmann area is associated with Primary auditory cortex?

390. Areas 44 and 45

What Brodmann area is associated with Broca's area?

391. These are the gross pathologic


changes associated with
Alzheimer's disease.

What dementia is associated with dilated ventricles with diffuse cortical


atrophy, decreased parietal lobe blood flow, and a decrease in choline
acetyl transferase activity?

392. The AR form is malignant and


AD is benign.

Is the AD or AR form of osteopetrosis malignant?

393. Arginine

What AA is a precursor of the following substances? N2O

394. Arginine

What AA is broken down into N2O, causing an increase in cyclic


guanosine monophosphate (cGMP) of smooth muscle, hence
vasodilation?

395. Arginine

What AA has a pKa of 13?

396. Argyll Robertson pupils

What is the term for pupils that react normally to accommodation but
have bilateral loss of constriction in response to light?

397. Argyll Robertson pupils accompany a loss of both direct


and consensual light reflexes, but the accommodationconvergence reaction remains intact. It can also be
seen in patients with pineal tumors or multiple
sclerosis.

What is the term for the type of pupil seen in


neurosyphilis, and what ocular reflexes are
lost?

398. Arnold-Chiari malformation type 2

What CNS developmental abnormality is


associated with downward displacement of the
cerebellar vermis and medulla compressing the
fourth ventricle and leading to obstructive
hydrocephalus?

399. Arnold-Chiari malformation type 2

What CNS developmental abnormality is


associated with 90% of syringomyelia?

400. Aromatase

What enzyme converts androgens to


estrogens?

401. Arsenic

Name the most common type or cause. Acute


metal poisoning in the United States

402. Arsenic, thorotrast, and vinyl chloride

What three chemical agents are associated


with angiosarcomas of the liver?

403. Arterial PCO2 levels are proportional to cerebral blood


flow.

Under resting conditions, what is the main


determinant of cerebral blood flow?

404. Arterioles

What component of the vascular system is


most sensitive to the effects of calcium channel
blockers?

405. Arterioles have the largest drop, whereas the vena


cava has the smallest pressure drop in systemic
circulation.

In the systemic circulation, what blood vessels


have the largest pressure drop? Smallest
pressure drop?

406. Arthroconidia with hyphae

A suspected dermatophyte infection is stained


with KOH. What spore type do you expect to
see?

407. ASA

What is the only NSAID that causes irreversible


inhibition of the cyclooxygenase pathway?

408. ASA (salicylate)

What commonly encountered overdose produces headache,


tinnitus, respiratory alkalosis, metabolic acidosis, confusion,
vomiting, and tachypnea?

409. Asbestosis

What pneumoconiosis is associated with exposure to the


following occupations or materials? Shipyards, brake linings,
insulation, and old building construction

410. Asbestosis

Name the most common type or cause. Nonorganic


pneumoconiosis

411.

With what two pathologies is a honeycomb lung associated?

Asbestosis and silicosis

412. Ascaris lumbricoides

What is the most common one? Helminthic parasite


worldwide

413. Ascending limb

What is the site of action of the following? Loop diuretics

414. Aschoff bodies of rheumatic fever

What foci of fibrinoid necrosis are surrounded by lymphocytes


and macrophages throughout all the layers of the heart?

415. Ascorbic acid (vitamin C) deficiency


(These are the signs of scurvy.)

What vitamin deficiency is evidenced by poor wound healing,


loose teeth, bleeding gums, petechiae, and ecchymosis?

416. ASD

What is the most common one? Congenital heart defect in


adults

417. As flow increases, the urine becomes


more dilute because of decreased time
for H2O reabsorption.

How are flow through the loop of Henle and concentration of


urine related?

418. Asparagine

What AA undergoes N-glycosylation?

419. Aspartame

What artificial sweetener must patients with PKU avoid?

420. Aspartate and carbomoyl PO4-

What are the two ways that nitrogen can enter into the urea
cycle?

421. Aspartate transcarbamylase

What enzyme catalyzes the rate-limiting step in pyrimidine


synthesis?

422. Aspartic acid and glutamic acid

What two AAs have a pKa of 4?

423. Aspiration

Name the most common type or cause. Lung abscess

424. Aspiration pneumonia

What is the most common one? Complication of nasogastric


tube feeding

425. As sample size increases, the lower the


chance of error.

What is the relationship between chance of error and


Sample size?

426. As the isotype is switched, the Ab


specificity does not change because it
does not affect the variable chains.

What happens to the Ab specificity when class switching


occurs in mature B cells?

427. As the name indicates, there is no


change in volume but there is an
increase in pressure.

What happens to intraventricular pressure and volume during


isovolumetric contraction?

428. As the standard deviation increases, the


greater the chance of error.

What is the relationship between chance of error and


Standard deviation?

429. Astrocytes

What cells contribute to the blood-brain barrier and proliferate


in response to CNS injury?

430. Astrocytoma

What CNS tumor cells stain positive for glial fibrillary acidic
protein (GFAP)?

431. Astrocytoma

Name the cancer associated with the following oncogenes.


(Some may have more than one answer) sis

432. At 8 to 9 years of age. Prior to this age


they view death as a form of punishment.

At what age do children begin to understand the irreversibility


of death?

433. A-T are linked by 2 hydrogen bonds, C-G


by 3 hydrogen bonds.

How many hydrogen bonds link A-T? C-G?

434. Ataxia-telangiectasia

What AR disorder is seen by age 1 to 2 with recurrent


sinopulmonary infections, uncoordinated muscle movements,
and dilation of the blood vessels?

435. At birth (reflex)

At what age does a child develop Endogenous smile?

436. AT II

What hormone constricts afferent and efferent arterioles


(efferent more so) in an effort to preserve glomerular capillary
pressure as the renal blood flow decreases?

437. AT II

What component of the renin-angiotensin-aldosterone


axis increases sodium reabsorption in the proximal
convoluted tubules and increases thirst drive?

438. At least 5 lobes

How many segments in a neutrophilic nucleus are


necessary for it to be called hypersegmented?

439. At least 15%

How far below ideal body weight are patients with


anorexia nervosa?

440. At least 30% blast in the bone marrow

What percentage of the bone marrow must be composed


of blast for leukemia to be considered?

441. At least six

How many caf-au-lait spots are necessary for the


diagnosis of neurofibromatosis type 1?

442. ATP/ADP Translocase

What component of the ETC is inhibited by the


following? Atractyloside

443. Atracurium

Which nicotinic receptor antagonist is inactivated to


laudanosine?

444. Atrial contraction Atrial, Contraction, Venous

On the venous pressure curve, what do the following


waves represent? A wave?

445. Atrial filling (venous filling) Atrial, Contraction,


Venous

On the venous pressure curve, what do the following


waves represent? V wave?

446. Atrophy. Disuse can also be due to


immobilization, ischemia, aging, and a host of
other causes.

What is the term to describe a decrease in the cell size


and function usually associated with disuse?

447. Atropine, 2-PAM (pralidoxime)

Name the antidote. Organophosphates

448. Atropine and 2-PAM (pralidoxime)

What is the antidote for organophosphate ingestion?

449. Atropine with pralidoxime

Name the antidote. AChE inhibitors

450. At the beginning of splay is when the renal


threshold for glucose occurs and the excess
begins to spill over into the urine.

On a graphical representation of filtration, reabsorption,


and excretion, when does glucose first appear in the
urine?

451. At the end of the PCT


25% of Na+, Cl-, K+ is
left

What are the following changes seen in the luminal fluid by the time it leaves
the PCT of the nephron? Percentage of Na+, Cl-, K+ left in the lumen

452. At the end of the PCT


25% of the original
volume is left

What are the following changes seen in the luminal fluid by the time it leaves
the PCT of the nephron? Percentage of original filtered volume left in the
lumen

453. Auspitz sign

What is the term for the sign revealed when a psoriatic scale is removed and
pinpoint bleeding occurs?

454. Autism. Head-banging,


rocking, and selfinjurious behaviors are
also common in autism.

What axis I disorder is characterized by pronoun reversal, preference for


inanimate objects, obliviousness to the external environment, lack of
separation anxiety, and abnormalities in language development?

455. Autograft

Name the type of graft described by these transplants: From one site to
another on the same person

456. Autoimmune hemolytic


anemia

What form of anemia is associated with IgG Abs against Rh antigens, positive
direct Coombs test, and splenomegaly?

457. Autoimmune hemolytic


anemia

What type II hypersensitivity disorder is defined as Autoantibodies directed


against RBC Ag I?

458. Autoimmune
thrombocytopenia
purpura

What type II hypersensitivity disorder is defined as Autoantibodies directed


against platelet integrin?

459. Autosomal dominant

Name the pattern of genetic transmission characterized thus: both M and F


are affected; M may transmit to M; each generation has at least one affected
parent; and one mutant allele may produce the disease.

460. Autosomal recessive

Name the pattern of genetic transmission: both M and F are affected; M-to-M
transmission may be present; both parents must be carriers; the trait skips
generations; two mutant alleles are needed for disease; and affected children
may be born of unaffected adults?

461. Average

What is the label given to an individual whose IQ is 90 to 109

462. Aversive conditioning

What classical conditioning therapy or modification is described as Pairing


noxious stimuli to an inappropriate behavior?

463. Avidity (more than one binding


site)

What is the term for the strength of association between multiple


Abbinding sites and multiple antigenic determinants?

464. Avidity. There is a positive


correlation between valence
numbers and avidity.

What is the term for the strength of the association between Ags and
Abs?

465. Avoidant

Name the cluster C personality disorder: Sensitive to criticism, shy,


anxious; socially isolated but yearns to be in the crowd

466. Axillary nerve and posterior


humeral artery

What nerve and artery could be affected in a humeral neck fracture?

467. Axis I

The DSM-IV-TR is scored on the basis of five axes of diagnosis. In


what axis would you place Clinical disorders (e.g., schizophrenia)?

468. Axis II

The DSM-IV-TR is scored on the basis of five axes of diagnosis. In


what axis would you place Personality and mental disorders?

469. Axis III

The DSM-IV-TR is scored on the basis of five axes of diagnosis. In


what axis would you place Medical or physical ailments?

470. Axis IV

The DSM-IV-TR is scored on the basis of five axes of diagnosis. In


what axis would you place Psychosocial and environmental
problems (stressors)?

471. Axis V

The DSM-IV-TR is scored on the basis of five axes of diagnosis. In


what axis would you place Global assessment of function?

472. Axon hillock

Where is the action potential generated on a neuron?

473. Azathioprine

What immunosuppressive agent is converted to 6-mercaptopurine?

474. Azithromycin

Which macrolide is given as 1200 mg/week for the prophylaxis of


Mycobacterium avium intracellulare complex?

475. The Babinski reflex is present in UMN lesions.


Muscle atrophy due to disuse, hyperreflexia,
spastic paralysis, increased muscle tone, and
weakness are commonly seen in UMN
lesions.

What reflex, seen in lesions of the corticospinal tract, is


an extension of the great toe with fanning the of
remaining toes?

476. Bacillus anthracis

What bacterium is characterized by large boxcar-shaped


gram-positive rods and is spore-forming, aerobic, and
associated with cutaneous infections and woolsorter's
disease?

477. Bacillus anthracis

What bacterium is responsible for woolsorter's disease?

478. Bacillus cereus

What organism would you suspect in a patient with


diarrhea after eating rice?

479. Bacteria

Which of the following characteristics accurately describe


fungi, bacteria, viruses, or parasites? Small prokaryotic
cells; no histones; 70S ribosomes; no sterols in cell
membrane; peptidoglycans in cell wall; replicate by
binary fission

480. Bacterial meningitis

What condition results in the following CSF results?


Opening pressure 400 mm H2O; WBCs 8500 (90%
PMNs); glucose 15; protein 120

481. Bactericidal

What form of antimicrobial therapy is best for an


immunocompromised patient?

482. Bainbridge reflex

What cardiac reflex is characterized by stretch receptors


in the right atrium, afferent and efferent limbs via the
vagus nerve, and increased stretch leading to an
increase in heart rate via inhibition of parasympathetic
stimulation?

483. Barbiturate overdose and benzodiazepine


withdrawal carry the highest mortality rates.

Which overdose carries a higher mortality rate, that of


benzodiazepines or barbiturates?

484. Barr bodies in females, T-cell receptor loci, and


immunoglobulin light and heavy chain loci

What are the three exceptions to the rule of


codominant gene expression?

485. Barrett esophagus has an increased risk of developing


adenocarcinoma of the esophagus.

What is the term for squamous to columnar


metaplasia of the distal esophagus secondary to
chronic inflammation?

486. Basal cell carcinoma

What is the most common one? Skin tumor in


the United States

487. Basal cell carcinoma

What is the most common one? Tumor on sunexposed sites

488. Basal ganglia

Name the area of the cerebral cortex with the


function described: Initiation and control of
movements

489. Baseline apical intrapleural pressure is -10 cm H2O


(more negative than the mean) resulting in a force to
expand the alveoli.

At the apex of the lung, what is the baseline


intrapleural pressure, and what force does it
exert on the alveoli?

490. The basilar artery is formed at the pontomedullary


junction.

What artery is formed by the union of the two


vertebral arteries?

491. Basophilic stippling

What is the term for cytoplasmic remnants of


RNA in RBCs, seen in lead poisoning?

492. B cells are mainly found in the peripheral white pulp


and germinal centers in the spleen.

What cell type is found in the peripheral white


pulp of the spleen?

493. B cells; macrophages and dendritic cells are


nonspecific.

What is the only specific Ag-presenting cell?

494. Bcl-2

What gene inhibits apoptosis by preventing the


release of cytochrome c from mitochondria?

495. Because creatinine is filtered and a small amount is


secreted

Why is the clearance of creatinine always slightly


greater than the clearance of inulin and GFR?

496. Because incidence is defined as new events; treatment does not


decrease the number of new events. It does decrease the number of
individuals with the event (prevalence would decrease).

Why isn't the incidence of a


disease decreased when a new
treatment is initiated?

497. Because it is the largest vessel and has the highest velocity in
systemic circulation

Why will turbulence first appear in


the aorta in patients with anemia?

498. Because RPF is markedly decreased, while GFR is only minimally


diminished; this results in an increase in FF (remember FF =
GFR/RPF).

Why is there an increase in FF if


the GFR is decreased under
sympathetic stimulation?

499. Because the chronic inhibition of dopamine (PIF) on the release of


prolactin from the anterior pituitary gland is removed, thereby
increasing the secretion of prolactin.

Why is there an increase in


prolactin if the hypothalamicpituitary axis was severed?

500. Because the large drop in TPR is accompanied by a large increase


in cardiac output, resulting in a minimal change in BP.

Why is there a minimal change in


BP during exercise if there is a
large drop in TPR?

501. Because the venous system is more compliant than the arterial
vessels, small changes in pressure result in large changes in blood
volume.

Regarding the venous system,


what happens to blood volume if
there is a small change in
pressure?

502. Because they do not block D2A receptors, they are antagonists of
5HT2 receptors.

Why are clozapine and olanzapine


unlikely to result in EPS effects?

503. Being a female physician increases the risk of suicide nearly four
times the general population.

Does being a female physician


increase or decrease the risk of
suicide?

504. Being awake

Name the stages of sleep with


these EEG patterns: Alpha
waves

505. Bell pad

What has proved to be the best


way to extinguish enuresis?

506. Bender Visual Motor Gestalt


Test

What neuropsychologic test shows nine designs to the patient, then


asks for recall of as many as possible?

507. Benign lentigo

What skin condition is a localized proliferation of melanocytes


presenting as small, oval, light brown macules?

508. Benign nevus (mole)

What is the term for a benign melanocytic tumor associated with sun
exposure that presents as tan-to-brown colored and has sharply
defined well-circumscribed borders?

509. Berger disease (IgA


nephropathy)

What type of GN, associated with celiac disease and dermatitis


herpetiformis, has mesangial deposits of IgA, C3, properdin, IgG, and
IgM?

510. Bernard-Soulier syndrome

What AR syndrome is due to a deficiency of glycoprotein Ib, resulting


in a defect in platelet adhesion?

511.

What aneurysm of the circle of Willis is associated with polycystic


kidney disease?

Berry aneurysm

512. Berry aneurysm in the circle of


Willis

Name the cerebral vessel associated with the following vascular


pathologies. Subarachnoid hemorrhage

513. Berylliosis

What pneumoconiosis is associated with exposure to the following


occupations or materials? Aerospace industry, nuclear reactors

514. Beryllium

What chemical can be dangerous if you work in the aerospace industry


or in nuclear plants?

515. Beta2-integrins

Which integrin mediates the adhesion to endothelial cells for migration


in and out of the blood during an immune response?

516. The Beta-2-microglobin. It is the


separately encoded Beta-chain
for class I Ags.

What molecule differentiates the MHC class I from II Ag? (Hint: it's in
the light chain.)

517. Between the renal arteries and


at the bifurcation of the
abdominal aorta

What is the most common one? Sites for abdominal aorta aneurysm

518. The bifurcation of the abdominal


aorta

What is the most common one? Site for atherosclerotic plaques in


the abdominal aorta

519. Bilateral degeneration of the dorsal columns in the spinal


cord secondary to syphilis is known as tabes dorsalis. A
high-step gait is seen in patients with tabes dorsalis
because of the inability to feel the ground beneath their
feet.

What sensory system is affected in the late


spinal cord manifestation of syphilis?

520. Bile acid sequestrants (cholestyramine and colestipol)


increase VLDL and triglyceride levels; therefore, they are
not used if a patient has hypertriglyceridemia.

Which class of antihyperlipidemics would you


avoid for a patient with elevated triglyceride
levels?

521. Bile salts are actively reabsorbed in the distal ileum.

Where in the GI tract does the reabsorption of


bile salts take place?

522. Bioavailability

What is the term for the proportion of a drug


that reaches systemic circulation?

523. Biofeedback

What operant conditioning therapy or


modification is described as Providing the
person with information regarding his or her
internal responses to stimuli with methods of
controlling them?

524. The biofeedback model is based on the parasympathetic


nervous system.

What part of the ANS is affected in the


biofeedback model of operant conditioning?

525. Biotin, ATP, and CO2

In the mitochondria, what complex is needed


for pyruvate carboxylase to catalyze the
reaction from pyruvate to OAA?

526. Biotin, ATP, and CO2

What complex is needed for propionyl CoA


carboxylase?

527. Biotin. Avidin is found in raw egg whites.

Avidin decreases the absorption of what


vitamin?

528. Biotin (only if eaten in large quantities)

What water-soluble-vitamin deficiency may


result from eating raw eggs?

529. Bipolar disorder (manic-depressive


disorder)

Increased self-esteem, flight of ideas, decreased sleep, increased


libido, weight loss, and erratic behavior are all symptoms of what
disorder?

530. Birds

What is the reservoir for the togavirus?

531. Birth defects, low birth weight (<


1500 g) with neonatal respiratory
distress syndrome (NRDS), and
SIDS

What are the top three causes of infant mortality?

532. Bisphosphonates

What class of drugs is used in the treatment of demineralization of


the bone?

533. Bitemporal heteronymous


hemianopsia

Name the ocular lesion; be specific. Optic chiasm lesion

534. Bladder cancer

Name the cancer associated with the following chemical agents.


(Some may have more than one answer.) Naphthylamine

535. Bladder carcinoma

RBCs in the urine

536. Blastoconidia

What type of spore is defined as an asexual budding daughter


yeast cell?

537. Blastomyces dermatitidis

Which dimorphic fungus is found as hyphae with nondescript


conidia in rotting wood in the Upper Great Lakes, Ohio,
Mississippi, eastern seaboard of the United States, and southern
Canada?

538. Blocking

Name these immature defense mechanisms: Inability to


remember a known fact (aware of forgetting)

539. Blood

What body fluid preferentially breaks down esters?

540. The blood withdrawn is the unconditioned stimulus,


inducing the unconditioned response (fainting). The
needle is part of the blood-drawing procedure and is the
conditioned stimulus (unconditioned and conditioned
stimuli are paired) resulting in the conditioned response
(fainting at the sight of the needle).

Which is the conditioned response, the


conditioned stimulus, the unconditioned
response, the unconditioned stimulus in this
case? A patient has blood withdrawn and
faints. The next time she goes to have blood
taken, she faints at the sight of the needle.

541. B lymphocyte

What cell type involves humoral immunity?

542. Body dysmorphic disorder

What somatoform disorder is described as


Unrealistic negative opinion of personal
appearance, seeing self as ugly?

543. Bone deposition increases with increased Ca2+ or PO 4concentrations, whereas resorption (breakdown) is
increased when there are low levels of Ca2+ or PO4-.

Is bone deposition or resorption due to


increased interstitial Ca2+concentrations?

544. Borderline

Name the cluster B personality disorder: In a


constant state of crisis, promiscuous, unable
to tolerate anxiety-causing situations, afraid
of being alone, and having intense but brief
relationships

545. Borderline

What is the label given to an individual whose


IQ is 70 to 79

546. Bordetella pertussis

What small gram-negative aerobic rod


requires Regan-Lowe or Bordet- Gengou
medium for growth?

547. Borrelia burgdorferi

What spirochete is responsible for Lyme


disease?

548. Borrelia recurrentis

Which organism causes multiple infections by


antigen switching?

549. Both

What type of fiber or fibers are carried in (answer


motor, sensory, or both) Ventral rami?

550. Both

What type of fiber or fibers are carried in (answer


motor, sensory, or both) Dorsal rami?

551. Both

What type of fiber or fibers are carried in (answer


motor, sensory, or both) Spinal nerve?

552. Both are AR with conjugated hyperbilirubinemia, but


Dubin-Johnson syndrome is differentiated from
Rotor by the black pigmentation of the liver.

Is Dubin-Johnson or Rotor syndrome associated


with black pigmentation of the liver?

553. Both Neisseria and Moraxella are gram-negative


cocci.

What are the two exceptions to the rule "all cocci


are gram positive"?

554. Bouchard nodes in the PIP joints; Heberden nodes


in the DIP joints.

What is the term for osteophyte formation at the


proximal interphalangeal (PIP) joints in
osteoarthritis? In the distal interphalangeal (DIP)
joints?

555. Boutonnire deformities

What is the term for flexion of the PIP and


extension of the DIP joints seen in rheumatoid
arthritis?

556. BPH

Name the most common type or cause. Urinary


tract obstruction

557. BPH

What is the most common one? Proliferative


abnormality of an internal organ

558. BPH. Although an argument can be made for prostatic cancer


(you should expect a much higher PSA result), these are
buzzwords for BPH. Prostatic cancer is usually silent until late
in the disease, when obstructive symptoms begin to occur.

A 70-year-old man complains of urinary


urgency, nocturia, hesitancy, postvoid
dribbling, urinary retention, and a PSA
result of 6.5 ng/mL. What is your
diagnosis?

559. Brain abscess

What condition results in the following


CSF results? Opening pressure 450
mm H2O; 5 WBCs (90% lymphocytes);
normal glucose and protein levels

560. Brain, spleen, and kidney

What are the three most common sites


for left-sided heart embolisms to
metastasize?

561. Breast

What is the most common one?


Primary tumor metastasis to the bone

562. Breast

What is the most common one?


Malignant tumor in women

563. Breast and lung cancer, respectively.

What are the two leading causes of


cancer-related death in women?

564. Breast cancer

Name the cancer associated with the


following oncogenes. (Some may have
more than one answer) hst-1, int-2,
erb-2, and erb-3

565. Breast cancer

What is the most common diagnosis


made (or missed!) resulting in a
malpractice suit?

566. Breast cancer

What do the risk factors late menopause, early menarche,


obesity, nulliparity, excessive estrogen, genetic factor p53, and
brc-abl characterize?

567. Breathing high-pressure nitrogen over


a long time and sudden
decompression result in the bends.

What two factors lead to the development of the bends (caisson


disease)?

568. Bridging veins draining into the sagittal


sinus

Name the cerebral vessel associated with the following vascular


pathologies. Subdural hemorrhage

569. Brief psychotic disorder. (In


schizophreniform disorder the
symptoms last longer than 6 months.)

What is the term for a schizophrenic episode lasting longer than


30 days with full return to former functioning capacity?

570. Broca's aphasia

Name the aphasia based on these characteristics: Nonfluent


speech, telegraphic and ungrammatical; lesion in Brodmann's
area 44; unimpaired comprehension

571. Bronchial asthma (eosinophil


membranes)

Charcot-Leyden crystals

572. Bronchial asthma (whorled mucous


plugs)

Curschmann spirals

573. Bronchiectasis

What is the term for chronic necrotizing pulmonary infections


resulting in permanent airway dilation and associated with
Kartagener syndrome?

574. Bronchioloalveolar carcinoma

What subset of adenocarcinoma arises from the terminal


bronchioles and/or alveolar walls?

575. Bronchoconstriction is associated with parasympathetic stimulation


(ACh), and catecholamine stimulation is associated with
bronchodilation (why epinephrine is used in emergency treatment of
bronchial asthma.)

Is ACh associated with


bronchoconstriction or
bronchodilation?

576. Bronchogenic carcinoma

What is the most common one?


Tumor in individuals exposed to
asbestos

577. Bronchogenic carcinoma

Name the cancer associated with


the following chemical agents.
(Some may have more than one
answer.) Chromium and nickel

578. Bronchogenic carcinoma

Name the cancer associated with


the following chemical agents.
(Some may have more than one
answer.) Polycyclic aromatic
hydrocarbons

579. Brother Fox (Eichner vs. Dillon). The substituted standard could not
apply because the patient had never been competent, so no one
knew what the patient would have wanted. Therefore, the decision
was based on what a "reasonable" person would have wanted.

What case is best known for use


of the "best interest standard"?

580. Brucella

What small, facultative gramnegative intracellular rod's


transmission is associated with
unpasteurized dairy products and
undulant fever?

581. Brucella and Listeria (has tumbling motility)

What two bacteria are associated


with drinking unpasteurized milk?

582. Brushfield spots

What is the term for the speckled appearance of the iris in


patients with Down syndrome?

583. Bruton X-linked


hypogammaglobinemia. Tyrosine
kinase deficiency leads to inadequate
B cell maturation.

What B cell disorder is characterized by pre-B cells in the bone


marrow, no circulating B cells in plasma, normal cell-mediated
immunity, low Igs, and appearance by 6 months of age?

584. Budd-Chiari syndrome

What is the term for the syndrome consisting of hepatomegaly,


ascites, and abdominal pain due to hepatic vein thrombosis?

585. Buerger disease (thromboangiitis


obliterans)

What small- to medium- sized vasculitis is seen in a 35-year-old


man who is a heavy smoker presenting with claudication
symptoms in the upper and lower extremities?

586. Buffers (remember that buffers are


best when they are used in a pH
range near its pK)

What is the term for chemicals that keep the pH constant despite
the formation of acids and bases during metabolism?

587. Bulbus cordis

From what embryonic structure are the following structures


derived? The aortic vestibule and the conus arteriosus

588. Bulla

What is the term for a raised fluid-filled cavity greater than 0.5 cm
that lies between the layers of the skin?

589. Bunyavirus

What family do the following viruses belong to? Hantavirus

590. Bunyavirus

What family do the following viruses belong to? California


encephalitis

591. Burkitt lymphoma

Starry sky appearance of macrophages is pathognomonic of what


lymphoma?

592. Burkitt lymphoma

Name the cancer associated with the following oncogenes.


(Some may have more than one answer) c-myc

593. Burkitt lymphoma

Starry sky pattern

594. Burr cells (echinocytes)

What is the term for RBCs with smooth undulations on the


surface of their membrane, commonly seen in uremia?

595. By causing rebound


insomnia and
decrease in REM
sleep

How does ceasation of barbiturate use affect sleep?

596. C1, C2, or C4


deficiency

What complement factor deficiency leads to Leukocyte adhesion deficiency with


poor opsonization?

597. C1 esterase inhibitor


(C1INH)

Hereditary angioneurotic edema (AD) produces local edema in organs (e.g., GI,
skin, respiratory tract). What enzyme deficiency causes increased capillary
permeability due to a release of vasoactive peptides?

598. C1 inhibitor (C1-INH)

What complement factor deficiency leads to Hereditary angioedema?

599. C1 through C4

What nerve roots constitute the cervical plexus?

600. C2 deficiency

What is the most common complement deficiency?

601. C3a, C4a, and C5a

What three complement fragments are also anaphylatoxins?

602. C3a, C4a, C5a

What complement factor or factors are associated with Anaphylaxis?

603. C3b

What complement factor or factors are associated with Opsonization?

604. C3, C4, and C5 keep


the diaphragm alive!

Ventral rami of what cervical nerves constitute the phrenic nerve?

605. C3 deficiency

What complement factor deficiency leads to Increased susceptibility to pyogenic


infections?

606. C4 (the upper border


of the thyroid
cartilage)

At what vertebral level does the common carotid artery bifurcate?

607. C5a

What is the name of the major chemotactic agent released from The blood
serum? (Hint: it is a complement factor.)

608. C5a

What complement factor or factors are associated with Chemotaxis?

609. C5b-C9

What components of the complement cascade form the MAC?

610. C5-C8 deficiency

What complement factor deficiency leads to Recurrent gonococcal infections?

611.

What complement factor or factors are associated with Membrane attack


complex (MAC)?

C5-C9

612. Caisson disease

What disease caused by decompression sickness leads to


multiple foci of ischemic necrosis that affect the head of the
femur, humerus, and tibia?

613. Calcarine sulcus

What sulcus divides the occipital lobe horizontally into a


superior cuneus and inferior lingual gyrus?

614. Calcitriol

What hormone is characterized by the following renal effects?


Calcium excretion, phosphate excretion

615. Calcium and ATP are required for


packaged macromolecules to be
extruded from the cell.

What two factors are required for effective exocytosis?

616. Calcium influx secondary to slow


channel opening

What triggers phase 0 of the action potential in a ventricular


pacemaker cell?

617. Calcium oxalate

What is the most common one? Kidney stone type

618. Calcium oxylate

What is the most common one? Renal calculus type

619. Calicivirus

What virus is associated with the Norwalk agent?

620. Camper's fascia; Scarpa's fascia is


devoid of fat. (Remember campers are
fat.)

What is the name of the superficial subcutaneous fascia of the


abdomen containing fat?

621. Campylobacter jejuni

What microaerophile is a motile gram-negative curved rod with


polar flagella that causes infectious diarrhea at low doses
(<500)?

622. Cancer

What is the second leading cause of death in the United


States?

623. Cancer

What is the second leading cause of death in children aged 1 to


14 in the United States?

624. Cancer of the lung, stomach, colon,


and breast

Name the cancer associated with the following tumor markers.


(Some may have more than one answer.) CEA

625. Candida

What is the most common fungal infection in HIV?

626. Candida albicans

What fungus causes endocarditis in IV drug users?

627. Capillaries

What vessels have the largest total cross-sectional area in


systemic circulation?

628. Capillary flow and pressure increase


with arteriolar dilation and decrease
with arteriolar constriction.

What happens to flow and pressure in capillaries with arteriolar


dilation? Arteriolar constriction?

629. Caplan syndrome

What syndrome is rheumatoid arthritis with pneumoconiosis?

630. Caput medusa, internal hemorrhoids,


esophageal varices, retroperitoneal
varices, and splenomegaly

What are five clinical signs of portal HTN?

631. Carbamazepine

What antiepileptic agent has SIADH as a side effect?

632. Carbamazepine

What is the DOC for trigeminal neuralgia?

633. Carbamazepine

Which anticonvulsant used in the treatment of bipolar disorder is


refractory to lithium?

634. Carbamazepine

Which anticonvulsant can cause teratogenic craniofacial


abnormalities and spina bifida?

635. Carbamoyl phosphate synthetase I

What enzyme catalyzes the rate-limiting step of the urea cycle?

636. Carbidopa and benserazide

What two drugs block dopa-decarboxylase in the periphery to


decrease the conversion of L-dopa to dopamine?

637. Carbidopa and benserazide

Inhibition of peripheral aromatic amino acid decarboxylase and


increased CNS availability of L-dopa are results of what drug
used in the treatment of Parkinson's disease?

638. Carbonic anhydrase

What enzyme is essential for the conversion of CO2 to HCO3-?

639. Carbonic anhydrase inhibitors


(acetazolamide and dorzolamide)

Which class of diuretics is used to treat metabolic acidosis, acute


mountain sicknesses, and glaucoma and to aid in the elimination
of acidic drug overdoses?

640. Carcinoid tumor

What neuroendocrine tumor produces excess serotonin; is


associated with diarrhea, flushing, bronchospasms, wheezing; and
is diagnosed by findings of elevated urinary 5-HIAA levels?

641. Cardiac muscle

Name the muscle type based on these descriptions: Continuous


involuntary contraction, uninuclear striated branched fibers, actin
and myosin overlapping for banding pattern, dyadic T tubules,
intercalated discs, troponin and desmin as a Z disc intermediate
filament.

642. Cardiac muscle

Name the muscle type based on the histological features: Actin


and myosin in sarcomeres; striated; uninuclear; gap junctions;
troponin:calcium binding complex; T tubules and SR forming
dyadic contacts; voltage-gated calcium channels

643. Carnitine acyl CoA transferase II

Which shuttle is used to bring fatty acyl CoA from the cytoplasm
for ketogenesis?

644. Carnitine acyltransferase I

What is the rate-limiting step of the following? -Oxidation

645. Carnitine acyltransferase-I

What enzyme catalyzes the rate-limiting step in fatty acid


oxidation?

646. The carotid body, which monitors


arterial blood directly

What peripheral chemoreceptor receives the most blood per gram


of weight in the body?

647. The carotid sinus is a pressuresensitive (low) receptor, while the


carotid body is an oxygen-sensitive
(low) receptor. (Remember "Sinus
Pressure").

Is the carotid sinus sensitive to pressure or oxygen?

648. The carotid sinus reflex

What reflex increases TPR in an attempt to maintain BP during a


hemorrhage?

649. Case control studies cannot assess


incidence or prevalence, but they
can determine causal relationships.

Can incidence, prevalence, and cause and effect be assessed in


Case control studies?

650. Case fatality rate

What type of mortality rate is defined


as the number of deaths From a
specific cause per number of
persons with the disease?

651. Caseous necrosis

Name the type of necrosis. Soft,


friable, cottage-cheese appearing;
characteristically seen in TB

652. The cat

What is the essential reservoir host


for Toxoplasma gondii?

653. Catalase-negative bacteria secrete H2O2 as a byproduct


(remember, catalase breaks down H2O2), allowing the neutrophils
to use it as the substrate for the other toxic metabolites. Patients
with CGD are prone to catalase-positive bacterial infections.

Why are patients with Chronic


Granulomatous Disease not prone to
develop infections from catalasenegative bacteria?

654. Catatonic schizophrenia

What subtype of schizophrenia is


characterized by Stuporous mute
echopraxia and automatic
obedience, waxy flexibility with
rigidity of posture?

655. Cauda equina

What is the name of the spinal cord


passing within the subarachnoid
space and forming the spinal nerves
that exit the lumbar and sacral
foramina?

656. Cause-specific mortality rate

What type of mortality rate is defined


as the number of deaths From a
specific cause per population?

657. CCK stimulates the pancreas to release amylase, lipase, and


proteases for digestion.

What is the pancreatic action of


CCK?

658. CD3

Name the T-cell CD marker: Expressed on all T cells


and is needed as a signal transducer for the T cell
receptor

659. CD3

What cell surface marker do all T cells have?

660. CD3 molecule. It transmits signals to the inside


of the T cell to trigger activation

What molecule that is needed to trigger T cell


activation is noncovalently linked to TCR?

661. CD4

Name the T-cell CD marker: Interacts with MHC class


II molecules

662. CD4+T cells

Development of what T cell line follows low affinity for


self-MHC class II Ags in the thymus?

663. CD4+ T cells (helper)

What subset of T cells recognizes the MHC class II


Ags?

664. The CD4 T cell; the APC is the first cell in the
immune response.

What is the second cell involved in the immune


response?

665. CD8

Name the T-cell CD marker: Interacts with MHC class


I molecules

666. CD8+ T cells

What T cell line arises from low affinity for self-MHC


class I Ags in the thymus?

667. CD8+ T cells (cytotoxic) Remember, 81=8


(CD8MHC class I=8); 42=8 (CD4 MHC
class II 8)

What subset of T cells recognizes the MHC class I


Ags?

668. CD14

What endotoxin receptor is the best marker for


macrophages?

669. CD15 and CD30

What two CD cell surface markers do Reed- Sternberg


cells stain positive for?

670. CD16 and CD56

What receptors are the best markers for NK cells?

671. CD19

Name the B-cell CD marker: Used clinically to count


B cells in blood

672. CD19

What cell surface marker is found on blood B cells?

673. CD21; it is a complement receptor for cleaved


C3

Name the B-cell CD marker: Receptor for EBV

674. CD28

Name the T-cell CD marker: Is a costimulatory


molecule in T cell activation

675. CD40

What cell surface marker is found on activated


helper T cells?

676. CD40

Name the B-cell CD marker: Required for


class switching signals from T cells

677. CD40 ligand

Name the T-cell CD marker: Essential for Ab


isotype switching (for B cell binding)

678. cDNA libraries are derived from mRNA, are continuous,


and contain no introns or regulatory elements.

What type of DNA library is made from the


mRNA from a tissue expressing a particular
gene?

679. cDNA, when it is made from mRNA

What form of continuous DNA, used in


cloning, has no introns or regulatory
elements?

680. Cefamandole, cefoperazone, and moxalactam

What three cephalosporins inhibit vitamin Kdependent factors?

681. Cefuroxime and cefaclor

Which two cephalosporins cross the bloodbrain barrier?

682. Celecoxib

What selective Cox-2 inhibitor should be


avoided in patients with a history of
sulfonamide allergy?

683. Celiac, aorticorenal, and superior mesenteric ganglias.


(Remember all " Splanchnics" are Sympathetic except
for the Pelvic splanchnics, which are Preganglionic
Parasympathetic fibers.)

Name the three postganglionic sympathetic


ganglia that receive input from thoracic
splanchnics.

684. Celiac sprue (gluten-sensitive enteropathy)

What malabsorption syndrome produces


abdominal distention, bloating, flatulence,
diarrhea, steatorrhea, and weight loss shortly
after eating bread products?

685. Cell-mediated immunity

What form of immunity is responsible for


removal of intracellular infections?

686. Central/neurogenic diabetes insipidus; in the


nephrogenic form there is sufficient ADH available,
but the renal collecting ducts are impermeable to its
actions.

What form of diabetes insipidus is due to an


insufficient amount of ADH for the renal collecting
ducts?

687. Central. Remember, there is a deficiency in ADH


production in the central form.

To differentiate central from nephrogenic diabetes


insipidus, after an injection of ADH, which will
show a decreased urine flow?

688. Central sulcus (sulcus of Rolando)

What fissure of the cerebral cortex runs


perpendicular to the lateral fissure and separates
the frontal and the parietal lobes?

689. Centriacinar worse in upper lobes; panacinar worse


in base of lower lobes

In which form of emphysema, panacinar or


centriacinar, is the effect worse in the apical
segments of the upper lobes?

690. Ceramide

What is the precursor of all sphingolipids?

691. Cerebellum

Name the area of the cerebral cortex with the


function described: Skill-based memory, verbal
recall, balance, refined voluntary movements

692. Cerebral aqueduct

What connects the third and the fourth ventricles?

693. Cervical neoplasia

What is the most common one? Primary


malignant tumor of the female genital tract in the
world

694. Chdiak-Higashi syndrome

In what rare AR disorder do you see neutropenia,


defective degranulation, and delayed microbial
killing due to a problem in chemotaxis and
migration?

695. Chemotaxis

What is the name of the process in which cells


migrate toward an attractant along a
concentration gradient?

696. Chemotaxis

What is the term for the unidirectional attraction of cells toward a


chemical mediator released during inflammation?

697. CHF

What is the term for the heart's inability to maintain perfusion and
meet the metabolic demands of tissues and organs?

698. CHF, cirrhosis, and nephrosis

What are the three causes of transudate?

699. Chief cell adenoma (80%)

What is the leading cause of primary hyperparathyroidism?

700. Chief cells

What are the cells of the parathyroid gland that produce PTH?

701. Chief cells

What cell type of the body or fundus of the stomach secretes


pepsinogen?

702. The chief cells of the parathyroid gland


release PTH in response to
hypocalcemia.

What cells of the parathyroid gland are simulated in response to


hypocalcemia?

703. Childhood

What stage of male development is characterized by the


following LH and testosterone levels? Both LH and
testosterone levels drop and remain low.

704. Chi-square.

What statistical method do you use when analyzing Crosssectional studies?

705. Chi-square (when you are in doubt


and have nominal data, use chisquare)

What statistical test, using nominal data only, checks whether


two variables are independent events?

706. Chlamydia

Name the most common reported STD. In females

707. Chlamydia; gonorrhea is second


(Remember, it is mandatory to report
STDs to the CDC.)

What is the most common disease or infection reported to the


CDC?

708. Chlamydia psittaci

Name the MCC. Pneumonia in a patient with atypical bird


exposure and hepatitis

709. Chlamydia trachomatis

Name the most common cause. Blindness worldwide

710. Chloral hydrate

Which sedative-hypnotic is contraindicated for patients taking


warfarin?

711.

Chloramphenicol

What bacteriostatic agent's side effect


profile includes grey baby syndrome and
aplastic anemia?

712. Chloramphenicol

Name the antimicrobial agent whose major


side effect is listed. Grey baby syndrome

713. Chlortetracycline

What tetracycline is associated with


hepatotoxicity?

714. Cholera toxin

What toxin ADP-ribosylates via Gs protein


to increase cAMP?

715. Cholera toxin irreversibly activates the cAMP-dependent


chloride pumps of the small and large intestine, producing a
large volume of chloride-rich diarrhea.

What is the site of action of cholera toxin?

716. Cholesterol biosynthesis

What pathway uses HMG CoA synthetase


in the cytoplasm?

717. Cholesterolosis

What condition results in a strawberry


gallbladder?

718. Chondroitin sulfate and keratan sulfate

What are the proteoglycans of cartilage


and bone?

719. Chordae tendineae

What attaches the cusps of the valves to


the papillary muscles in the heart?

720. Chorda tympani of CN VII

What nerve supplies taste sensation to the


anterior two-thirds of the tongue?

721. Choriocarcinoma

What is the most common one?


Malignant germ cell tumor in women

722. Choriocarcinomas and trophoblastic tumors

Name the cancer associated with the


following tumor markers. (Some may have
more than one answer.) Beta-hCG

723. Chromaffin cells (adrenal medulla)

What cells of the adrenal gland are neural crest derivatives?

724. Chromium (Cr)

What mineral is associated with impaired glucose tolerance?

725. Chromomycosis

What fungus is seen as colored cauliflower lesions?

726. Chromophobe adenoma

What is the most common one? Pituitary tumor

727. Chromosome 1

Name the associated chromosome. Gaucher disease

728. Chromosome 3p

Name the associated chromosome. VON Hippel-Lindau disease

729. Chromosome 4p

Name the associated chromosome. Huntington disease

730. Chromosome 5

Cri-du-chat syndrome results in a terminal deletion of the short arm


of what chromosome?

731. Chromosome 5, 21

Name the associated chromosomal translocation. Adult familial


polyposis

732. Chromosome 5p

Name the associated chromosome. Cri-du-chat

733. Chromosome 5q21

On what chromosome is the adenomatous polyposis coli gene?

734. Chromosome 6

Which chromosome is associated with MHC genes?

735. Chromosome 7

Name the associated chromosome. Cystic fibrosis

736. Chromosome 8, 14

Name the associated chromosomal translocation. Burkitt


lymphoma

737. Chromosome 9, 22 (Philadelphia


chromosome)

Name the associated chromosomal translocation. CML

738. Chromosome 11, 22

Name the associated chromosomal translocation. Ewing sarcoma

739. Chromosome 11p

Name the associated chromosome. Niemann-Pick disease

740. Chromosome 11p

Name the associated chromosome. Albinism

741. Chromosome 12

Name the associated chromosome. Chronic lymphocytic leukemia


(CLL)

742. Chromosome 13

Name the associated chromosome. Patau syndrome

743. Chromosome 14, 18

Name the associated chromosomal translocation. Follicular


lymphoma

744. Chromosome 15

Name the associated chromosome. Marfan disease

745. Chromosome 15

What chromosome is autism linked to?

746. Chromosome 15,


17

Name the associated chromosomal translocation. Acute promyelocytic leukemia


(M3)

747. Chromosome 15,


17

Name the associated chromosomal translocation. M3 AML

748. Chromosome
15q

Name the associated chromosome. Tay-Sachs disease

749. Chromosome 17

Name the associated chromosome. Neurofibromatosis type 1

750. Chromosome 18

Name the associated chromosome. Edward syndrome

751. Chromosome 19

Familial hypercholesteremia is due to a mutation in the LDL receptor gene. What


chromosome carries it?

752. Chromosome 19

Name the associated chromosome. Familial hypercholesterolemia

753. Chromosome
20q (deficiency
in adenosine
deaminase)

What chromosome is associated with the AR form of SCID?

754. Chromosome 21

Name the associated chromosome. Down syndrome

755. Chromosome
22q

Name the associated chromosome. Neurofibromatosis type 2

756. Chronic alcoholic

Mallory bodies

757. Chronic endstage renal


disease

Waxy casts

758. Chronic rejection

Based on the following information, is the renal transplantation rejection acute,


chronic, or hyperacute? Months to years after transplantation; gradual onset of HTN,
oliguria, and azotemia; seen as intimal fibrosis of the blood vessels and interstitial
lymphocytes

759. Churg-Strauss
syndrome

What variant of polyarteritis nodosa is associated with bronchial asthma, granulomas,


and eosinophilia?

760. Chylomicrons

Name the lipoprotein based on the following characteristics. apo E, apo C-II, apoB48

761. Cilia

What cell surface modification of ependymal cells and respiratory epithelium has a 9 +
2 microtubular configuration and movement as its function?

762. Ciliary ganglion

What ganglion supplies the postganglionic parasympathetic fibers to the ciliary


muscles of the eye?

763. Ciliary ganglion producing a tonic pupil

What ocular ganglion is affected if the pupil on the affected


side sluggishly responds to light with normal
accommodation?

764. Ciliary ganglion. (These fibers are carried


in CN III. Remember it like this:-ili-in
ciliary ganglion looks like the III of CN III.)

What is the name of the postganglionic parasympathetic


ganglion that innervates The papillary sphincter and ciliary
muscle of the eye?

765. Cingulate gyrus herniation (subfalcine)

Which cerebral herniation results in compression of the


anterior cerebral artery?

766. Circumvallate papillae

What papillae are responsible for sweet taste?

767. Cirrhosis

What disorder is associated with spider angiomas, palmar


erythema, gynecomastia, testicular atrophy, encephalopathy,
abnormalities in clotting factors, and portal HTN?

768. Cirrhosis and hemolytic anemia, which


are risk factors for pigmented gallstones.

Which of the following is not a risk factor for cholesterol


gallstones: pregnancy, OCP use, female gender, hemolytic
anemia, cirrhosis, and obesity? (May be more than one
answer.)

769. Citrate, via the citrate shuttle

As what compound do the carbons for fatty acid synthesis


leave the mitochondria?

770. Clarke's nucleus

What nucleus, found in the intermediate zone of the spinal


cord, sends unconscious proprioception to the cerebellum?

771. Class 1 (1A, 1B, and 1C)

Which class of antiarrhythmics blocks sodium channels?

772. Class II

Which class of antiarrhythmics are -blockers?

773. Class III

Which class of antiarrhythmics are potassium channel


blockers?

774. Class IV

Which class of antiarrhythmics are calcium channel blockers?

775. Clavicle, acromion, and glenoid fossa of


the scapula and the humerus

Name the bony articulations of the following sites. Be specific.


Shoulder

776. Clearance

What is the term for drug removal per unit of time in a given
volume of blood?

777. Clearance

What term describes the volume of plasma


from which a substance is removed over
time?

778. Clear cell

What is the most common one? Renal cell


cancer type

779. Cleft lip

What results when the maxillary prominence


fails to fuse with the medial nasal
prominence?

780. Cleft palate

What results when the palatine prominences


fail to fuse with the other side?

781. Climbing fibers;, they are monosynaptic input on Purkinje


cells. Mossy fibers, also excitatory, are axons of all other
sources and synapse on granule cells.

What excitatory fibers arise from the inferior


olivary nuclei on the contralateral side of the
body?

782. Clindamycin

Which antimicrobial agent is used with


pyrimethamine to treat toxoplasmosis?

783. CLL (B-cell origin)

Which B-cell neoplasm has the following cell


surface markers: CD19, CD20, CD5 (T-cell
marker), CD23; and are CD10-negative?

784. Clofazimine

What is the drug of choice in the treatment of


the lepra reaction?

785. Clonidine

Which centrally acting 2-adrenergic agonist


is used in the treatment protocol of opioid
withdrawal?

786. Clonidine

Which drug is used to treat opioid withdrawal,


ADHD, and sometimes Tourette's syndrome?

787. Clonorchis sinensis

What parasitic infection is associated with


cholangiocarcinoma?

788. Clostridium botulinum

What bacterium found in poorly preserved


canned food causes flaccid paralysis?

789. Clostridium botulinum

What gram-positive spore-forming anaerobic rod blocks the


release of ACh at the NMJ, resulting in reversible flaccid
paralysis?

790. Clostridium botulinum

Which gram-positive bacteria infection of infancy is associated


with ingestion of honey?

791. Clostridium perfrigens

Gas gangrene is associated with which Clostridium species?

792. Clostridium perfringens

What organism is associated with the following types of


diarrhea? Watery diarrhea from beef, poultry, or gravies

793. Clostridium perfringens

What is the only nonmotile pathogenic Clostridium species?

794. Clostridium tetani

What large, spore-forming, gram-positive anaerobic rod is


associated with infections due to puncture wounds and trauma?

795. Closure of the aortic valve indicates


the termination of the ejection phase
and the beginning of the isovolumetric
relaxation phase of the cardiac cycle.

The closure of what valve indicates the beginning of the


isovolumetric relaxation phase of the cardiac cycle?

796. Clozapine

What neuroleptic agent causes agranulocytosis but does not


induce tardive dyskinesia?

797. Clozapine

What is the only neuroleptic agent that does not cause


hyperprolactinemia?

798. CML

What leukemia is characterized by Philadelphia chromosomal


translocation (9;22); massive splenomegaly; peripheral
leukocytosis (commonly > 100, 00); decreased LAP levels; and
nonspecific symptoms of fatigue, malaise, weight loss, and
anorexia?

799. CML and ALL

Name the cancer associated with the following oncogenes.


(Some may have more than one answer) abl

800. CMV; remember, EBV is associated


with heterophile-positive
mononucleosis.

What DNA virus is associated with heterophile-negative


mononucleosis?

801. CMV retinitis and HSV-2

What are the two most common viral infections in HIV?

802. CMV; Toxoplasma also causes intracerebral


calcifications but it is a parasite.

What viral infection is known to cause intracerebral


calcifications?

803. CMV, Toxoplasma gondii, and Listeria

Which three organisms cause heterophilic negative


mononucleosis?

804. CN I

Name the structure that enters or exits the following


foramina: Cribriform plate

805. CN II and ophthalmic artery

Name the structure that enters or exits the following


foramina: Optic canal

806. CN III

With which CN are preganglionic parasympathetic


axons arising from the Edinger- Westphal nucleus
associated?

807. CN III and IV

What CNs are affected if there is a lesion in The


midbrain?

808. CN III and IV

What CNs arise from The midbrain?

809. CN I, II, III, VI, and XII Add 1 + 1 = 2, 1 + 2 = 3,


1 + 2 + 3 = 6, 1 + 2 + 3 + 6 = 12

Which CNs are found in the midline of the brainstem?

810. CN III, IV, and VI

What three CNs are associated with conjugate eye


movements?

811.

What CNs are responsible for the sensor and motor


components of the light reflex?

CN II is the sensory limb and CN III is the motor


component through parasympathetic
stimulation.

812. CN III, VII, IX, and X

What four CN carry preganglionic parasympathetic


fibers?

813. CN IV

What is the only CN to arise from the dorsal surface of


the midbrain?

814. CN IX

What CN is associated with the Third pharyngeal


arch?

815. CN IX and X

What two CNs are responsible for the carotid body and
sinus reflexes?

816. CN IX, X, and XII

What CNs are affected if there is a lesion in The


upper medulla?

817. CN IX, X, and XII CN XI arises from the cervical


spinal cord.

What CNs arise from The medulla?

818. CN IX, X, and XI; sigmoid sinus

Name the structure that enters or exits the


following foramina: Jugular foramen

819. CN V

What CN is associated with the First


pharyngeal arch?

820. CN V

What CNs are affected if there is a lesion in


The upper pons?

821. CN V1 carries the sensory and CN VII carries the motor


component of the blink reflex.

What nerves carry the sensory and motor


components of the blink reflex?

822. CN V1, the occulomotor division of the trigeminal nerve,


is the sensory component of the corneal reflex.

What CN transmits sensory information from


the cornea?

823. CN V2

Name the structure that enters or exits the


following foramina: Foramen rotundum

824. CN V3 and the lesser petrosal nerve

Name the structure that enters or exits the


following foramina: Foramen ovale

825. CN VII

What CN is associated with the Second


pharyngeal arch?

826. CN VII

Name the structure that enters or exits the


following foramina: Stylomastoid foramen

827. CN VII and VIII

Name the structure that enters or exits the


following foramina: Internal auditory meatus

828. CN VII, IX, and X; taste and general sensation for the
tongue is sent to the solitary nucleus.

Which three CNs send sensory information to


the solitary nucleus?

829. CN VI, VII, and VIII

What CNs are affected if there is a lesion in


Pontomedullary junction?

830. CN V, VI, VII, and VIII

What CNs arise from The pons?

831. CN X

What CN is associated with the Sixth


pharyngeal arch?

832. CN X

What CN is associated with the Fourth


pharyngeal arch?

833. CN XI and X

What nerve or nerves supply general sensation


and taste to the posterior third of the tongue?

834. CN XII

Name the structure that enters or exits the


following foramina: Hypoglossal canal

835. CN XI, vertebral arteries

Name the structure that enters or exits the


following foramina: Foramen magnum

836. CN X (Remember, the vagus nerve supplies the


parasympathetic information from the tip of the pharynx
to the end of the midgut and all between.)

What CN carries preganglionic


parasympathetic fibers that innervate the
viscera of the neck, thorax, foregut, and
midgut?

837. CO

Cherry red intoxication is associated with what


form of poisoning?

838. Coagulative

What is the most common one? Form of


necrosis

839. Coagulative necrosis

Name the type of necrosis. The most common


form of necrosis; denatured and coagulated
proteins in the cytoplasm

840. Coarctation of the aorta

What is the most common one? Cardiac


anomaly in Turner syndrome

841. Cocaine

What is the only local anesthetic to produce


vasoconstriction?

842. Cocaine

What illegal drug can cause rhabdomyolysis,


MI, cerebral infarct, and lethal cardiac
arrhythmias?

843. Coccidioides immitis

Which dimorphic fungus is found as


Arthroconidia in desert sand of the
southwestern United States (e.g., San Joaquin
Valley)?

844. The coding strand is


identical to mRNA, and the
template strand is
complementary and
antiparallel.

Is the coding or the template strand of DNA identical to mRNA (excluding


the T/U difference)?

845. Cohort studies determine


incidence and causality, not
prevalence.

Can incidence, prevalence, and cause and effect be assessed in Cohort


studies?

846. Collecting ducts, which


make them readily
permeable to water
reabsorption.

What area of the nephron is sensitive to the effects of ADH?

847. Coma, convulsions, and


cardiotoxicity

What are the three C's of a TCAD overdose?

848. Combined metabolic and


respiratory acidosis

If the pH is low with increased CO2 levels and decreased HCO3- levels,
what is the acid-base disturbance?

849. Common peroneal nerve

What lower extremity nerve is described by the following motor loss? Loss
of eversion; inversion, dorsiflexion, and plantarflexion of the foot

850. Common peroneal nerve

Fracture of the fibular neck, resulting in foot drop, is an injury of what


nerve?

851. Competitive antagonist

Is the following description characteristic of competitive or noncompetitive


antagonists: parallel shift to the right on a dose-response curve, complete
reversibility with increase of the dose of the agonist drug, and capacity to
decrease the potency of the agonist drug?

852. Complex 4

What complex of the ETC contains Cu2+?

853. Complex I

What component of the ETC is inhibited by the following? Barbiturates

854. Complex I

What component of the ETC is inhibited by the following? Rotenone

855. Compliance; the more compliant a


lung is, the easier it is to inflate.

What term is an index of the effort needed to expand the lungs


(i.e., overcomes recoil)?

856. Compliance (think of it as


distensibility)

What is the term to describe how easily a vessel stretches?

857. Compulsions. They are actions done


to fix the bad thoughts. Obsessions
are the thoughts.

What is the term for repetitive actions blocking recurring bad


thoughts?

858. Concentration force and electrical


force

What two forces affect movement of ions across a membrane?

859. The concentration of plasma proteins


determines effective osmolarity
because capillary membranes are
freely permeable to all substances
except proteins.

What determines the effective osmolarity of the ICF and the ECF
compartments?

860. Concrete operations (6-12 years)

At what stage of cognitive development (according to Piaget) do


children See death as irreversible?

861. Conduction aphasia

Name the aphasia based on these characteristics: Lesion is in


the parietal lobe or arcuate fibers because the connection
between Broca's and Wernicke's area is severed; word
comprehension preserved; inability to write or speak the
statement (can't tell you what you said)

862. Cones (C for color and cones)

What cells of the retina sees in color and needs bright light to be
activated?

863. Confabulation; it is commonly seen in


Korsakoff's syndrome.

What is the term for making up stories regarding past experiences


because of an inability to retrieve them?

864. Confidence interval. It is a way of


admitting estimation for the
population.

What specifies how accurately the sample values and the true
values of the population lie within a given range?

865. Confounding bias

When more than one explanation can account for the end result,
what form of bias occurs?

866. Congenital heart disease

Name the most common type or cause. Childhood heart disease


in the United States

867. Conidia

What type of spore is asexual and formed of hyphae?

868. Conjugated (direct) hyperbilirubinemia

What type of jaundice is seen in Rotor's syndrome?

869. Conjugated (direct) hyperbilirubinemia, a


transport defect

What type of jaundice is seen in Dubin-Johnson


syndrome?

870. Conn syndrome (primary hyperaldosteronism)

What syndrome that is due to an adrenal gland


adenoma produces excess aldosterone resulting in
HTN, hypokalemia, and low rennin levels?

871. Constipation and miosis

What are the two side effects of opioids to which the


user will not develop tolerance?

872. Constriction of afferent arteriole

What has occurred to the renal arterioles based on the


following changes in the GFR, RPF, FF, and
glomerular capillary pressure? GFR , RPF , FF
normal, capillary pressure

873. Constriction of efferent arteriole

What has occurred to the renal arterioles based on the


following changes in the GFR, RPF, FF, and
glomerular capillary pressure? GFR , RPF , FF ,
capillary pressure

874. Constrictor pupillae and ciliary muscles

Which muscles of the eye are under parasympathetic


control?

875. Contralateral loss below and bilateral loss at the


level of the lesion

Describe the loss for each of the following in a


hemisection of the spinal cord. (Brown-Sequard
syndrome) Spinothalamic tract?

876. Contralateral muscle weakness when above the


decussation, whereas an UMN injury below the
pyramidal decussation results in ipsilateral
muscle weakness.

What sided muscle weakness is seen in an UMN


corticospinal tract injury above the pyramidal
decussation?

877. Contralateral. The spinothalamic tract enters the spinal


cord and immediately synapses in the dorsal horn, crosses
over, and ascends contralateral in the spinal cord,
brainstem, thalamus, and postcentral gyrus.

Will a unilateral lesion in the spinothalamic


tract result in a contralateral or ipsilateral
loss of pain and temperature?

878. Conus arteriosus of the right ventricle and right and left
auricles

What part of the heart forms Superior


border?

879. The conus medullaris terminates at the level of the second


lumbar vertebra.

In an adult, where does the spinal cord


terminate and what is it called?

880. Convergence

Contracting both medial rectus muscles


simultaneously makes the images of near
objects remain on the same part of the
retina. What term describes this process?

881. Conversion disorder

What somatoform disorder is described as


La belle indiffrence, suggestive of true
physical ailment because of alteration of
function?

882. The conversion of CHO to pregnenolone via the enzyme


desmolase

What is the rate-limiting step in the


production of steroids?

883. The conversion of tyrosine to dopamine in the cytoplasm

What is the rate-limiting step in the synthetic


pathway of NE at the adrenergic nerve
terminal?

884. Coombs test

What test is used to detect anti-RBC Abs


seen in hemolytic anemia?

885. Copper

Leukopenia, neutropenia, and mental


deterioration are signs of what mineral
deficiency?

886. Copper

What mineral is a component of cytochrome


a/a3?

887. Copper (Cu+)

What element is needed for the proper


alignment of tropocollagen molecules?

888. Copper deficiency

What mineral deficiency involves blood vessel fragility?

889. Coprophilia

What paraphilia is defined as Combining sex with


defecation?

890. Cori cycle

What cycle is responsible for converting to glucose in the


liver the lactate produced in the RBCs?

891. Coronary circulation

Where are the lowest resting PO2 levels in a resting


individual?

892. Coronavirus

Name the most common cause. A cold in the winter and


summer

893. Correlation. No, correlation does not imply


causation.

What is the degree to which two measures are related?


Does it imply causation?

894. Corticospinal tract

What tract is responsible for voluntary refined


movements of distal extremities?

895. Corticosteroid

What is the DOC for the treatment of the late phase


response in an asthmatic attack?

896. Cortisol, a 21-carbon steroid, has a -OH


group at position 17.

What is the only 17-hydroxysteroid with hormonal


activity?

897. Cortisol levels increase. Lymphocyte levels


decrease in sleep-deprived individuals.

What happens to cortisol levels in sleep-deprived


individuals?

898. Cortisol Remember, from the outer cortex to


the inner layer, Salt, Sugar, Sex. The adrenal
cortex gets sweeter as you go deeper.

What is the major hormone produced in the following


areas of the adrenal cortex? Zona fasciculata

899. Corynebacterium diphtheriae

What non-spore-forming gram-positive aerobic rod


produces bull neck, sore throat with pseudomembranes,
myocarditis, and sometimes respiratory obstructions?

900. Corynebacterium diphtheriae

Which bacteria are associated with the following pigment


production? Black-gray pigmentation

901. The courts. These are legal, not medical


terms.

Who decides competency and sanity?

902. Courvoisier's law

What law states that an enlarged, palpable


gallbladder is more likely due to cancer than stone
obstruction?

903. Coxiella burnetii

What organism causes Q fever?

904. Coxiella burnetii

What is the only Rickettsia that is stable in the


environment?

905. Coxsackie A

What picornavirus is associated with hand-footand-mouth disease?

906. Coxsackie B

What is the most common one? Viral cause of


myocarditis

907. C-peptide levels

What serves as a marker of endogenous insulin


secretion?

908. Craniopharyngioma

What CNS tumor arises from Rathke's pouch?

909. Creatinine

Serum concentration of what substance is used as


a clinical measure of a patient's GFR?

910. Creutzfeldt-Jakob disease

What prion-associated CNS pathology produces


rapidly progressive dementia with myoclonus,
involuntary movements, and death within 6 to 12
months?

911.

What form of dementia is characterized by onset at


age 40 to 50, rapid progression, infection by a
prion, and death within 2 years?

Creutzfeldt-Jakob's disease. Patients first develop


vague somatic complaints and anxiety, rapidly
followed by dysarthria, myoclonus, ataxia, and
choreoathetosis.

912. Cricothyroid muscle; all other laryngeal muscles are


innervated by the recurrent laryngeal nerve.

What is the only laryngeal muscle innervated by


the external laryngeal nerve?

913. Cricothyroid muscles

Name the laryngeal muscle described by the


following: Tenses the vocal ligaments, increasing
the distance between the cartilages, thereby
increasing the pitch

914. Crigler-Najjar syndrome

In what disease is there a genetic absence of UDP-glucuronate transferase,


resulting in an increase in free unconjugated bilirubin?

915. Criterion-referenced
tests. You need a
certain number correct
to pass (e.g., the
USMLE).

Objective tests that base the result of the examination on a preset standard use
what form of reference?

916. Crohn disease

What transmural inflammatory bowel disease can be found from the mouth to
anus, has noncaseating granulomas, is discontinuous (skip lesions), and has a
cobblestone appearance, thickening of the bowel wall, linear fissures, and
aphthous ulcers with normal mucosa between?

917. Crohn disease

What GI pathology is associated with a positive string sign, an increase in the


number of bloody stools, RLQ pain, skip lesions, terminal ileum most
commonly affected, occurrence in women more than men, and an increased
thickness of the bowel?

918. Crossover study

In what study, for ethical reasons, is no group left out of intervention?

919. Cross-sectional studies


determine prevalence,
not incidence or cause
and effect.

Can incidence, prevalence, and cause and effect be assessed in Crosssectional studies?

920. Crude mortality rate

What type of mortality rate is defined as the number of deaths In the


population?

921. Cryptococcus

What is the MCC of the following meningitides? In patients who have AIDS or
are immunocompromised

922. Cryptococcus

What is the only encapsulated fungal pathogen?

923. Cryptococcus

What fungus is urease positive?

924. Cryptococcus
neoformans

Which encapsulated fungus is found in soil enriched with pigeon droppings?

925. Cryptococcus
neoformans

What fungus is characterized by India ink staining of the CSF that produces
colorless cells with a halo on a black background?

926. Cryptorchidism; normally the testes descend into the scrotum within
3 months of birth.

What is the term for failure of the


testes to descend into the scrotum?

927. CSF H+ levels, with acidosis being the main central drive, resulting
in hyperventilation (the opposite being true with alkalosis)

What is the central


chemoreceptor's main drive for
ventilation?

928. The cupola of the lung is posterior to the subclavian artery and vein.
It is the reason one must be cautious when performing subclavian
venipuncture.

Where is the cupola of the lung in


relation to the subclavian artery
and vein?

929. Curling ulcers (think curling iron = burn)

What is the term for gastric ulcers


associated with severely burned or
traumatic patients?

930. Cushing's disease (pituitary) has elevated ACTH and cortisol


suppression with dexamethasone, whereas Cushing's syndrome
(adrenal adenoma) has decreased ACTH and no cortisol
suppression with dexamethasone.

Does Cushing syndrome or


Cushing disease have elevated
ACTH levels and cortisol
suppression with dexamethasone?

931. Cushing's ulcers

What is the term for gastric ulcers


associated with increased
intracranial pressure?

932. Cut down (ever tried and failed?) Annoyed (criticism makes angry?)
Guilty (about drinking behavior?) Eye opener (drinking to shake out
the cobwebs?)

What are the CAGE questions?

933. C wave

What wave is the cause of the


following venous pulse deflections?
The bulging of the tricuspid valve
into the right atrium

934. Cyanide

What form of poisoning is


associated with bitter almondscented breath?

935. Cyanocobalamin (B12)

What vitamin requires IF for


absorption?

936. Cyanocobalamin (B12)


deficiency (Folic acid
deficiency has only
homocystinuria as a sign.)

What vitamin deficiency produces homocystinuria and methylmalonic


aciduria?

937. Cyclothymia (nonpsychotic


bipolar). Patients are ego
syntonic.

What disorder is characterized by an alternating pattern of depressed


mood with periods of hypomania for more than 2 years?

938. Cyproheptadine

What antihistamine is used in the treatment of serotonergic crisis?

939. Cystadenocarcinoma

What is the most common one? Malignant carcinoma of the ovaries

940. Cystadenocarcinoma

What ovarian carcinoma is characterized by psammoma bodies?

941. Cystadenoma

What is the most common one? Benign ovarian tumor

942. Cysticerci

What is the term for cestode-encysted larvae found in intermediate hosts?

943. Cystic fibrosis

What is the most common lethal AR disorder affecting white Americans?

944. Cystic fibrosis. (Parents are


usually the first to find out
because the baby tastes
salty.)

What AR disease involves a defect in AA 508 on chromosome 7, causing


a defect in Cl- transportation that leads to recurrent pulmonary infections
and an increase in viscid mucoid secretions along with pancreatic
insufficiencies?

945. Cytochrome a/a3

What component of the ETC is inhibited by the following? Azide

946. Cytochrome a/a3

What component of the ETC is inhibited by the following? CO

947. Cytochrome a/a3

What component of the ETC is inhibited by the following? Cyanide

948. Cytochrome b/c1

What component of the ETC is inhibited by the following? Antimycin A

949. Cytochrome c

What is released from the mitochondria to trigger apoptosis?

950. Cytokine. If a cytokine


affects another class of
leukocytes, it is called an
interleukin.

What is the term for a substance secreted by a leukocyte in response to a


stimulus?

951. Cytosine

What pyrimidine base is found In both DNA and RNA?

952. D1 receptor; inhibition of the direct pathway occurs


through the D2 receptors.

Which dopamine receptor excites the direct


pathway of the basal ganglia?

953. D2 and 5HT3 receptors

What two receptors in the chemoreceptor trigger


zone lead to vomiting when stimulated?

954. Dantrolene

Which antispasmodic blocks the release of Ca


2+from the SR and is used in the treatment of
malignant hyperthermia?

955. Dapsone

Which bacteriostatic drug whose major side


effect is the lepra reaction inhibits folic acid
synthesis?

956. Day 14

What day of the menstrual cycle does ovulation


take place?

957. The DCML system. (Remember, everything but pain


and temperature.)

What ascending sensory system carries joint


position, vibratory and pressure sensation, and
discriminative touch from the trunk and limbs?

958. The death of a spouse. The higher the score, the


greater the risk of developing an illness in the next 6
months.

What is the most stressful event as determined


by the Holmes and Rahe scale?

959. Decrease (5-HT and dopamine levels do the same)

What happens to NE levels in Major


depression?

960. Decreased pulmonary arterial pressure (low


perfusion) and less-distensible vessels (high
resistance) result in decreased blood flow at the apex.

What two factors result in the apex of the lung


being hypoperfused?

961. Decreased REM sleep and REM rebound during


withdrawal

How is sleep affected in a person with alcohol


intoxication?

962. A decrease in Ca2+ and an increase in PO4-

What two factors cause PTH to be secreted?

963. A decrease in cutaneous blood flow results from


constriction of the arterioles, and decreased
cutaneous blood volume results from constriction of
the venous plexus.

How does sympathetic stimulation to the skin


result in decreased blood flow and decreased
blood volume? (Hint: what vessels are
stimulated, and how?)

964. Decrease in surface area and increase in membrane thickness


(Palv O2 > PaO2)

What are two causes of diffusion


impairment in the lungs?

965. Decreases

With a decrease in arterial


diastolic pressure, what happens
to Stroke volume?

966. Decreases

With a decrease in arterial


diastolic pressure, what happens
to Heart rate?

967. Decreases

With a decrease in arterial


diastolic pressure, what happens
to TPR?

968. Decreases

With an increase in arterial systolic


pressure, what happens to
Vessel compliance?

969. Decreases

With an increase in arterial systolic


pressure, what happens to Heart
rate?

970. Decreases Remember, the carotid sinus reflex attempts to


compensate by increasing both TPR and heart rate.

When a person goes from supine


to standing, what happens to the
following? Cardiac output

971. Decreases Remember, the carotid sinus reflex attempts to


compensate by increasing both TPR and heart rate. -------------------------------------------------------------------------------

When a person goes from supine


to standing, what happens to the
following? BP

972. Decreasing potassium conductance, which results in increased


excitability

What triggers phase 4 of the action


potential in a ventricular
pacemaker cell?

973. The deepest sleep levels (stage 3 and 4) occur mostly in the first
half of sleep.

When does most of the NREM


sleep (stage 3 and 4) occur, in the
first or second half of sleep?

974. Deferoxamine, penicillamine

Name the antidote. Iron

975. Deficiency in surfactant

What protein deficiency results in respiratory distress


syndrome of newborns?

976. delta-Cells; somatostatin has an inhibitory


effect on alpha- and Beta-islet cells.

What pancreatic islet cell secretes somatostatin?

977. Deltoid

What muscle acts in all ranges of motion of the arm?

978. Demeclocycline

Which tetracycline is used in the treatment of SIADH?

979. Dementia Urinary incontinence Gait apraxia


(NPH wet, wacky, wobbly)

What is the triad of NPH?

980. Dendrites receive information, whereas axons


send information.

What part of a neuron receives information?

981. Dendritic cells, macrophages, and thymic


epithelial cells

What three cells are essential for T-cell differentiation in


the thymus?

982. Denial

Name these narcissistic defense mechanisms: Not


allowing reality to penetrate because afraid of becoming
aware of painful aspect of reality.

983. Denial Anger Bargaining Depression


Acceptance No, they can be skipped,
repeated, and completed out of sequence.

What are the five Kbler-Ross stages of death and


dying? Must they be completed in order?

984. The dentate nucleus

What deep cerebellar nuclei receive Purkinje cell


projections in The intermediate hemispheres?

985. Dependent

Name the cluster C personality disorder: Gets others


to assume responsibility, is subordinate, and is fearful of
being alone and caring for self

986. Depolarization (i.e., Na+ influx)

What is the term for the negative resting membrane


potential moving toward threshold?

987. Depolarization is from apex to base and from


endocardium to epicardium.

How does ventricular depolarization take place, base to


apex or vice versa?

988. Depression

Name the most common cause. Insomnia

989. de Quervain thyroiditis

What thyroiditis presents as a tender, enlarged,


firm thyroid gland, usually preceded by an
upper respiratory viral illness?

990. Desipramine

What TCAD causes sudden cardiac death in


children?

991. DHEA (androgens) Remember, from the outer cortex to


the inner layer, Salt, Sugar, Sex. The adrenal cortex
gets sweeter as you go deeper.

What is the major hormone produced in the


following areas of the adrenal cortex? Zona
reticularis

992. Diabetes mellitus

Name the most common cause. Blindness in


the United States

993. Diacylglycerols and phosphatidic acid

From which two substances are phospholipids


made?

994. Diastole. During systole the left ventricle contracts,


resulting in intramyocardial vessel compression and
therefore very little blood flow in the coronary
circulation.

Is most of the coronary artery blood flow during


systole or diastole?

995. Diazepam is longest acting and midazolam is shortest


acting.

What are the longest-acting and shortestacting benzodiazepines?

996. Diazoxide

Which direct-acting vasodilator is used


clinically to treat hypertensive emergencies,
insulinomas, and as a uterine smooth muscle
relaxant?

997. Diethylcarbamazine

What is the drug of choice for filariasis and


onchocerciasis?

998. Diffuse proliferative GN

What is the most common one? Renal


pathology in patients with SLE

999. Diffusion rate is indirectly proportional to membrane


thickness and is directly proportional to membranes
surface area.

To what is the diffusion rate indirectly


proportional?

1000.

DiGeorge's syndrome

What immunologic syndrome is due to a


pharyngeal pouch 3 and 4 failure?

1001.

DiGeorge syndrome

What syndrome is characterized by


embryologic failure of the third and fourth
pharyngeal pouches resulting in
hypocalcemia, tetany, and T-cell
deficiency?

1002.

DiGeorge syndrome, which is due to a failure of the third


and fourth pharyngeal pouch development. Remember, B
cell deficiencies are associated with extracellular infection.
T cell deficiencies are associated with intracellular
infections

What T cell deficiency syndrome is


associated with facial anomalies,
hypoparathyroidism, thymic hypoplasia,
and recurrent viral and fungal infections?

1003.

Digoxin is renally eliminated; digitoxin is hepatically


eliminated.

Is digoxin or digitoxin renally eliminated?

1004.

Dihydrofolate reductase

What enzyme of pyrimidine synthesis is


inhibited by the following? Methotrexate

1005.

Dihydrofolate reductase

What enzyme is blocked by methotrexate/


trimethoprim?

1006.

Dihydrofolate reductase

What enzyme is inhibited by trimethoprim?

1007.

Dihydropteroate synthetase

What enzyme is inhibited by


sulfonamides?

1008.

Dihydrotestosterone

What is the most potent male sex


hormone?

1009.

Dilation of afferent arteriole

What has occurred to the renal arterioles


based on the following changes in the
GFR, RPF, FF, and glomerular capillary
pressure? GFR , RPF , FF normal,
capillary pressure

1010.

Dilation of efferent arteriole

What has occurred to the renal arterioles based on the


following changes in the GFR, RPF, FF, and glomerular
capillary pressure? GFR , RPF , FF , capillary pressure

1011.

Dilation of the internal mammary


arteries results in erosions on the inner
surface of the ribs and is seen as
notching.

Notching of the ribs, seen on chest radiograph in patients with


postductal coarctation of the aorta, is due to collateralization of
what arteries?

1012.

Dilator pupillae muscle

Which muscle of the eye is under sympathetic control?

1013.

Dimercaprol

Name the antidote. Gold

1014.

Dimercaprol, D-penicillamine

Name the antidote. Arsenic

1015.

Dimercaprol, penicillamine

Name the antidote. Mercury

1016.

Dimorphic

What is the term for fungi that can convert from hyphal to
yeast forms?

1017.

Diphenhydramine

What agent is used IM to treat acute dystonias?

1018.

Diphyllobothrium latum

What parasite found in raw fish can produce vitamin B12


deficiency?

1019.

Diphyllobothrium latum

What organism is associated with megaloblastic anemia?

1020.

Direct; in males less than 50 years old


indirect hernias are the most common
type.

What is the most common one? Type of hernia seen in


males more than 50 years old

1021.

Direct pathway; a good example is


Parkinson's disease.

In what pathway of the basal ganglia do lesions result in an


underactive cortex with hypokinetic, slow, or absent
spontaneous movement?

1022.

Disorganized schizophrenia

What subtype of schizophrenia is characterized by Childlike


behaviors, unorganized speech and behaviors, poor
grooming, incongruous smiling and laughter, and the worst
prognosis?

1023.

Displacement

Use of an outlet for emotions (stuff flows downhill).

1024.

Dissecting aortic aneurysm. MI is also


high on the list, but these are buzzwords
to look for dissection.

A marfanoid patient presents with tearing retrosternal chest


pain radiating to her back. What is your first diagnosis?

1025.

Disseminated intravascular coagulation


(DIC)

What disorder is associated with decreased platelet count,


prolonged PT and PTT, decreased fibrinogen, and increased
fibrin split products (D-dimers)?

1026.

Dissociation

Name these anxiety defense mechanisms: Separating


oneself from the experience. The facts are accepted but the
form is changed for protection.

1027.

Dissociative fugue. (Patients are


unaware of memory loss.)

What is the term to describe inability to recall the past and


possible assumption of a completely new identity?

1028.

Distal convoluted tubules

What is the site of action of the following? Aldosterone


antagonists

1029.

Disulfiram

What drug is used to prevent alcohol consumption by


blocking aldehyde dehydrogenase?

1030.

Diverticulosis

Name the most common type or cause. Rectal bleeding

1031.

DLCO, an index of lung surface area and membrane thickness, is


decreased in fibrosis because of increased membrane thickness and
decreased in emphysema because of increased surface area without
increase in capillary recruitment; in exercise there is an increase in surface
area due to capillary recruitment.

Why does carbon


monoxide diffusion in the
lung (DLCO) decrease in
emphysema and fibrosis
but increase during
exercise?

1032.

DM, hypercholesterolemia, smoking, and HTN are major risk factors.


Being male, obesity, sedentary lifestyle, homocysteine elevation, oral
contraceptive pills, and genetics are minor risk factors for atherosclerosis.

Name four major risk


factors for atherosclerosis.

1033.

DNA ligase

What protein catalyzes the


formation of the last PDE
bond between the Okazaki
fragments to produce a
continuous strand?

1034.

DNA polymerase I

What enzyme has a 5' to 3'


synthesis of the Okazaki
fragments, 3' exonuclease
activity, and 5'
exonuclease activity?

1035.

DNA polymerase-

Name the eukaryotic DNA


polymerase based on the
following information:
Synthesizes the lagging
strand during replication

1036.

DNA polymerase-

Name the eukaryotic DNA polymerase based on the following information:


Replicates mitochondrial DNA

1037.

DNA polymerase-

Name the eukaryotic DNA polymerase based on the following information:


Synthesizes the leading strand during replication

1038.

Dominant parietal lobe


(Gerstmann's
syndrome)

Name the area of the cerebral cortex affected by the description of the effects,
symptoms, and results of the lesion. Agraphia, acalculia, finger agnosia, rightleft disorientation

1039.

Dominant temporal
lobe

Name the area of the cerebral cortex affected by the description of the effects,
symptoms, and results of the lesion. Euphoria, delusions, thought disorders,
Wernicke's aphasia, auditory hallucinations (Hint: the lesion is in the left
hemisphere)

1040.

Dopamine is converted
into NE in the vesicle
via the enzyme
dopamine-Betahydroxylase.

In an adrenergic nerve terminal, where is dopamine converted to NE? By what


enzyme?

1041.

Dopamine levels rise


with waking; dopamine
is associated with
wakefulness.

What happens to dopamine levels when we awaken?

1042.

Dorsal motor nucleus


of CN X

What parasympathetic nucleus is found on the floor of the fourth ventricle and
supplies preganglionic fibers innervating the terminal ganglias of the thorax,
foregut, and midgut?

1043.

Dorsal prefrontal
cortex

Name the area of the cerebral cortex affected by the description of the effects,
symptoms, and results of the lesion. Apathy, poor grooming, poor ability to
think abstractly, decreased drive, poor attention span (Hint: if the lesion is in the
dominant hemisphere, the patient will develop Broca's aphasia)

1044.

Dorsal spinocerebellar tract; the


accessory cuneate nucleus is the second
nucleus for the cuneocerebellar tract.

Clarke's nucleus is the second ascending sensory neuron


of which spinocerebellar tract?

1045.

The dose of glipizide must be decreased


in hepatic dysfunction. Glyburide's dose
must be decreased in renal dysfunction.

The dose of which second-generation sulfonylurea agent


must be decreased for patients with hepatic dysfunction?

1046.

Double-blind study. It is the most


scientifically rigorous study known.

What type of random controlled test is least subjective to


bias?

1047.

Down syndrome

What is the most common chromosomal disorder?

1048.

Down syndrome

What is the MCC of inherited mental retardation?

1049.

Down syndrome (it is slightly more


common than fragile X syndrome.)

What is the most common genetic cause of mental


retardation?

1050.

Down syndrome (trisomy 21)

Name the most common one. Chromosomal disorder

1051.

Down syndrome (trisomy 21)

What disease that involves mental retardation, flat face,


muscle hypotonia, and a double-bubble sign on radiograph
poses an increased risk of Alzheimer's disease and ALL?

1052.

Doxacurium

What is the most potent NMJ blocker and also has no


cardiovascular side effects?

1053.

Doxycycline

Which tetracycline is used to treat prostatitis because it


concentrates strongly in prostatic fluid?

1054.

Doxycycline and minocycline

Which two tetracyclines have the highest plasma binding?

1055.

D-Penicillamine

Name the antidote. Copper

1056.

d-Tubocurarine

Which nicotinic receptor antagonist causes hypotension


and is associated with malignant hyperthermia?

1057.

Duchenne muscular dystrophy

Calf pseudohypertrophy

1058.

Duchenne muscular dystrophy

What disease is seen in children younger than 5 years of age


and is characterized by X-linked recessive cardiac
myopathies, calf pseudohypertrophy, lordosis, protuberant
belly, an increase then a decrease in CPK, and death
commonly in the second decade of life?

1059.

Duchenne muscular dystrophy

What is the most common one? Form of muscular dystrophy

1060.

Duchenne muscular dystrophy


(Remember, Becker's is slower in
progress, less severe, later in onset,
and lacks cardiac involvement.)

What X-linked recessive disorder that is due to an abnormality


in the dystrophin gene, has onset by age 5 with progressive
proximal muscle weakness, calf pseudohypertrophy, and
elevated CPK levels?

1061.

Duct cells secrete HCO3-, electrolytes,


and water. The acini secrete the
enzymes necessary for carbohydrate,
nucleic acid, protein cleavage, and
emulsification of fats.

Do the duct or the acini cells of the pancreas secrete HCO3-?

1062.

Ductus epididymis, which is lined by


pseudostratified epithelium with
stereocilia.

Where do sperm go for maturation?

1063.

Duodenal atresia

What is the name for failed recanalization of the duodenum


resulting in polyhydramnios, bile-containing vomitus, and a
distended stomach?

1064.

Duodenal ulcer

What type of PUD is classically described by the onset of


burning epigastric pain 1 to 3 hours after eating that is
relieved by food?

1065.

Duodenal ulcers (4 times as common


as gastric)

What is the most common one? Type of PUD

1066.

Duodenum

In what area of the GI tract are water-soluble vitamins


absorbed?

1067.

Duodenum

In what area of the GI tract does iron get absorbed?

1068.

Duodenum

What segment of the small intestine is associated with


Brunner's glands?

1069.

. Duodenum (all but the first part) 2. Ascending Colon 3.


Ureters 4. Pancreas 5. Supra renal glands (adrenals) 6.
Descending colon 7. Aorta 8. Kidneys 9. Rectum 10. IVC D
CUPS DAKRI is the mnemonic, everything else is covered
with peritoneum

Name the 10 retroperitoneal organs.

1070.

During a lumbar puncture the needle passes through the


interlaminar space in the midline of L3-L4, with the tip of the
iliac crest in the flexed position as the landmark. Order of
puncture: 1. Skin 2. Superficial fascia 3. Deep fascia 4.
Supraspinous ligament 5. Interspinous ligament 6. Interlaminar
space 7. Epidural space 8. Dura mater 9. Arachnoid mater 10.
Subarachnoid space. (They ask this in some variation every
year, so know it.)

Arrange the following layers in the


correct sequence through which a
needle must pass in a lumbar
puncture. Skin Subarachnoid space
Interspinous ligament Dura mater
Deep fascia Epidural space
Superficial fascia Interlaminar space
Supraspinous ligament Arachnoid
mater

1071.

During REM sleep

In what stage of sleep is it easiest to


arouse a sleeping individual?

1072.

During stage 3 and 4 (remember, it is called deep sleep.)

In what stage of sleep is it hardest to


arouse a sleeping individual?

1073.

Dynorphin

What is the neurotransmitter at the Kreceptor?

1074.

Dysgerminoma

What is the most common one?


Germ cell tumor of the ovary

1075.

Dyslexia

Name the most common type.


Learning disability

1076.

Dysmetria; this is seen in a finger-to-nose test.

What is the name for inability to stop a


movement at the intended target?

1077.

Dyspareunia. It is a common complaint in women who have


been raped or sexually abused.

What is the term for recurrent and


persistent pain before, after, or during
sexual intercourse?

1078.

Dysplasia

What is the term for nonneoplastic abnormal


proliferation of cell size, shape, and cellular
organization?

1079.

Dysthymia, which is also known as nonpsychotic


depression. (Think of it as the car running but not
well.)

What disorder is characterized by a depressed


mood and a loss of interest or pleasure for more
than 2 years?

1080.

Dystrophic calcification

What is the term for precipitation of calcium


phosphate in dying or necrotic tissue?

1081.

Ea (activation energy)

What is the only factor of enzyme kinetics that the


enzyme affects?

1082.

Early collecting duct

What is the site of action of the following? K+sparing diuretics

1083.

Early distal tubule

What is the site of action of the following?


Thiazide diuretics

1084.

Eaton-Lambert syndrome

What syndrome is due to Abs directed to calcium


channels and causes muscle weakness that
improves with repeated use?

1085.

EBV

What virus is associated with both nasopharyngeal


carcinoma and Burkitt lymphoma?

1086.

EBV

With what virus are Downey type II cells


associated?

1087.

EBV

What virus is associated with the endemic form of


Burkitt lymphoma?

1088.

EBV infections resulting in infectious


mononucleosis can be diagnosed by the
Monospot test. (Remember, it may be negative in
the first week of the illness, so retest if you have a
high index of suspicion.)

A 20-year-old college student has fever, grey-white


membranes over the tonsils, posterior auricular
lymphadenitis, and hepatosplenomegaly. What is
your diagnosis? What test do you order to confirm
your diagnosis?

1089.

The ECF compartment always enlarges when there is a net gain


in total body water and decreases when there is a loss of total
body water. Hydration status is named in terms of the ECF
compartment.

What happens to extracellular


volume with a net gain in body fluid?

1090.

Echinococcus multilocularis

Which cestode infection results in


alveolar hydatid cyst disease?

1091.

Eclipse period

What is the term of the viral growth


period when no viruses can be found
intracellularly?

1092.

Edinger-Westphal nucleus (via CN III)

What nucleus supplies the


preganglionic parasympathetic fibers
to the ciliary ganglion?

1093.

Edrophonium

What drug is used to differentiate a


cholinergic crisis from myasthenia
gravis?

1094.

Edward syndrome (trisomy 18)

In what disease do you see


horseshoe kidneys, rockerbottom
feet, low-set ears, micrognathia, and
mental retardation?

1095.

eEF-1

What eukaryotic translation enzyme


is associated with the following:
Elongation

1096.

eEF-2 is the site where Pseudomonas and Diphtheria toxins


work.

What elongation factor is inactivated


by ADP ribosylation, preventing
translation?

1097.

EF-1 and GTP

What factors are needed for


elongation in eukaryotes?

1098.

EF-2 and GTP

What factors are needed for


translocation in eukaryotes?

1099.

Effective dose for 50% of drug takers


(median effective dose)

What do the following values indicate? ED50

1100.

Efficacy

What is the term for how well a drug produces its desired
response?

1101.

EF-G of the 50S subunit

What prokaryotic positioning enzyme in translation is blocked


by the following? Erythromycin

1102.

EF-Tu and EF-Ts of the 30S ribosomal


subunit

What prokaryotic positioning enzyme in translation is blocked


by the following? Tetracycline

1103.

EF-Tu or EF-ts and GTP

What factors are needed for elongation in prokaryotes?

1104.

Ego

What Freudian psyche component is described as Reality,


rationality, language basis?

1105.

Ego dystonic

What is the term to describe homosexuals who Are


uncomfortable with their own person and disagree with their
sense of self?

1106.

Ego syntonic

What is the term to describe homosexuals who Are


comfortable with their own person and agree with their sense
of self?

1107.

eIF-2 in the P site

What eukaryotic translation enzyme is associated with the


following: Initiation

1108.

Eight pairs through seven cervical


vertebrae. Totaling 31 pairs of spinal
nerves.

How many pairs of spinal nerves are associated with Cervical


vertebrae?

1109.

Eisenmenger's syndrome

After a longstanding left-to-right shunt reverses, causing


cyanosis, and becomes a right-to-left shunt, what is it termed?

1110.

Eisenmenger syndrome, which can


also occur with any left-to-right shunt

What is the term for a large VSD that leads to pulmonary HTN,
RVH, and cyanosis due to right-to-left reversal of the shunt?

1111.

ELISA is used as a screening test because it is


very sensitive; Western blot is used as a
confirmatory test because it detects antibodies
(protein) to the HIV virus.

What is the primary screening test used to detect


HIV-infected individuals? Confirmatory test?

1112.

ELISA. It detects anti-p24 IgG.

What test is used to screen for HIV?

1113.

ELISA screens and Western blot confirms the


diagnosis.

Does ELISA or Western blot confirm whether a


patient is HIV-positive?

1114.

ELISA test

What test uses p24 protein when diagnosing HIV?

1115.

Elongation factor-G and GTP

What factors are needed for translation in


prokaryotes?

1116.

Embolic

Are hemorrhagic cerebral infarcts more commonly


associated with embolic or thrombotic occlusions?

1117.

Embolism

What is the term for occlusion of a blood vessel due


to an intravascular mass that has been carried
downstream?

1118.

Emphysema

What pulmonary disease, most commonly associated


with smoking, results in enlarged, overinflated lungs
owing to the destruction of the alveolar walls with
diminished elastic recoil?

1119.

Emphysematous bleb

Name the most common type or cause.


Spontaneous pneumothorax

1120.

Empyema

What is the term for pus in the pleural space?

1121.

Encapsulated bacteria

Patients with sickle cell anemia are at increased risk


for infection from what type of organisms?

1122.

Endocardial cushion defect

Name the most common one. Heart defect in Down


syndrome

1123.

Endometrial cancer

What is the most common one? Malignancy of the


female genital tract

1124.

Endometriosis

What female pathology is associated with endometrial glands and stroma outside
the uterus commonly affecting the ovaries as chocolate cysts?

1125.

The energy of
activation

What determines the rate of reaction?

1126.

The energy of
activation (Ea)

What determines the rate of a reaction?

1127.

Enkephalin

What is the neurotransmitter at the -receptor?

1128.

Enkephalins

What naturally occurring substances mimic the effects of opioids?

1129.

Entamoeba
histolytica

What is the most common one? Organism associated with liver abscesses

1130.

Entamoeba
histolytica (treat
with
metronidazole)

What protozoal parasite results in dysentery with blood and pus in the stool, is
transmitted via fecal-oral route, is diagnosed by cysts or trophozoites in the stool,
and forms liver abscesses and inverted flask-shaped lesions in the large intestine?

1131.

Enterobius
vermicularis

What is the most common one? Helminth parasitic infection in the United States

1132.

Enterobius
vermicularis; the
treatment is
albendazole.

What nematode is known as pinworms? What is the treatment?

1133.

Enterococcus
faecalis

What is the MCC of the following endocarditis scenarios? Following biliary


infections

1134.

Enterococcus
(Streptococcus
faecalis)

Which streptococcal species is characterized by being catalase negative, turning


bile esculin agar black, producing a positive PYR test, and resulting in biliary and
urinary tract infections?

1135.

Enteroendocrine
(EE) cells; they
also secrete
secretin.

What cell of the duodenum secretes CCK?

1136.

Enterohemorhagic
Escherichia coli

What organism is associated with the following types of diarrhea? Bloody diarrhea
associated with hamburger ingestion

1137.

Enterohepatic
cycling

How are drugs that are excreted via the biliary system resorbed by the GI tract?

1138.

Enterokinase

What enzyme is needed to


activate the following reactions?
Trypsinogen to trypsin

1139.

Enteropathogenic Escherichia coli (Rotavirus is the MCC overall.)

Name the most common type or


cause. Infantile bacterial
diarrhea

1140.

The entire tubule barring the thick ascending limb

What is the site of action of the


following? Osmotic diuretics

1141.

env structural protein

What HIV structural gene


produces GP120 and GP41?

1142.

The enzyme lysyl oxidase requires Cu2+and O2 to function properly.

What mineral is required for


cross-linking of collagen
molecules into fibrils?

1143.

The enzyme phenyl ethanolamine N-methyltransferase (PNMT),


used in the conversion of epinephrine, is regulated by cortisol.
Removing the anterior pituitary gland decreases ACTH and therefore
cortisol.

Why is there a decrease in the


production in epinephrine when
the anterior pituitary gland is
removed?

1144.

Eosin

Do the following structures pick


up stain from hematoxylin or
eosin? Cytoplasm

1145.

Eosin

Do the following structures pick


up stain from hematoxylin or
eosin? RBCs

1146.

Eosin

Do the following structures pick


up stain from hematoxylin or
eosin? Collagen

1147.

Eosin

Do the following structures pick


up stain from hematoxylin or
eosin? Fibrin

1148.

Eosin

Do the following structures pick up stain from hematoxylin or eosin? Thyroid


colloid

1149.

Eosinophil

What cell type is commonly elevated in asthma?

1150.

Eosinophil chemotactic
factor A

What mast cell mediator is a chemotactic agent?

1151.

Eosinophilic exudates

Name the type of exudate, given the following examples. Parasitic infection

1152.

Eosinophils

What chronic inflammatory WBC is associated with IgE-mediated allergic


reactions and parasitic infections?

1153.

Ependymal cells

What cells lining the ventricles have cilia on their luminal surface to move
CSF?

1154.

Ependymoma

What is the most common one? Intraspinal tumor

1155.

Ependymoma

What is the most common one? Intramedullary spinal cord tumor in adults

1156.

Ephedrine

What drug that penetrates the blood-brain barrier is found in asthma


preparations and used as a nasal decongestant?

1157.

Epidermophyton

Which genus of dermatophytes is associated with the following sites of


infection? Nails and skin only

1158.

Epidural hematoma

Lucid interval

1159.

Epinephrine

What are the primary neurotransmitters at the following sites? Chromaffin


cells of the adrenal medulla

1160.

Epinephrine

Name the hormoneglucagon, insulin, or epinephrine: Glycogenolytic,


gluconeogenic, lipolytic, glycolytic, and stimulated by hypoglycemia

1161.

Epispadia

What is the term for the external urethra opening onto the dorsal surface of
the penis?

1162.

Epstein-cardiac anomaly
of the tricuspid valve

What is the teratogenic effect associated with lithium?

1163.

Erectile dysfunction

What are the pharmacologic effects seen sexually with Neuroleptics?

1164.

Ergosterol

What is the major cell membrane sterol found in fungi?

1165.

Erythema marginatum

What type of erythema do you see in Rheumatic fever?

1166.

Erythema multiforme

What type of erythema do you see in StevensJohnson syndrome?

1167.

Erythema nodosum

What type of erythema do you see in Ulcerative


colitis?

1168.

Erythromycin

Which macrolide is used in the treatment of


gastroparesis?

1169.

Erythromycin

Name the antimicrobial agent whose major side


effect is listed. Cholestatic hepatitis

1170.

Escherichia coli

What facultative gram-negative anaerobic rod is


motile, ferments lactose, and is the MCC of UTIs?

1171.

Escherichia coli

Name the most common type or cause. UTIs

1172.

Escherichia coli

What is the most common one? Organism that


causes pyelonephritis

1173.

Escherichia coli

Name the most common type or cause. Cystitis

1174.

Escherichia coli and Pseudomonas sp.

What are the two MCC of acute epididymitis in


males? More than 35 years old

1175.

Esophagus and esophageal nerve plexus (CN X)


Remember: 1 at T8, 2 at T10, and 3 at T12

What structure or structures cross the diaphragm at


T10 level?

1176.

Esophagus, SVC, vagus nerve, azygos vein,


thoracic duct, thymus, and phrenic nerve

What seven structures are found in more than one


mediastinum?

1177.

Estradiol

What hormone level peaks 1 day before the surge of


LH and FSH in the female cycle?

1178.

Estriol

What form of estrogen is of placental origin?

1179.

Estrogen

Circulating levels of what hormone cause the


cervical mucus to be thin and watery, allowing sperm
an easier entry into the uterus?

1180.

Estrogen can result in a hypercoagulable state


because of the decrease in AT III and increase in
factors II, VII, IX, and X.

What components of the coagulation cascade are


affected by estrogen?

1181.

Estrogens are 18-carbon


steroids. (Removal of one carbon
from an androgen produces an
estrogen.)

How many carbons do estrogens have?

1182.

Estrogen; the first 14 days of the


female reproductive cycle mark
the proliferative phase.

What hormone, produced by the granulose cell, stimulates the


endometrium to enter the proliferative phase?

1183.

Estrone

What type of estrogen is produced in peripheral tissues from


androgens?

1184.

Ethacrynic acid

Irreversible ototoxicity is associated with which loop diuretic?

1185.

Ethambutol

What antitubercular agent causes loss of red-green visual


discrimination?

1186.

Ethyl alcohol

Name the antidote. Ethylene glycol

1187.

Ethyl alcohol

Name the antidote. Methyl alcohol

1188.

Euchromatin, the dark stuff in the


nucleus on an electron
microscope image.

What type of chromatin is transcriptionally active?

1189.

Ewing sarcoma

Pseudorosettes

1190.

Ewing's sarcoma

What malignant neoplasm of the bone is associated with HomerWright pseudorosettes; onion skinning of the periosteum on
radiographs of the femur, pelvis, and tibia; and chromosome 11;22
translocation?

1191.

Excess estradiol in the blood

What is a sign of a Sertoli cell tumor in a man?

1192.

Excision endonuclease, which


removes thiamine dimers from
DNA

What DNA excision and repair enzyme is deficient in patients with


xeroderma pigmentosum?

1193.

Excretion is greater than filtration


for net secretion to occur.

Is excretion greater than or less than filtration for net secretion to


occur?

1194.

Exhibitionism

What paraphilia is defined as Having a recurrent desire to expose


the genitals to strangers?

1195.

Exogenous steroids suppress LH release and


result in Leydig cell atrophy. Testosterone,
produced by Leydig cells, is needed for
spermatogenesis.

Why is spermatogenesis decreased with anabolic


steroid therapy?

1196.

Expiratory reserve volume (ERV)

Name the lung measurement based on the following


descriptions: Additional volume that can be expired
after normal expiration

1197.

Exposure

What classical conditioning therapy or modification is


described as Forcing patients to confront their fears by
being exposed to them until they are extinguished?

1198.

Expressive aphasia

What aphasia produces a nonfluent pattern of speech


with the abilty to understand written and spoken
language seen in lesions in the dominant hemisphere?

1199.

External 3', 5' PDE bonds

What bonds are broken by exonucleases?

1200.

External abdominal oblique

What abdominal muscle contributes to the anterior layer


of the rectus sheath, forms the inguinal ligament, and in
men gives rise to the external spermatic fascia of the
spermatic cord?

1201.

Extinction

Based on operant conditioning, what type of


reinforcement is described when Removing a stimulus
stops a behavior?

1202.

Extinction

What operant conditioning therapy or modification is


described as Stopping the reinforcement that is
leading to an undesired behavior?

1203.

Extrahepatic biliary atresia

What disorder is associated with jaundice, white stools,


and dark urine due to biliary duct occlusion secondary to
incomplete recanalization?

1204.

Extravascular hemolysis if it occurs in the


spleen; if in the liver, it results in
hepatomegaly.

Is splenomegaly more commonly associated with


intravascular or extravascular hemolysis?

1205.

Facial expression (the second is vocal intonation)

What is the primary method of nonverbal


communication of emotional states?

1206.

Factitious disorder

What disorder is described as having Conscious


symptoms with unconscious motivation?

1207.

Factors II, VII, IX, and X and proteins C and S.

Name the six vitamin K-dependent coagulation


factors.

1208.

Factors II, VII, IX, X, and proteins C and S

What are the vitamin K-dependent coagulation


factors?

1209.

Factor XII for the intrinsic; factor VII for the extrinsic
pathway

What factor gets activated in the intrinsic pathway


of the coagulation cascade? Extrinsic pathway?

1210.

Fading

What operant conditioning therapy or modification


is described as Removing a reinforcement
(without the patient knowing) gradually over time
to stop a condition?

1211.

FALSE

True or false? Miniature end-plate potentials


(MEPPs) generate action potentials.

1212.

FALSE

True or false? Bile pigments and bile salts are


reabsorbed in the gallbladder.

1213.

FALSE

True or false? Pregnancy ensures emancipation.

1214.

False. Adenovirus vaccine is a live pathogenic virus


in an enteric coated capsule. All of the others are
inactivated vaccines.

True or false? All of the following are inactivated


vaccines available in the United States: influenza,
Vibrio cholera, hepatitis A, rabies, and
adenovirus.

1215.

False. All except multiparity are risk factors for


breast cancer. Nulliparity, increasing age, and family
history in first-degree relative are also risk factors.
Memorize this list!

True or false? All of the following are risk factors


for breast cancer: early menses, late menopause,
history of breast cancer, obesity, and multiparity.

1216.

False. ASO titers are elevated, but serum complement levels


are decreased.

True or false? Elevated ASO titers and


serum complement levels are
associated with poststreptococcal GN.

1217.

False. Atelectasis is a reversible collapse of a lung.

True or false? Atelectasis is an


irreversible collapse of a lung.

1218.

False. Bacteriocidals kill; bacteriostatics slow growth.

True or false? Antimicrobial agents


that slow bacterial growth are
bactericidal.

1219.

False. B cells recognize unprocessed Ags, but T cells can


recognize only processed Ags.

True or false? T cells can recognize,


bind, and internalize unprocessed
Ags.

1220.

False. Cardiac output refers to circulating blood volume. The


blood in the systemic veins and the pulmonary circuits are
storage reserves and therefore are not considered in cardiac
output.

True or false? The blood stored in the


systemic veins and the pulmonary
circuit are considered part of the
cardiac output.

1221.

False. Childbirth carries five times as much risk of serious


psychiatric illness as abortion.

True or false? Serious psychiatric


illness is more common after abortion
than childbirth.

1222.

False. CSF is a clear isotonic solution with lower


concentrations of K+ and HCO3-. It does have higher
concentrations of Cl- and Mg2+.

True or false? CSF is a clear,


hypertonic solution with higher
concentrations of K + and HCO3-,
than the serum.

1223.

False. Direct tests detect Ags; indirect tests detect Abs.

True or false? Direct fluorescent Ab


test is used to detect Abs in a patient?

1224.

False. Dopamine is an inhibitory neurotransmitter in the CNS.

True or false? Dopamine is an


excitatory neurotransmitter in the
CNS.

1225.

False. Elevated calcitonin levels are seen in medullary


carcinoma of the thyroid. Psammoma bodies are seen in
papillary carcinoma of the thyroid and ovaries, as well as
meningiomas.

True or false? Psammoma bodies are


seen in medullary carcinoma of the
thyroid.

1226.

False. Ergotamines are used in the treatment of migraines for


their vasoconstrictive action in the cerebral circulation.

True or false? Ergotamines, because


of their vasodilation action, cause
migraines.

1227.

False. Females are more likely to develop femoral hernias then


males (remember Female's Femoral).

True or false? Males are more likely to


develop femoral hernias than females.

1228.

False. Fewer Ags are needed to trigger a secondary response.

True or false? More Ag is needed to


produce a secondary immune
response than a first immune
response.

1229.

False. Flumazenil is unable to block the effects of barbiturates.

True or false? Flumazenil is used in


the treatment of barbiturate overdose.

1230.

False. Glutamic acid is excitatory, but GABA is inhibitory.

True or false? Both GABA and


glutamic acid are excitatory
neurotransmitters in the CNS.

1231.

False. Histidine operon is activated when there are low


intracellular levels of histidine.

True or false? Histidine activates the


histidine operon.

1232.

False. hyperkalemia decreases digoxin's activity, while


hypokalemia, hypomagnesemia, and hypercalcemia all
increase digoxin toxicity.

True or false? Hyperkalemia increases


the effect of digoxin.

1233.

False. Increased LAP is seen in stress reactions and helps


differentiate benign conditions from CML, which has low LAP
levels.

True or false? Increased leukocyte


alkaline phosphatase (LAP) is
associated with CML.

1234.

False. Interferons are produced by virally infected cells to


inhibit viral replication via RNA endonucleases. They do not act
directly on the virus, nor are they virus specific.

True or false? Interferons are


eukaryotic proteins that inhibit viral
replication by being virus specific.

1235.

False. It closes just after birth because the change in


pulmonary circulation causes increased left atrial pressure.

True or false? The foramen ovale


closes just prior to birth.

1236.

False. It does increase uterine synthesis of prostaglandins,


which increase uterine contractions.

True or false? Oxytocin initiates


rhythmic contractions associated with
labor.

1237.

False. It has glycogenolytic and lipolytic actions but


not proteolytic.

True or false? Epinephrine has proteolytic


metabolic effects.

1238.

False. It is an enzyme secreted by the lining of the


small intestine.

True or false? Enterokinase is a brush border


enzyme.

1239.

False. It is a passive process due to decreased sex


hormones.

True or false? Menstruation is an active process


due to increased gonadal sex hormones?

1240.

False; it is for the production of toxic metabolites.

True or false? The increased oxygen consumption


after phagocytosis is for ATP production.

1241.

False. It is reversible because the Ags and Abs are


not linked covalently.

True or false? Ag-Ab binding is irreversible

1242.

False. Meningococcus is encapsulated;


Gonococcus is not.

True or false? Gonococcus is encapsulated.

1243.

False. Metformin does not have weight gain as a


potential side effect (unlike sulfonylureas).

True or false? Metformin is contraindicated in


obese patients because of weight gain as its side
effect.

1244.

False-negative rate

What rate is indicated by 1- sensitivity?

1245.

False-positive rate

What rate is indicated by 1-specificity?

1246.

False. Right-sided valves are the first to open and


last to close.

True or false? Right-sided valves close before the


valves on the left side of the heart.

1247.

False. Risk of breast and endometrial cancer but


not ovarian cancer increases with the use of
estrogen.

True or false? Estrogen use increases the risk of


developing ovarian cancer.

1248.

False. Separation, divorce, being widowed, and


unemployment increase your risk, but being single
does not.

True or false? Being single increases your risk of


suicide.

1249.

False. Staphylococci have catalase; streptococci do


not.

True or false? Streptococci have catalase.

1250.

False. That is precisely where COMT is found; it is


not found in adrenergic nerve terminals.

True or false? Catechol-O-methyl transferase


(COMT) is not found in smooth muscle, liver, and
the kidneys.

1251.

False. The disease has no associated pathology; the


phenomenon is arterial insufficiency due to an underlying
disease.

True or false? Raynaud's phenomenon has


no underlying pathology.

1252.

False. The faster the rate of absorption, the smaller the


Tmax and larger the Cmax (bioinequivalence). The
slower the rate of absorption, the larger the Tmax and
smaller the Cmax.

True or false? The faster the rate of


absorption, the smaller the time of maximum
concentration (Tmax) and the maximum
concentration (Cmax).

1253.

False. The gallbladder does not produce bile, but it


concentrates bile via active sodium transport; water
follows the sodium.

True or false? The gallbladder functions to


produce bile.

1254.

False. The PPD tests exposure to TB.

True or false? A positive PPD skin test


indicates the patient has active pulmonary
disease.

1255.

False. These beliefs are characteristic of people with an


external locus of control.

True or false? According to social learning


theory, people who believe that luck, chance,
or the actions of others control their fate
have an internal locus of control.

1256.

False. They are all risk factors for endometrial carcinoma


except multiparity. Nulliparity, estrogen-producing tumors,
and estrogen replacement therapy are also risk factors
for endometrial carcinoma.

True or false? Obesity, DM, HTN, multiparity,


early menarche, and late menopause are all
risk factors for endometrial carcinoma.

1257.

False. They are bacteriostatic.

True or false? Tetracyclines are bactericidal.

1258.

False. Thyroid size is a measure of TSH levels (which


are goitrogenic).

True or false? Thyroid size is a measure of


its function.

1259.

False. Water readily diffuses across the blood-brain


barrier, but glucose requires carrier-mediated transport.

True or false? Glucose readily diffuses


across the blood-brain barrier.

1260.

False. You need clear evidence the patient is not


competent; if you are unsure, assume the patient is
competent.

True or false? A patient has to prove his or


her competency.

1261.

Familial hypercholesterolemia

What AD disease associated with chromosome 19 involves


a defect in the LDL receptors that leads to skin and tendon
xanthomas?

1262.

Familial polyposis coli

What AD syndrome involves 1000 or more edematous


polyps, most commonly affects the colorectal area, and is
associated with chromosome 5q21?

1263.

Fanconi-like syndrome

What syndrome is characterized by the triad of renal tubular


acidosis, nephrosis, and amino aciduria?

1264.

Farnesyl pyrophosphate (FPP)

What intermediate of cholesterol synthesis anchors proteins


in the membranes and forms CoA?

1265.

The fasciculus cuneatus

What tract carries the ipsilateral dorsal column fibers from


the upper limbs in the spinal cord?

1266.

The fasciculus gracilis (Graceful), which


lies closest to the midline of the spinal
cord.

What tract carries the ipsilateral dorsal column fibers from


the lower limbs in the spinal cord?

1267.

The fast component is directed toward


the affected side of a cerebellar lesion.

If a patient with a cerebellar lesion has nystagmus, which


way is the fast component directed, toward or away from the
lesion?

1268.

The fastigial nucleus

What deep cerebellar nuclei receive Purkinje cell projections


in The vermis?

1269.

Fat necrosis

Name the type of necrosis. Due to lipase activity and has a


chalky white appearance

1270.

Fatty acids cannot cross the blood-brain


barrier; therefore, they cannot be used as
an energy source.

During a prolonged fast, why is the brain unable to use fatty


acids?

1271.

Fc portion of IgG, C3b, and mannosebinding proteins

Name three opsonins.

1272.

The Fc region of IgG and C3b

What are the two opsonizing factors?

1273.

Female pseudohermaphrodite

What condition results from a 46XX karyotype and female


internal organs with virilized external genitalia?

1274.

Femoral nerve

What lower extremity nerve is described by the following


motor loss? Loss of knee extension, weakened hip flexion

1275.

Fentanyl

What drug is given transdermally for chronic pain but can


cause chest wall rigidity if given IV?

1276.

Ferritin

What is the physiologic storage form of iron?

1277.

Ferrochelatase

What enzyme is deficient in hereditary protoporphyria?

1278.

Fetal alcohol syndrome

Name the most common type. Mental retardation

1279.

Fever, night sweats, and hemoptysis

What is the classic triad of TB?

1280.

Fibrillary protein, amyloid protein, and


glycosaminoglycans (heparin sulfate
mainly)

What are the three main components of amyloid?

1281.

Fibrinoid necrosis

Name the type of necrosis. Histologically resembles fibrin

1282.

Fibrinous exudates

Name the type of exudate, given the following examples.


Uremic pericarditis

1283.

Fibroadenoma

What is the most common one? Benign breast tumor in


females less than 35 years old

1284.

Fibroadenoma

What is the most common one? Benign tumor of the


breast

1285.

Fibrocystic change of the breast

Blue-domed cysts

1286.

Fibrocystic change of the breast

Name the most common type or cause. Breast lump in


females

1287.

Fibrocystic change of the breast

What is the most common one? Benign lesion that affects


the breast

1288.

Fibrocystic change of the breast. This highlights


the distinguishing features from breast cancer,
which is commonly unilateral, single nodule, no
variation with pregnancy.

A 30-year-old woman goes to your office with


bilateral multiple breast nodules that vary with
menstruation and have cyclical pain and
engorgement. What is your diagnosis?

1289.

Fibrous pericardium

What portion of the pericardium adheres to the


tunica adventitia of the great vessels?

1290.

Fifth MS CARD is my mnemonic for the aortic arch


derivatives

From what aortic arch are the following structures


derived? Degenerates

1291.

Filiform papillae

What papillae are touch receptors on the tongue


and send their sensations via CN V3 (mandibular
division)?

1292.

Filovirus

What family do the following viruses belong to?


Ebola

1293.

Filtered and secreted: Cx > Cin (i.e., PAH). Filtered


and reabsorbed: Cx < Cin (i.e., glucose), where Cx
= clearance of a substance and Cin = clearance of
inulin.

Is the clearance for a substance greater than or


less than for inulin if it is freely filtered and
secreted? If it is freely filtered and reabsorbed?

1294.

Filtration; if hydrostatic pressure is less than


oncotic pressure, reabsorption is promoted.

If capillary hydrostatic pressure is greater than


oncotic pressure, is filtration or reabsorption
promoted?

1295.

Filtration is greater than excretion for net


reabsorption to occur.

Is filtration greater than or less than excretion for


net reabsorption to occur?

1296.

Finasteride

What agent used in the treatment of BPH and


baldness is a 5--reductase inhibitor?

1297.

First M PITS for pharyngeal pouch derivatives

From what pharyngeal pouch is the following


structure derived? Middle ear

1298.

First MS CARD is my mnemonic for the aortic arch


derivatives

From what aortic arch are the following structures


derived? Maxillary artery

1299.

First-order elimination

What type of drug


elimination is
characterized by the
following? Elimination
directly proportional to
plasma level; constant
fraction eliminated per unit
time; fixed half-life

1300.

First-pass effect

When a drug is
administered orally and
enters into portal
circulation, it undergoes
hepatic metabolism. What
is the name of this effect?

1301.

First-pass metabolism and acid lability

What two factors influence


low oral bioavailability?

1302.

First pharyngeal groove; all others degenerate.

From what pharyngeal


groove is the external
auditory meatus derived?

1303.

The first sensory neuron is in the dorsal root ganglia. It carries ascending
sensory information in the dorsal root of a spinal nerve, eventually
synapsing with second sensory neuron. In the brainstem (DCML) and the
spinal cord (spinothalamic) the second neuron cell body sends its axons to
synapse in the thalamus. The third sensory neuron cell body is a thalamic
nuclei that sends its fibers to the primary somatosensory cortex.

Where are the cell bodies


for the DCML and
spinothalamic sensory
systems?

1304.

The first trimester

In what trimester is the fetus most vulnerable to congenital rubella


syndrome?

1305.

Fitz-Hugh-Curtis syndrome

What syndrome occurs when pelvic inflammatory disease


ascends to surround the liver capsule in violin string adhesions?

1306.

Five pairs through five lumbar


vertebrae. Totaling 31 pairs of spinal
nerves.

How many pairs of spinal nerves are associated with Lumbar


vertebrae?

1307.

Five pairs through five sacral


vertebrae. Totaling 31 pairs of spinal
nerves.

How many pairs of spinal nerves are associated with Sacral


vertebrae?

1308.

Fixation (arrested development)

What is the term for failure to give up infantile patterns of behavior


for mature ones?

1309.

Fixed interval

What type of scheduled reinforcement states that after a desired


response, the reinforcement is given On a set time schedule?

1310.

Fixed ratio (rewards set behaviors)

What type of scheduled reinforcement states that after a desired


response, the reinforcement is given After a set number of
responses?

1311.

Flea-bitten kidney (can also be seen


in pyelonephritis)

What is the term for the appearance of the kidney in malignant


hypertension (it has petechiae on its surface)?

1312.

Flocculonodular lobe (one of my


favorite words in all of medicine!)

In a topographical arrangement of the cerebellar homunculus


map, what area or lobe Controls balance and eye movements?

1313.

Flow cytometric analysis

What test, by using specific Abs to different receptors, allows for


rapid analysis of cell types in a blood sample?

1314.

Fluconazole

What is the only antifungal agent to penetrate CSF?

1315.

Fluconazole

What is the DOC for coccidioidomycosis?

1316.

Fluconazole

What is the DOC for esophageal candidiasis?

1317.

Flucytosine

What antifungal agent is activated by fungal cytosine deaminase


to form 5-FU?

1318.

Flumazenil

What is the name of the benzodiazepine antagonist used in the


treatment of an overdose?

1319.

Flumazenil

Name the antidote. Benzodiazepines

1320.

Flurazepam

Which benzodiazepine has the longest half-life?

1321.

Flutamide

What androgen receptor blocker is used in the treatment of


prostatic cancer?

1322.

F-Met-Peptides

What is the name of the major chemotactic agent released from


Bacteria?

1323.

Fo/F1 complex

What component of the ETC is inhibited by the following?


Oligomycin

1324.

Folate deficiency (very common


in the elderly)

The "tea-and-toast" diet is classically associated with what cause of


megaloblastic anemia?

1325.

Folate, vitamin B12, and vitamin


B6

What three vitamin deficiencies are associated with


homocystinemia?

1326.

The folding of an AA chain

What is determined by the secondary structure of an AA?

1327.

Folic acid

What water-soluble-vitamin deficiency is associated with neural


tube defects in the fetus?

1328.

Folic acid deficiency

Megaloblastic anemia and thrombocytopenia are signs of what


vitamin deficiency?

1329.

Follicular adenoma

What is the most common one? Thyroid adenoma

1330.

Follicular lymphoma

What is the most common form of non-Hodgkin lymphoma in the


United States?

1331.

Follicular phase

What phase of the female cycle occurs during days 1 to 15?

1332.

Follicular phase is estrogen-dependent with increased FSH


levels, while the luteal phase is progesterone-dependent.

What phase of the menstrual cycle is


dominated by estrogen?
Progesterone?

1333.

Foods rich in tyramine (e.g., cheese, dried fish, sauerkraut,


chocolate, avocados, and red wine) should be avoided.
Hypertensive crisis occurs when tyramine and MAOIs are
mixed.

What type of foods should patients


taking MAOIs avoid? Why?

1334.

Foramen of Monro

What connects the lateral ventricles


to the third ventricle?

1335.

Foramen of Winslow

What foramen must be traversed for


entry into the lesser peritoneal sac?

1336.

For males, HTN; for females, pregnancy.

What is the most common primary


diagnosis resulting in an office visit
for males? For females?

1337.

For males it's prostate cancer, and for females it's breast cancer.

What is the second leading cause of


cancer deaths in males? In females?

1338.

Formal operations (> 12 years)

At what stage of cognitive


development (according to Piaget) do
children Have abstract thinking?

1339.

Formation of an identity through issues of independence and


rebellion; they define who they are.

What is the key issue surrounding


teenagers' maturation?

1340.

For the USMLE Step 1 the answer is no, but if the information
would do more harm than good, withhold. This is very rare but it
does occur.

If the family member of a patient


asked you to withhold information,
would you?

1341.

Foscarnet

What antiviral agent for ganciclovirresistant infections has limited utility


because of its nephrotoxic side
effect?

1342.

Fossa ovale

What are the adult remnants of the following


structures? Foramen ovale

1343.

Four

How many mature sperm are produced by one type B


spermatogonium?

1344.

Four high energy bonds, two from ATP in AA


activation and two from GTP

How many high-energy bonds does the cycle of


elongation cost?

1345.

Four Ms and a U 1. Median 2. Medial


antebrachial 3. Medial pectoral 4. Medial
brachial cutaneus 5. Ulnar

What are the five branches of the median cord of the


brachial plexus?

1346.

Fourth M PITS for pharyngeal pouch derivatives

From what pharyngeal pouch is the following


structure derived? Superior parathyroid gland and
ultimobranchial body of the thyroid

1347.

Fourth MS CARD is my mnemonic for the aortic


arch derivatives

From what aortic arch are the following structures


derived? Arch of the aorta and right subclavian
artery

1348.

Fovea

What area of the retina consists of only cones and


has the greatest visual acuity?

1349.

Fragile X syndrome

What disease with familial mental retardation


produces large, everted ears and macro-orchidism?

1350.

Frameshift

Name the type of mutation: The deletion or addition


of a base

1351.

Francisella tularensis

What small gram-negative facultative intracellular rod


is transmitted to human host by Dermacentor tick
bite?

1352.

Free form determines hormone activity and is


responsible for the negative feedback loop.

Is the bound form or free form of a lipid-soluble


hormone responsible for the negative feedback
activity?

1353.

Free hormone levels remain constant, and the


bound hormone level changes with a decrease
in binding hormones.

What happens to free hormone levels when the liver


decreases production and release of binding
proteins?

1354.

Free ionized Ca2+

What is the physiologically active form of Ca2+?

1355.

Free polysomes. Membraneassociated polysomes are


the site of protein synthesis
destined to leave the cell.

What organelle synthesizes proteins that are intended to stay within the
cell?

1356.

Free, unprocessed Ag

What type of Ag do B cells recognize?

1357.

Friedreich ataxia

What AR CNS disorder presents early in childhood with gait ataxia, loss
of deep tendon reflexes, impaired vibratory sensation, hand clumsiness,
and loss of position sense?

1358.

From age 5 onward IQ


stabilizes.

At what age does IQ stabilize?

1359.

From the adrenal medulla;


NE is mainly derived from
the postsynaptic sympathetic
neurons.

Where does most circulating plasma epinephrine originate?

1360.

From the wall of the yolk sac

Where do the primordial germ cells arise?

1361.

Frontal eye field (Brodmann


area 8)

What area of the brain acts as the center for contralateral horizontal
gaze?

1362.

Frontal lobe

Name the area of the cerebral cortex with the function described:
Speech; critical for personality, concentration, initiating and stopping
tasks (do one thing and begin a new without completion of the first),
abstract thought, and memory and higher-order mental functions

1363.

Frontal lobe syndrome


(lesion in the prefrontal
cortex)

What syndrome causes inability to concentrate, easy distractibility,


apathy, and regression to an infantile suckling or grasping reflex?

1364.

Frotteurism

What paraphilia is defined as A male rubbing his genitals on a fully


clothed female to achieve orgasm?

1365.

Fructose-1, 6bisphosphatase

What enzyme catalyzes the rate-limiting step in gluconeogenesis?

1366.

Fructose; both glucose and galactose are


actively absorbed via secondary active
transport.

Which CHO is independently absorbed from the small


intestine?

1367.

FSH

What hormone acts on Granulosa cells?

1368.

FSH and testosterone

What two substances stimulate Sertoli cells?

1369.

Functionally they are part of the left lobe of


the liver because they receive their blood
supply from the left hepatic artery.
Anatomically they are considered part of the
right lobe of the liver.

Are the quadrate and caudate lobes of the liver


functionally part of the left or right lobe?

1370.

Functional residual capacity (FRC)

Name the lung measurement based on the following


descriptions: Volume in the lungs at the end of passive
expiration

1371.

Fungi

Which of the following characteristics accurately describe


fungi, bacteria, viruses, or parasites? Eukaryotic cell; 3
to 10 microns; 80S ribosomes; chitinous cell wall;
ergosterol in cell membrane; replicate via cytokinesis
with mitosis and meiosis

1372.

Fungiform papillae

What papillae send their senses via chorda tympani of


CN VII?

1373.

Fusobacterium

Which organism causes trench mouth?

1374.

G0 phase

What phase of the cell cycle are the following


antineoplastic agents specific for? Nitrosoureas

1375.

G0 phase

What phase of the cell cycle are the following


antineoplastic agents specific for? Cisplatin

1376.

G0 phase

What phase of the cell cycle are the following


antineoplastic agents specific for? Dacarbazine

1377.

G0 phase

What phase of the cell cycle are the following


antineoplastic agents specific for? Alkylating agents

1378.

G0 phase

Name the phase of the eukaryotic cell cycle: Cells cease replicating (i.e., nerve
cell)

1379.

G1 phase; G2 and S
phase are diploid
(2N).

What phase of Interphase is haploid (N)?

1380.

G1 phase (gap 1)

Name the phase of the eukaryotic cell cycle: Period of cellular growth
(translation and transcription) before DNA synthesis

1381.

G2 phase

What phase of the cell cycle are the following antineoplastic agents specific for?
Bleomycin

1382.

G2 phase (gap 2)

Name the phase of the eukaryotic cell cycle: Period of cellular growth
(translation and transcription) after DNA synthesis

1383.

G-6-PD

What is the rate-limiting enzyme of the HMP shunt?

1384.

G-6-PD deficiency

Heinz bodies

1385.

G-6-PD deficiency

In what X-linked recessive disease is there a decrease in the HMP shunt, along
with Heinz body formation?

1386.

G-6-PD deficiency

Eating fava beans can produce the Mediterranean type of what deficiency?

1387.

G-6-PD deficiency;
pyruvate kinase
deficiency is second.

What is the most common genetic deficiency resulting in hemolytic anemia?

1388.

Gabapentin

Which anticonvulsant is used in the treatment of bipolar disorder and neuropathic


pain?

1389.

GABA, quantitatively

What is the most common one? Neurotransmitter in the brain

1390.

gag gene

HIV's capsid, core nucleocapsid, and matrix proteins are products of what
structural gene?

1391.

Gain of hypertonic
fluid (mannitol or
hypertonic saline)

Describe what type of fluid is either gained or lost with the following changes in
body hydration for the ECF volume, ICF volume, and body osmolarity,
respectively: ECF, increase; ICF, decrease; body, increase

1392.

Gain of hypotonic
fluid (water
intoxication or
hypotonic saline)

Describe what type of fluid is either gained or lost with the following changes in
body hydration for the ECF volume, ICF volume, and body osmolarity, respectively:
ECF, increase; ICF, increase; body, decrease

1393.

Gain of isotonic
fluid (isotonic
saline)

Describe what type of fluid is either gained or lost with the following changes in
body hydration for the ECF volume, ICF volume, and body osmolarity, respectively:
ECF: increase; ICF: no change; body: no change

1394.

Galactitol

To what does aldose reductase convert galactose?

1395.

Galactose and
glucose

What two monosaccharides are produced when lactose is hydrolyzed?

1396.

Galactoside
permease

Regarding the Lac operon, for what do the following genes code? Y gene

1397.

Gametocytes

What form of the Plasmodium species is ingested by mosquitoes?

1398.

gamma and delta


chains

What are the two chains of the TCR that are mainly found on the skin and mucosal
surfaces?

1399.

Ganciclovir

What is the DOC for CMV retinitis?

1400.

Ganglioside

What sphingolipid cannot be produced without sialic acid and amino sugars?

1401.

Gangrenous
necrosis

Name the type of necrosis. Seen as dead tissue with coagulative necrosis

1402.

Gap junctions

What is the name of hydrophilic pores that allow the direct passage of ions and
particles between two adjacent cells?

1403.

Gardnerella
vaginalis

Clue cells are associated with which organism that causes vaginal discharge?

1404.

Gardner
syndrome

What AD GI neoplasia produces multiple adenomatous polyps, osteomas, fibromas,


and epidural inclusion cysts?

1405.

Gastric cancer

Name the cancer associated with the following chemical agents. (Some may have
more than one answer.) Nitrosamines

1406.

Gastric carcinoma

Signet ring cells

1407.

Gastric carcinoma

A Japanese man has weight loss, anorexia, early satiety, epigastric abdominal pain,
and a palpable left supraclavicular lymph node. On endoscopy you find a large,
irregular ulcer with elevated margins on the lesser curvature of the stomach. What
is your diagnosis?

1408.

Gastric ulcer

What type of peptic ulcer disease is characterized by the onset of


burning epigastric pain immediately after eating?

1409.

Gastrinoma

What pancreatic islet cell tumor is associated with MEN I


syndrome?

1410.

Gastrin-releasing peptide (GRP)


stimulates G cells to release
Gastrin. (All G's)

What parasympathetic neurotransmitter of the GI tract stimulates


the release of gastrin?

1411.

Gaucher disease

What disorder is due to a deficiency in the enzyme


glucocerebrosidase?

1412.

Gaucher disease

What is the most common lysosomal storage disorder?

1413.

G-C rich sequences, because they


have 3 hydrogen bonds, where A-T
has 2 hydrogen bonds, resulting in
higher melting points.

Would a G-C or an A-T rich dsDNA sequence have a higher


melting point? Why?

1414.

A gene (a rather simple definition


but accurate)

What is the term for a unit of DNA that encodes a particular protein
or RNA molecule?

1415.

Generalized anxiety disorder

What disorder, experienced more than half of the time for a 6month period, is described as being fearful, worrisome, or
impatient and having sleep disturbances, poor concentration,
hyperactivity, and an overall sense of autonomic hyperactivity?

1416.

Genomic libraries are made from


nuclear DNA, are fragmented, and
contain all sequences found in the
particular genome copied.

Do genomic or cDNA libraries contain introns, exons, promoters,


enhancers, and are they fragmented?

1417.

Gerstmann's syndrome; spoken


language is usually understood.

What syndrome is described by a lesion in the angular gyrus (area


39) resulting in alexia, agraphia, acalculia, finger agnosia, and
right-left disorientation?

1418.

Gestational trophoblastic neoplasia


(GTN or choriocarcinoma)

What malignant tumor of the trophoblast causes high levels of


hCG and may occur after a hydatidiform mole, abortion, or normal
pregnancy?

1419.

GH

T3 increases bone ossification through synergistic effect with what


hormone?

1420.

GH and prolactin are produced by


acidophils; all others are by
basophils.

What two pituitary hormones are produced by acidophils?

1421.

GH deficiency

Insulin-induced hypoglycemia is the most reliable (by far not the


safest) test for what hormone deficiency?

1422.

GH, especially IGF-1. GH also


increases the incorporation of
thymidine in DNA synthesis and
uridine in RNA synthesis.

What hormone has the following effects: chondrogenic in the


epiphyseal end plates of bones; increases AA transport for protein
synthesis; increases hydroxyproline (collagen); and increases
chondroitin sulfate synthesis?

1423.

A Ghon focus is a TB tubercle,


whereas a Ghon complex is a focus
with hilar lymph node involvement.

What is the difference between a Ghon focus and a Ghon


complex?

1424.

GH-producing adenoma

With which pituitary adenoma is an elevated somatomedin C level


associated?

1425.

GH. The largest output of GH in a


24-hour period is during stage 4
sleep.

What hormone's release is strongly associated with stage 4


sleep?

1426.

Giant cell bone tumor


(osteoclastoma)

What malignant neoplasm of the bone has a soap bubble


appearance on radiograph?

1427.

Giant cell tumor of the bone

Soap bubble appearance on radiograph

1428.

Giardia lamblia (treat with


metronidazole)

What protozoal parasite forms flasked-shaped lesions in the


duodenum, is transmitted via fecal-oral route, and is commonly
seen in campers who drank stream water?

1429.

Gigantism

What does prepubertal hypersecretion of growth hormone lead


to?

1430.

Gilbert's syndrome

What disease has a genetically low level of UDPglucuronate


transferase, resulting in elevated free unconjugated bilirubin?

1431.

Glanzmann syndrome

What AR disorder is due to a deficiency in glycoprotein IIb-IIIa,


resulting in a defect in platelet aggregation?

1432.

Glioblastoma multiforme

What is the most common one? Primary CNS tumor

1433.

Glioblastoma multiforme

Pseudopalisades

1434.

Glioblastoma multiforme

What is the most common one? Primary CNS tumor in adults

1435.

Global aphasia

Name the aphasia based on these characteristics: Both Broca's


and Wernicke's areas damaged by lesion in the presylvian speech
area; trouble repeating statements; poor comprehension with
telegraphic speech

1436.

Glomerular capillary pressure


(increased glomerular capillary
pressure, increased GFR and
vice versa)

What is the main factor determining GFR?

1437.

Glossopharyngeal nerve

What CN is associated with the sensory innervation of


Oropharynx?

1438.

Glucagon

Name the hormoneglucagon, insulin, or epinephrine:


Glycogenolytic, gluconeogenic, lipolytic, glycolytic, proteolytic, and
stimulated by hypoglycemia and AAs

1439.

Glucagon and epinephrine

What two stress hormones are under the permissive action of


cortisol?

1440.

Glucocerebrosidase

What lysosomal enzyme is deficient in Gaucher's disease?

1441.

Glucocorticoids and
amphetamines

What two classes of drugs can cause schizoid behavior?

1442.

Glucokinase

What glycolytic enzyme has a high Vmax, high Km, and low affinity
for glucose?

1443.

Glucose-6-phosphatase

What enzyme is deficient in the following glycogen storage disease?


von Gierke's disease

1444.

Glucose-6-phosphatase

What gluconeogenic enzyme is absent in muscle, accounting for its


inability to use glycogen as a source for blood glucose?

1445.

Glucose and galactose

What two sugars can be used to produce cerebrosides?

1446.

Glucosyl cerebroside

What substrate gets built up in Gaucher's disease?

1447.

Glucuronidation

What enzyme system is the most common form of phase II


biotransformation?

1448.

GLUT 2

Name the GLUT transporter based on the following: Found in liver


and pancreatic -cells

1449.

GLUT 3 and 4

Name the GLUT transporter based on the following: Found in most


tissues, including brain and RBCs

1450.

GLUT 4

Name the GLUT transporter based on the following: Found in


skeletal muscle and adipose tissues

1451.

Glutamate

What AA is a precursor of the following substances? GABA

1452.

Glutamate

What neurotransmitter inhibits the optic nerve bipolar cell and shuts
off in response to light?

1453.

Glutamate dehydrogenase

What enzyme catalyzes the reversible oxidative deamination of


glutamate and produces the TCA cycle intermediate ketoglutarate?

1454.

Glutamine

What AA is the major carrier of nitrogen byproducts from most


tissues in the body?

1455.

Glutamine and aspartate

What two AAs act as excitatory transmitters in the CNS, generating


EPSPs?

1456.

Gluteus maximus

What is the largest muscle in the body?

1457.

Glyceraldehyde dehydrogenase

What is the enzyme for the oxidative reaction in glycolysis?

1458.

Glycerol kinase

What liver enzyme, for triglyceride synthesis, converts glycerol to


glycerol-3-phosphate?

1459.

Glycine and succinyl-CoA

What are the two precursors of heme?

1460.

Glycine/arginine

What AA is a precursor of the following substances? Creatine

1461.

Glycogen phosphorylase

What enzyme catalyzes the rate-limiting step in glycogenolysis?

1462.

Glycogen synthase

What is the rate-limiting enzyme of glycogen synthesis?

1463.

GM2 ganglioside Caused by a


deficiency of -hexosaminidase
A

What substrate builds up in Tay-Sachs disease?

1464.

GnRH

What hypothalamic hormone is synthesized in


the preoptic nucleus?

1465.

GnRH constant infusion

What hormone disorder is characterized by the


following abnormalities in sex steroids, LH,
and FSH? Sex steroids , LH , FSH ?

1466.

GnRH pulsatile infusion

What is it called when levels of sex steroids


increase, LH increases, and FSH increases?

1467.

Golgi tendon organs are stimulated by Ib afferent


neurons in response to an increase in force or
tension. The inverse muscle reflex protects muscle
from being torn; it limits the tension on the muscle.

What encapsulated group of nerve endings seen


at the muscle-tendon junction responds to an
increase in tension generated in that muscle?
(This is dropping a box that is too heavy to carry.)

1468.

Gonads

What embryonic structure forms the adult male


structure? Testes, seminiferous tubules, and
rete testes

1469.

Gonads

What embryonic structure forms the adult female


structures? Ovary, follicles, rete ovarii

1470.

Gonorrhea

Name the most common reported STD. In


males

1471.

Goodpasture disease

Name the nephritic disease based on the


immunofluorescent staining. Smooth and linear
pattern of IgG and C3 in the GBM

1472.

Goodpasture syndrome

What syndrome is due to anti-GBM Abs directed


against the lung and kidneys?

1473.

Goodpasture syndrome

What type II hypersensitivity disorder is defined


as Autoantibodies against the type IV collagen
in the basement membrane of the kidneys and
lungs?

1474.

Good Samaritan Law. (Physicians are not required to


stop and help.)

What law was adopted to shield physicians from


liability when helping at the scene of an
accident?

1475.

Gout

What pathology is associated with podagra,


tophi in the ear, and PMNs with monosodium
urate crystals?

1476.

Gout

Upon seeing negatively birefringent needleshaped crystals from a joint aspiration of the
great toe, what form of arthritis do you diagnose?

1477.

GP41

What glycoprotein in the HIV virus is used for


fusion?

1478.

GP120

What glycoprotein in the HIV virus attaches to


CD4?

1479.

gp120

What CD4 T-cell receptor does the HIV virus


bind to?

1480.

GP Ib

What glycoprotein allows platelets to adhere to


von Willebrand factor?

1481.

GP IIb and IIIa receptors

What are the glycoprotein receptors that


platelets and fibrinogen cross-link to when
forming a thrombus?

1482.

GP IIb/IIIa, which is why GP IIb/IIIa inhibitors are


used in the treatment of acute coronary syndromes

What glycoprotein allows platelets to adhere to


each other through the use of fibrinogen?

1483.

Gradient-time system

What form of renal tubular reabsorption is


characterized by high back leak, low affinity for
substance, and absence of saturation and is
surmised to be a constant percentage of a
reabsorbed filtered substance?

1484.

The granule cell is the only excitatory neuron in the


cerebellar cortex, and it uses glutamate as its
neurotransmitter. All the other cells in the cerebellum
are inhibitory neurons, and they use GABA as their
neurotransmitter.

What is the only excitatory neuron in the


cerebellar cortex, and what is its
neurotransmitter?

1485.

The granule cell. Its neurotransmitter is


glutamic acid, which is also the principal
neurotransmitter of the visual pathways.

What is the most abundant neuron in the cerebellum?

1486.

Granulosa cell

What cell converts androgens to estrogens?

1487.

Granulosa cells

What follicular cell possesses FSH receptors and converts


androgens into estradiol?

1488.

Granulosa cell tumor of the ovary

What estrogen-producing tumor of the female genital tract


is characterized by Call-Exner bodies?

1489.

Granulosa/thecal cell tumor of the ovary

Call-Exner bodies

1490.

Granulose cells secrete progesterone.


After fertilization the granulose cells form
from follicular cells.

After fertilization, what cells of the corpus luteum Secrete


progesterone?

1491.

Grasp proceeds release

In regard to motor development during infancy, choose the


motor response that happens first. Release or grasp

1492.

Graves disease

What type II hypersensitivity disorder is defined as


Autoantibodies directed against the TSH receptor?

1493.

Graves disease

What disease has IgG autoantibodies, occurs in women


more than men, and includes exophthalmos, pretibial
myxedema, nervousness, heart palpitations, and fatigue?

1494.

Graves disease

What disorder leads to IgG autoantibodies to the TSH


receptor?

1495.

Graves disease (Increased T4 decreases


TRH and TSH through negative feedback.)

What thyroid abnormality has the following? TRH


decreased, TSH decreased, T4 increased

1496.

The greater the cell diameter, the greater


the conduction velocity.

How does cell diameter affect the conduction velocity of an


action potential?

1497.

The greater the myelination, the greater


the conduction velocity.

How does myelination affect conduction velocity of an


action potential?

1498.

Griseofulvin

What oral antifungal agent is used to treat dermatophyte infections


by disrupting microtubule structure and depositing keratin?

1499.

Group A -hemolytic streptococci

Rheumatic fever most commonly follows pharyngeal infections with


what bacteria?

1500.

Guanethidine and bretylium

What two drugs inhibit the release of neurotransmitters from


storage granules?

1501.

Gubernaculum

What embryonic structure forms the adult male structure?


Gubernaculum testes

1502.

Gubernaculum

What embryonic structure forms the adult female structures?


Ovarian and round ligaments

1503.

Guillain-Barr syndrome

What syndrome has loss of deep tendon reflexes, muscle


weakness, and ascending paralysis preceded by a viral illness?

1504.

The gut rotates clockwise around


the superior mesenteric artery.

What direction does the primitive gut rotate? What is its axis of
rotation?

1505.

H1

Are the following responses associated with histamines H1 or H2


receptor activation? Bronchial smooth muscle activity

1506.

H1

Are the following responses associated with histamines H1 or H2


receptor activation? Pain and pruritus

1507.

H1

Are the following responses associated with histamines H1 or H2


receptor activation? Edema

1508.

H1 histones

What histone binds two nucleosomes together?

1509.

H2

Are the following responses associated with histamines H1 or H2


receptor activation? Inotropic action and automaticity

1510.

H2

Are the following responses associated with histamines H1 or H2


receptor activation? SA nodal activity

1511.

H2

Are the following responses associated with histamines H1 or H2


receptor activation? Gastric acid secretion

1512.

H2PO4- (dihydrogen phosphate)

What is the titratable acid form of H+ in the urine?

1513.

Haemophilus ducreyi

Name the most common type or cause.


Chancre

1514.

Haemophilus influenzae type B

What is the MCC of the following


meningitides? In those 12 months to 6
years of age

1515.

Haemophilus influenzae type B

Name the MCC. Epiglottitis in an


unvaccinated child

1516.

The hairpin loop made by reverse transcriptase at the 3' end


of the first strand is the primer.

What is the primer for the synthesis of the


second strand in production of cDNA from
mRNA?

1517.

Hairy cell leukemia

What B-cell neoplasm is seen in males


with massive splenomegaly, produces dry
tap on bone marrow aspirations, and
stains positive for tartrate-resistant acid
phosphatase (TRAP)?

1518.

The hallmarks of LMN lesion injury are absent or decreased


reflexes, muscle fasciculations, decreased muscle tone, and
muscle atrophy What two areas of the skin do flaccid
paralysis). Don't forget, LMN lesions are ipsilateral at the
level of the lesion!

What happens to muscle tone and stretch


reflexes when there is a LMN lesion?

1519.

Hallucinations are sensory impressions (without a stimulus);


illusions are misperceptions of real stimuli; and delusions
are false beliefs that are not shared by the culture.

How do delusions, illusions, and


hallucinations differ?

1520.

Haloperidol and fluphenazine (but use of any high-potency


phenothiazine runs the risk of EPS)

Which two antipsychotic drugs create a


high risk for developing extrapyramidal
syndrome (EPS) side effects?

1521.

The Halsted-Reitan battery. It consists of finger oscillation,


speech sound perception, rhythm, tactual, and category
testing.

What neuropsychologic test has five


basic scales testing for the presence and
localization of brain dysfunction?

1522.

Hapten; if it is coupled with a


carrier, it may become
immunogenic.

What is the term for an immunogenic agent that is too small to elicit an
immune response?

1523.

Hapten (not immunogenic)

What is the term for a single isolated antigenic determinant?

1524.

Hashimoto thyroiditis

Name the most common type or cause. Noniatrogenic hypothyroidism


in the United States

1525.

HBc Ab

What Ab is an indication of recurrent disease for hepatitis?

1526.

HBcAb and HBeAb. You see


the antibodies c and e.

In the window phase of a hepatitis B infection, which Abs do you see?

1527.

HBcAb IgG and HBsAb IgG

Which hepatitis B serology markersHBcAb IgG, HBcAb IgM, HBeAg,


HBsAb IgG, HBsAg, HBV-DNAare associated with the following
periods? Prior infection

1528.

HBcAb IgG, HBV-DNA,


HBeAg, HBsAg

Which hepatitis B serology markersHBcAb IgG, HBcAb IgM, HBeAg,


HBsAb IgG, HBsAg, HBV-DNAare associated with the following
periods? Chronic infection

1529.

HBcAb IgM

Which hepatitis B serology markersHBcAb IgG, HBcAb IgM, HBeAg,


HBsAb IgG, HBsAg, HBV-DNAare associated with the following
periods? Window period

1530.

HBcAb IgM, HBV-DNA,


HBeAg, HBsAg

Which hepatitis B serology markersHBcAb IgG, HBcAb IgM, HBeAg,


HBsAb IgG, HBsAg, HBV-DNAare associated with the following
periods? Acute infection

1531.

HBeAb

What Ab is an indication of low transmissibility for hepatitis?

1532.

HBeAb

Which hepatitis B Ab indicates low transmissibility?

1533.

HBeAg

What Ag is needed to diagnose an infectious patient with hepatitis B?

1534.

HBeAg

Which hepatitis B Ag correlates with infectivity and viral proliferation?

1535.

HBsAb IgG

Which hepatitis B serology markersHBcAb IgG, HBcAb IgM, HBeAg,


HBsAb IgG, HBsAg, HBV-DNAare associated with the following
periods? Immunization

1536.

HBsAg and HBeAg

What two Ags must be positive for a patient to have


chronic active hepatitis?

1537.

HBsAg as its envelope

What does hepatitis D virus need from hepatitis B


virus to be infective?

1538.

HBsAg (incubation period)

What is the first Ag seen in an individual with


hepatitis?

1539.

hCG

What hormone, produced by the syncytiotrophoblast,


stimulates the production of progesterone by the
corpus luteum?

1540.

hCG can be detected in the blood by day 8 and


in the urine by day 10.

How early can a pregnancy be detected by hCG


assays in the blood? In urine?

1541.

hCS and serum estriol, which are produced by


the fetal liver and placenta, respectively, are
used as estimates of fetal well-being.

What hormones, secreted in proportion to the size of


the placenta, are an index of fetal well-being?

1542.

HDL

Name the lipoprotein based on the following


characteristics. apo A-1, apo E, apo C-II

1543.

Head of the pancreas

What is the most common one? Site of pancreatic


cancer

1544.

Healing by secondary intention

What is characterized by an intense inflammatory


reaction, an increase in the amounts of granulation
tissue and wound contraction by myofibroblasts?

1545.

Heart disease

What is the MCC of death in the United States?

1546.

Heart, esophagus, and colon. Remember, you


get megas: cardiomegaly, megaesophagus,
and megacolon.

What three organs can be affected by Trypanosoma


cruzi?

1547.

Heart rate determines the diastolic interval, and


contractility determines the systolic interval.

Does the heart rate determine the diastolic or systolic


interval?

1548.

Heinz bodies

Oxidation of Hgb forms what bodies in patients with G6-PD deficiency?

1549.

Helicase (requires ATP for energy)

What DNA replication enzyme breaks the hydrogen


bond of base pairing, forming two replication forks?

1550.

Helicobacter pylori

What motile, gram-negative spiral bacillus with flagella is oxidase


positive, urease positive, and associated with gastritis, peptic
ulcer disease, and stomach cancer?

1551.

Helicobacter pylori, which is also


associated with an increased risk of
gastric carcinoma

What urease-producing gram-negative curved rod is associated


with PUD and chronic gastritis?

1552.

Hemangioblastoma

What vascular tumor associated with von Hippel-Lindau syndrome


involves the cerebellum, brainstem, spinal cord, and retina?

1553.

Hemangioma (benign)

What is the most common one? Primary tumor of the liver

1554.

Hematogenous

What is the most common one? Route of spread in pyogenic


osteomyelitis

1555.

Hematogenous

What is the most common one? Route for infectious arthritis

1556.

Hematoxylin

Do the following structures pick up stain from hematoxylin or


eosin? Nuclei

1557.

Hematoxylin

Do the following structures pick up stain from hematoxylin or


eosin? Calcium

1558.

Hematoxylin

Do the following structures pick up stain from hematoxylin or


eosin? Bacteria

1559.

Hematoxylin

Do the following structures pick up stain from hematoxylin or


eosin? Nucleoli

1560.

Hemiazygous vein

What vein drains the lower third of the thoracic wall?

1561.

Hemiballismus

What is the name of violent projectile movements of a limb


resulting from a lesion in the subthalamic nuclei of the basal
ganglia?

1562.

Hemochromatosis

What is the term for increased iron deposition resulting in


micronodular cirrhosis, CHF, diabetes, and bronzing of the skin?

1563.

Hemolytic uremic syndrome

What platelet disorder is characteristically seen in children


following a bout of gastroenteritis with bloody diarrhea?

1564.

Hemophilia A

What form of hemophilia is X-linked recessive and due to a


deficiency in factor VIII?

1565.

Hemophilia B

What form of hemophilia is X-linked recessive and is due to a


deficiency in factor IX?

1566.

Hemorrhagic exudates

Name the type of exudate, given the following examples.


Rickettsial infection

1567.

Hemosiderin

What Hgb-derived endogenous pigment is found in areas of


hemorrhage or bruises?

1568.

Hemostasis

What is the term for formation of a stable fibrin-platelet plug to


stop bleeding?

1569.

Henoch-Schnlein purpura

With what disease do you see IgA deposits in small vessels of


the skin and the kidneys?

1570.

Hepadnavirus

What is the only DNA virus that has the reverse transcriptase
enzyme?

1571.

Hepadnavirus

Name the DNA virus: Partially dsDNA circular; enveloped;


virion-associated polymerases; has RNA intermediate;
replicates in the nucleus

1572.

Heparin, intrinsic pathway, PTT;


warfarin, extrinsic pathway, PT
(mnemonic: hPeT, wPiTT)

Do you monitor heparin's therapeutic levels by PT or PTT?

1573.

Hepatic adenomas

What liver tumor is associated with oral contraceptive pill


use?

1574.

Hepatic glycogen phosphorylase

What enzyme is deficient in the following glycogen storage


disease? Hers' disease

1575.

Hepatitis A

Name the hepatitis virus based on the following information.


Naked capsid RNA picornavirus

1576.

Hepatitis A

Which hepatitis virus is in the Picornaviridae family?

1577.

Hepatitis A (infectious)

Which type of hepatitis is a picornavirus?

1578.

Hepatitis B

Name the hepatitis virus based on the following information.


Enveloped DNA hepadnavirus

1579.

Hepatitis B

Which type of hepatitis can cause hepatocellular carcinoma?

1580.

Hepatitis B and hepatitis C

What are the two hepatitis viruses that can be chronic and can
lead eventually to hepatocellular carcinoma?

1581.

Hepatitis C

Name the hepatitis virus based on the following information.


Enveloped RNA flavivirus

1582.

Hepatitis C

Which hepatitis virus is in the Flaviviridae family?

1583.

Hepatitis D

Which hepatitis strain is a defective virus that can replicate only


inside HBV-infected cells?

1584.

Hepatitis D

Name the hepatitis virus based on the following information.


Small circular RNA virus with defective envelope

1585.

Hepatitis E

What type of hepatitis has the highest mortality rate among


pregnant women?

1586.

Hepatitis E

Name the hepatitis virus based on the following information.


Naked capsid RNA calicivirus

1587.

Hepatitis E (enteric)

Which type of hepatitis is a calicivirus?

1588.

Hepatitis, with or without necrosis

What toxicities are caused by the following agents? Halothane

1589.

Hepatocellular carcinoma

What is the most common one? Primary malignant tumor in


the liver

1590.

Hepatocellular carcinoma

Name the cancer associated with the following chemical


agents. (Some may have more than one answer.) Aromatic
amines and azo dyes

1591.

Hepatocellular carcinoma is
associated with hepatitis B and C
infections, not with EBV.

Which carcinomaBurkitt's lymphoma, nasopharyngeal


carcinoma, hepatocellular carcinoma, or thymic carcinomais
not associated with EBV?

1592.

Hepatocytes lack the enzyme succinyl


CoA acetoacetyl CoA transferase
(thiophorase).

Why is the liver unable to metabolize ketone bodies?

1593.

Hepatoma and nonseminomatous


testicular germ cell tumors

Name the cancer associated with the following tumor markers.


(Some may have more than one answer.) alpha-Fetoprotein

1594.

Hereditary spherocytosis

What normochromic, normocytic AD anemia has


splenomegaly and increased osmotic fragility?

1595.

Heroin

What illicit drug can cause amyloidosis and focal


segmental glomerulosclerosis in the kidney?

1596.

Herpes I

What virus lies dormant in the Trigeminal ganglia?

1597.

Herpes II

What virus lies dormant in the Sensory ganglia of S2


and S3?

1598.

Herpesvirus

Cowdry type A bodies

1599.

Herpes virus

Name the DNA virus Linear dsDNA; nuclear envelope;


icosahedral; replicates in the nucleus

1600.

Herpes virus I and II

What viruses are associated with Cowdry type A


intranuclear inclusions?

1601.

Heterochromatin, the light stuff in the nucleus


on an electron microscope image.

What type of chromatin is transcriptionally inactive?

1602.

Hexosaminidase A

What lysosomal enzyme is deficient in Tay-Sachs


disease?

1603.

Hgb F

What type of Hgb is increased in patients with sickle cell


anemia who take hydroxyurea?

1604.

HGPRT

What enzyme of the purine salvage pathway is deficient


in the following? Lesch-Nyhan syndrome

1605.

HHV 6

What DNA virus is associated with exanthem subitum


(roseola)?

1606.

HHV-8

What virus is associated with body cavity large B-cell


lymphomas?

1607.

HHV 8

What herpes virus is associated with Kaposi sarcoma?

1608.

High average

What is the label given to an individual whose IQ is


110 to 119

1609.

A high-resistance system is formed when


resistors are added in a series.

What type of resistance system, high or low, is formed


when resistors are added in a series?

1610.

High volume, watery solution;


sympathetic stimulation results
in thick, mucoid saliva.

What type of saliva is produced under parasympathetic stimulation?

1611.

Hirschsprung disease

Name the most common type or cause. Neonatal bowel obstruction

1612.

Hirschsprung disease
(aganglionic megacolon)

What is the term for a congenital absence of the ganglionic cells of the
Auerbach and Meissner plexus in the rectum and sigmoid colon?

1613.

Hirschsprung's disease
(colonic gangliosus)

What disease results in a failure of neural crest cells to migrate to the


myenteric plexus of the sigmoid colon and rectum?

1614.

Histamine

What neurotransmitter is associated with sedation and weight gain?

1615.

Histamine

What are the primary neurotransmitters at the following sites? The


hypothalamus

1616.

Histamine

IgE-mediated mast cell release, C3a and C5a, and IL-1 all trigger the
release of what vasoactive amine?

1617.

Histidine

What AA is a precursor of the following substances? Histamine

1618.

Histidine, because of the


imidazole ring found in the R
group, is basic.

What AA is classified as basic even though its pK is 6.5 to 7?

1619.

Histiocytes

Name the macrophage based on its location: Connective tissue


macrophages

1620.

Histiocytosis X

Birbeck granules

1621.

Histiocytosis X

What malignant tumor of the skin is associated with Birbeck granules?

1622.

Histone acetylases is a
regulator favoring gene
expression. All of the others
favor inactivation.

Which of the followingDNA methylating enzymes, scaffolding


proteins, histone acetylases, or deacetylasesis a regulator of
eukaryotic gene expression?

1623.

Histoplasma capsulatum

Which dimorphic fungus is endemic in the Ohio, Mississippi, Missouri,


and Eastern Great Lakes, is found in soil with bird and bat feces, and
is associated with infections in spelunkers and chicken coop cleaners?

1624.

Histoplasma capsulatum infects the cells of


the RES and can result in
hepatosplenomegaly.

What facultative intracellular fungus is associated with


hepatosplenomegaly?

1625.

Histrionic

Name the cluster B personality disorder: Colorful,


dramatic, extroverted, seductive, and unable to hold longterm relationships

1626.

HIV

What enveloped RNA retrovirus infects CD4 T cells and


uses the enzyme reverse transcriptase?

1627.

HIV and poxvirus

What two viruses get their envelope not from budding but
from coding?

1628.

HMG CoA reductase

What is the rate-limiting step of the following? Cholesterol


synthesis

1629.

HMG-CoA reductase

What enzyme of cholesterol synthesis is inhibited by the


statins?

1630.

HMG-CoA reductase

What enzyme catalyzes the rate-limiting step in cholesterol


metabolism?

1631.

HMG CoA synthase

What is the rate-limiting step of the following?


Ketogenolysis

1632.

HMP shunt

What cytoplasmic pathway produces NADPH and is a


source of ribose 5-phosphate?

1633.

HMP shunt

Via what pathway is glycolysis increased after


phagocytosis?

1634.

hnRNA

Name the RNA subtype based on the following: Found


only in the nucleus of eukaryotic cells and are precursors
of mRNA

1635.

Hodgkin lymphoma

Reed-Sternberg cells

1636.

Homeodomain proteins

What group of eukaryotic regulatory proteins has a major


factor in controlling the gene expression embryonically?

1637.

Homicide. It is also the leading cause of


death in black females aged 15 to 24.

What is the MCC of death in black males aged 15 to 24?

1638.

Homocysteine methyl transferase

What is the only enzyme in the body that uses N-5-methyl


folate?

1639.

Homocysteine methyl transferase and methylmalonyl


CoA transferase

What two enzymes are vitamin B12-dependent

1640.

Homogentisate oxidase deficiency is seen in patients


with alcaptonuria.

What enzyme deficiency results in darkening of


the urine when exposed to air?

1641.

Homogentisic oxidase

What enzyme is deficient in alkaptonuria?

1642.

Hormone secretion for T-cell differentiation and T-cell


education to recognize self from nonself

What are the two functions of the thymus in Tcell differentiation?

1643.

Hormone-sensitive lipase

What hormone is activated in adipose tissue


when blood glucose levels decrease?

1644.

Hormone-sensitive lipase, which breaks down


triglyceride into glycerol and free fatty acid

What hormone, stimulated by epinephrine,


results in an increase in lipolysis?

1645.

Hot pink with PAS and grey to black with silver stain

What color do fungi stain with PAS? Silver


stain?

1646.

Housemaid's knee

What is the name of inflammation of the


prepatellar bursa?

1647.

Howell-Jolly bodies

What is the term for RBC remnants of nuclear


chromatin in asplenic patients?

1648.

HPV

What virus produces koilocytic cells on a


Papanicolaou (Pap) smear?

1649.

HPV

What is the most common STD?

1650.

HPVs 16 and 18

What viruses are associated with cervical


carcinoma?

1651.

HPV serotypes 1 and 4

Which serotypes of HPV are associated with


plantar warts?

1652.

HPV serotypes 6 and 11

Which HPV serotypes are associated with


condyloma acuminatum?

1653.

HPV serotypes 16, 18, 31, and 33

What HPV serotypes are associated with


increased risk of cervical cancer?

1654.

HTLV-1

What retrovirus is associated with adult T-cell leukemia?

1655.

HTN

What is the most common condition leading to an intracerebral


bleed?

1656.

Human chorionic
somatomammotropin (hCS) or
human placental lactogen (hPL)

What hormone is secreted by the placenta late in pregnancy,


stimulates mammary growth during pregnancy, mobilizes energy
stores from the mother so that the fetus can use them, and has an
amino acid sequence like GH?

1657.

Humerus (between the medial


epicondyle and the trochlea)

What bone houses the ulnar groove?

1658.

Humerus with ulna (major) and


radius (minor)

Name the bony articulations of the following sites. Be specific.


Elbow

1659.

Humor

Name these mature defense mechanisms: Easing anxiety with


laughter

1660.

Huntington disease

What AD disease is associated with chromosome 4p; does not


present until the person is in his or her 30s; and involves atrophy of
the caudate nucleus, dilatation of the lateral and third ventricles, and
signs of extrapyramidal lesions?

1661.

Huntington disease

What AD disorder is characterized by degeneration of GABA


neurons in the caudate nucleus resulting in atrophy, chorea,
dementia, and personality changes?

1662.

Huntington's chorea. (Death in


15-20 years, often via suicide.)

What AD dementia has a defect in chromosome 4, onset between


the ages of 30 and 40, choreoathetosis, and progressive
deterioration to an infantile state?

1663.

Huntington's chorea; patients


have chorea, athetoid
movements, progressive
dementia, and behavioral
problems.

What chromosome 4, AD disorder is a degeneration of GABA


neurons in the striatum of the indirect pathway of the basal ganglia?

1664.

Hurler syndrome and Hunter


syndrome, respectively

What syndrome results if the enzyme -1-iduronidase is deficient? Liduronate sulfatase deficiency?

1665.

Hyaluronic acid (all sulfates bind


to the core protein)

What is the only glycosaminoglycan (GAG) that binds to the linker


portion of the proteoglycan?

1666.

Hydatidiform mole

Preeclampsia in the first trimester, hCG levels above 100,


00 mIU/mL, and an enlarged bleeding uterus are clinical
signs of what?

1667.

Hydatidiform mole

What cystic swelling of the chorionic villi is the most


common precursor of choriocarcinoma?

1668.

Hydralazine

Which direct-acting vasodilator is associated with SLE-like


syndrome in slow acetylators?

1669.

Hydralazine, isoniazid, and procainamide


(HIP)

What three drugs are associated with SLE-like syndrome


in slow acetylators?

1670.

Hydrolysis

What form of oxidation takes place with the addition of a


water molecule and breakage of bonds?

1671.

Hydrophobic (water-insoluble) end faces


the aqueous environment and the
hydrophilic (water-soluble) end faces the
interior of the cell.

Is the hydrophobic or hydrophilic end of the phospholipids


of the cell membrane facing the aqueous environment?

1672.

Hydrostatic pressure of the glomerular


capillaries (promotes filtration)

Under normal conditions, what is the main factor that


determines GFR?

1673.

Hydroxyapatite

What is the major inorganic component of bone?

1674.

Hydroxyproline

What form of AA is found only in collagen?

1675.

Hydroxyproline

Increased urinary excretion of what substance is used to


detect excess bone demineralization?

1676.

Hydroxyproline (breakdown product of


collagen)

Excess bone demineralization and remodeling can be


detected by checking urine levels of what substance?

1677.

Hyperacute rejection

Based on the following information, is the renal


transplantation rejection acute, chronic, or hyperacute?
Immediately after transplantation; seen as a neutrophilic
vasculitis with thrombosis

1678.

Hypercortisolism (Cushing
syndrome)

Thin extremities, fat collection on the upper back and abdomen,


hypertension, hypokalemic alkalosis, acne, hirsutism, wide purple
striae, osteoporosis, hyperlipidemia, hyperglycemia with insulin
resistance, and protein depletion are all characteristics of what
disorder?

1679.

Hypermyelination of the
corpus striatum and the
thalamus (seen in cerebral
palsy)

What causes slow writhing movements (athetosis)?

1680.

Hyperplasia

What is the term for an increase in the number of cells in a tissue?

1681.

Hyperpolarization (i.e., K+
influx)

What is the term for the negative resting membrane potential becoming
more negative?

1682.

Hypersegmented neutrophils

What is the first sign of megaloblastic anemia on a peripheral blood


smear?

1683.

Hypertension

What is the primary predisposing factor for vascular dementia?

1684.

Hypertrophic cardiomyopathy
(hypertrophic subaortic
stenosis, or IHSS)

What cardiomyopathy is due to a ventricular outflow obstruction as a


result of septal hypertrophy and leads to sudden cardiac death in young
athletes?

1685.

Hypertrophic pyloric stenosis


due to hypertrophy of the
muscularis externa, resulting
in a narrowed pyloric outlet

Projectile nonbilious vomiting and a small knot at the right costal margin
(olive sign) are hallmarks of what embryonic disorder?

1686.

Hypertrophic scar

What is the term for excessive production of collagen that flattens out
and does not extend beyond the site of the injury?

1687.

Hypertrophy

What is the term to describe the increase in organ size due to the
increase in cell size and function?

1688.

Hypervariable region

What region of the Ig does not change with class switching?

1689.

Hypervariable region (three


per light chain; three per
heavy chain)

Which region of the variable domain comprises the Ag-binding site of


the Ab?

1690.

Hyperventilation, which decreases CO2, shifting


the reaction to the left and decreasing H+

What is the respiratory compensation mechanism


for metabolic acidosis?

1691.

Hypnopompic hallucinations occur during


awakening, whereas hypnagogic hallucinations
occur while one is falling asleep.

What type of hallucination occurs during


awakening?

1692.

Hypnozoite

What form of Plasmodium species affects the liver?

1693.

Hypoactive sexual desire disorder

What is the term for a deficiency or absence of


sexual fantasies or desires?

1694.

Hypocalcemia and hypophosphatemia

1--Hydroxylase activity is increased in response


to what two physiologic states? (hint: think of
vitamin D activity)

1695.

Hypochondriasis (they will begin with "I think I


have...")

What somatoform disorder is described as


Preoccupied with illness or death, persisting
despite reassurance, lasting longer than 6
months?

1696.

Hypoglycemia for secretion and hyperglycemia for


inhibition

What is the most potent stimulus for glucagon


secretion? Inhibition?

1697.

Hypokalemic metabolic acidosis occurs in colonic


diarrhea because of the net secretion of HCO3and potassium into the colonic lumen.

What acid-base disturbance occurs in colonic


diarrhea

1698.

Hypokalemic metabolic alkalosis occurs from


vomiting because of the loss of H+, K+, and Cl-.

What acid-base disturbance is produced from


vomiting?

1699.

Hypoparathyroidism

What do low levels of Ca2+ and PO4- along with


neuromuscular irritability indicate?

1700.

Hypospadia

What is the term for the external urethra opening


onto the ventral surface of the penis?

1701.

Hypothyroidism. (TSH levels must be monitored.)

What is associated with prolonged lithium use?

1702.

Hypotonia (rag doll


appearance)

What is the term to describe the soft, flabby feel and diminished
reflexes seen in patients with acute cerebellar injury to the deep
cerebellar nuclei?

1703.

Hypotonic, because NaCl is


reabsorbed in the salivary
ducts

Are salivary secretions hypertonic, hypotonic, or isotonic?

1704.

Hypoventilation results in an
increase in PCO2 levels and
therefore an increase in blood
flow.

What is the effect of hypoventilation on cerebral blood flow?

1705.

Hypoventilation, which
increases CO2, shifting the
reaction to the right and
increasing H+

What is the respiratory compensation mechanism for metabolic


alkalosis?

1706.

Hypoxanthine (remember, IMP


is a precursor for AMP and
GMP)

What purine base is contained in inosine monophosphate?

1707.

Hypoxia

Name the most common type or cause. Cellular injury

1708.

Iatrogenesis

Name the most common type or cause. Hypothyroidism in the United


States

1709.

I-cell disease is a result of a


genetic defect affecting the
phosphorylation of mannose
residues.

What genetic defect is characterized by coarse facial features, gingival


hyperplasia, macroglossia, psychomotor and growth retardation, club
foot, claw hand, cardiorespiratory failure, and death in the first decade
of life?

1710.

ICF volume decreases when


there is an increase in
osmolarity and vice versa.

What happens to intracellular volume when there is an increase in


osmolarity?

1711.

Id

Per Freud, with what part of the unconscious are sex and aggression
(instincts) associated?

1712.

Idiotypes

What are defined by Ag-binding specificity?

1713.

IDL

Name the lipoprotein based on the following characteristics. apo E

1714.

Id (pleasure principle)

What Freudian psyche component is


described as The urges, sex
aggression, and "primitive"
processes?

1715.

IF-2 of the 30S subunit

What prokaryotic positioning


enzyme in translation is blocked by
the following? Streptomycin

1716.

If a carboxyl group is the R group, it is acidic; if an amino group is


the R group, it is said to be basic.

What is found in the R group if the


AA is acidic? Basic?

1717.

IF and GTP (eIF for eukaryotes)

What is needed to initiate


translation?

1718.

If positive, hypotonic urine (osmolarity <300 mOsm/L); if


negative, hypertonic urine (osmolarity > 300 mOsm/L)>>

If free water clearance (CH2O) is


positive, what type of urine is
formed? And if it is negative?

1719.

If the polyuria begins before the collecting ducts, the patient is


dehydrated and electrolyte depleted. If the polyuria originates
from the collecting ducts, the patient is dehydrated with normal
electrolytes.

What is the origin of the polyuria if a


patient is dehydrated and electrolyte
depleted?

1720.

If the ratio of the filtrate to plasma concentration of a substance is


equal, the substance is freely filtered by the kidney.

If the ratio of a substance's filtrate


and plasma concentrations are
equal, what is that substance's affect
on the kidney?

1721.

IgA

What Ig is associated with mucosal


surfaces and external secretions?

1722.

IgA

What is the major Ig of the


secondary immune response in the
mucosal barriers?

1723.

IgA

Which immunoglobulin is secreted by the plasma cells in the


gastrointestinal tract?

1724.

IgA

What Ig prevents bacterial adherence to mucosal surfaces?

1725.

IgA

What Ig activates the alternate pathway, neutralizes bacterial


endotoxins and viruses, and prevents bacterial adherence?

1726.

IgA

What Ig is the major protective factor in colostrum?

1727.

IgA deficiency; patients commonly


present with recurrent
sinopulmonary infections and GI
disturbances.

What is the most common Ig deficiency?

1728.

IgA, IgD, IgE, IgG, and IgM

What are the five Ig isotypes?

1729.

IgA nephropathy

Name the most common type or cause. GN in the world

1730.

IgA nephropathy

What form of nephritic syndrome is associated with celiac sprue


and Henoch- Schnlein purpura?

1731.

IgA nephropathy (Berger disease)

Name the nephritic disease based on the immunofluorescent


staining. Mesangial deposits of IgA and C3

1732.

IgD; IgM is also correct.

What Ig is associated with Ag recognition receptor on the surface


of mature B cells?

1733.

IgE

What Ig is responsible for Antibody-Dependent Cell-mediated


Cytotoxicity of parasites, has a high-affinity Fc receptor on mast
cells and basophils, and is responsible for the allergic response?

1734.

IgE

What Ig is associated with ADCC for parasites?

1735.

IgE. It attaches via receptor for the


Fc region of the heavy epsilon chain

What Ig is associated with mast cell and basophil binding?

1736.

IGF-1

What is the major stimulus for cell division in chondroblasts?

1737.

IGF-1 (somatomedin C)

What somatomedin serves as a 24-hour marker of GH secretion?

1738.

IgG

What Ig mediates ADCC via K cells, opsonizes, and is the Ig of


the secondary immune response?

1739.

IgG

What is the major Ig of the secondary immune response?

1740.

IgG

What is the only Ig that crosses the placenta?

1741.

IgG

What Ig is responsible for activation of complement,


opsonization, and ADCC and is actively transported across the
placenta?

1742.

IgG

What is the most common circulation Ig in the plasma?

1743.

IgG3

What subtype of IgG does not bind to staphylococcal A protein?

1744.

IgG4

What subtype of IgG does not activate complement cascade?

1745.

IgG4

Which IgG cannot activate complement?

1746.

IgM

What is the Ig associated with the primary immune response?

1747.

IgM

What is the first Ab a baby makes?

1748.

IgM

What Ig activates the complement cascade most efficiently?

1749.

IgM

What is the major Ab of the primary immune response?

1750.

IgM; IgD follows shortly thereafter.

What is the first membrane-bound Ig on B cell membranes?

1751.

IL-2. T cells express IL-2 receptors on


their surface to induce self-expression.

What IL do T cells secrete to induce T-and B-cell division?

1752.

IL-3 (3 face down is an M)

What IL is important in myeloid cell development?

1753.

IL-4

Which IL is associated with increases of IgG and IgE?

1754.

IL-4, IL-10, and IL-13

What cytokines do Th2 cells secrete to inhibit Th1 cell


function?

1755.

IL-4 is secreted to activate B cells. This begins the


second step in the immune response, known as
Activation. CD4 T cells secrete INF-alpha to activate
macrophages

What cytokine do CD4 T cells secrete to


activate B cells when the specific peptide in
the groove of the MHC II molecule interacts
with the TCR?

1756.

IL-5. It also stimulates eosinophil proliferation.

Which IL increases IgA synthesis?

1757.

IL-7 (A 7 upside down is an L; L is for Lymphoid)

What IL is essential for lymphoid cell


development?

1758.

IL-8 (IL-1 and TNF-gamma also)

What is the name of the major chemotactic


agent released from Macrophages?

1759.

IL-8. It not only is chemotactic, it also acts as an


adhesive for neutrophils.

What IL, produced by macrophages, is


chemotactic for neutrophils?

1760.

IL-10

What IL down-regulates cell mediated


immunity?

1761.

Ileum

What area of the small intestine is


characterized by Peyer's patches?

1762.

Imipenem and meropenem

What bactericidal agents are resistant to lactamases and are used to treat in-hospital
life-threatening infections?

1763.

Immature B cells

During what stage of B-cell development is


IgM first seen on the surface?

1764.

Immunogenicity; antigenicity refers to Ab/lymphocyte


reaction to a specific substance.

What is the term for the inherent ability to


induce a specific immune response?

1765.

Impaired ejaculation

What are the pharmacologic effects seen


sexually with 1-Blockers?

1766.

Impotence

What are the pharmacologic effects seen


sexually with -Blockers?

1767.

Inactivated

Do local anesthetics bind to the activated or


inactivated sodium channels?

1768.

Inactivated; glucose results in decreased cAMP levels and


therefore blocks protein binding between cAMP and CAP.

Is the Lac operon activated or


inactivated in the presence of both
glucose and lactose?

1769.

In bone; nearly 99% of Ca2+ is stored in the bone as


hydroxyapatite.

Where is most of the body's Ca2+


stored?

1770.

In children, the fourth ventricle; in adults, the lateral ventricle


or spinal cord

What is the most common one?


Location of ependymomas in children

1771.

Incidence rate

What is the term for the number of new


events occurring in a population divided
by the population at risk?

1772.

Increase (5-HT and dopamine levels do the same)

What happens to NE levels in Bipolar


disorder?

1773.

Increased ACh to decreased NE levels. (NE pathway begins in


the pons and regulates REM sleep.)

What is the biochemical trigger for REM


sleep?

1774.

Increased androgen secretion at puberty drives the increased


GH secretion.

During puberty, what is the main drive


for the increased GH secretion?

1775.

Increased CO2, H+, temperature, and 2, 3-BPG levels all shift


the curve to the right, thereby making the O2 easier to remove
(decreased affinity) from the Hgb molecule.

What four factors shift the Hgb-O2


dissociation curve to the right? What is
the consequence of this shift?

1776.

Increased erection and libido

What are the pharmacologic effects


seen sexually with Dopamine
agonists?

1777.

Increased GABA levels decrease the likelihood of learned


helplessness.

Increased levels of what


neurotransmitter, in the hippocampus,
decrease the likelihood of learned
helplessness?

1778.

The increased H+ moves intracellularly and is buffered


by K+ leaving the cells, resulting in intracellular depletion
and serum excess. (Intracellular hypokalemia is the
reason you supplement potassium in diabetic
ketoacidosis, even though the serum levels are
elevated.)

Why is there a transcellular shift in K+ levels


in a diabetic patient who becomes acidotic?

1779.

Increased pulmonary arterial pressure (high perfusion)


and more distensible vessels (low resistance) result in
increased blood flow at the base.

What two factors result in the base of the


lung being hyperperfused?

1780.

Increased REM sleep, decreased REM latency, and


decreased stage 4 sleep, leading to early morning
awakening

What happens to REM, REM latency, and


stage 4 sleep during major depression?

1781.

Increased secretion of GH postpuberty leading to


acromegaly.

What hormone excess brings about


abnormal glucose tolerance testing, impaired
cardiac function, decreased body fat,
increased body protein, prognathism, coarse
facial features, and enlargements of the
hands and feet?

1782.

Increase enzyme concentrations

What is the only way to increase maximum


velocity (Vmax)?

1783.

Increase in ECF osmolarity means a in decrease in ICF


osmolarity, so cells shrink.

When the ECF osmolarity increases, what


happens to cell size?

1784.

Increase in erythropoietin and increase in 2, 3-BPG, also


called 2, 3-diphosphoglycerate (2, 3-P2Gri) (increase in
glycolysis)

What two compensatory mechanisms occur


to reverse hypoxia at high altitudes?

1785.

An increase in serum glucose levels

What is the most important stimulus for the


secretion of insulin?

1786.

An increase in the afterload decreases velocity; they are


inversely related. (V equals 1 divided by afterload.)

Regarding skeletal muscle mechanics, what


is the relationship between velocity and
afterload?

1787.

Increase in urinary excretion of HCO3-, shifting the


reaction to the right and increasing H+

What is the renal compensation mechanism


for alkalosis?

1788.

Increases

With an increase in arterial systolic


pressure, what happens to Stroke
volume?

1789.

Increases Remember, the carotid sinus reflex attempts to


compensate by increasing both TPR and heart rate. -------------------------------------------------------------------------------

When a person goes from supine to


standing, what happens to the following?
Dependent venous blood volume

1790.

Increases Remember, the carotid sinus reflex attempts to


compensate by increasing both TPR and heart rate. -------------------------------------------------------------------------------

When a person goes from supine to


standing, what happens to the following?
Dependent venous pressure

1791.

Increase the concentration of enzymes

What is the only way to increase the Vmax


of a reaction?

1792.

Increasing blood flow is the only way to increase O2


delivery in the coronary circulation because extraction is
nearly maximal during resting conditions.

What is the only way to increase O2


delivery in the coronary circulation?

1793.

Indirect hernia passes in the inguinal canal; a direct hernia


passes directly through Hesselbach's triangle.

What type of hernia is described as


passing through the deep lateral ring of the
inguinal canal?

1794.

Indirect or unconjugated

What type of bilirubin is found in neonatal


jaundice?

1795.

Indirect pathway (Tourette syndrome for example)

In what pathway of the basal ganglia do


lesions result in hyperactive cortex with
hyperkinetic, chorea, athetosis, tics, and
dystonia?

1796.

In early-morning sleep, usually between the sixth and


eighth hours

When does cortisol secretion peak?

1797.

In eukaryotes transcription occurs in the nucleus and


translation in the cytoplasm.

Why are eukaryotes unable to perform


transcription and translation at the same
time like prokaryotes?

1798.

Infant Doe. Generally, parents cannot forego lifesaving


treatment, but this case states that there are exceptions to
the rule.

What case is known as "let nature take its


course"?

1799.

Infection

Name the most common type or


cause. Hematuria

1800.

Inferior Oblique (CN III) (LR6 SO4)3

What ocular muscle Elevates and


abducts the eyeball?

1801.

Inferior pancreaticoduodenal, middle colic, right colic, ileocolic,


and 10 to 15 intestinal arteries

What are the five branches of the


superior mesenteric artery?

1802.

Inferior phrenic artery

What is the first branch of the


abdominal artery?

1803.

Inferior rectus (CN III) (LR6 SO4)3

What ocular muscle Depresses and


adducts the eyeball?

1804.

INF-gamma

What cytokine do Th1 cells secrete to


inhibit Th2 cell function?

1805.

Infiltrating ductal carcinoma

What is the most common one?


Histologic variant of breast cancer

1806.

Infliximab

What is the name of the monoclonal


antibody used in the treatment of
Crohn disease and rheumatoid
arthritis?

1807.

Infrapatellar bursa

What bursa is inflamed in clergyman's


knee?

1808.

Inhaled anesthetics with low lipid solubility have low potency


(directly related), so they have rapid induction and rapid
recovery; vice versa for drugs with high lipid solubility.

How are lipid solubility and potency


related for inhaled anesthetics? How
do they affect onset and recovery?

1809.

Inhibin, mllerian-inhibiting factor, and androgen binding


protein

What three factors do Sertoli cells


produce for normal male
development?

1810.

Inhibited orgasm

What are the pharmacologic effects


seen sexually with Serotonin?

1811.

Initiate and manage gross skeletal muscle movement control

What is the function of the basal


ganglia?

1812.

In men it is prostate cancer, and


in women it is breast cancer.

What is the most common cancer diagnosed in men? In women?

1813.

The inner cortex (paracortex)


contains the T cells, so it is
considered the thymic-dependent
area.

What area of the lymph node is considered the thymic-dependent


area?

1814.

The inner membrane of the


mitochondria (cristae)

Where are the enzymes for the ETC and oxidative phosphorylation
found?

1815.

Inspiratory capacity

Name the lung measurement based on the following descriptions:


The maximal volume inspired from FRC

1816.

Inspiratory reserve volume (IRV)

Name the lung measurement based on the following descriptions:


Additional air that can be taken in after normal inspiration

1817.

Insulin

What hormone is secreted into the plasma in response to a meal


rich in protein or CHO?

1818.

Insulin

Name the hormoneglucagon, insulin, or epinephrine:


Glycogenic, gluconeogenic, lipogenic, proteogenic, glycolytic, and
stimulated by hyperglycemia, AAs, fatty acids, ketosis, ACh, GH,
and Beta-agonist

1819.

Insulin increases intracellular K+


stores while decreasing serum K+
levels.

What is the effect of insulin on intracellular K+ stores?

1820.

Insulin increases total body stores


of protein, fat, and CHOs. When
you think insulin, you think
storage.

What is the effect of insulin on protein storage?

1821.

Insulinoma

What is the most common one? Pancreatic islet cell tumor

1822.

Insulinoma

What pancreatic islet cell tumor is associated with hypoglycemia,


sweating, hunger, confusion, and increased C-peptide levels?

1823.

Integrase

What enzyme does HIV use to integrate the proviral dsDNA into the
host?

1824.

Integument (skin and its derivatives)

What is the largest organ in the body?

1825.

Intellectualization

Replacing normal affect with "brain power"

1826.

Intention tremor; it is a sign of


cerebellar lesions. A tremor at rest (i.e.,
pill rolling) is seen in basal ganglia
lesions.

What is the name of the tremor that occurs during movements


and is absent while the person is at rest?

1827.

Intercellular adhesion molecule (ICAM)


1

Which integrin mediates adhesion by binding to lymphocyte


function- associated Ag 1 (LFA-1) and MAC-1 leukocyte
receptors?

1828.

Intermediate part of the hemispheres

In a topographical arrangement of the cerebellar homunculus


map, what area or lobe Controls distal musculature?

1829.

Internal 3', 5' PDE bonds

What bonds are broken by endonucleases?

1830.

Internal abdominal oblique

What abdominal muscle runs in a posteroinferior direction,


splits to contribute to the rectus sheath, contributes to the
formation of the conjoint tendon, and in men gives rise to the
middle spermatic fascia and the cremasteric muscle of the
spermatic cord?

1831.

Internal carotid artery

Ophthalmic artery is a branch of what artery?

1832.

Internal carotid artery

The ophthalmic artery is a branch of what vessel?

1833.

Internal carotid artery and sympathetic


plexus

Name the structure that enters or exits the following foramina:


Carotid canal

1834.

An internal elastic lamina

What does the tunica intima of arteries have that veins do not?

1835.

Internal iliac nodes

What is the lymphatic drainage of the pelvic organs?

1836.

The internal laryngeal nerve supplies


sensory information from above the
vocal cords while the recurrent
laryngeal nerve supplies sensory
information below.

What nerves provide sensory innervation above the vocal


cords? Below the vocal cords?

1837.

The interposed nucleus

What deep cerebellar nuclei receive Purkinje


cell projections in The lateral cerebellar
hemispheres?

1838.

Interval scale (a ruler, for example)

What type of scale is graded into equal


increments, showing not only any difference but
how much?

1839.

In the cytoplasm of the presynaptic nerve terminal; it is


catalyzed by choline acetyltransferase.

Where does the synthesis of ACh occur?

1840.

In the dorsal horn of the spinal cord on the primary


afferent neurons

Where in the spinal cord are the presynaptic


opioid receptors?

1841.

In the first 3 months

When do hCG concentrations peak in


pregnancy?

1842.

In the first and second bronchi

Where is most of the airway resistance in the


respiratory system?

1843.

In the JG cells of the kidney

Where is renin produced?

1844.

In the medulla; all the other structures are cortical.

Where in the kidney are the long loops of Henle


and the terminal regions of the collecting
ducts?

1845.

In the mouth with salivary -amylase (ptyalin)

Where does CHO digestion begin?

1846.

In the peroxisome until it is 10 carbons long; the rest is


completed in the mitochondria.

Where does Beta-oxidation of very long chain


fatty acids begin?

1847.

In the PNS, myelin is produced by Schwann cells, in


the CNS by oligodendrocytes.

Myelin is produced by which cells in the PNS?


In the CNS?

1848.

In the RBC; remember, you need carbonic anhydrase


for the conversion, and plasma does not have this
enzyme.

Where does the conversion of CO2 into HCO3take place?

1849.

In the supraoptic nuclei of the hypothalamus; it is


stored in the posterior pituitary gland.

Where is ADH synthesized?

1850.

In the tuberoinfundibular pathway, dopamine


release will decrease prolactin release, which is
why dopamine agonists are used in the
treaatment of hyperprolactinemic states.

What hormone can be affected by dopamine


release in the tuberoinfundibular pathway?

1851.

In the ventral horn of the spinal cord. UMN cell


bodies are in the precentral gyrus of the frontal
lobe.

Where are the LMN cell bodies of the corticospinal


tract?

1852.

Intraductal papilloma

What benign solitary papillary growth within the


lactiferous ducts of the breast commonly produces
bloody nipple discharge?

1853.

Intrapleural pressure at the base is -2.5 cm H2O


(more positive than the mean), resulting in a force
to collapse the alveoli.

At the base of the lung, what is the baseline


intrapleural pressure, and what force does it exert
on the alveoli?

1854.

Intrapleural pressure decreases (becomes more


negative).

What happens to intrapleural pressure when the


diaphragm is ontracted during inspiration?

1855.

Intrinsic factor (IF)

What substance is secreted by parietal glands and


is required for life?

1856.

Intrinsic. The extrinsic pathway is activated by the


release of tissue factors.

What pathway of the coagulation cascade is


activated when it is in contact with foreign
surfaces?

1857.

Introjection (a sports fan is a good example)

Name these immature defense mechanisms:


Taking others' beliefs, thoughts, and external stimuli
and making them part of the self. (Hint: if it's done
consciously, it is called imitation.)

1858.

Intussusception

What is the term for telescoping of the proximal


bowel into the distal segment presenting as
abdominal pain, currant jelly stools, and intestinal
obstruction?

1859.

Inulin

The clearance of what substance is the gold


standard of GFR?

1860.

Invariant chain

Which protein prevents internal binding of self proteins


within an MHC class II cell?

1861.

Invariant chain. This is essential because the


CD4 T cells have antigen receptors only for
peptides bound to the MHC II molecule.
(MHC restriction)

In MHC class II molecules, what chain blocks access to


the peptide-binding groove during transportation within
the cell, ensuring that the MHC class II-peptide complex
is transported to the surface?

1862.

Invasive ductal carcinoma

What is the most common one? Malignant tumor of the


breast

1863.

Inversely proportional: the higher the


potency, the lower the anticholinergic side
effects.

Regarding neuroleptics, what is the relationship between


potency and anticholinergic side effects?

1864.

Iodine (I)

What mineral is associated with hypothyroidism?

1865.

Iodine, resulting in congenital hypothyroidism

What intrauterine deficiency leads to failure to thrive,


mental retardation, motor incoordination, and stunted
growth?

1866.

Ionized drugs are water soluble, and since


only lipid-soluble drugs can cross
biomembranes, an ionized drug cannot cross
them without the help of a carrier.

If a drug is ionized, is it water or lipid soluble? Can it


cross a biomembrane?

1867.

Ions diffuse in the direction to bring the


membrane potential toward the equilibrium
potential.

In reference to membrane potential (Em) and equilibrium


potential (Ex), which way do ions diffuse?

1868.

Ipratropium

What is the DOC for -blocker-induced


bronchospasms?

1869.

Ipratropium

What antimuscarinic is used as an inhalant for asthma?

1870.

Ipsilateral flaccid paralysis

Describe the loss for each of the following in a


hemisection of the spinal cord. (Brown-Sequard
syndrome) LMN?

1871.

Ipsilateral loss at and below the level of the


lesion

Describe the loss for each of the following in a


hemisection of the spinal cord. (Brown-Sequard
syndrome) Dorsal column tract?

1872.

Ipsilateral loss below the level of the


lesion

Describe the loss for each of the following in a hemisection of


the spinal cord. (Brown-Sequard syndrome) Corticospinal
tract?

1873.

IQ

What is defined as a general estimate of the functional


capacities of a human?

1874.

Iron deficiency anemia

Name the hypochromic microcytic anemia based on the


following laboratory values. Decreased iron, percent saturation,
and ferritin; increased TIBC

1875.

Iron (Fe)

What mineral, via excessive depositions in the liver, causes


hemochromatosis?

1876.

Ischemia

Name the most common type or cause. Hypoxia

1877.

Ischemic

Name the most common type or cause. Acute tubular necrosis

1878.

Ischemic heart disease (MIs)

Name the most common type or cause. Death in the United


States

1879.

Isocitrate dehydrogenase

What enzyme catalyzes the rate-limiting step in the TCA cycle?

1880.

Isocitrate dehydrogenase, ketoglutarate dehydrogenase, and


malate dehydrogenase

What enzyme of the TCA cycle catalyzes the production of the


following: NADH (hint: 3 enzymes)

1881.

Isograft

Name the type of graft described by these transplants: Between


genetically identical individuals

1882.

Isolation of affect

Fact without feeling (la belle indiffrence)

1883.

Isoleucine, leucine, and valine

What three AAs must patients with maple syrup urine disease
not eat?

1884.

Isoleucine, threonine, tryptophan,


tyrosine, and phenylalanine

What are the five AAs that are both ketogenic and glucogenic?

1885.

Isoniazid

What antitubercular agent requires vitamin B6 supplementation?

1886.

Isoniazid treatment

What is the most common cause of vitamin B6 deficiency?

1887.

Isotonic contraction

What type of muscle contraction occurs when the muscle


shortens and lifts the load placed on it?

1888.

Isotypes

What is the term for different classes and subclasses of the


same gene products?

1889.

Isovolumetric contraction

What is the most energydemanding phase of the


cardiac cycle?

1890.

It both begins and ends at T4 (sternal angle [of Louis]).

At what vertebral levels does


the aortic arch begin and end?

1891.

It causes a buildup of dATP, which inhibits ribonucleotide reductase


and leads to a decrease in deoxynucleoside triphosphate, a precursor
of DNA, resulting in overall bone marrow suppression.

How can a deficiency in


adenosine deaminase be a
bone marrow suppressor?

1892.

It decreases insulin secretion.

How does the sympathetic


nervous system affect insulin
secretion?

1893.

It forms a germinal tube at 37C.

What does Candida albicans


do that distinguishes it from
other fungi?

1894.

It increases Ca2+ excretion from the kidney and increases bone


mineralization.

What are the two actions of


calcitonin?

1895.

It increases REM and total sleep time.

How does L-tryptophan affect


sleep?

1896.

It is best to begin with open-ended questions, allowing patients to


describe in their own words what troubles them. You can then move to
closed-ended questions when narrowing the diagnosis.

What type of questions should


you begin with when a patient
seeks your medical opinion?

1897.

It is produced by the preload on the muscle prior to contraction.

Regarding muscle mechanics,


how is passive tension
produced?

1898.

It is rare with normal grief; however, it is relatively common in


depression

Is suicidal ideation a
component of normal grief?

1899.

It normally does not play a role in filtration but


becomes important when there is an obstruction
downstream.

When does the hydrostatic pressure in


Bowman's capsule play a role in opposing
filtration?

1900.

Itraconazole

What is the DOC for blastomycoses and


sporotrichoses?

1901.

It repeats every 90 to 120 minutes and correlates with


elevated levels of motilin.

A migrating myoelectric complex is a propulsive


movement of undigested material of undigested
material from the stomach to the small intestine
to the colon. During a fast, what is the time
interval of its repeats?

1902.

IVC Remember: 1 at T8, 2 at T10, and 3 at T12

What structure or structures cross the


diaphragm at T8 level?

1903.

Ixodes (also the vector for Lyme disease)

What tick is the vector for babesiosis?

1904.

Janeway lesions

Name the following descriptions associated with


bacterial endocarditis: Painless hyperemic
lesions on the palms and soles

1905.

JC virus

What virus is associated with progressive


multifocal leukoencephalopathy?

1906.

Jejunum (upper)

What area of the GI tract has the highest


activity of brush border enzymes?

1907.

The JG cells

What modified smooth muscle cells of the


kidney monitor BP in the afferent arteriole?

1908.

Job syndrome

What disease involves cold skin abscesses due


to a defect in neutrophil chemotaxis and a
serum IgE level higher than 2000?

1909.

The junction where the anterior communicating and


anterior cerebral arteries join. As the aneurysm
expands, it compresses the fibers from the upper
temporal fields of the optic chiasm, producing
bitemporal inferior quadrantanopia

What is the most common site for an aneurysm


in cerebral circulation?

1910.

Juxtaglomerular (JG) cell

What cell of the nephron is responsible for renin production


and secretion?

1911.

K1 capsule

What capsular serotype is associated with Escherichia


coli- induced meningitis?

1912.

Kartagener's syndrome (also known as


immotile cilia syndrome) ------------------------------------------------------------------------------

What syndrome is characterized by dynein arm


abnormality resulting in chronic sinusitis, recurrent
pulmonary infections, and infertility?

1913.

Kawasaki disease

What coronary artery vasculitis is seen in Japanese infants


and children less than 4 years old with acute febrile illness,
conjunctivitis, maculopapular rash, and lymphadenopathy?

1914.

Kayser-Fleischer ring

What is the name of a thin brown ring around the outer


edge of the cornea, seen in Wilson's disease?

1915.

Kayser-Fleischer rings

What is the term for the copper corneal deposits found in


Wilson's disease?

1916.

Keloid

What is the term for an abnormal amount of collagen type


III that produces a large bulging scar, seen primarily in
blacks?

1917.

Keratinocytes, the most numerous cells in


the epidermis, carry melanin and produce
keratin.

What cells of the epidermis carry the pigment melanin?

1918.

Ketamine

What is the only anesthetic that causes cardiovascular


stimulation?

1919.

Ketoconazole

What antifungal agent is used in the treatment of androgen


receptor-positive cancers?

1920.

Kidney

What is the most common one? Organ involved in


amyloidosis

1921.

Kidney

What is the most common organ involved in amyloidosis?

1922.

The kidneys excrete the excess nitrogen


from the body as urea in the urine.

What organ is responsible for the elimination of excess


nitrogen from the body?

1923.

Kimmelstiel-Wilson disease

What renal disease in diabetic patients is seen as a halo of


capillaries around the mesangial nodules?

1924.

Kinesins. Dynein
generates retrograde
transportation of
information.

What generate anterograde transport of information in a neuron?

1925.

Klatskin tumor

What is the tumor at the bifurcation of the right and left hepatic ducts?

1926.

Klebsiella pneumoniae

Name the MCC. Aspiration pneumonia in an alcoholic

1927.

Klebsiella pneumoniae

What encapsulated gram-negative, lactose-fermenting rod is associated


with pneumonia in patients with alcoholism, diabetes, and chronic lung
disease?

1928.

Klinefelter syndrome

What syndrome has elevated FSH and LH levels with decreased


testosterone levels and 47XXY karyotype?

1929.

Klinefelter syndrome

Name the most common one. Chromosomal disorder involving sex


chromosomes

1930.

Klver-Bucy syndrome

What syndrome is described as bilateral lesions of the amygdala and the


hippocampus resulting in placidity, anterograde amnesia, oral exploratory
behavior, hypersexuality, and psychic blindness?

1931.

Klver-Bucy syndrome

What syndrome is characterized by bilateral medial temporal lobe lesion,


placidity, hyperorality, hypersexuality, hyperreactivity to visual stimuli, and
visual agnosia?

1932.

Knee

What is the most common one? Joint affected in pseudogout

1933.

Krukenberg tumor

What is the name of the tumor when gastric carcinoma spreads to the
ovaries?

1934.

Kupffer cells

Name the macrophage based on its location: Liver macrophages

1935.

Kupffer cells

Name the macrophages by location: Liver

1936.

L2 to L4 (L2 to L4, thigh;


L4 to S3, leg)

What segments of the lumbosacral plexus form the following nerves?


Femoral nerve

1937.

L2 to L4 (L2 to L4, thigh;


L4 to S3, leg)

What segments of the lumbosacral plexus form the following nerves?


Obturator nerve

1938.

L4 to S3 (L2 to L4, thigh;


L4 to S3, leg)

What segments of the lumbosacral plexus form the following nerves?


Common peroneal nerve

1939.

L4 to S3 (L2 to L4, thigh; L4 to S3, leg)

What segments of the lumbosacral plexus form


the following nerves? Tibial nerve

1940.

Labetalol

What -blocker is also an -blocker?

1941.

Labile

Name the type of regeneration (i.e., labile, stable,


or permanent) based on the following examples.
Epidermis

1942.

Labile

Name the type of regeneration (i.e., labile, stable,


or permanent) based on the following examples.
Mucosal epithelium

1943.

Labile

Name the type of regeneration (i.e., labile, stable,


or permanent) based on the following examples.
Hematopoietic cells

1944.

Labioscrotal swelling

What embryonic structure forms the adult female


structures? Labia majora

1945.

Labioscrotal swelling

What embryonic structure forms the adult male


structure? Scrotum

1946.

The lack of glutathione peroxidase activity results in


a decrease in NADPH production, leaving
glutathione in the reduced state.

What causes the lysis of RBCs by oxidizing


agents in a G-6-PD deficiency?

1947.

Lac repressor protein

Regarding the Lac operon, for what do the


following genes code? I gene

1948.

Lactose must be present and glucose must be


absent

What two requirements must be met for the Lac


operon to be activated?

1949.

Lactulose

Which laxative is used in the treatment of hepatic


encephalopathy ?

1950.

"Lady between two Majors": latissimus dorsi,


pectoralis major, and teres major

What muscles insert in or on the intertubercular


groove of the humerus?

1951.

Laminin

What component of the basement membrane binds to


collagen type IV and heparin sulfate and is a cell surface
receptor?

1952.

Langerhans cells

Name the macrophage based on its location: Epidermal


macrophages

1953.

Langerhans cells (found in the stratum


spinosum)

What cell type of the epidermis functions as antigenpresenting cells?

1954.

Large intestine. Water is passively removed


from the lumen.

What segment of the gastrointestinal tract lacks villi, has


crypts, and actively transports sodium out of its lumen?

1955.

The larger the volume of distribution, the


lower the plasma concentration of a drug.

What happens to plasma concentration of a drug if there


is a large volume of distribution?

1956.

Large segment deletions

Name the type of mutation: Unequal crossover in


meiosis with loss of protein function

1957.

LARP: Left goes Anterior and Right goes


Posterior (because of the rotation of the gut;
remember your embryology!)

What is the anatomic positioning of the right and left


gastric nerve plexus of the esophagus as they pass
through the diaphragm?

1958.

Latency stage (6-12 years)

In what stage of psychosexual development, according to


Freud, do children resolve the Oedipus complex?

1959.

Lateral collateral ligament

What structure of the knee is described thus? Prevents


adduction

1960.

Lateral compartment of the leg, superficial


peroneal nerve

Name the compartment of the lower extremity and the


nerve based on its movements. Plantar flex the foot and
evert the foot

1961.

Lateral cricoarytenoid muscles

Name the laryngeal muscle described by the following:


Adducts the vocal ligaments, closes the air passageway
during swallowing, and allows phonation

1962.

Lateral fissure (fissure of


Sylvius)

What fissure of the cerebral cortex separates the frontal and temporal
lobes rostrally and partially separates the parietal and temporal lobes?

1963.

Lateral hypothalamic zone;


lesions here result in aphagia.
(Notice the difference between
the feeding center and the
satiety center; they are in
different zones.)

What area of the hypothalamus is the feeding center?

1964.

Lateral medullary
(Wallenberg's) syndrome

What syndrome is associated with the following brainstem lesions?


Occlusion of the PICA, resulting in ipsilateral limb ataxia, ipsilateral
facial pain and temperature loss, contralateral pain and body
temperature loss, ipsilateral Horner's syndrome, and ipsilateral
paralysis of the vocal cords, palate droop, dysphagia, nystagmus,
vomiting, and vertigo

1965.

Lateral part of the hemispheres

In a topographical arrangement of the cerebellar homunculus map,


what area or lobe Is involved in motor planning?

1966.

Lateral pontine syndrome

What syndrome is associated with the following brainstem lesions?


AICA or superior cerebellar artery occlusion, resulting in ipsilateral
limb ataxia, ipsilateral facial pain and temperature loss, contralateral
loss of pain and temperature to the body, ipsilateral Horner's
syndrome, ipsilateral facial paralysis, and hearing loss

1967.

Lateral rectus (CN VI) (LR6


SO4)3

What ocular muscle Abducts the eyeball and is involved in horizontal


conjugate gaze?

1968.

The lateral vestibular nucleus

What deep cerebellar nuclei receive Purkinje cell projections in The


flocculonodular lobe?

1969.

Latissimus dorsi

What muscle or muscles are innervated by the following nerves?


Thoracodorsal nerve

1970.

LC4, LD4, and LE 4

What three LTs are associated with bronchospasms and an increase


in vessel permeability and vasoconstriction?

1971.

LDL

Name the lipoprotein based on the following characteristics.


apo B-100

1972.

LDL

What lipoprotein is formed if an IDL particle acquires


cholesterol from a HDL particle?

1973.

Lead

What type of metal poisoning causes mental retardation,


somnolence, convulsions, and encephalopathy?

1974.

Lead

Name the most common type or cause. Chronic metal


poisoning

1975.

Lead poisoning

What metal poisoning produces microcytic anemia with


basophilic stippling?

1976.

Lead-time bias (remember, patients don't


live longer with the disease; they are
diagnosed sooner.)

What form of bias is due to false estimates of survival


rates?

1977.

Left abducens nerve

Where is the lesion that produces these symptoms when a


patient is asked to look to the left? Left eye can't look to
the left

1978.

Left abducens nucleus or right cerebral


cortex

Where is the lesion that produces these symptoms when a


patient is asked to look to the left? Neither eye can look left
with a slow drift to the right

1979.

Left anterior descending artery

What artery travels with the following veins? Great cardiac


vein

1980.

Left atrial myxoma

What is the most common one? Cardiac tumor

1981.

Left atrium

What part of the heart forms Posterior wall?

1982.

Left atrium and tip of the right atrium

What part of the heart forms Base?

1983.

Left CN XII lesion would result in the


tongue pointing to the left (points at the
affected side).

What direction would the tongue protrude in a left CN XII


lesion?

1984.

A left CN X lesion results in the uvula


deviating to the right. (Uvula points away
from the affected side.)

Which direction does the uvula deviate in a left vagus nerve


lesion?

1985.

Left colic, superior rectal, and sigmoidal


arteries

What are the three branches of the inferior mesenteric


artery?

1986.

Left coronary artery

What artery supplies the left ventricle, left atrium, and


interventricular septum?

1987.

Left eye anopsia (left nasal and temporal


hemianopsia)

Name the ocular lesion; be specific. Left optic nerve


lesion

1988.

The left gastric, splenic, and common hepatic


arteries

What are the three branches of the celiac trunk?

1989.

Left heart failure

Name the most common type or cause. Right heart


failure

1990.

Left homonymous hemianopsia

Name the ocular lesion; be specific. Right calcarine


cortex lesion

1991.

Left homonymous hemianopsia

Name the ocular lesion; be specific. A right LGB lesion


(in the thalamus)

1992.

Left homonymous hemianopsia

Name the ocular lesion; be specific. Left Meyer's loop


lesion of the optic radiations.

1993.

The left is a branch of the aortic arch, while


the right is a branch of the brachiocephalic
trunk.

The left subclavian artery is a branch of what artery?

1994.

Left posterolateral side of the diaphragm,


usually resulting in pulmonary hypoplasia.

What is the most common one? Site of congenital


diaphragmatic hernias

1995.

Left: stomach, spleen, and left kidney; right:


liver, duodenum, and right kidney

What three structures are in contact with the left colic


flexure? With the right colic flexure?

1996.

Left ventricle and auricle of left atrium

What part of the heart forms Left border?

1997.

Left ventricle and tip of right ventricle

What part of the heart forms Diaphragmatic wall?

1998.

Legionella

What bacterium is diagnosed using the Dieterle silver


stain?

1999.

Legionella (think air conditioners)

What water-associated organism is a weakly stained


gram-negative rod that requires cysteine and iron for
growth?

2000.

Leiomyoma

What is the most common one? Tumor of the female


genitourinary tract

2001.

Leiomyoma

What is the most common one? Benign GI tumor

2002.

Leiomyoma (fibroids)

What is the most common one? Tumor of the female genital tract

2003.

Leishmania donovani (kala azar


is also known as visceral
leishmaniasis)

Which hemoflagellate species causes kala azar?

2004.

Lente insulin and either NPH or


protamine zinc insulin (PZI)

What two forms of insulin, if mixed together, precipitate zinc?

2005.

Lente or NPH insulin (neutral


protamine Hagedorn insulin)

Name the insulin preparation based on the peak effect and duration of
action. Peak, 8 to 12 hours; duration, 18 to 24 hours

2006.

Lentigo maligna melanoma

Which form of melanoma carries the best prognosis?

2007.

Leptospira

Which organism causes Weil's disease?

2008.

Lesch-Nyhan syndrome

What X-linked recessive disease involves mental retardation, selfmutilation, choreoathetosis, spasticity, a decrease in HGPRT, and an
increase in uricemia?

2009.

Lesch-Nyhan syndrome

What X-linked recessive disorder is characterized by hyperuricemia,


spastic cerebral palsy, mental retardation, and self-mutilation?

2010.

Lesser curvature of the antrum


of the stomach

What is the most common one? Location of a gastric ulcer

2011.

Lethal dose for 50% of drug


takers (median lethal dose)

What do the following values indicate? LD50

2012.

Leucine and lysine

What are the two ketogenic AAs?

2013.

Leukemias

Name the cancer associated with the following chemical agents.


(Some may have more than one answer.) Benzenes

2014.

Leukemias and lymphomas

Name the cancer associated with the following chemical agents.


(Some may have more than one answer.) Alkylating agents

2015.

Leukotriene B4 (LTB4)

What is the name of the major chemotactic agent released from


Neutrophils?

2016.

Leuprolide

What GnRH analog is used as a repository form of treatment in


prostatic cancer?

2017.

Levator palpebrae superioris

What muscle is most superior in the orbit?

2018.

Lewy bodies

What is the term for the round intracytoplasmic eosinophilic


inclusions containing -synuclein found in the dopaminergic
neurons of the substantia nigra?

2019.

Leydig cells produce testosterone. LH


stimulates Leydig cells. (Both start with
L.)

What cell of the male reproductive system produces


testosterone?

2020.

Leydig cell tumor

What stromal tumor in males is characterized histologically


with crystalloids of Reinke?

2021.

Leydig cell tumor

What sex cell tumor causes precocious puberty,


masculinization, gynecomastia in adults, and crystalloids of
Reinke?

2022.

Leydig cell tumor

Reinke crystals

2023.

LGB (think EYES)

Name the thalamic nucleus based on its input and output.


Input from the optic tract; output projects to the primary
visual cortex of the occipital lobe

2024.

LH

The surge of what hormone induces ovulation?

2025.

LH

For what hormone do Leydig cells have receptors?

2026.

LH has no effect on the production of


adrenal androgens; ACTH stimulates
adrenal androgen production.

What is the effect of LH on the production of adrenal


androgens?

2027.

Libman-Sacks endocarditis

What form of endocarditis do patients with SLE commonly


encounter?

2028.

Libman-Sacks endocarditis

What is the most common one? Cardiac pathology in


patients with SLE

2029.

Ligament of Treitz

The duodenal-jejunal flexure is suspended from the


posterior abdominal wall by what?

2030.

Ligament teres

What are the adult remnants of the following structures?


Left umbilical vein

2031.

Ligamentum arteriosum

What are the adult remnants of the following structures?


Ductus arteriosus

2032.

Ligamentum venosum

What are the adult remnants of the following structures? Ductus


venosus

2033.

Light regulates the activity of the


pineal gland via the retinalsuprachiasmatic- pineal pathway.

Does light or darkness regulate the pineal gland?

2034.

Limbic system

Name the area of the cerebral cortex with the function described:
Motivation, memory, emotions, violent behaviors, sociosexual
behaviors, conditioned responses

2035.

Limbic system

Name the area of the cerebral cortex affected by the description of


the effects, symptoms, and results of the lesion. Apathy,
aggression, inability to learn new material, and memory problems

2036.

The limbic system

What area of the brain is responsible for emotion, feeding, mating,


attention, and memory?

2037.

Lingual nerve of CN V3

What nerve supplies general sensation to the anterior two-thirds of


the tongue?

2038.

Linoleic acid and linolenic acid

What are the two essential fatty acids?

2039.

Lipid-soluble hormones are


considered slow-acting
hormones.

What type of hormone is described as having intracellular receptors,


being synthesized as needed, mostly bound to proteins, and having
its activity determined by free hormone levels?

2040.

Lipoid nephrosis

Name the most common type or cause. Nephrotic syndrome in


children

2041.

Lipoprotein lipase

What enzyme, induced by insulin and activated by apo C-II, is


required for chylomicron and VLDL metabolism?

2042.

Lipoxygenase pathway, from


arachidonic acid

What is the name of the pathway that produces leukotrienes?

2043.

Liquefaction necrosis

Name the type of necrosis. Seen as dead tissue with liquefactive


necrosis?

2044.

Liquids, CHO, protein, fat

Place in order from fastest to slowest the rate of gastric emptying for
CHO, fat, liquids, and proteins.

2045.

Lisch nodules

What is the term for pigmented iris hamartomas seen in patients


with neurofibromatosis type 1?

2046.

Lispro insulin

Name the insulin preparation based on the peak effect and duration of
action. Peak, 0.3 to 2 hours; duration, 3 to 4 hours

2047.

Listeria

What gram-positive rod is distinguished by its tumbling motility?

2048.

Listeria monocytogenes

What small gram-positive, non-spore-forming rod is a facultative


intracellular parasite that grows in the cold and is associated with
unpasteurized milk products?

2049.

Listeria monocytogenes

What is the MCC of the following meningitides? In renal transplant


patients

2050.

Listeria monocytogenes

Name the most common type or cause. Meningitis in renal transplantation


patients

2051.

Lithium

What is the DOC for bipolar disorder?

2052.

Lithium

Which medication used in the treatment of bipolar disorder decreases the


release of T 4 from the thyroid gland?

2053.

Liver

What organ functions to keep blood glucose levels normal through both
well-fed and fasting states and produces ketones in response to increased
fatty acid oxidation?

2054.

Liver

What organ must metastasize for carcinoid heart disease to occur?

2055.

Liver (hepatocytes)

What is the primary target for the action of glucagon?

2056.

The liver (in the


mitochondria)

What is the only organ in the body that can produce ketone bodies?

2057.

Liver, kidney, and GI


epithelium

What are the three glycogenic organs?

2058.

Liver, kidneys, and


pituitary gland via 5'
deiodinase enzyme

What three organs are responsible for peripheral conversion of T4 to T3?

2059.

Log-kill hypothesis
(follows first order
kinetics)

What hypothesis states that an antineoplastic agent kills a fixed percentage


of tumor cells, not a fixed number?

2060.

Long-term memory is impaired in


hippocampal lesions; it is spared in
medial temporal lobe lesions.

How can you differentiate between a medial temporal lobe


and a hippocampal lesion based on memory impairment?

2061.

Long thoracic nerve. To avoid confusing


long thoracic nerve and lateral thoracic
artery: long has an n for nerve; lateral
has an a for artery.

Damage to what nerve will give you winged scapula?

2062.

Look for broken blood vessels in the


baby's eyes.

In what organ system would you attempt to localize a sign for


shaken baby syndrome"? What do you look for?

2063.

Loose association

What is the term to describe jumping from one topic to the


next without any connection?

2064.

A loss of afferent activity indicates a


decrease in BP, and an increase in
afferent activity indicates an increase in
BP.

What does a loss of afferent activity from the carotid sinus


onto the medulla signal?

2065.

Loss of hypotonic fluid (alcohol,


diabetes insipidus, dehydration)

Describe what type of fluid is either gained or lost with the


following changes in body hydration for the ECF volume, ICF
volume, and body osmolarity, respectively: ECF, decrease;
ICF, decrease; body: increase

2066.

Loss of isotonic fluid (diarrhea, vomiting,


hemorrhage)

Describe what type of fluid is either gained or lost with the


following changes in body hydration for the ECF volume, ICF
volume, and body osmolarity, respectively: ECF, decrease;
ICF, no change; body, no change

2067.

Low average

What is the label given to an individual whose IQ is 80 to 89

2068.

Lower extremity and lower trunk information travels in the


dorsal spinocerebellar tract. The upper trunk and extremity
information travels in the cuneocerebellar tract.
(Cuneocerebellar and fasciculus cuneatus both apply to
upper extremities.)

What tract carries unconscious


proprioceptive information from the Golgi
tendon organs and muscle spindles to the
cerebellum, helping monitor and modulate
muscle movements?

2069.

Lower lobe

In which region of the lung are 75% of the


pulmonary infarcts seen?

2070.

Low interstitial free Ca2+ concentrations

What is the only important physiological


signal regulating the release of PTH?

2071.

Low molecular weight heparin (LMWH)

What class of heparin is active against


factor Xa and has no effect on PT or PTT?

2072.

A low-resistance system is formed by resistors added in


parallel.

What type of resistance system (i.e., high


or low) is formed when resistors are added
in parallel?

2073.

LTB4

Name the product or products of


arachidonic acid: Chemotactic for
neutrophils

2074.

LTB4

Which leukotrieneLTA4, LTB4, LTC4, or


LTD4is not associated with anaphylaxis
and bronchoconstriction?

2075.

LTB4

Which leukotriene is chemotactic for


neutrophils?

2076.

LTC4 and LTD4

Which leukotrienes are associated with


the late-phase inflammatory response?

2077.

LTC4, LTD4, and LTE4

Name the product or products of


arachidonic acid: Vasodilation,
bronchospasm, and increased vascular
permeability

2078.

Lumbar splanchnics

What preganglionic sympathetic fibers are responsible for


innervating the smooth muscle and glands of the pelvis and
the hindgut?

2079.

Lung and bronchus cancer

What is the second most common cancer diagnosed in both


males and females?

2080.

Lung cancer

What is the MCC of cancer mortality in males and females?

2081.

Lung cancer

Name the cancer associated with the following oncogenes.


(Some may have more than one answer) Ki-ras and erb-2

2082.

Lung recoil, being a force to collapse the


lung, increases as the lung enlarges
during inspiration.

What is the name of the force that develops in the wall of the
lungs as they expand?

2083.

The lung will expand; also the opposite


is true.

What happens to the lung if the intrapleural pressure exceeds


lung recoil?

2084.

Luteal phase

What is the term for days 15 to 28 in the female cycle?

2085.

LVEDV and LVEDP (left ventricular enddiastolic volume and end-diastolic


pressure, respectively)

What are the two best indices of left ventricular preload?

2086.

Lymph nodes

What lymphoid organ has the following characteristics: outer


and inner cortical areas, encapsulation, germinal centers,
and high endothelial venules?

2087.

Lymphoblastic lymphoma

What rapidly progressive and aggressive T-cell lymphoma


affects young males with a mediastinal mass (thymic)?

2088.

Lysine and arginine

What are the two most common AAs found in histones?

2089.

Lysine and tyrosine

What two AAs have a pKa of 10?

2090.

Lysosomal -1, 4-glucosidase

What enzyme is deficient in the following glycogen storage


disease? Pompe's disease

2091.

M 1 and nicotinic receptors are


excitatory in the CNS and M 2 receptor
is inhibitory.

Which CNS ACh receptor or receptors are excitatory?


Inhibitory?

2092.

M2

Which muscarinic receptor uses a decrease


in adenyl cyclase as its second messenger?

2093.

M3 (promyelocytic leukemia)

Which subtype of AML is most commonly


associated with Auer rods?

2094.

M12 strains

Which M-protein strain of Streptococcus


pyogenes is associated with acute
glomerulonephritis?

2095.

(MA/CA) x 100 = IQ score, where MA = mental age and


CA = chronological age

What is the formula to calculate IQ?

2096.

Macrophages

What cell in chronic inflammation is derived


from blood monocytes?

2097.

Macula densa

What cells of the kidney are extravascular


chemoreceptors for decreased Na+, Cl-, and
NaCl?

2098.

Macula densa

What cells of the nephron function as


sodium concentration sensors of the tubular
fluid?

2099.

Macula densa

Which extravascular chemoreceptor detects


low NaCl concentrations?

2100.

Macule (e.g., a freckle)

What is flat, circumscribed nonpalpable


pigmented change up to 1 cm?

2101.

Magnesium

What is the DOC for treating torsade de


pointes?

2102.

Magnesium ammonium phosphate (struvite)

What renal calculus is associated with ureasplitting bacteria?

2103.

The main drive shifts from central chemoreceptors (CSF


H+) to peripheral chemoreceptors monitoring low PO2
levels.

In high altitudes, what is the main drive for


ventilation?

2104.

Major

Are the following major or minor Jones


criteria of rheumatic fever? Erythema
marginatum

2105.

Major

Are the following major or minor Jones criteria of rheumatic fever?


Sydenham chorea

2106.

Major

Are the following major or minor Jones criteria of rheumatic fever?


Pericarditis

2107.

Major

Are the following major or minor Jones criteria of rheumatic fever?


Subcutaneous nodules

2108.

Major

Are the following major or minor Jones criteria of rheumatic fever?


Migratory polyarthritis

2109.

Major basic protein

What substance found in eosinophils is toxic to parasitic worms?

2110.

Malassezia furfur (treat with


selenium sulfide)

What fungus is characterized by hypopigmented spots on the thorax,


spaghetti-and-meatball KOH staining, and pityriasis or tinea versicolor?

2111.

Male pseudohermaphrodite
(dude looks like a lady!)

What condition is characterized by a 46XY karyotype, testes present, and


ambiguous or female external genitalia?

2112.

Male pseudo-intersexuality
(hermaphrodite); these
individuals are 46XY.

What disorder is due to a 5--reductase deficiency, resulting in testicular


tissue and stunted male external genitalia?

2113.

Malignancy

What disorder is associated with loss of polarity, anaplasia,


pleomorphism, discohesiveness, increase in the nuclear:cytoplasmic
ratio, hyperchromasia, and increase in the rate of mitosis?

2114.

Malignant mesothelioma

In what rare lung malignancy have 90% of patients had an occupational


exposure to asbestos?

2115.

Malingering

What disorder is described as having Conscious symptoms with


conscious motivation?

2116.

Mallory-Weiss syndrome

What is the term for severe and protracted vomiting resulting in linear
lacerations at the gastroesophageal junction?

2117.

Malpighian layer (made up


of the stratum basale and
stratum spinosum)

On what layer of the epidermis does all mitosis occur?

2118.

Mannitol

What IV-only agent inhibits water reabsorption in the PCTs and is used to
treat increased intracranial pressure, increased intraocular pressure, and
acute renal failure?

2119.

Mantle cell lymphoma

Name the cancer associated with the following oncogenes. (Some may
have more than one answer) cyclin D

2120.

Mantle cell lymphoma

What lymphoma is characterized by CD19, CD20, CD5; CD23-negative;


and chromosome 11;14 translocations?

2121.

Marfan syndrome

Name the AD disease associated with chromosome 15 in which the patient


has long extremities, lax joints, pigeon chest, and posterior mitral leaflet
prolapse and is prone to developing dissecting aortic aneurysm.

2122.

Marfan syndrome

What syndrome arises from mutation in the fibrillin gene (FBN1) on


chromosome 15q21?

2123.

Marijuana

What drug causes a sixfold increase in schizophrenia, can impair motor


activity, and can cause lung problems?

2124.

Marrow failure, cancer,


and leukemia

What are the three causes of normochromic normocytic anemia with a


normal MCV and a low reticulocyte count?

2125.

Masochism

What paraphilia is defined as Deriving sexual pleasure from being abused


or in pain?

2126.

Massive influx of calcium

Influx of what ion is associated with irreversible cell injury?

2127.

Mast cells

What is the term to describe basophils that have left the bloodstream and
entered a tissue?

2128.

Mast cells

What is the term for tissue-based basophils?

2129.

Mature B cell; the


memory B cell can have
IgG, IgA, or IgE on its
surface.

At what stage of B-cell development can IgM or IgD be expressed on the cell
surface?

2130.

Maxillary division of CN V
and glossopharyngeal
nerves

What CN is associated with the sensory innervation of Nasopharynx?

2131.

McArdle's syndrome

What glycogen storage disease is due to the following enzyme deficiencies?


Muscle phosphorylase

2132.

M-cells

What cell transports IgA, is secreted by plasma cells, and is in Peyer's patches to
the gastrointestinal lumen?

2133.

Mean

What is the term to describe the average?

2134.

Mean corpuscular
hemoglobin
concentration
(MCHC)

What is the name for the following RBC indices? Average Hgb concentration/given
volume of packed RBCs

2135.

Mean corpuscular
hemoglobin
(MCH)

What is the name for the following RBC indices? Average mass of the Hgb
molecule/RBC

2136.

Mean corpuscular
volume (MCV)

What is the name for the following RBC indices? Average volume of a RBC

2137.

Measles (rubeola)

What negative sense RNA virus is associated with cough, coryza, and conjunctivitis
with photophobia?

2138.

Measurement bias

What type of bias is it when the information is distorted because of the way it is
gathered?

2139.

Mebendazole

What inhibitor of microtubule synthesis is the drug of choice for whipworm and
pinworm?

2140.

Meckel
diverticulum

What congenital small bowel outpouching is a remnant of the vitelline duct?

2141.

Meclizine

Which antihistamine is one of the DOCs for treatment of vertigo?

2142.

Medial collateral
ligament

What structure of the knee is described thus? Prevents abduction

2143.

Medial
compartment of
the thigh,
obturator nerve

Name the compartment of the lower extremity and the nerve based on its
movements. Adduct the thigh and flex the hip

2144.

Medial medullary
syndrome

What syndrome is associated with the following brainstem lesions? Vertebral


artery or anterior spinal artery occlusion, resulting in contralateral corticospinal tract
and medial lemniscus tract deficits and an ipsilateral CN XII lesion

2145.

Medial meniscus

What structure of the knee is described thus? C-shaped shock absorber; aids in
attachment of the tibia to the femur via the medial collateral ligament

2146.

Medial
midbrain
(Weber's)
syndrome

What syndrome is associated with the following brainstem lesions? Posterior cerebral
artery occlusion resulting in a contralateral corticospinal tract signs, contralateral
corticobulbar signs to the lower face, and ipsilateral CN III palsy

2147.

Medial
nuclear group
(limbic
system)

Name the thalamic nucleus based on its input and output. Input from the amygdala,
prefrontal cortex, and temporal lobe; output to the prefrontal lobe and the cingulated
gyrus

2148.

Medial
pontine
syndrome

What syndrome is associated with the following brainstem lesions? Contralateral


corticospinal and medial lemniscus tract deficits and an ipsilatera medial strabismus
secondary to a lesion in CN VI

2149.

Medial rectus
(CN III) (LR6
SO4)3

What ocular muscle Adducts the eyeball and is involved in horizontal conjugate gaze?

2150.

Medial
umbilical
ligaments

What are the adult remnants of the following structures? Right and left umbilical
arteries

2151.

Median nerve

What nerve is compromised in carpal tunnel syndrome?

2152.

Median nerve

What nerve is associated with the following functions? Flex the wrist and digits,
pronate the wrist and the LOAF (Lumbricales, Opponens pollicis, Abductor pollicis
brevis, Flexor pollicis brevis) muscles of the hand

2153.

Median nerve
lesion

What nerve lesion presents with ape or simian hand as its sign?

2154.

Median (think
of it as the
halfway point)

What is the term for the point on a scale that divides the population into two equal parts?

2155.

Medulla
blastoma

What is the most common one? Primary CNS tumor in children

2156.

Medullary
carcinoma of
the thyroid

What thyroid carcinoma secretes calcitonin and arises from the parafollicular C cells?

2157.

Medullary
carcinoma of
the thyroid

Name the cancer associated with the following tumor markers. (Some may have more
than one answer.) Calcitonin

2158.

Mees lines

What is the term for transverse bands on the fingernails seen in


patients with chronic arsenic poisoning?

2159.

Megaloblastic anemia (decreased


absorption of vitamin B12 and folic
acid)

What blood disorder is a side effect of metformin?

2160.

Megaloblasts

Macro-ovalocytes in the peripheral blood smear are formed from


what cell in the bone marrow?

2161.

Meiosis I; disjunction with


centromere splitting occurs during
meiosis II.

The separation of 46 homologous chromosomes without splitting


of the centromeres occurs during what phase of meiosis?

2162.

Melanin

What endogenous pigment found in the substantia nigra and


melanocytes is formed by the oxidation of tyrosine to
dihydroxyphenylalanine?

2163.

Melanocytes

What cell type of the epidermis originates from the neural crest?

2164.

Melanoma

Name the cancer associated with the following oncogenes. (Some


may have more than one answer) CDK4

2165.

Melanoma, cancer of the stomach


and bladder

Name the cancer associated with the following oncogenes. (Some


may have more than one answer) hst-1 and int-2

2166.

Melanosis coli

What is the term for black pigmentation of the colon associated


with laxative abuse?

2167.

Melasma

What skin condition has irregular blotchy patches of


hyperpigmentation on the face commonly associated with OCP
use and pregnancy?

2168.

Melatonin. It is a light-sensitive
hormone that is associated with
sleepiness.

What pineal hormone's release is inhibited by daylight and


increased dramatically during sleep?

2169.

Melatonin, serotonin, and CCK

Name the three hormones produced by pinealocytes.

2170.

Membrane conductance (think


conductance = channels open)

What is the term that refers to the number of channels open in a


cell membrane?

2171.

Membrane depolarization is the stimulus to


open these channels, which are closed in
resting conditions.

What is the signal to open the voltage-gated


transmembrane sodium channels?

2172.

Membrane depolarization is the stimulus to


open these slow channels, and if they are
prevented from opening, it will slow down
the repolarization phase.

What is the signal to open the voltage-gated


transmembrane potassium channels?

2173.

Membranoproliferative glomerulonephritis
(MGN)

Name the most common type or cause. Nephrotic


syndrome

2174.

Membranoproliferative glomerulonephritis
(MPGN)

What GN is highly associated with hepatitis B and C


infections?

2175.

Membranous glomerulonephritis

What renal pathology involves uniform thickening of the


glomerular capillary wall, granular appearance under the
microscope, and effacement of foot processes?

2176.

Membranous septal defects are


interventricular; a persistent patent ovale
results in an interatrial septal defect.

Is a membranous septal defect more commonly


interventricular or interatrial?

2177.

Mntrier's disease

What protein-losing enteropathy has grossly enlarged


rugal fold in the body and fundus of the stomach in
middle-aged males, resulting in decreased acid
production and an increased risk of gastric cancer?

2178.

MEN II and III syndromes

Name the cancer associated with the following


oncogenes. (Some may have more than one answer) et

2179.

MEN IIa (or Sipple syndrome)

Which subset of MEN syndrome is associated with the


following? Medullary carcinoma of the thyroid,
pheochromocytoma, and parathyroid adenomas (or
hyperplasia)

2180.

MEN III (or IIb)

Which subset of MEN syndrome is associated with the


following? Medullary carcinoma of the thyroid,
pheochromocytoma, and mucocutaneous neuromas

2181.

Meningioma

What slow-growing primary CNS tumor that affects mostly


females is associated with psammoma bodies?

2182.

Meningocele All except occulta cause


elevated-fetoprotein levels.

Name the form of spina bifida. Meninges project through a


vertebral defect

2183.

Meningococcus (Gonococcus does


not)

Which gram-negative diplococcus ferments maltose?

2184.

Meningococcus grows on chocolate


agar, and Gonococcus grows on
Thayer-Martin medium.

Which gram-negative diplococcus grows on chocolate agar?


Thayer- Martin medium?

2185.

Meningomyelocele All except occulta


cause elevated-fetoprotein levels.

Name the form of spina bifida. Meninges and spinal cord


project through a vertebral defect

2186.

MEN I (or Wermer syndrome)

Which subset of MEN syndrome is associated with the


following? Parathyroid, pancreatic, and pituitary gland tumors
and Zollinger-Ellison syndrome

2187.

Menses

What is the term for days 1 to 7 of the female cycle?

2188.

Menses. (Ovulation occurs 14 days


before the beginning of menses.)

What is the term for the first 3 to 5 days of the female


reproductive cycle?

2189.

Mentally disabled

What is the label given to an individual whose IQ is Below 69

2190.

Mercury

What type of acute metal poisoning involves stomach and colon


erosion and acute tubular necrosis?

2191.

Merkel cells (Merkel's tactile cells)

What cells of the epidermis, derived from the neural crest, act
as mechanoreceptors?

2192.

Mesangial macrophages

Name the macrophages by location: Kidney

2193.

Mesencephalic nucleus

What component of the trigeminal nuclei Forms the sensory


component of the jaw jerk reflex?

2194.

Mesencephalon

Name the primary vesicle the following structures are derived from
(proencephalon, mesencephalon, or rhombencephalon). Midbrain

2195.

Mesencephalon

Name the primary vesicle the following structures are derived from
(proencephalon, mesencephalon, or rhombencephalon). Cerebral
aqueduct

2196.

Mesocortical area

In what area of the brain can an excess of dopamine lead to psychotic


symptoms?

2197.

Mesolimbic system

What area of the brain is linked to emotion and movement?

2198.

Mesonephric duct

What embryonic structure forms the adult male structure? Epididymis,


ductus deferens, seminal vesicle, and ejaculatory duct

2199.

Mesonephric duct

What embryonic structure forms the adult female structures? Gartner's


duct

2200.

Mesonephric duct (ureteric


bud)

What embryonic structure forms the following adult structures?


Collecting ducts, calyces, renal pelvis, and ureter

2201.

Mesothelioma and
bronchogenic carcinoma

Name the cancer associated with the following chemical agents. (Some
may have more than one answer.) Asbestos

2202.

Metabolic acidosis (summary:


low pH, high H+, and low
HCO3-)

What primary acid-base disturbance is caused by a gain in fixed acid


forcing the reaction to shift to the left, decreasing HCO3- and slightly
increasing CO2?

2203.

Metabolic alkalosis
(summary: high pH, low H+
and high HCO3-)

What primary acid-base disturbance is caused by a loss in fixed acid


forcing the reaction to shift to the right, thereby increasing HCO3levels?

2204.

Metabolic rate and alveolar


ventilation (main factor)

What are the two factors that affect alveolar PCO2 levels?

2205.

Metanephros

What embryonic structure forms the following adult structures?


Nephrons, kidney

2206.

Metaphase II

At ovulation, in what stage of meiosis II is the secondary oocyte


arrested?

2207.

Metaphase of meiosis II (in


the oocyte of the graafian
follicle)

What is the second arrested stage of development in the female


reproductive cycle?

2208.

Metaplasia (usually to a more protective


cell type)

What is the term for a reversible change in one cell type to


another?

2209.

Metaproterenol and albuterol

What two 2-agonists are used to produce bronchodilation?

2210.

Metaraminol (1, 1 )

What 1-agonist is used to treat paroxysmal atrial


tachycardia with hypotension?

2211.

Metastatic

What is the most common one? Brain tumor

2212.

Metastatic

What is the most common one? Bone tumor

2213.

Metastatic calcification

What is the term for hypercalcemia resulting in precipitation


of calcium phosphate in normal tissue?

2214.

Metastatic carcinoma

What is the most common one? Liver tumor

2215.

Metastatic carcinomas

What is the most common one? Neoplastic tumor in the


lungs

2216.

Metformin

What oral hypoglycemic agent should be used with caution


in patients with CHF because it causes lactic acidosis?

2217.

Methanol

Which form of alcohol toxicity results in ocular damage?

2218.

Methanol (wood alcohol)

What form of alcohol causes blindness?

2219.

Methicillin

Which penicillin can cause interstitial nephritis?

2220.

Methionine (start) and tryptophan are the


only two AAs with only one codon.

What two AAs do not have more than one codon?

2221.

Methyldopa

Which centrally acting 2-adrenergic agonist is safe for use


in renal dysfunction and in HTN during pregnancy?

2222.

Methylene blue

Name the antidote. Nitrates

2223.

Methylphenidate (Ritalin)

What is the drug of choice for treating ADHD?

2224.

Metronidazole

Name the antimicrobial agent whose major side effect is


listed. Teratogenicity

2225.

Metronidazole

What is the DOC for the following protozoal infections?


Trichomoniasis

2226.

Metronidazole

What is the DOC for the following


protozoal infections? Giardiasis

2227.

Metronidazole

What is the DOC for the following


protozoal infections? Amebiasis

2228.

Metronidazole

What is the DOC for treating Entamoeba


histolytica, Giardia, Trichomonas, Bacillus
fragilis, and Clostridium difficile infections?

2229.

M:F 4:1 committing, but M:F ratio of attempts is 1:3 (males


commit more but females try it more)

What is the male-to-female ratio for


committing suicide?

2230.

MGB

Which thalamic nucleus receives auditory


input from the inferior colliculus?

2231.

MGB (think EARS)

Name the thalamic nucleus based on its


input and output. Input from inferior
colliculus; output to primary auditory
cortex

2232.

MGN

Name the most common type or cause.


Nephritic syndrome in adults

2233.

MHC class I Ags (the endogenous pathway). This allows


the body to eliminate tumor cells, virus-infected cells
anything the body recognizes as nonself via CD8+ T cells.

What MHC class functions as a target for


elimination of abnormal host cells?

2234.

MHC class I antigens; they are also found on the surface of


platelets.

What MHC class of antigens do all


nucleated cells carry on their surface
membranes?

2235.

MHC class II Ags. This is accomplished via CD4 T cells.

What MHC class acts to remove foreign


Ags from the body?

2236.

MI

Name the MCC of death. In diabetic


individuals

2237.

Microcytotoxic assay

What assay is used to identify MHC class


I molecules?

2238.

Microglia

Name the macrophage based on its location: Brain macrophages

2239.

Microglia. All others are


neuroectodermal derivatives.

What is the only neuroglial cell of mesodermal origin?

2240.

Microglial cells

Name the macrophages by location: CNS

2241.

Microglia (Microglia and


mesoderm both begin with M)

What glial cell is derived from mesoderm and acts as a scavenger,


cleaning up cellular debris after injury?

2242.

Microsporum

Which genus of dermatophytes is associated with the following sites of


infection? Hair and skin only

2243.

Microvillus

What cell membrane structure increases the surface area of a cell and
has actin randomly assorted within its structure?

2244.

Midazolam

Which benzodiazepine is most commonly used IV in conscious


sedation protocols?

2245.

Middle

Name the compartment of the mediastinum associated with the


following thoracic structures: Heart and pericardium

2246.

Middle

Name the compartment of the mediastinum associated with the


following thoracic structures: Ascending aorta

2247.

Middle

Name the compartment of the mediastinum associated with the


following thoracic structures: IVC

2248.

Middle

Name the compartment of the mediastinum associated with the


following thoracic structures: Pulmonary artery and veins

2249.

Middle cerebral artery

What artery supplies most of the lateral surfaces of the cerebral


hemispheres?

2250.

Middle cerebral artery

What is the most common one? Site of a cerebral infarct

2251.

Middle meningeal artery

Name the structure that enters or exits the following foramina:


Foramen spinosum

2252.

Middle meningeal artery

Name the cerebral vessel associated with the following vascular


pathologies. Epidural hemorrhage

2253.

Middle meningeal artery

What vessel is lacerated in an epidural hematoma?

2254.

MIF

What hormone, produced by Sertoli cells, if absent would result in the formation of
internal female structures?

2255.

Mikulicz syndrome

What condition is manifested by bilateral sarcoidosis of the parotid glands,


submaxillary gland, and submandibular gland with posterior uveal tract
involvement?

2256.

Mild (50-70)

What level of mental retardation is characterized by Being self-supportive with


minimal guidance and able to be gainfully employed (includes 85% of the mentally
retarded)?

2257.

Minimal change
disease

Name the most common type or cause. Nephritic syndrome in children

2258.

Minimal change
disease

What nephrotic syndrome has effacement of the epithelial foot processes without
immune complex deposition?

2259.

Minocycline

What tetracycline is associated with vestibular toxicity?

2260.

Minor

Are the following major or minor Jones criteria of rheumatic fever? Elevated acute
phase reactants (e.g., ESR)

2261.

Minor

Are the following major or minor Jones criteria of rheumatic fever? Arthralgias

2262.

Minor

Are the following major or minor Jones criteria of rheumatic fever? Fever

2263.

Minoxidil (orally
and topically,
respectively!)

Which -blocking agent is used to treat refractory hypertension and baldness?

2264.

Misoprostol

What medication is a PGE1 analog that results in increased secretions of mucus


and HCO3-in the GI tract?

2265.

Missense

Name the type of mutation: New codon specifies a different AA

2266.

Mitochondria

In what organelle does the TCA cycle occur?

2267.

Mitochondria

What organelles make ATP, have their own dsDNA, and can synthesize protein?

2268.

Mitochondrial
inheritance

What pattern of genetic transmission is characterized by no transmission from M,


maternal inheritance, and the potential for the disease to affect both sons and
daughters of affected F?

2269.

Mitochondria-linked disorders are


always inherited from the mother.

Who is responsible for passing on mitochondrial DNA genetic


disorders?

2270.

Mitosis

What phase of the cell cycle are the following antineoplastic


agents specific for? Vincristine

2271.

Mitosis

What phase of the cell cycle are the following antineoplastic


agents specific for? Vinblastine

2272.

Mitosis

What phase of the cell cycle are the following antineoplastic


agents specific for? Paclitaxel

2273.

Mitral (bicuspid) valve

What is the only valve in the heart with two cusps?

2274.

Mitral insufficiency

Name the valve abnormality based on the following criteria:


Back-filling into the left atrium during systole; increased v-wave,
preload, left atrial volume, and left ventricular filling

2275.

Mitral stenosis

Name the valve abnormality based on the following criteria:


Diastolic murmur, increased right ventricular pressure, left atrial
pressure, and atrial to ventricular pressure gradient; decreased left
ventricular filling pressure

2276.

Mitral valve closure indicates the


termination of the ventricular filling
phase and beginning of
isovolumetric contraction.

The closure of what valve indicates the beginning of isovolumetric


contraction?

2277.

Mitral valve prolapse

What is your diagnosis of a young, thin asymptomatic female with


a midsystolic click on cardiac auscultation?

2278.

Mitral valve stenosis

What is the most common one? Valve abnormality associated


with rheumatic fever

2279.

Mixed (both cholesterol and


calcium)

What is the most common one? Stone type associated with


cholecystitis

2280.

Mode

The most frequent number occurring in a population is what?

2281.

Moderate (35-49)

What level of mental retardation is characterized by Can


work in sheltered workshops and learn simple tasks but need
supervision?

2282.

Molindone

What is the only neuroleptic that does not cause an increase


in weight or appetite?

2283.

Molluscum contagiosum

What virus causes small pink benign wartlike tumors and is


associated with HIV-positive patients?

2284.

Molybdenum (Mb)

What mineral is an important component of the enzyme


xanthine oxidase?

2285.

Monosodium urate crystals

What type of crystals are associated with gout?

2286.

More Ab is produced in less time in a


secondary immune response (shorter
lag period).

Are more Abs produced in a primary or a secondary immune


response?

2287.

More than 75% of the vessel

How much of a vessel must be stenosed to cause sudden


cardiac death?

2288.

Motilin

What hormone causes contractions of smooth muscle,


regulates interdigestive motility, and prepares the intestine for
the next meal?

2289.

Motor

What type of fiber or fibers are carried in (answer motor,


sensory, or both) Ventral root?

2290.

Motor aspect

What component of the corneal reflex is lost in a CN VII


deficit?

2291.

Motor nucleus of CN V

What component of the trigeminal nuclei Supplies the


muscles of mastication?

2292.

The mouth

What structure is derived from the prochordal plate?

2293.

Moving objects, along with large bright


objects with curves and complex
designs.

Do newborns have a preference for still or moving objects?

2294.

mRNA

Name the RNA subtype based on the following: Only type of


RNA that is translated

2295.

MstII; changing codon 6 (from A to T)


destroys the restriction site.

What restriction endonuclease site is destroyed in sickle globin allele?

2296.

Multiple myeloma

What is the most common one? Tumor arising within the bone

2297.

Multiple myeloma

Bence-Jones proteinuria

2298.

Multiple myeloma

Russell bodies

2299.

Multiple myeloma

A 60-year-old man has back pain (compression spinal fracture), hypercalcemia,


increased serum protein, Bence- Jones proteinuria, and monoclonal M-spike on
serum electrophoresis. What is your diagnosis?

2300.

Multiple sclerosis

What CNS demyelinating disease is characterized by diplopia, ataxia,


paresthesias, monocular blindness and weakness, or spastic paresis?

2301.

Mumps

Name the most common cause. Infectious pancreatitis

2302.

Mumps

What negative sense RNA virus is associated with parotitis, pancreatitis, and
orchitis?

2303.

Mumps and
influenza virus

Which two negative sense RNA viruses have neuraminidase enzymes?

2304.

Mnchhausen's
syndrome
(factitious
disorder)

In which syndrome does a person present with intentionally produced physical


ailments with the intent to assume the sick role?

2305.

Mural infarct

What is the term for hypoperfusion of an area involving only the inner layers?

2306.

Muscle glycogen
phosphorylase

What enzyme is deficient in the following glycogen storage disease? McArdle's


disease

2307.

The muscle's
ATPase activity

What determines the Vmax of skeletal muscle?

2308.

Muscles of the
internal eye

What adult structures are derived from preotic somites?

2309.

Musculocutaneous
nerve

What nerve is associated with the following functions? Flex the shoulder, flex the
elbow, and supinate the elbow

2310.

Musculophrenic
and superior
epigastric arteries

At the level of rib 6, the internal thoracic artery divides into what two arteries?

2311.

Myasthenia gravis

What myopathy due to autoantibodies to ACh receptors can present with thymic
abnormalities, red cell aplasia, and muscle weakness?

2312.

Myasthenia gravis

What autoimmune disorder is due to Abs directed to ACh receptors at the NMJ?

2313.

Myasthenia gravis

A 20-year-old woman goes to the ER with ptosis, diplopia, weakness in


her jaw muscles when chewing, and muscle weakness with repeated
use. What is your diagnosis?

2314.

Myasthenia gravis

What type II hypersensitivity disorder is defined as Autoantibodies


directed against ACh receptors?

2315.

Mycobacterium leprae

Which acid-fast rod is an obligate intracellular parasite?

2316.

Mycobacterium marinum

A tropical fish enthusiast has granulomatous lesions and cellulitis; what


is the most likely offending organism?

2317.

Mycoplasma

What genus is known as the smallest free living bacteria? (hint: has no
cell wall and has sterols in the membrane)

2318.

Mycoplasma pneumoniae

Name the MCC. Walking pneumonia, seen in teens and military


recruits

2319.

Mycoplasma pneumoniae

What atypical pneumonia can be diagnosed with elevated cold


agglutinin titers?

2320.

Mycosis fungoides (cutaneous


T-cell lymphoma)

What skin carcinoma is a superficial dermal infiltrate of T lymphocytes


seen in males more than 40 years old and presents as scaly red
patches or plaques?

2321.

Myelofibrosis with myeloid


metaplasia

What myeloid disorder is characterized by dry bone marrow


aspirations, splenomegaly, leukoerythroblastosis, teardrop RBCs, and
hyperuricemia due to increased cell turnover?

2322.

Myeloperoxidase

What oxygen-dependent killing enzyme requires hydrogen peroxide


and halide (Cl-) to produce hypochlorous acid?

2323.

Myeloschisis All except occulta


cause elevated-fetoprotein
levels.

Name the form of spina bifida. An open neural tube lying on the
surface of the back

2324.

The myotatic reflex is


responsible for the tension
present in all resting muscle.

What basic reflex regulates muscle tone by contracting muscles in


response to stretch of that muscle?

2325.

N-5-methyl THF

In what form is excess folate stored in the body?

2326.

N-Acetylcysteine

Name the antidote. Acetaminophen

2327.

NaCl is removed from the lumen to


dilute the fluid leaving the loop of
Henle.

What is pumped from the lumen of the ascending loop of Henle


to decrease the osmolarity?

2328.

NADPH and molecular O 2

What are the two absolute requirements for the cytochrome


P450 enzyme system?

2329.

NADPH, O2, and cytochrome b5

What three things are needed to produce a double bond in a


fatty acid chain in the endoplasmic reticulum?

2330.

NADPH oxidase

What enzyme is deficient in chronic granulomatous disease of


childhood?

2331.

NADPH oxidase is deficient, resulting


in an inability to produce toxic
metabolites.

What enzyme is deficient in patients with CGD?

2332.

Naloxone, naltrexone

Name the antidote. Opioids

2333.

Naloxone or naltrexone

Which drug is used to treat respiratory depression associated


with an overdose of opioids?

2334.

Narcissistic

Name the cluster B personality disorder: Grandiose sense of


self-importance; demands constant attention; fragile selfesteem; can be charismatic

2335.

Nasal polyps, rhinitis, and aspirin


hypersensitivity

What are the three components to the asthma triad?

2336.

Nasopharynx

What region of the pharynx does the eustachian tube enter?

2337.

Native Americans

What ethnic group has the highest adolescent suicide rate?

2338.

NAVEL: Femoral Nerve, Artery, Vein,


Empty space, and
Lymphatics/Lacunar ligament

Name the components of the femoral canal, working laterally to


medially.

2339.

NE

What neurotransmitter is presynaptically inhibited by reserpine


and guanethidine?

2340.

NE

What are the primary neurotransmitters at the following sites?


Postganglionic sympathetic neurons

2341.

Nearly 25% of patients taking lithium develop polyuria and


polydipsia.

What renal side effect is


commonly seen in patients taking
lithium?

2342.

Nearly 50% of all reported cases of elderly abuse are due to


neglect. Physical, psychological, and financial are other forms of
elderly abuse with an overall prevalence rate of 5% to 10%.

What is the most common form of


elderly abuse?

2343.

Necator americanus is treated with mebendazole and iron therapy.

What nematode is known as


hookworms? What is the
treatment?

2344.

Necrophilia

What paraphilia is defined as


Having sex with cadavers?

2345.

Necrotizing enterocolitis

What is the most common one?


Acquired GI emergency of infancy

2346.

Need 3 panic attacks over 3 weeks (remember, they come out of


the blue.)

How many attacks are needed


over how much time before panic
disorder is diagnosed?

2347.

nef and tat genes

What two HIV regulatory genes


down-regulate MHC class I
expression in the host?

2348.

Negative

Are the following conditions


associated with a negative or
positive nitrogen balance?
Kwashiorkor

2349.

Negative

Are the following conditions


associated with a negative or
positive nitrogen balance?
Infection

2350.

Negative

Are the following conditions associated with a negative or positive


nitrogen balance? AA deficiency

2351.

Negative

Are the following conditions associated with a negative or positive


nitrogen balance? Starvation

2352.

Negative

Are the following conditions associated with a negative or positive


nitrogen balance? Uncontrolled DM

2353.

The negative charge inhibits the


filtration of protein anions.

What is the role of the negative charge on the filtering membrane of


the glomerular capillaries?

2354.

Negative correlation

What type of correlation is defined as One variable that diminishes


in the presence of the other?

2355.

Negative predictive value

The probability that a person with a negative test result is truly


disease free refers to what value?

2356.

Negative reinforcement

Based on operant conditioning, what type of reinforcement is


described when Removing a stimulus reinforces a behavior?

2357.

Negative selection. This helps to


prevent autoimmune diseases.

What is the term for thymic induction of T cells with high-affinity Ag


receptors for self that are programmed to undergo apoptosis?

2358.

Neisseria gonorrhea (history of


STD in patient with monoarticular
infectious arthritis: think
gonococcus)

A 20-year-old woman who was recently diagnosed with a sexually


transmitted disease goes to the ER with a tender, painful, swollen,
and erythematous knee (monoarticular). What organism is the likely
culprit?

2359.

Neisseria gonorrhoeae

A urethral swab of a patient shows gram-negative diplococci in


PMNs; what organism do you diagnose?

2360.

Neisseria gonorrhoeae and


Chlamydia trachomatis

What are the two MCC of acute epididymitis in males? Less than
35 years old

2361.

Neisseria meningitides

What is the MCC of the following meningitides? In military recruits

2362.

Neisseria meningitidis

Which organism releases endotoxins prior to cell death?

2363.

Neologisms. Thomas Jefferson noted,


"Necessity obliges us to neologize."
(Abnormal use of neologisms is known as
neolalism.)

What is the term for new made-up words?

2364.

Neomycin

Which aminoglycoside is used only topically because it is


too toxic for systemic use?

2365.

Neonatal respiratory distress syndrome


(NRDS)

Name the MCC of death. In neonates

2366.

Nephroblastoma

What is the most common one? Solid tumor in the body

2367.

Nephrotoxicity

What toxicities are caused by the following agents?


Methoxyflurane

2368.

Nephrotoxic oxylate stones

What type of damage to the kidneys is caused by drinking


ethylene glycol (antifreeze)?

2369.

Nerve fibers with small diameter and high


firing rates are most sensitive to local
blockade

Which size nerve fibers (small or large diameter) are more


sensitive to local anesthetic blockade?

2370.

Neuroblastoma

What is the most common one? Solid tumor in children

2371.

Neuroblastoma

Name the cancer associated with the following oncogenes.


(Some may have more than one answer) N-myc

2372.

Neuroblastoma

What disease arises from the adrenal medulla, displaces


and crosses the midline, metastasizes early, is the most
common solid tumor, and is seen in the 2-to 4-year-old age
group?

2373.

Neuroblastoma

Homer-Wright rosettes

2374.

Neurofibromatosis

Caf-au-lait spot on the skin

2375.

Neurofibromatosis I (chromosome 22q is


with neurofibromatosis II and no Lisch
nodules)

What disease has multiple schwannomas, caf-au-lait


spots on the skin, and Lisch nodules and is associated with
chromosome 17q?

2376.

Neuroleptic malignant syndrome is a potentially lifethreatening condition which is treated with


symptomatic support, bromocriptine, and dantrolene.

A schizophrenic patient who is taking haloperidol


is brought to your ER with muscle rigidity, a
temperature of 104 F, altered mental status,
and hypotension. What is your diagnosis and
what is the treatment?

2377.

Neurologic symptoms Renal failure


Thrombocytopenia Fever Microangiopathic hemolytic
anemia (Don't forget it. When I was an intern, my
senior resident asked me this question more times
than I would like to remember.)

What is the pentad of TTP?

2378.

Neutropenia, splenomegaly, and rheumatoid arthritis

What is the triad of Felty syndrome?

2379.

Neutrophils, eosinophils, and basophils

What are the three polymorphonuclear


leukocytes? Be specific.

2380.

NE, via its vasoconstrictive action on blood vessels

What neurotransmitter is essential for


maintaining a normal BP when an individual is
standing?

2381.

Nevirapine; it cuts the rate from 20% to 10%. AZT is


also used, cutting the rate from 20% to 10%.

What drug is being given to HIV-positive


mothers during labor and to the children after
birth to decrease the risk of mother-to-child HIV
transmission?

2382.

N-formyl-methionine LTB4 C5a IL-8

Name four chemotactic factors for neutrophils.

2383.

NH4+(ammonium)

What acid form of H+ in the urine cannot be


titrated?

2384.

Niacin (B3)

What water-soluble vitamin deficiency results in


pellagra?

2385.

Niclosamide

What is the drug of choice, which inhibits


oxidative phosphorylation in cestodes, for
taeniasis?

2386.

Nicotine (but it sure has a nasty withdrawal!)

What is the only drug that does not have an


intoxication?

2387.

Nicotinic acid, because its side effects


include hyperuricemia and exacerbation of
ulcers.

Which antihyperlipidemic agent would you not prescribe


for a patient with a history of gout or PUD?

2388.

Niemann-Pick and Tay-Sachs diseases

What two lysosomal storage diseases have cherry-red


spots on the retina?

2389.

Niemann-Pick disease

What AR disease involves a decreased amount of


sphingomyelinase, massive organomegaly, zebra bodies,
and foamy histiocytes and is associated with chromosome
11p?

2390.

Nigrostriatal pathways (basal ganglia)

Movement disorders are associated with what dopamine


pathway (what part of the brain)?

2391.

Nimodipine

Which calcium channel blocker is used to treat


subarachnoid hemorrhages by preventing
posthemorrhagic vasospasms?

2392.

Nissl substances; there is a great deal of


RER in neuron cell bodies, indicating high
protein synthesis.

What is the name of RER in neurons?

2393.

Nitrates

What class of antianginal activates the NO pathway to


produce endothelial vasodilation?

2394.

Nitroblue tetrazolium reduction test (NBT).


It is negative in patients with CGD because
there is no production of oxygen radicals.

What test is done to diagnose CGD?

2395.

Nitrofurantoin

Name the antimicrobial agent whose major side effect is


listed. Hemolytic anemia

2396.

Nitroprusside

What antihypertensive agent produces cyanide and


thiocyanate as byproducts?

2397.

No, but it does interfere with performing


complex tasks and decreases attention to
detail. (Be careful post call!)

Does REM deprivation interfere with performance on


simple tasks?

2398.

Nocardia asteroides

What aerobic branching rod that is gram positive and


partially acid-fast is associated with cavitary
bronchopulmonary disease in immunosuppressed
patients?

2399.

No change in length

What happens to the following during skeletal muscle


contraction? Actin and myosin lengths

2400.

No change in length

What happens to the following during skeletal muscle


contraction? A band

2401.

No CHO, AA, ketones, or peptides are left in


the tubular lumen.

What are the following changes seen in the luminal fluid


by the time it leaves the PCT of the nephron?
Concentration of CHO, AA, ketones, peptides

2402.

Nodes of Ranvier

What is the region of an axon where no myelin is found?

2403.

Nodular melanoma

Which form of melanoma carries the worst prognosis?

2404.

Nodular sclerosis

What is the only subtype of Hodgkin's lymphoma that is


most commonly seen in females?

2405.

Nodular sclerosis

What is the most common one? Subtype of Hodgkin


lymphoma

2406.

No enzymes are needed. When the stop


codon reaches the A site, it results in
termination.

What eukaryotic translation enzyme is associated with


the following: Termination

2407.

No. Heparin is safe in pregnancy, but


warfarin is contraindicated in pregnancy
because of its ability to cross the placenta.

Is heparin contraindicated in pregnancy?

2408.

No. It is abnormal only if it interferes with


normal sexual or occupational function.

Is masturbation considered an abnormal sexual practice?

2409.

No, it is never acceptable to lie.

Is it acceptable to lie, even if it protects a colleague from


malpractice?

2410.

No, it is not a mandatory reportable


offense (if you can believe it). Child and
elderly abuse are mandatory reportable
offenses.

Is spousal abuse a mandatory reportable offense?

2411.

No, it is used in chronic treatment of


gout to decrease the uric acid pools in
the body.

Is allopurinol used in the acute treatment of gout?

2412.

Nominal scale (categorical, e.g., male or


female)

What scale separates things into groups without defining the


relationship between them?

2413.

Nondominant parietal lobe

Name the area of the cerebral cortex affected by the


description of the effects, symptoms, and results of the lesion.
Denial of illness, hemineglect, construction apraxia (can't
arrange matchsticks)

2414.

Nondominant temporal lobe

Name the area of the cerebral cortex affected by the


description of the effects, symptoms, and results of the lesion.
Dysphoria, irritability, musical and visual abilities decreased

2415.

None. Denaturation of dsDNA breaks


hydrogen bonds, not covalent bonds.

How many covalent bonds per purine-pyrimidine base pairing


are broken during denaturation of dsDNA?

2416.

None; it degenerates.

What CN is associated with the Fifth pharyngeal arch?

2417.

None; they are not formed until a girl


reaches puberty.

How many oogonia are present at birth?

2418.

None; they are the storage sites for


ADH and oxytocin.

What hormones are produced in the median eminence region


of the hypothalamus and the posterior pituitary gland?

2419.

Non-Hodgkin lymphoma

What hematologic malignancy is treated with the monoclonal


antibody rituximab?

2420.

Nonsense

Name the type of mutation: New codon specifies for a stop


codon

2421.

No. Prions are infectious proteins, so


antibiotics are useless.

Are antibiotics helpful in treating a disease caused by a


prion?

2422.

NO!! Remember, only a judge can commit a patient. A physician can


detain a patient (maximum is for 48 hours).

Can a physician commit a


patient?

2423.

No. Remember, tests do not diagnose, they confirm or refute your


diagnosis. Also, diagnosis of MI requires two of three criteria: chest
pain consistent and characteristic of MI, elevated cardiac enzymes
consistent with MI, and ST segment elevation of 2 mm or more in at
least two contiguous leads.

Can an acute MI be diagnosed


only by looking at an ECG?

2424.

Normally corticobulbar fiber innervation of the CNs is bilateral (the LMN


receives information from both the left and right cerebral cortex), but
with CN VII the LMN of the upper face receives bilateral input but the
lower facial LMNs receive only contralateral input. -------------------------------------------------------------------------------

How do the corticobulbar


fibres of CN VII differ from the
rest of the CNs?

2425.

Normal thyroid hormones levels in the plasma are necessary for proper
secretion of GH. Hypothyroid patients have decreased GH secretions.

What hormone is necessary


for normal GH secretion?

2426.

Normal values

What condition results in the


following CSF results?
Opening pressure 70 to 180
mm H2O; 0-10 WBCs
(monocytes); glucose 45 to 85,
protein 15 to 45

2427.

Norm reference (i.e., 75% of the students in the class will pass)

When the results of a test are


compared to findings for a
normative group, what form of
reference does the objective
test use?

2428.

Northern blot

What blotting technique uses


the following for analysis?
RNA

2429.

No, therapeutic indices can tell which drug is safer, not more toxic.

Can you tell which drug is


more toxic based on
therapeutic indices alone?

2430.

The notochord

What embryonic structure, around day 19, tells the ectoderm


above it to differentiate into neural tissue?

2431.

No, unsaturated fatty acids have double


bonds.

Does a saturated fatty acid have double bonds?

2432.

No, you never refuse to treat a patient


simply because he or she can't pay.
You are a patient advocate.

If a patient cannot pay, can you refuse services?

2433.

NREM sleep. Night terrors are dreams


that we are unable to recall.

With what stage of sleep are night terrors associated?

2434.

NREM sleep. Remember awake body,


sleeping brain

What general pattern of sleep is described by slowing of EEG


rhythms (high voltage and slower synchronization), muscle
contractions, and lack of eye movement or mental activity?

2435.

Nucleolus. Ribosomal assembly also


takes place in the nucleolus.

What organelle is responsible for ribosomal RNA synthesis?

2436.

Nucleus ambiguus, resulting in the


uvula deviating away from the side of
the lesion.

A unilateral lesion in what nucleus will produce ipsilateral


paralysis of the soft palate?

2437.

Nucleus pulposus

What portion of the intervertebral disk is a remnant of the


notochord?

2438.

Null hypothesis (what you hope to


disprove)

What hypothesis states that the findings of a test are a result


of chance?

2439.

The number of Ags that the Ab can bind

What is the valence of an Ig molecule equal to?

2440.

The number of cross-bridges cycling


during contraction: the greater the
number, the greater the force of
contraction.

What determines the overall force generated by the


ventricular muscle during systole?

2441.

Nystagmus and ataxia

What are the first signs of phenobarbital overdose?

2442.

Nystagmus and ataxia

What are the first signs of overdose from phenobarbitals?

2443.

Nystagmus is named by the fast component, which is the


corrective attempt made by the cerebral cortex in response to
the initial slow phase.

Is nystagmus defined by the fast or slow


component?

2444.

O2 affinity increases with a decrease in the p50, making O2


more difficult to remove from the Hgb molecule.

What happens to O2 affinity with a


decrease in p50?

2445.

O2 has a higher driving force but is only one-twenty-fourth as


soluble as CO2. CO 2 has a very small partial pressure
difference across the alveolar membrane (47-40 = 7 mmHg),
but it is extremely soluble and therefore diffuses readily
across the membrane.

Does O2 or CO2 have a higher driving


force across the alveolar membrane?

2446.

Obsessive-compulsive

Name the cluster C personality disorder:


Orderly, inflexible, perfectionist; makes
rules, lists, order; doesn't like change,
has a poor sense of humor, and needs
to keep a routine

2447.

Obstructive lung disorders. The opposite changes (where you


see decrease exchange it for increase and vice versa) are
seen in a restrictive pattern.

Are the following parameters associated


with an obstructive or restrictive lung
disorder: decreased FEV1, FVC, peak
flow, and FEV1/FVC; increased TLC,
FRC, and RV?

2448.

Obturator nerve

What lower extremity nerve is described


by the following motor loss? Loss of
adduction of the thigh

2449.

Occipital lobe

Name the area of the cerebral cortex


with the function described: Recall of
objects, distances, and scenes; visual
input processed here

2450.

Occipital lobe (Anton's syndrome if it is due to


bilateral posterior cerebral artery occlusions)

Name the area of the cerebral cortex affected by the


description of the effects, symptoms, and results of
the lesion. Denies being blind, cortical blindness

2451.

Occulta All except occulta cause elevatedfetoprotein levels.

Name the form of spina bifida. Defect in the


vertebral arch

2452.

Oddly enough, it is true. Cryptorchidism,


Klinefelter syndrome, testicular feminization, and
family history of testicular cancer are all risk
factors.

True or false? Being a white male increases your


risk factor for testicular cancer.

2453.

Odds ratio. (Case control studies deal with


prevalence.)

What statistical method do you use when analyzing


Case control studies?

2454.

Oligodendroglioma

What slow-growing CNS tumor in 30-to 50-year-old


patients with a long history of seizures has fried egg
cellular appearance in a network of chicken wire?

2455.

Omega-3 fatty acids

What type of fatty acid is associated with a decrease


in serum triglycerides and cardiovascular disease?

2456.

Omega-3; linoleic is omega-6

Is linolenic acid an omega-3 or omega-6 fatty acid?

2457.

Omphalocele and gastroschisis

What two pathologic conditions occur when the gut


does not return to the embryo?

2458.

Oncotic pressure increases because of the


removal of water.

What happens to capillary oncotic pressure with


dehydration?

2459.

The oncotic pressure of plasma promotes


reabsorption and is directly proportional to the
filtration fraction.

Does the oncotic pressure of plasma promote


filtration or reabsorption?

2460.

One, and it provides the energy for mechanical


contraction.

How many ATPs are hydrolyzed every time a


skeletal muscle cross-bridge completes a single
cycle?

2461.

One day after the LH surge and 2 days after the


estrogen peak.

How many days after the LH surge is ovulation?

2462.

One pair with three to five coccygeal vertebrae.


Totaling 31 pairs of spinal nerves.

How many pairs of spinal nerves are associated with


Coccygeal vertebrae?

2463.

The one start codon, AUG, in eukaryotes codes


for methionine and in prokaryotes
formylmethionine.

What is the start codon, and what does it code for in


eukaryotes? Prokaryotes?

2464.

One-way ANOVA

What statistical test compares the means of many


groups (>2) of a single nominal variable by using an
interval variable?

2465.

Onion skinning

What is the term for fibrinoid necrosis of the arterioles


in the kidney secondary to malignant hypertension?

2466.

Onset of bleeding

What event signifies the first day of the menstrual


cycle?

2467.

The onset of symptoms for Neisseria gonorrhea


conjunctivitis is 2 to 5 days, whereas onset of
symptoms for Chlamydia trachomatis is 5 to 10
days.

Based on the onset of the symptoms, how are


bacterial conjunctivitis from Neisseria and Chlamydia
differentiated?

2468.

On the superior body of the posterior wall of the


uterus

What is the most common site for implantation of the


blastocyst?

2469.

Opening of the aortic valve terminates the


isovolumetric phase and begins the ejection
phase of the cardiac cycle.

The opening of what valve indicates the beginning of


the ejection phase of the cardiac cycle?

2470.

Opening of the mitral valve indicates the


termination of the isovolumetric relaxation
phase and the beginning of the ventricular filling
phase.

The opening of what valve indicates the termination of


isovolumetric relaxation phase of the cardiac cycle?

2471.

Operant conditioning (reinforcement is after a


response)

What form of conditioning is defined as a new


response to an old stimulus resulting in a
consequence?

2472.

Operon

In prokaryotes, what is the term for a set of structural


genes that code for a select group of proteins and the
regulatory elements required for the expression of
such gene?

2473.

Opioids cause cerebral


vasodilation and can result in
increased intracerebral
pressure.

Why should opioid analgesics be avoided for patients with head


trauma?

2474.

Opposite the second upper


molar tooth

Where does the parotid (Stensen's) duct enter the oral cavity?

2475.

Opsonization

By which process do Abs make microorganisms more easily ingested


via phagocytosis?

2476.

The optic disk is the blind spot.

What area of the posterior aspect of the eye has no photoreceptors?

2477.

Orbitomedial frontal lobe

Name the area of the cerebral cortex affected by the description of the
effects, symptoms, and results of the lesion. Withdrawn, fearful,
explosive moods, violent outbursts, and loss of inhibitions

2478.

Order of attachment is site 1, 2,


3, 4, and for release is 4, 3, 2,
1.

What is the order of attachment of O2 to Hgb-binding sites in the lung?


Order of release from the binding sites in the tissue?

2479.

Ordinal scale (e.g.,


faster/slower, taller/shorter)

What scale assesses a rank order classification but does not tell the
difference between the two groups?

2480.

Orotic acid (purine metabolism)

What is the pyrimidine intermediate that joins PRPP (5Phosphoribosyl-1-Pyrophosphate)?

2481.

Osler nodes

Name the following descriptions associated with bacterial endocarditis:


Painful subcutaneous nodules on fingers and toes

2482.

Osmosis; water will diffuse


from higher to lower water
concentrations.

What is the term for diffusion of water across a semipermeable or


selectively permeable membrane?

2483.

Osteitis fibrosa cystica (von


Recklinghausen disease)

What bone disorder is characterized by brown tumors, bone pain,


deformities, and fractures due to excessive PTH?

2484.

Osteoarthritis

What is the most common one? Form of arthritis

2485.

Osteoarthritis

What disorder causes joint stiffness that worsens


with repetitive motion, crepitus, effusions, and
swelling and commonly affects the knees, hips, and
spine?

2486.

Osteoblast (Remember, blasts make, clasts take)

What cell type in the bone is responsible for bone


deposition?

2487.

Osteoblasts (Remember, they modulate the


function of osteoclasts.)

What bone cell has receptors for PTH?

2488.

Osteoblasts, which in turn stimulate osteoclasts


to break down bone, releasing Ca2+ into the
interstitium. (Remember, blasts make, clasts
take.)

What cell type of the bone has PTH receptors?

2489.

Osteoclasts

What cell in bone is a part of the mononuclear


phagocytic system?

2490.

Osteoclasts

Name the macrophage based on its location: Bone


macrophages

2491.

Osteocyte

What type of cell is surrounded by mineralized


bone?

2492.

Osteogenesis imperfecta

Blue sclera is seen in what hereditary bone


disorder?

2493.

Osteogenesis imperfecta

Blue sclera

2494.

Osteoma

What benign bone tumor is associated with Gardner


syndrome?

2495.

Osteomalacia. Osteoporosis has normal levels of


calcium, phosphorus, and alkaline phosphatase.

Are elevated alkaline phosphatase and decreased


phosphorus and calcium levels more consistent with
osteoporosis or osteomalacia?

2496.

Osteopetrosis (Albers-Schnberg disease)

What hereditary bone disorder is due to decreased


osteoclast function, resulting in thick, sclerotic
bones that fracture easily?

2497.

Osteoporosis

What is the most common one? Bone disorder in


the United States

2498.

Osteosarcoma

What malignant bone tumor is characterized by


Codman triangle (periosteal elevation) on
radiograph?

2499.

Osteosarcoma

What is the most common one? Malignant


tumor in the bone of teenagers

2500.

Osteosarcoma

Codman triangle on radiograph

2501.

Osteosarcoma

What malignant bone tumor is associated with


familial retinoblastoma?

2502.

Osteosarcoma

What is the most common primary malignant


tumor in bone?

2503.

Ostium secundum defects

What is the most common one? Type of ASD

2504.

The otic ganglion. (These fibers are carried in CN IX.


Remember it like this: the -oti-is in both otic ganglion
and parotid gland.)

What is the name of the postganglionic


parasympathetic ganglion that innervates The
parotid gland?

2505.

The outer cortex contains most of the germinal


centers and therefore also most B cells.

What area of the lymph node contains germinal


centers?

2506.

Outside the cell

Where is tropocollagen aggregated to form a


collagen fibril?

2507.

Ovarian cancer

What is the most common one? Female


genital tract malignancy resulting in death

2508.

Ovarian cancer

Name the cancer associated with the following


tumor markers. (Some may have more than
one answer.) CA-125

2509.

Ovarian fibroma

What is the most common one? Stromal


tumor of the ovary

2510.

Ovulation

What is the term for the day after the LH surge


in the female cycle?

2511.

Oxazepam, temazepam, and lorazepam (OTL)


(mnemonic: Outside The Liver). They undergo
glucuronide conjugation, not via the cytochrome
p450 system.

What are the three benzodiazepines that do not


undergo microsomal oxidation?

2512.

Oxazepam, temazepam, and lorazepam ( OTL,


Outside The Liver)

Name the three benzodiazepines that are not


metabolized by the cytochrome P450 enzyme
system.

2513.

Oxygen

Name the antidote. Carbon monoxide (CO)

2514.

Oxytocin

Near the end of pregnancy, what hormone's


receptors increase in the myometrium because of
elevated plasma estrogen levels?

2515.

Oxytocin

What hormone causes milk letdown?

2516.

P = .05; P < .05 rejects the null hypothesis

What P value defines whether the hull hypothesis


should or should not be rejected?

2517.

P1 protein

What protein allows Mycoplasma to attach to the


respiratory epithelium?

2518.

P24

What protein of the HIV virus does ELISA detect to


determine whether a patient is HIV positive?

2519.

The p50 value does not change in either anemia or


polycythemia; the main change is the carrying
capacity of the blood.

Is there a shift in p50 values with anemia?


Polycythemia?

2520.

p-53

What gene stimulates apoptosis when DNA repair


is unable to be done?

2521.

Paget disease of the breast

What breast malignancy has tumor cells with a


halo surrounding the nucleus and is an ulceration
of the nipple and areola with crusting, fissuring,
and oozing?

2522.

Paget disease of the breast

What breast pathology involves malignant cells


with halos invading the epidermis of the skin?

2523.

Paget disease (osteitis deformans)

What disorder of bone remodeling results in


thick, weak bones and is associated with highoutput cardiac failure?

2524.

Palatoglossus muscle is innervated by CN X; all


other tongue muscles are innervated by CN XII.

What is the only tongue muscle innervated by


CN X?

2525.

Palindrome

What is the term to describe the 5'-3' sequence


of one strand being the same as the opposite 5'3' strand?

2526.

The Palmar interosseus ADducts, whereas the Dorsal


interosseus ABducts (PAD and DAB)

What muscles in the hand adduct the fingers?

2527.

Palmitate

Name the end product or products: Fatty acid


synthesis

2528.

Palms and soles of the feet. Sebaceous glands are


associated with hair follicles, which are lacking on the
palms and soles of the feet.

What two areas of the skin do not contain


sebaceous glands?

2529.

Palms-up before palms-down maneuvers

In regard to motor development during infancy,


choose the motor response that happens first.
Palms up or down

2530.

PALS

What is the T-cell area of the spleen?

2531.

PALS (Parietolateral lymphocytic sheath)

What is the name of the T cell-rich area of the


spleen?

2532.

Panacinar

Which form of emphysema is associated with an


alpha1-antitrypsin deficiency?

2533.

Pancreas and colon

Name the cancer associated with the following


oncogenes. (Some may have more than one
answer) Ki-ras

2534.

Pancreatic cancer

Name the cancer associated with the following


tumor markers. (Some may have more than one
answer.) CA-19.9 and CEA

2535.

Paneth cells

What cell of the duodenum contains high concentrations


of lysozymes and has phagocytic activity?

2536.

Pantothenic acid

What vitamin is a component of the enzymes fatty acid


synthase and acyl CoA?

2537.

Papillary carcinoma

What is the most common one? Thyroid cancer

2538.

Papillary carcinoma of the ovary

Orphan Annie cells

2539.

Papillary carcinoma of the thyroid

What is the most common one? Malignant thyroid


tumor

2540.

Papillary carcinoma of the thyroid

What thyroid carcinoma is associated with radiation


exposure, psammoma bodies, and Orphan Annie eye
nuclei?

2541.

Papovavirus

Name the DNA virus: Circular dsDNA; naked;


icosahedral; replicates in the nucleus

2542.

Papule

What is a palpable, elevated solid mass up to 0.5 cm?

2543.

Para-aminohippurate (PAH)

The clearance of what substance is the gold standard of


renal plasma flow?

2544.

Paracortex

What is the name of the T cell-rich area of the lymph


node?

2545.

Paradoxic emboli most commonly enter the


arteries through a patent septal defect in the
heart.

What is the term for a venous embolus in the arterial


system?

2546.

Parafollicular C cells

What cells of the thyroid gland secrete calcitonin?

2547.

The parafollicular cells of the thyroid (C cells)


release calcitonin in response to
hypercalcemia.

What cells of the thyroid gland are stimulated in


response to hypercalcemia?

2548.

Parainfluenza virus

Name the most common cause. Croup

2549.

Parallel circuit

In what type of circuit is the total resistance always less


than that of the individual resistors?

2550.

Paramesonephric ducts

What embryonic structure forms the adult female


structures? Uterine tube, uterus, cervix, and upper third
of the vagina

2551.

Paramyxovirus

What family do the following viruses belong to? RSV

2552.

Paramyxovirus

What family do the following viruses belong to? Measles

2553.

Paranoid

Name these cluster A personality disorders: Baseline


mistrust; carries grudges; afraid to open up; uses projection
as defense mechanism; lacks hallucinations or delusions

2554.

Paranoid schizophrenia

What subtype of schizophrenia is characterized by


Delusions of persecution and/or grandeur, auditory
hallucinations, late onset, and the best prognosis?

2555.

Parasites

Which of the following characteristics accurately describe


fungi, bacteria, viruses, or parasites? Eukaryotic cell, 15 to
25 microns, 80S ribosomes, no cell walls, replicates via
cytokinesis with mitosis and meiosis

2556.

Parasympathetic

What part of the ANS (i.e., PNS or CNS) controls the


constriction of the pupil in response to light?

2557.

Parasympathetics (parasympathetics
point, sympathetics shoot)

What component of the ANS is responsible for dilation of the


blood vessels in the erectile tissue of the penis, resulting in
an erection?

2558.

Parasympathetic stimulation, via the


vagus nerve, results in
bronchoconstriction, whereas
sympathetic stimulation results in
bronchodilation.

What component of the ANS, when stimulated, results in


bronchoconstriction?

2559.

Parental assignment and culture (not


biology)

What are the strongest determinants of gender identity?

2560.

Parietal cells (Remember, they secrete


HCl, too.)

What cell type of the body or fundus of the stomach secretes


IF?

2561.

Parietal lobe

Name the area of the cerebral cortex with the function


described: Intellectual processing of sensory information,
with the left (dominant) processing verbal information, the
right processing visual-spatial orientation

2562.

Parkinson disease

Lewy bodies

2563.

Parkinson's disease

Shuffling gait, cogwheel rigidity, masklike facies, pill-rolling tremor, and


bradykinesia describe what form of dementia?

2564.

Parkinson's disease (I can't


underestimate all of the
buzzwords in this question.
Remember it.)

What direct-pathway basal ganglia disease is described by masklike


facies, stooped posture, cogwheel rigidity, pill-rolling tremor at rest,
and a gait characterized by shuffling and chasing the center of
gravity?

2565.

Paroxysmal nocturnal
hemoglobinuria

What form of anemia is diagnosed with sucrose lysis test and Ham
test?

2566.

Parvovirus

Name the DNA virus: ssDNA; naked; icosahedral; replicates in the


nucleus

2567.

Parvovirus B 19

What viral infection in patients with sickle cell anemia results in


aplastic crisis?

2568.

Parvovirus B-19

What DNA viral disease is associated with aplastic crisis in patients


with sickle cell anemia?

2569.

Parvovirus (it is the only ssDNA


virus)

What ssDNA virus must make dsDNA before it makes mRNA?

2570.

Passive-aggressive

Setting up to be let down (it is unconscious; if conscious, you're just


rude)

2571.

Pasteurella, Brucella,
Legionella, and Francisella (all
of the-ellas)

Which four bacteria require cysteine for growth?

2572.

Pasteurella multocida

What organism is commonly associated with a cellulitis from an


animal bite?

2573.

Patau syndrome (trisomy 13)

What disease involves microcephaly, mental retardation, cleft lip or


palate, and dextrocardia?

2574.

A patent processus vaginalis

What remains patent in a hydrocele of the testis, allowing peritoneal


fluid to form into a cyst?

2575.

Patent urachus

What is the name for failure of the allantois to close, resulting in a


urachal fistula or sinus?

2576.

The pathologic problem with CO poisoning is that CO has 240


times as much affinity for Hgb molecule as does O2, reducing the
carrying capacity and shifting the curve to the left, making it difficult
to remove the CO molecule from Hgb.

Which way does the Hgb-O2


dissociation curve shift in patients
with CO poisoning?

2577.

The patient decides about the health care she does or does not get
even if it harms the fetus. This also means she can refuse blood
transfusions even if it harms the fetus.

What is the central issue


regarding the Roe vs. Wade
decision (legalization of abortion)?

2578.

Patients with cystic fibrosis have a mutation in the chloride channel


protein in the CFTR gene on chromosome 7.

What chromosome is mutant in


patients with cystic fibrosis?

2579.

Patients with unilateral cerebellar lesions fall toward the side of the
lesion.

Will a patient with a unilateral


lesion in the cerebellum fall
toward or away from the affected
side?

2580.

PCR

What test uses very small


amounts of DNA that can be
amplified and analyzed without
the use of Southern blotting or
cloning?

2581.

PCT

What is the longest and most


convoluted segment of the
nephron?

2582.

PDA

What is the most common one?


Cardiac anomaly in children

2583.

Pearson correlation

What type of correlation compares


two interval variables?

2584.

Peau d'orange

What is the term for inflamed,


thickened skin on the breast with
dimpling associated with cancer?

2585.

Pedophilia

What paraphilia is defined as Sexual urges toward children?

2586.

Pedophilia

What is the most common sexual assault?

2587.

Pelvic splanchnics (They all begin


with P.)

What splanchnic carries preganglionic parasympathetic fibers that


innervate the hindgut and the pelvic viscera?

2588.

Penicillamine, EDTA (calcium


disodium edetate), or dimercaprol

Name the antidote. Lead

2589.

Penicillin G (benzylpenicillin)

What form of penicillin is used in the treatment of life-threatening


illnesses?

2590.

Penicillin V

What form of penicillin is stable in acid environments?

2591.

Pentoxifylline

What hematologic agent is used in the treatment of intermittent


claudication because it decreases blood viscosity and increases
RBC membrane flexibility and stability?

2592.

Pergolide

What is the DOC for hyperprolactinemia?

2593.

Periaortic lymph nodes

What is the most common one? Lymph node affected in nonHodgkin lymphoma

2594.

Permanent

Name the type of regeneration (i.e., labile, stable, or permanent)


based on the following examples. Skeletal muscle

2595.

Permanent

Name the type of regeneration (i.e., labile, stable, or permanent)


based on the following examples. CNS neurons

2596.

Permanent

Name the type of regeneration (i.e., labile, stable, or permanent)


based on the following examples. Cardiac muscle

2597.

Pernicious anemia

What disorder is characterized by autoantibodies to IF?

2598.

Pernicious anemia (secondary to


a lack of vitamin B12 absorption)

What type of anemia is the result of a deficiency in intrinsic factor?

2599.

Peroxidase, which is also needed


for iodination and coupling inside
the follicular cell

What thyroid enzyme is needed for oxidation of I- to I'?

2600.

Persistent truncus arteriosus

What right-to-left shunt occurs when only


one vessel receives blood from both the
right and left ventricles?

2601.

Pertussis toxin

What toxin ADP-ribosylates via Gi to


increase cAMP?

2602.

Peutz-Jeghers syndrome

What AD syndrome produces


hamartomatous polyps in the small
intestine and pigmentation of the lips and
oral mucosa?

2603.

Peyer's patch

What lymphoid organ is characterized by


germinal centers, plasma cells that
secrete IgA, and no encapsulation?

2604.

PGD2, PGE2, and PGF 2

Name the product or products of


arachidonic acid: Vasodilation

2605.

PGE 1, PGI 2 (most potent), and TXA 2

What three PGs are potent platelet


aggregators?

2606.

PGE2

Name the product or products of


arachidonic acid: Pain and fever

2607.

PGE2 and PGF2

What two PGs are used to induce labor?

2608.

PGE (along with low oxygen tension)

What prostaglandin is associated with


maintaining patency of the ductus
arteriosus?

2609.

PGE and intrauterine or neonatal asphyxia maintain patency


of the ductus arteriosus. Indomethacin, ACh, and
catecholamines promote closure of the ductus arteriosus.

Which PG is associated with maintaining


a PDA?

2610.

PGI 2

Which PG maintains patency of the


ductus arteriosus and is used in the
treatment of primary pulmonary HTN?

2611.

PGI2

Name the product or products of arachidonic acid: Vasodilation and inhibition of


platelet aggregation produced by vascular endothelium

2612.

Phallic stage (46 years)

At what stage of psychosexual development (according to Freud) do children fear


castration?

2613.

Phallus

What embryonic structure forms the adult female structures? Glans clitoris, corpus
cavernosus, and spongiosum

2614.

Phallus

What embryonic structure forms the adult male structure? Corpus cavernosus,
corpus spongiosum, and glans and body of the penis

2615.

Phallus,
urogenital folds,
and labioscrotal
swellings

What embryonic structure forms the following adult structures? External genitalia

2616.

Phase 0

Name the phase of the ventricular muscle action potential based on the following
information: Fast channels open, then quickly close, and sodium influx results in
depolarization

2617.

Phase 1

Name the phase of the ventricular muscle action potential based on the following
information: Slight repolarization secondary to potassium and closure of the sodium
channels

2618.

Phase 1

Testing of a drug in a small group of volunteers without the disease is a component of


which phase of clinical drug testing?

2619.

Phase 2

Name the phase of the ventricular muscle action potential based on the following
information: Slow channels open, allowing calcium influx; voltage-gated potassium
channels closed; potassium efflux through ungated channels; plateau stage

2620.

Phase 3

Name the phase of the ventricular muscle action potential based on the following
information: Slow channels close, voltage-gated potassium channels reopen with a
large influx of potassium, and the cell quickly repolarizes

2621.

Phase 4

Which phase of clinical drug testing includes postmarket reporting of adverse


reactions?

2622.

Phase 4; ACh hyperpolarizes the cell via


increasing potassium conductance, taking
longer to reach threshold and slowing the
rate of firing.

In a ventricular pacemaker cell, what phase of the action


potential is affected by ACh?

2623.

Phase 4; NE increases the slope of the


prepotential, allowing threshold to be
reached sooner, and increases the rate of
firing.

In a ventricular pacemaker cell, what phase of the action


potential is affected by NE?

2624.

Phase I

What phase of Food and Drug Administration approval


tests The safety in healthy volunteers?

2625.

Phase I

Are hydrolysis, oxidation, and reduction phase I or II


biotransformations?

2626.

Phase II

What phase of Food and Drug Administration approval


tests The protocol and dose levels in a small group of
patient volunteers?

2627.

Phase III. It is considered the definitive test.

What phase of Food and Drug Administration approval


tests The efficacy and occurrence of side effects in
large group of patient volunteers?

2628.

Phentolamine and phenoxybenzamine

What mixed -antagonists are used for patients with


pheochromocytoma?

2629.

Phenylalanine hydroxylase

What enzyme is deficient in patients with PKU?

2630.

Phenylephrine

What 1-agonist, not inactivated by catechol-O-methyl


transferase (COMT), is used as a decongestant and for
treatment of paroxysmal atrial tachycardia?

2631.

Phenytoin

A patient with a history of epilepsy goes to the ER with


sedation, gait ataxia, diplopia, and vertical nystagmus.
Which anticonvulsant overdose resulted in the patient's
symptoms?

2632.

Phenytoin (cyclosporine and nifedipine also


result in gingival hyperplasia)

Which anticonvulsant causes gingival hyperplasia?

2633.

Pheochromocytoma

What catecholamine-hypersecreting tumor, a secondary cause of


HTN, results in headache, diaphoresis, anxiety, tachycardia, and
palpitations?

2634.

pH of 2 and 9

All AAs have titration plateaus at what pH values?

2635.

The pH of CSF is 7.33, acidotic.

Is the pH of CSF acidotic, alkalotic, or neutral?

2636.

Phosphofructokinase-1 and costs


1 ATP

What is the rate-limiting enzyme on glycolysis?

2637.

Phosphorylation of mannose
residues

What signals are used to direct an enzyme to a lysosome?

2638.

Phototherapy

What is the treatment for physiologic jaundice of newborns?

2639.

Physiologic antagonism

Are two different receptors with two different agonists characteristic


of pharmacologic or physiologic antagonism?

2640.

Physiologic dead space is the


total dead space of the respiratory
system.

Anatomical and alveolar dead spaces together constitute what


space?

2641.

Physostigmine

Name the antidote. Anticholinergics

2642.

Physostigmine

Name the antidote. Atropine

2643.

Pia mater

What protective covering adheres to the spinal cord and CNS


tissue?

2644.

PICA

What branch off the vertebral artery supplies The cerebellum and
the dorsolateral part of the medulla?

2645.

Pick's disease

What rare form of dementia is associated with personality changes


and affects the frontal and temporal lobes?

2646.

Picornaviridae

To what viral family does the polio virus belong?

2647.

pI (isoelectric point)

What is the term for the pH at which the structure carries no


charge?

2648.

Pimozide

What is the drug of choice for treating Tourette syndrome?

2649.

Pinpoint pupils, decreased


respiratory rate, and coma

What are the three signs of morphine overdose?

2650.

Pirenzepine

What drug is an M1-specific antispasmodic?

2651.

Pituitary adenoma

Name the most common type or cause. Cushing syndrome

2652.

Pituitary hypogonadism

What hormone disorder is characterized by the following


abnormalities in sex steroids, LH, and FSH? Sex
steroids , LH , FSH

2653.

pK (think of it as where half is base and half is


acid)

What is the term for the pH range where the


dissociation of H+ occurs?

2654.

PKU

What disorder is due to a deficiency in the enzyme


phenylalanine hydroxylase?

2655.

The placenta

From the fourth month of fetal life to term, what secretes


the progesterone and estrogen to maintains the uterus?

2656.

Placental alkaline phosphatase

What tumor marker is associated with seminomas?

2657.

Planning and fine-tuning of voluntary skeletal


muscle contractions. (Think coordination.)
Remember, the function of the basal ganglia
is to initiate gross voluntary skeletal muscle
control.

What is the function of the cerebellum?

2658.

Plasma cell

What type of cell can never leave the lymph node?

2659.

Plasma cell (mature B lymphocyte)

What is the name of the B cell that secretes Ig?

2660.

Plasma concentration and excretion rate

What two factors determine the clearance of a


substance?

2661.

Plasmin

What protein causes fibrinolysis?

2662.

Plasmodium falciparum

What type of Plasmodium affects RBCs of all ages?

2663.

Plasmodium falciparum

Name the Plasmodium species based on the following


information: No persistent liver stage or relapse; blood
smear shows multiple ring forms and crescent-shaped
gametes; irregular febrile pattern; associated with
cerebral malaria

2664.

Plasmodium malariae

What type of Plasmodium affects Only mature RBCs?

2665.

Plasmodium
malariae

Name the Plasmodium species based on the following information:No persistent liver
stage or relapse; blood smear shows rosette schizonts; 72-hour fever spike pattern

2666.

Plasmodium
malariae; the
others are
tertian.

What is the only Plasmodium that is quartan?

2667.

Plasmodium
ovale

Name the Plasmodium species based on the following information:Persistent


hypnozoite liver stage with relapses; blood smear shows amoeboid trophozoites with
oval, jagged infected RBCs; 48-hour fever spike pattern

2668.

Plasmodium
vivax

What type of Plasmodium affects Only reticulocytes?

2669.

Plasmodium
vivax

Name the Plasmodium species based on the following information: Persistent


hypnozoite liver stage with relapses; blood smear shows amoeboid trophozoites; 48hour fever spike pattern; the most prevalent form worldwide

2670.

PlummerVinson
syndrome

What syndrome is seen in iron-deficient middle-aged women with esophageal webs?

2671.

Pneumococcus

Name the most common type or cause. Meningitis in adults

2672.

Pneumococcus

Name the most common type or cause. Sinusitis and otitis media in children

2673.

Pneumococcus
(Streptococcus
pneumoniae)

Describe the organism based on the following information: Alpha-Hemolytic


Streptococcus; lysed by bile; sensitive to Optochin

2674.

Pneumocystis
carinii

Name the MCC. Pneumonia in an HIV-positive patient with CD4 less than 200

2675.

Pneumocystis
carinii

What obligate extracellular fungus is silver stain-positive and is associated with


pneumonia in patients with AIDS?

2676.

Pneumocystis
carinii

What is the most common infectious agent in HIV?

2677.

Pneumotaxic
center (short,
fast breaths)

What respiratory center in the rostral pons has an inhibitory affect on the apneustic
center?

2678.

Pneumothorax

What is the term for air in the pleural space?

2679.

Poliomyelitis; it is a bilateral
LMN lesion.

What is the name of bilateral flaccid paralysis, hyporeflexia, and


hypotonia due to a viral infection of the ventral horn of the spinal cord?

2680.

Polyarteritis nodosa

What vasculitis is characterized by systemic vasculitis in small to


medium- size vessels (except the lung); affecting young males; 30%
HBsAg-positive; P-ANCA and autoantibodies against myeloperoxidase?

2681.

The poly(A) site on the DNA

What causes transcription to stop in eukaryotes?

2682.

Polycystic kidney disease of


childhood

What bilateral AR disorder seen in infancy as progressive renal failure


has multiple small cysts at right angles to the cortical surface?

2683.

Polycystic ovaries

Name the most common type or cause. Anovulation

2684.

Polycystic ovary disease


(Stein-Leventhal syndrome)

What female genital tract disorder is characterized by obesity, hirsutism,


infertility, amenorrhea, elevated LH and testosterone levels, and low FSH
levels?

2685.

Polycythemia vera
(Remember, polycythemia
vera is a risk factor for acute
leukemias.)

What myeloid disorder is characterized by increased hematocrit, blood


viscosity, basophils, and eosinophils; intense pruritus; and gastric ulcers
due to histamine release from basophils, increased LAP, and plethora?

2686.

Polysome. Ribosomes read


from the 5' to the 3' end of
the mRNA.

A single mRNA strand translated by a ribosome is termed what?

2687.

Pompe's disease

What glycogen storage disease is due to the following enzyme


deficiencies? Lysosomal glucosidase (acid maltase)

2688.

Pons

Name the area of the cerebral cortex with the function described:
Important for REM sleep; origin of NE pathway

2689.

Pontocerebellar angle
syndrome

What syndrome is associated with the following brainstem lesions?


Slow-growing acoustic neuroma producing CN VII deficiencies

2690.

The popliteal artery, the deepest structure in the


popliteal fossa, risks injury in a supracondylar
fracture of the femur.

What vascular injury may result from a


supracondylar fracture of the femur?

2691.

Popliteus

What muscle laterally rotates the femur to unlock


the knee?

2692.

Porcelain gallbladder

What is the term for calcification of the


gallbladder seen on radiograph due to chronic
cholecystitis or adenocarcinoma of the
gallbladder?

2693.

Positive

Are the following conditions associated with a


negative or positive nitrogen balance?
Pregnancy

2694.

Positive

Are the following conditions associated with a


negative or positive nitrogen balance? Growth

2695.

Positive

Are the following conditions associated with a


negative or positive nitrogen balance?
Recovery from injury

2696.

A positive AV difference indicates that a substance is


extracted by the organ, and a negative difference
indicates that it is produced by the organ.

If the AV difference is positive, is the substance


extracted or produced by the organ?

2697.

Positive correlation

What type of correlation is defined as Two


variables that go together in the same direction?

2698.

Positive predictive value

The probability that a person with a positive test


result is truly positive refers to what value?

2699.

Positive reinforcement

Based on operant conditioning, what type of


reinforcement is described when Adding a
stimulus reinforces a behavior?

2700.

Positive sense RNA can be used as mRNA. Negative


sense RNA cannot be used as mRNA; it requires
special RNA-dependent RNA polymerases.

If a virus has positive sense RNA, can it be used


as mRNA or is a template needed?

2701.

Positive sense RNA means it can serve as mRNA


and therefore has 5'-3' polarity

What is the polarity (i.e., 5'-3' or 3'-5') of a


positive sense RNA?

2702.

Postductal coarctation of the aorta (adult)

What vascular pathology is associated with HTN


in the upper extremities, hypotension in the lower
extremities, and a radial-femoral delay?

2703.

Posterior

Name the compartment of the mediastinum


associated with the following thoracic structures:
Descending aorta

2704.

Posterior

Name the compartment of the mediastinum


associated with the following thoracic structures:
Splanchnic nerves

2705.

Posterior

Name the compartment of the mediastinum


associated with the following thoracic structures:
Vagus nerve

2706.

Posterior chamber

What chamber of the eye lies between the iris


and the lens?

2707.

Posterior compartment of the leg, tibial nerve

Name the compartment of the lower extremity


and the nerve based on its movements. Plantar
flex the foot, flex the toes, and invert the foot

2708.

Posterior compartment of the thigh, tibial nerve

Name the compartment of the lower extremity


and the nerve based on its movements. Extend
the hip and flex the knee

2709.

Posterior cricoarytenoid muscles

Name the laryngeal muscle described by the


following: Only muscle to abduct the vocal
cords

2710.

Posterior cruciate ligament

What structure of the knee is described thus? Prevents posterior


displacement and has medial-to-lateral attachment on the tibia

2711.

Posterior hypothalamic zones;


lesions here result in
poikilothermy (environmental
control of one's body
temperature).

What area of the hypothalamus is responsible for recognizing a


decrease in body temperature and mediates the response to
conserve heat?

2712.

Posterior interventricular artery

What artery travels with the following veins? Middle cardiac vein

2713.

Postinfectious GN

What type of GN occurs most commonly in children after a


pharyngeal or skin infection; is immune complex-mediated; and is
seen as lumpy-bumpy subepithelial deposits?

2714.

Postinfectious GN

Name the nephritic disease based on the immunofluorescent


staining. Granular deposits of IgG, IgM, and C3 throughout the
glomerulus

2715.

Poststreptococcal GN

Two weeks after her son has a throat infection, a mother takes the
boy to the ER because he has fever, malaise, HTN, dark urine, and
periorbital edema. What is your diagnosis?

2716.

Posttraumatic stress disorder.


(Important: symptoms must be
exhibited for longer than 1
month.)

What is the term for headaches, inability to concentrate, sleep


disturbances; avoidance of associated stimuli; reliving events as
dreams or flashbacks following a psychologically stressful event
beyond the normal range of expectation?

2717.

Potency

What is the term for the amount of a drug that is needed to produce
the desired effect?

2718.

Pott disease

What is the term for the collapse of the vertebral body due to TB?

2719.

Potter facies

What is the term for flattened nose, low-set ears, and recessed chin
seen in patients with bilateral renal agenesis?

2720.

Poxvirus

Name the DNA virus Linear dsDNA; enveloped; virion-associated


polymerases; replicates in the cytoplasm :

2721.

Poxvirus

What is the only DNA virus that is not icosahedral?

2722.

Poxvirus replicates its DNA in the cytoplasm.

What is the only DNA virus that does not replicate its
DNA in the nucleus of the host cell?

2723.

PPRF

What area of the brain acts as the center for


ipsilateral horizontal gaze?

2724.

Prader-Willi syndrome and Angelman


syndrome, respectively

What syndrome results when there is a deletion to


paternal chromosome 15? Maternal?

2725.

Preductal (infantile)

What form of coarctation of the aorta is associated


with Turner syndrome?

2726.

Prefrontal cortex; it is in front of the premotor


area.

What frontal lobe cortex is associated with organizing


and planning the intellectual and emotional aspect of
behavior?

2727.

Preganglionic sympathetic neurons

What neuronal cell bodies are contained in the


intermediate zone of the spinal cord? (T1-L2)

2728.

Pregnancy

What is the leading cause of school dropout?

2729.

Preload. It is the load on a muscle Prior to


contraction.

What is the term for the load on a muscle in the


relaxed state?

2730.

Preload (the load on the muscle in the relaxed


state)

What term is described as the prestretch on the


ventricular muscle at the end of diastole?

2731.

Premature ejaculation

What is the term for ejaculation before or immediately


after vaginal penetration on a regular basis?

2732.

Preoperational (2-6 years)

At what stage of cognitive development (according to


Piaget) do children Lack law of conservation and be
egocentric?

2733.

Preoptic area of the hypothalamus; if the lesion


occurs after puberty, amenorrhea or impotence
will be seen.

If a lesion occurs before the onset of puberty and


arrests sexual development, what area of the
hypothalamus is affected?

2734.

Prevalence increases.

What happens to prevalence as incidence increases?

2735.

Prevalence increases. (Note:


Incidence does not change.)

What happens to prevalence as duration increases?

2736.

Prevalence increases. (Remember,


prevalence can decrease in only
two ways, recovery and death.)

What happens to prevalence as the number of long-term survivors


increases?

2737.

Prevalence rate

What is the term for the number of individuals who have an


attribute or disease at a particular point in time?

2738.

Previous suicide attempt

What is the primary risk factor for suicide?

2739.

Priapism

What are the pharmacologic effects seen sexually with


Trazodone?

2740.

Primary biliary cirrhosis

What autoimmune liver disease is characterized by affecting a


middle-aged woman with jaundice, pruritus, fatigue, xanthomas,
increased direct bilirubin levels, and antimitochondrial Abs?

2741.

Primary erectile disorder

What is the term to describe a man who has Never been able to
achieve an erection?

2742.

Primary follicle (in the white pulp)

What is the name of the B cell-rich area of the spleen?

2743.

Primary follicle of the cortex

What is the name of the B cell-rich area in the lymph node?

2744.

Primary Hemochromatosis

What disease is associated with the HLA-A3 allele

2745.

Primary hyperaldosteronism (Conn


syndrome)

What disorder of aldosterone secretion is characterized by


Increased total body sodium, ECF volume, plasma volume, BP,
and pH; decreased potassium, renin and AT II activity; no edema?

2746.

Primary hypercortisolism

What pathophysiologic disorder is characterized by the following


changes in cortisol and ACTH? Cortisol increased, ACTH
decreased

2747.

Primary hyperparathyroidism

What pathology is associated with elevated levels of Ca2+, cardiac


arrhythmias, bone resorption, kidney stones, and metastatic
calcifications?

2748.

Primary hyperparathyroidism

What endocrine abnormality is characterized by


the following changes in PTH, Ca2+, and
inorganic phosphate (Pi)? PTH increased, Ca2+
increased, Pi decreased

2749.

Primary hypoaldosteronism (Addison's disease)

What disorder of aldosterone secretion is


characterized by Decreased total body sodium,
ECF volume, plasma volume, BP, and pH;
increased potassium, renin, and AT II activity; no
edema?

2750.

Primary hypocortisolism (Addison's disease)

What pathophysiologic disorder is characterized


by the following changes in cortisol and ACTH?
Cortisol decreased, ACTH increased

2751.

Primary hypogonadism (postmenopausal women)

What hormone disorder is characterized by the


following abnormalities in sex steroids, LH, and
FSH? Sex steroids , LH , FSH ?

2752.

Primary hypoparathyroidism

What endocrine abnormality is characterized by


the following changes in PTH, Ca2+, and
inorganic phosphate (Pi)? PTH decreased, Ca2+
decreased, Pi increased

2753.

Primary hypothyroidism (Low T4 has a decreased


negative feedback loop, resulting in both the
hypothalamus and the anterior pituitary gland to
increase TRH and TSH release, respectively.)

What thyroid abnormality has the following? TRH


increased, TSH increased, T4 decreased

2754.

Primary intention

What type of healing occurs in a clean surgical


incision?

2755.

Primary polydipsia; patients with diabetes insipidus


will continue to produce large volumes of dilute
urine.

In the water deprivation test, does a patient with


reduced urine flow have primary polydipsia or
diabetes insipidus?

2756.

Primary sclerosing cholangitis

What chronic liver disease has a beaded


appearance of the bile ducts on cholangiogram?

2757.

Primary spermatocyte

Match the chromosome and haploid number with the stage of sperm
development, spermatid, spermatocyte (primary or secondary),
spermatogonia (type A or B): 46/4n

2758.

Primase

What enzyme produces an RNA primer in the 5'-3' direction and is


essential to DNA replication because DNA polymerases are unable to
synthesize DNA without an RNA primer?

2759.

Primitive atrium

From what embryonic structure are the following structures derived?


The right and left atria

2760.

Primitive neuroectodermal
tumors (i.e., medulloblastomas
and retinoblastomas)

What highly undifferentiated aggressive CNS tumor of primordial


neuroglial origin develops in children and is associated with
pseudorosettes?

2761.

The primitive streak grows


caudal to rostral.

What is the direction of growth for the primitive streak, caudal to rostral
or rostral to caudal?

2762.

Primitive ventricle

From what embryonic structure are the following structures derived?


The right and left ventricles

2763.

PR interval

What component of an ECG is associated with the following?


Conduction delay in the AV node

2764.

Prinzmetal variant angina

What form of angina is characterized by Coronary artery vasospasm,


symptom occurrence at rest, ST segment elevation (during episode),
and no signs on ECG?

2765.

Probenecid, a uricosuric agent

What is the drug of choice in treating a patient with hyperuricemia due


to underexcretion of uric acid?

2766.

Procainamide

What class 1A antiarrhythmic agent has SLE-like syndrome side effect


in slow acetylators?

2767.

Processed antigenic peptides


bound in the groove of the
MHC molecule

What type of Ag do T cells recognize?

2768.

Prodrugs

What class of pharmaceuticals are inactive until they are metabolized


to their active products?

2769.

Production of HCO3-, shifting the


reaction to the left and thereby
decreasing H+

What is the renal compensation mechanism for acidosis?

2770.

Production of oxygen free radicals

From what do catalase, superoxide dismutase, and glutathione


peroxidase defend the cell?

2771.

Proencephalon

Name the primary vesicle the following structures are derived from
(proencephalon, mesencephalon, or rhombencephalon).
Telencephalon

2772.

Proencephalon

Name the primary vesicle the following structures are derived from
(proencephalon, mesencephalon, or rhombencephalon). Lateral
ventricles

2773.

Proencephalon

Name the primary vesicle the following structures are derived from
(proencephalon, mesencephalon, or rhombencephalon).
Thalamus

2774.

Proencephalon

Name the primary vesicle the following structures are derived from
(proencephalon, mesencephalon, or rhombencephalon). Third
ventricle

2775.

Proencephalon

Name the primary vesicle the following structures are derived from
(proencephalon, mesencephalon, or rhombencephalon).
Cerebral hemispheres

2776.

Proencephalon

Name the primary vesicle the following structures are derived from
(proencephalon, mesencephalon, or rhombencephalon).
Diencephalon

2777.

Proencephalon diencephalon
derivative

Name the primary vesicle the following structures are derived from
(proencephalon, mesencephalon, or rhombencephalon).
Neurohypophysis

2778.

Proencephalon diencephalon
derivative

Name the primary vesicle the following structures are derived from
(proencephalon, mesencephalon, or rhombencephalon). Pineal
gland

2779.

Proencephalon diencephalon
derivative

Name the primary vesicle the following structures are derived from
(proencephalon, mesencephalon, or rhombencephalon). Eye

2780.

Proencephalon diencephalon
derivative

Name the primary vesicle the following structures are derived


from (proencephalon, mesencephalon, or rhombencephalon).
Hypothalamus

2781.

Profound (I.Q. range < 20)

What level of mental retardation is characterized by Needing a


highly structured environment with constant supervision?

2782.

Progestin

Is hirsutism a side effect of estrogen or progestin therapy?

2783.

Progressive multifocal
leukoencephalopathy

What encephalitis is associated with the JC virus?

2784.

Projection

Name these narcissistic defense mechanisms: Person takes his


or her own feelings, beliefs, wishes, and so on and thinks they
are someone else's. (e.g., a cheating man thinks his wife is
unfaithful)

2785.

Projective drawing. The artistic form


is irrelevant, but the size, placement,
erasures, and distortions are
relevant.

What type of test asks a patient to draw a scene, attempting to


find out the individual's unconscious perceptions in his or her
life?

2786.

Projective test. Most tests with a


wide range of possibilities for the
answers are projective.

What type of personality test is the Rorschach inkblot test,


objective or projective?

2787.

Prokaryotes. Polycistronic and


prokaryotes both start with P.

Which organisms have polycistronic mRNA?

2788.

Prolactin

What hormone level increases in the first 3 hours of sleep?

2789.

Prolactinoma

What is the most common one? Pituitary adenoma

2790.

Proline and lysine

What two AAs require vitamin C for hydroxylation?

2791.

Prolyl and lysyl hydroxylases

What two post-transcriptional enzymes in collagen synthesis


require ascorbic acid to function properly?

2792.

The promoter indicates where


transcription will begin.

What is the binding site for RNA polymerase?

2793.

Promotes systemic venous return into the


chest and increases the caliber and volume
of the pulmonary vessels

What changes does more negative intrathoracic


pressure cause to systemic venous return and to the
pulmonary vessels?

2794.

Pro-opiomelanocortin (POMC) is cleaved


into ACTH and -lipotropin.

Corticotropin-releasing hormone promotes the synthesis


and release of what prohormone?

2795.

Prophase of meiosis I (between 12th and


22nd week in utero)

When is the first arrested stage of development in the


female reproductive cycle?

2796.

Propionibacterium acnes

What is the most common causative organism of acne


vulgaris?

2797.

Propionic acid

What is the only fatty acid that is gluconeogenic?

2798.

Propofol

Which IV agent has the lowest incidence of postoperative


emesis and has the fastest rate of recovery?

2799.

Proportionate mortality rate

What type of mortality rate is defined as the number of


deaths From a specific cause per all deaths?

2800.

Propranolol

What -blocker is also a membrane stabilizer?

2801.

Prostate and lung cancer, respectively

What are the two leading causes of cancer-related death


in men?

2802.

Prostate cancer

Name the cancer associated with the following tumor


markers. (Some may have more than one answer.) PSA
and prostatic acid phosphatase

2803.

Prostatic carcinoma

What adenocarcinoma presents with elevated levels of


acid phosphatase, dihydrotestosterone, PSA, and bone
pain?

2804.

Prostatic carcinoma

What cancer of the male genitourinary system is


associated with osteoblastic bony metastasis?

2805.

Protamine sulfate

What agent antagonizes the effects of heparin?

2806.

Protamine sulfate

Name the antidote. Heparin

2807.

Protease, integrase, and reverse


transcriptase

Name three products of HIV's pol gene.

2808.

Protein A

What Staphylococcus aureus protein inhibits phagocytosis?

2809.

Proteus

What urease-positive non-lactose-fermenting gram-negative rod with swarmingtype motility is associated with staghorn renal calculi?

2810.

Proton pump
inhibitors

What class of drugs are the DOCs for peptic ulcer disease, Zollinger-Ellison
syndrome, and gastroesophageal reflux disease?

2811.

Proto-oncogenes

What is the term for normal cellular genes associated with growth and
differentiation?

2812.

Proud flesh

What is the term for excessive amounts of granulation tissue that can block reepithelialization and wound healing?

2813.

Proximal to distal
progression

In regard to motor development during infancy, choose the motor response that
happens first. Proximal or distal progression

2814.

Proximal tubule

What is the site of action for carbonic anhydrase inhibitors?

2815.

PRPP
aminotransferase

What enzyme of purine synthesis is inhibited by allopurinol and 6mercaptopurine?

2816.

PRPP
aminotransferase

What enzyme catalyzes the rate-limiting step in purine synthesis?

2817.

Pseudogout

What pathology is associated with deposition of calcium pyrophosphate in


patients older than 50 years?

2818.

Pseudogout

What form of arthritis is associated with calcium pyrophosphate crystals?

2819.

Pseudohyphae

What is the term for hyphae with constrictions at each septum that are commonly
seen in Candida albicans?

2820.

Pseudomembranous
colitis

What GI pathology can be caused by a patient taking clindamycin or lincomycin


or by Clostridium difficile, ischemia, Staphylococcus, Shigella, or Candida
infection?

2821.

Pseudomembranous
exudates

Name the type of exudate, given the following examples. Diphtheria infection

2822.

Pseudomonas

What are the most common causes of osteomyelitis In drug addicts?

2823.

Pseudomonas
aeruginosa

Name the most common type or cause. Infection in a patient on a ventilator

2824.

Pseudomonas aeruginosa

What is the most likely causative organism for a patient


with folliculitis after spending time in a hot tub?

2825.

Pseudomonas aeruginosa

What bacterium is a gram-negative, oxidase-positive


aerobic rod that produces a grapelike odor and pyocyanin
pigmentation?

2826.

Pseudomonas aeruginosa

Which bacteria are associated with the following pigment


production? Pyocyanin (blue-green)

2827.

Pseudomonas sp.

Name the MCC. Pneumonia in a neutropenic burn


patient

2828.

Psoas major

What muscle is the chief flexor of the hip?

2829.

Psoriasis

What diagnosis ensues from finding well-demarcated


erythematous plaques with silvery scales on the knees,
elbows, and scalp along with nail bed pitting and
discoloration?

2830.

Psoriasis, ankylosing spondylitis,


inflammatory bowel disease, and Reiter's
syndrome

What diseases are associated with the HLA-B27 allele

2831.

Pterygopalatine ganglion (I remember this


as the only ganglion left.)

What is the name of the postganglionic parasympathetic


ganglion that innervates The lacrimal gland and oral and
nasal mucosa?

2832.

PT for extrinsic and PTT for intrinsic


(remember: wPeT and hPiTT, which means
warfarin, extrinsic, PT; heparin, intrinsic,
PTT)

Does PT or PTT test the extrinsic coagulation pathway?

2833.

PTH

What hormone is characterized by the following renal


effects? Calcium reabsorption, phosphate excretion

2834.

PTH

What hormone is required for 1, 25-dihydroxy-vitamin D (1,


25-diOH-D) to have bone resorbing effects?

2835.

PTH increases Ca2+ reabsorption in the DCT of


the kidney and decreases PO4- reabsorption in
the PCT.

What are effects of PTH in the kidney?

2836.

Ptosis (eyelid drooping), miosis (pupillary


constriction), and anhydrosis (lack of sweating)
occur when the preganglionic sympathetic fibers
from T1-to T4 are obstructed.

What is the triad of Horner's syndrome?

2837.

Puberty

What stage of male development is characterized by


the following LH and testosterone levels? LH
pulsatile amplitude and levels increase, with
increased testosterone production.

2838.

Pubis, ilium, and ischium

What bones make up the acetabulum?

2839.

Public speaking (Remember: stage fright)

Name the most common type. Specific phobia

2840.

Puborectalis

What component of the pelvic diaphragm forms the


rectal sling (muscle of continence)?

2841.

Pulmonary alveolar macrophages

Name the macrophage based on its location: Lung


macrophages

2842.

Pulmonary embolism due to a DVT; this is not


absolute but a classic description.

After traveling in a plane across the Atlantic Ocean,


an obese male goes to the ER with swollen right leg
and sudden onset of shortness of breath. What do
you immediately diagnose?

2843.

Pulmonary infections

Name the MCC of death. In patients with cystic


fibrosis

2844.

Pulmonary shunt (i.e., pulmonary embolism)

What does failure of PaO2 to increase with


supplemental O2 indicate?

2845.

The pulmonary vein, which carries oxygenated


blood from the lung to the left atrium.

What is the only vein in the body with a high O2


content?

2846.

The pulsatile release of GnRH

What prevents the down-regulation of the receptors


on the gonadotrophs of the anterior pituitary gland?

2847.

Pulse pressure

What is the term for the difference between systolic and diastolic
pressures?

2848.

Punishment

Based on operant conditioning, what type of reinforcement is described


when Adding a stimulus stops a behavior?

2849.

Purine nucleoside
phosphorylase

What enzyme is deficient in selective T cell immunodeficiency?

2850.

Purine nucleoside
phosphorylase

What enzyme of the purine salvage pathway is deficient in the


following? Selective T-cell immunodeficiency

2851.

The Purkinje cell

What cell's axons are the only ones that leave the cerebellar cortex?

2852.

Purkinje cell is the fastest,


and the AV node is the
slowest.

What is the fastest-conducting fiber of the heart? Slowest conduction


fiber in the heart?

2853.

Purulent exudates

Name the type of exudate, given the following examples.


Meningococcal infection

2854.

P wave

What component of an ECG is associated with the following? Atrial


depolarization

2855.

Pyelonephritis

What is the term for ascending bacterial infection of the renal pelvis,
tubules, and interstitium causing costovertebral angle tenderness, fever,
chills, dysuria, frequency, and urgency?

2856.

Pyelonephritis

In what condition do you see dimpling on the kidney's surface?

2857.

Pygmalion effect
(experimenter expectancy).
This can be eliminated with
double-blind studies.

What form of bias occurs when the experimenter's expectation


inadvertently is expressed to the subjects, producing the desired
effects? How can it be eliminated?

2858.

Pyloric stenosis

A mother takes her 2-week-old infant to the ER because the baby


regurgitates and vomits after eating and has peristaltic waves visible on
the abdomen and a palpable mass in the right upper quadrant. What is
your diagnosis?

2859.

Pyrantel pamoate

What drug of choice for ascaris causes neuromuscular blockade of the


worm?

2860.

Pyridoxal phosphate is derived from vitamin B6


and is needed to transfer the amino groups of one
carbon skeleton to another.

What vitamin is necessary for the transfer of one


amino group from a carbon skeleton to another?

2861.

Pyridoxine

Name the antidote. Isoniazid

2862.

Pyridoxine (B6)

What vitamin is needed for the production of heme?

2863.

Pyridoxine (B6) deficiency

What vitamin deficiency causes a glove-andstocking neuropathy seen in alcoholics?

2864.

Pyruvate, acetyl CoA, and propionyl CoA


carboxylase

What are the three carboxylase enzymes that


require biotin?

2865.

Pyruvate carboxylase

What gluconeogenic mitochondrial enzyme requires


biotin?

2866.

QRS complex

What component of an ECG is associated with the


following? Ventricular depolarization

2867.

Quaternary structure

What structure of a protein describes the interaction


among subunits?

2868.

Quinolones

Name the antimicrobial agent whose major side


effect is listed. Cartilage abnormalities

2869.

Rabies

What negative sense RNA virus is associated with


intracytoplasmic inclusion bodies called Negri
bodies?

2870.

Radial Immuno Diffusion (RID) for Ig levels.

What is the most common precipitin test used in


clinical medicine?

2871.

Radial nerve

What nerve is associated with the following


functions? Supinate the wrist, extend the wrist and
digits, extend the shoulder and elbow

2872.

Radial nerve and the profunda brachii artery

What nerve and artery could be affected in a


midshaft humeral fracture?

2873.

Radius with scaphoid and lunate and ulna with


triquetrum and pisiform (Remember, for major
articulations, wrist/radius and humerus/ulna = elbow)

Name the bony articulations of the following


sites. Be specific. Wrist

2874.

Raloxifene

What estrogen receptor agonist in bone is


used in the treatment and prevention of
osteoporosis?

2875.

Range

What is the term for the difference between


the highest and the lowest score in a
population?

2876.

The raphe nuclei

What reticular nuclei synthesize serotonin


from L-tryptophan and plays a role in mood,
aggression, and inducing sleep?

2877.

Rapid cycling bipolar disorder

What is the name of depression and mania


alternating within a 48-to 72-hour period?

2878.

Rapid efflux of potassium

What triggers phase 3 of the action potential


in a ventricular pacemaker cell?

2879.

Rapidly progressive glomerulonephritis (RPGN)

What form of GN is characteristically


associated with crescent formation?

2880.

The rate of infusion

What determines the plasma level at steady


state?

2881.

The rate of -oxidation

What regulates the rate of ketone body


formation?

2882.

Rathke's pouch; they can result in compression of the


optic chiasm.

Craniopharyngiomas are remnants of what?

2883.

Rationalization

Name these anxiety defense mechanisms:


Use of explanations to justify unacceptable
behaviors.

2884.

Ratio scale

What scale has a true zero point, graded into


equal increments, and also orders them?

2885.

RBCs. That is why it is called red pulp.

Which major cell type is found in the red pulp of the


spleen?

2886.

Reaction formation

A complete opposite expression of your inward feeling


(e.g., arguing all the time with someone you are attracted
to when your feelings are not known)

2887.

Recall bias. These problems arise in


retrospective studies.

Failure to accurately recall the past leads to what form of


bias?

2888.

Receptive aphasia is due to a lesion in


Brodmann areas 22, 39, and 40; generally
the patient is unaware of the deficit.

What aphasia is seen as an inability to comprehend


spoken language and speaking in a word salad?

2889.

Recovery and death

What are the two ways to leave the prevalence pot?

2890.

The rectouterine pouch (of Douglas)

What is the most common one? Site for an ectopic


abdominal pregnancy

2891.

Rectum

What is the most common one? Site of ulcerative colitis

2892.

Red blood cell distribution width index (RDW)

What is the name for the following RBC indices? The


coefficient of variation of the RBC volume

2893.

Red muscle

What muscle type is characterized by low ATPase


activity, aerobic metabolism, myoglobin, association with
endurance, and small muscle mass?

2894.

Red pulp (Remember, Red pulp and RBCs


begin with R.)

What area of the spleen consists of splenic cords of


Billroth and phagocytoses RBCs?

2895.

The reduviid bug

What is the vector for Chagas disease?

2896.

Regardless of the rate of infusion, it takes 4


to 5 half-lives to reach steady state.

If you double the infusion rate of a drug, how long will it


take to reach steady state?

2897.

Regression

Name these immature defense mechanisms: Returning


to an earlier stage of development (e.g., enuresis)

2898.

Regular insulin

Name the insulin preparation based on the peak effect


and duration of action. Peak, 0.5 to 3 hours; duration, 5
to 7 hours

2899.

Reinforcement

What is the term for any stimulus that


increases the probability of a response
happening?

2900.

Reiter syndrome

What seronegative spondyloarthropathy is


seen in HLA-B27-positive young females
and presents with the triad of conjunctivitis,
urethritis, and arthritis affecting the knees
and ankles?

2901.

Reject it

If the P value is less than or equal to .05,


what do you do to the null hypothesis?

2902.

Relative refractory period

What period is described when a largerthan-normal stimulus is needed to produce


an action potential?

2903.

Relative risk and/or attributable risk. (Cohort studies deal


with incidence.)

What statistical method do you use when


analyzing Cohort studies?

2904.

Relaxation of the diaphragm increases the intrapleural


pressure (becomes more positive).

What happens to the intrapleural pressure


when the diaphragm relaxes?

2905.

Reliability (think of it as "nice grouping" or "precise")

What is the term for the ability of a test to


measure something consistently?

2906.

REM

Name the stages of sleep with these EEG


patterns: Sawtooth waves, random low
voltage pattern

2907.

REM and stage 4 sleep; they decrease.

What aspects of sleep are affected during


benzodiazepine use?

2908.

Remember the mnemonic PAM SMIDT P, (hyper)


Parathyroid/ Paget disease A, Addison's disease M, Milkalkali syndrome/metastatic cancer S, Sarcoidosis M,
Multiple myeloma I, Immobilization/idiopathic D, Vitamin D
intoxication T, Tumors

Name at least three causes of metastatic


calcification.

2909.

REM latency; normally it is about 90 minutes.

What is the period between falling asleep


and REM sleep called?

2910.

The removal of Na+ results in the renal tubule becoming


negatively charged. The negative luminal charge attracts
both K+ and H+ into the renal tubule and promotes
HCO3- to enter the ECF and results in hypokalemic
alkalosis.

By removing Na+ from the renal tubule and


pumping it back into the ECF compartment,
what does aldosterone do to the body's
acid-base stores?

2911.

REM sleep. Nightmares are frightening dreams that we


recall.

With what stage of sleep are nightmares


associated?

2912.

REM sleep occurs more often in the second half of sleep.


The amount of REM sleep increases as the night goes on.

When does most REM sleep occur, in the


first or second half of sleep?

2913.

REM sleep. Remember, awake brain in a sleeping body.

Aroused EEG pattern (fast low voltage and


desynchronization), saccadic eye
movements, ability to dream, and sexual
arousal are all associated with what general
pattern of sleep?

2914.

Renal cell carcinoma

Name the most common type or cause.


Painless hematuria

2915.

Renal circulation

Where is the greatest venous PO2 in


resting tissue?

2916.

The renal circulation has the smallest AV O2 (high venous


PO2) difference in the body because of the overperfusion
of the kidneys resulting from filtration.

What organ of the body has the smallest AV


oxygen difference?

2917.

Renal failure

Name the MCC of death. In SLE

2918.

Renal plasma flow (decrease flow, increase FF)

What is the main factor determining FF?

2919.

Renin

What substance do the JG cells of the


kidney secrete in response to low blood
pressure?

2920.

Renshaw neuron

What is the term for an inhibitory


interneuron?

2921.

Reovirus

What is the only dsRNA virus?

2922.

Repaglinide

What oral hypoglycemic agent stimulates


pancreatic -cell release of insulin and is given
just prior to meals because of its short halflife?

2923.

Repolarization is from base to apex and from


epicardium to endocardium.

How does ventricular repolarization take place,


base to apex or vice versa?

2924.

Repression

Unconsciously forgetting(forgetting that you


forgot something!)

2925.

Reserpine

What drug blocks intragranular uptake of NE?

2926.

Residual volume

What is the term for the amount of blood in the


ventricle after maximal contraction?

2927.

Residual volume (RV)

Name the lung measurement based on the


following descriptions: Amount of air in the
lungs after maximal expiration

2928.

Resistance and vessel length are proportionally


related. The greater the length of the vessel, the
greater the resistance is on the vessel.

How are resistance and length related


regarding flow?

2929.

Resistance decreases as resistors are added in


parallel.

In a parallel circuit, what happens to


resistance when a resistor is added in parallel

2930.

The resistance increases 16-fold.

If the radius of a vessel is decreased by half,


what happens to the resistance?

2931.

Resistance increases as resistors are added to the


circuit.

When referring to a series circuit, what


happens to resistance when a resistor is
added?

2932.

Resistance of the system increases. (Remember, when


resistors are connected in a series, the total of the
resistance is the sum of the individual resistances.)

What happens to the resistance of the system


when a resistor is added in a series?

2933.

The resistance will decrease onesixteenth of the original resistance.

If the radius of a vessel doubles, what happens to resistance?

2934.

Resorb CSF into the blood

What is the function of the arachnoid granulations?

2935.

Respiratory acidosis (summary:


high CO2, high H+, slightly high
HCO3-)

What primary acid-base disturbance is cause by a decrease in


alveolar ventilation (increasing CO2 levels) resulting in the
reaction shifting to the right and increasing H+ and HCO3- levels?

2936.

Respiratory alkalosis (summary: low


CO2, low H+, slightly low HCO3-)

What primary acid-base disturbance is caused by an increase in


alveolar ventilation (decreasing CO2 levels) resulting in the
reaction shifting to the left and decreasing both H+ and HCO3levels?

2937.

Respiratory depression

What is the major pulmonary side effect of -activators?

2938.

Respiratory distress syndrome;


treatment with cortisol and
thyroxine can increase production
of surfactant.

What syndrome is due to a deficiency of surfactant?

2939.

RES(Reticular endothelial system)

Where in the body is heme converted to bilirubin?

2940.

The resting membrane potential of


the cell is -90 mV because of the
intracellular proteins.

Why is the cell's resting membrane potential negative?

2941.

Reticulocyte

What is the term for a large, immature RBC that is spherical, blue,
and without a nucleus?

2942.

Retinoblastoma

What is the most common one? Eye tumor in children

2943.

Reverse transcriptase

What is the term for RNA-dependent DNA polymerase?

2944.

Reverse transcriptase

What HIV enzyme produces a dsDNA provirus?

2945.

Reverse transcription

What is the term for production of a DNA copy from an RNA


molecule?

2946.

Reye syndrome

What potentially fatal disease occurs in children who are given


aspirin during a viral illness?

2947.

Rhabdomyoma

What benign cardiac tumor is associated with tuberous


sclerosis?

2948.

Rhabdomyoma

What is the most common one? Cardiac tumor of infancy

2949.

Rhabdovirus

What is the name of the bullet-shaped virus?

2950.

Rhabdovirus

What family do the following viruses belong to? Rabies

2951.

Rheumatic fever

Aschoff bodies

2952.

Rheumatoid arthritis

What is the most common one? Inflammatory arthritis

2953.

Rheumatoid arthritis

What chronic systemic inflammatory disease commonly seen


in women aged 20 to 50 is a progressive, symmetric arthritis
affecting the hands, wrists, knees, and ankles that improves
with increased activity?

2954.

Rheumatoid arthritis and type 1


diabetes (with HLA-DR3)

What diseases are associated with HLA-DR4

2955.

Rhinovirus

Name the most common cause. A cold in the spring and fall

2956.

Rhinovirus and hepatitis A virus

What are the only two picornaviruses that do not lead to


aseptic meningitis?

2957.

Rhinoviruses for summer and fall;


coronaviruses for winter and spring

Which virus is the MCC of a cold in the summer and fall?


Winter and spring?

2958.

Rho (D) immune globulin (RhoGAM),


an anti-RhD IgG antibody, prevents
generation of RhD-specific memory B
cells in the mother.

What is given to pregnant women within 24 hours after birth to


eliminate Rh+ fetal blood cells from their circulation?

2959.

Rhombencephalon

Name the primary vesicle the following structures are derived


from (proencephalon, mesencephalon, or rhombencephalon).
Pons

2960.

Rhombencephalon

Name the primary vesicle the following structures are derived


from (proencephalon, mesencephalon, or rhombencephalon).
Myelencephalon

2961.

Rhombencephalon

Name the primary vesicle the following structures are


derived from (proencephalon, mesencephalon, or
rhombencephalon). Medulla

2962.

Rhombencephalon

Name the primary vesicle the following structures are


derived from (proencephalon, mesencephalon, or
rhombencephalon). Metencephalon

2963.

Rhombencephalon

Name the primary vesicle the following structures are


derived from (proencephalon, mesencephalon, or
rhombencephalon). Cerebellum

2964.

Rib 7 articulates with T7 and T8. Each rib


articulates with the corresponding numerical
vertebral body and the vertebral body below it.

With what thoracic vertebra or vertebrae does rib 7


articulate?

2965.

Ribavirin

What antiviral agent is used to treat RSV, influenza A


and B, Lassa fever, and hantavirus and as an adjunct to
IFN-in hepatitis C?

2966.

Riboflavin (B2)

What water-soluble-vitamin deficiency is associated


with cheilosis and magenta tongue?

2967.

Riboflavin (B2) deficiency

What vitamin deficiency produces angular stomatitis,


glossitis, and cheilosis?

2968.

Ribonucleotide reductase

What enzyme is blocked by hydroxyurea?

2969.

Ribonucleotide reductase

What enzyme of pyrimidine synthesis is inhibited by the


following? Hydroxyurea

2970.

Ribozymes

Name the RNA subtype based on the following: RNA


molecules with enzymatic activity

2971.

Rickets

What childhood pathology involves anterior bowing of


the tibia, epiphyseal enlargements, and costochondral
widening, with the endochondral bones being affected?

2972.

Rickets prior to fusion, osteomalacia if the


deficiency occurs after epiphyseal fusion.

What is the term for vitamin D deficiency prior to epiphyseal


fusion?

2973.

Riedel thyroiditis

What rare disorder presents as a large, hard, irregular


thyroid gland due to fibrous proliferation of connective tissue
in the thyroid gland and extends to adjacent structures?

2974.

Rifampin

What is the drug of choice for asymptomatic meningitis


carriers?

2975.

Rifampin

What antitubercular agent causes a red-orange tinge to


tears and urine?

2976.

The right and left pulmonary arteries, the


only arteries that carry deoxygenated
blood

What vessels carry deoxygenated blood into the lungs from


the right ventricle?

2977.

Right atrium

What part of the heart forms The right border?

2978.

Right brachiocephalic artery

Name the correct artery. The right recurrent laryngeal


nerve passes around it.

2979.

A right CN V lesion results in weakened


muscles of mastication, and the jaw
deviates to the right.

A motor lesion to the right CN V results in deviation of the


jaw to which side?

2980.

Right coronary artery

What artery supplies the right atrium, right ventricle,


sinoatrial and atrioventricular nodes?

2981.

Right coronary artery

What artery travels with the following veins? Small cardiac


vein

2982.

The right gonadal vein drains into the


inferior vena cava directly, and the left
gonadal vein drains into the left renal vein.

What vessel does the right gonadal vein drain into?

2983.

The right kidney is lower in the abdominal


cavity because of the amount of space the
liver occupies.

Which kidney is lower? Why?

2984.

Right medial longitudinal


fasciculus

Where is the lesion that produces these symptoms when a patient is


asked to look to the left? Right eye can't look left, left eye
nystagmus, and convergence is intact

2985.

Right nasal hemianopsia

Name the ocular lesion; be specific. A right lateral compression of


the optic chiasm (as in aneurysms in the internal carotid artery)

2986.

Right-sided

Is jugular venous distention a presentation of isolated left or right


heart failure?

2987.

Right vagus innervates the SA


node and the left vagus
innervates the AV node

Does the left or right vagus nerve innervate the SA node?

2988.

Right ventricle

What part of the heart forms Anterior wall?

2989.

Rigors, leukopenia, decrease in


blood pressure, and increase in
temperature

What is the tetrad of Jarisch-Herxheimer reaction?

2990.

Risperidone

What neuroleptic agent is also considered to be an antihistamine?

2991.

Ristocetin

What substance is used to test platelets' response in patients with


von Willebrand disease?

2992.

Ritodrine and terbutaline

What two 2-agonists cause myometrial relaxation?

2993.

RNA polymerase I

Name the eukaryotic RNA polymerase based on the following:


Synthesizes 28S, 18S and 5.8S rRNAs

2994.

RNA polymerase II

Name the eukaryotic RNA polymerase based on the following:


Synthesizes hnRNA, mRNA, and snRNA

2995.

RNA polymerase III

Name the eukaryotic RNA polymerase based on the following:


Synthesizes tRNA, snRNA, and the 5S rRNA

2996.

Rochalimaea quintana (now


called Bartonella quintana)

Which organism causes trench fever?

2997.

Rods

What cells of the retina see in black and white and are used for night
vision?

2998.

The Romberg sign is present if the patient sways or loses


balance when standing with eyes open. In a dorsal column
lesion, patients sway with eyes closed. (Don't forget this one.)

What bedside test is used to


differentiate a dorsal column lesion
from a lesion in the vermis of the
cerebral cortex?

2999.

Rotavirus

Name the most common type or


cause. Infantile diarrhea

3000.

Rotavirus

Name the most common type or


cause. Diarrhea in children

3001.

Rotavirus

What organism is associated with the


following types of diarrhea? Day careassociated diarrhea in infants

3002.

Roth spots

Name the following descriptions


associated with bacterial endocarditis:
Retinal emboli

3003.

Roth spots, and they are seen in bacterial endocarditis.

What is the term for white retinal spots


surrounded by hemorrhage? In what
condition are they seen?

3004.

Round and ovarian ligaments

What two ligaments of the uterus are


remnants of the gubernaculum?

3005.

rRNA

Name the RNA subtype based on the


following: The most abundant form of
RNA in the cell

3006.

RSV

Name the most common type or


cause. Bronchiolitis in children

3007.

RSV

Name the most common cause.


Hospitalization in children younger
than 1 year of age

3008.

RSV

Which virus is treated with the monoclonal antibody


palivizumab?

3009.

RSV

Name the most common type or cause. Viral pneumonia


leading to death

3010.

RU 486

What drug, if given during pregnancy, would cause the


uterus to exhibit signs of progesterone withdrawal and induce
an abortion?

3011.

Rubor (red), dolor (pain), calor (heat),


tumor (swelling); also sometimes there is
loss of function

What are the four signs of acute inflammation?

3012.

S1 nuclease

What enzyme is used to remove the hairpin loop during


production of cDNA from mRNA?

3013.

S2, S3, S4keeps the pee-pee off the


floor!

What spinal nerves contribute to the pelvic splanchnic


(parasympathetic) nerves that innervate the detrusor muscle
of the urinary bladder?

3014.

Saccule and utricle

What part of the inner ear contains the gravity receptors for
changes in the position of the head?

3015.

Sacrococcygeal teratoma

What tumor is derived from primitive streak remnants and


often contains bone, hair, or other tissue types?

3016.

Sad but true. Some women can have


multiple successive orgasms.

True or false? Only men have refractory sexual periods.

3017.

Sadism

What paraphilia is defined as Deriving sexual pleasure from


other peoples' pain?

3018.

Salmeterol

What 2-agonist is used as a prophylactic agent in the


treatment of asthma?

3019.

Salmonella

What Enterobacteriaceae are prone to produce osteomyelitis


in sickle cell patients?

3020.

Salmonella typhi

What Vi-encapsulated gram-negative motile anaerobic rod


that produces H2S is associated with enteric fever,
gastroenteritis, and septicemia?

3021.

Salpingitis

What is the term for pelvic inflammatory disease of the fallopian


tubes?

3022.

Saltatory conduction

What term describes how an action potential is propagated along an


axon?

3023.

The sandfly

What is the vector for Leishmania infections?

3024.

SA node; it is the reason it is


the primary pacemaker of the
heart.

What cell in the heart has the highest rate of automaticity?

3025.

Sarcoidosis

A 25-year-old black woman presents with nonproductive cough,


shortness of breath, fatigue, and malaise; she has bilateral hilar
lymphadenopathy on chest radiography and elevated ACE levels.
What do you diagnose?

3026.

Sarcoplasmic reticulum

What is the name of the SER of striated muscle?

3027.

The scaphoid is the most


commonly fractured and the
lunate is the most commonly
dislocated.

What carpal bone is most commonly fractured? Dislocated?

3028.

Scavenger receptor (SR-B1)

Via what cell surface receptor does HDL cholesterol from the periphery
enter hepatoceles?

3029.

Schatzki ring

What ring is a weblike narrowing of the gastroesophageal junction?

3030.

Schedule dependent;
antineoplastic agents that work
on nonproliferating cells are
dose dependent

Are antineoplastic agents that work on actively proliferating cells


schedule or dose dependent?

3031.

Schistocytes

What is the term for RBC fragments?

3032.

Schistosoma haematobium

What parasite is associated with squamous cell carcinoma of the


urinary bladder?

3033.

Schistosoma haematobium

Which trematode is associated with bladder carcinoma in Egypt and


Africa?

3034.

Schistosoma have separate


males and females.

What is the only trematode that is not hermaphroditic?

3035.

Schizoid

Name these cluster A personality disorders: Socially withdrawn, seen


as eccentric but happy to be alone

3036.

Schizotypal

Name these cluster A personality disorders: Odd, strange; has magical


thinking; socially isolated, paranoid, lacks close friends; has incongruous
affect

3037.

School phobia

To what does failure to resolve separation anxiety lead?

3038.

Schwannoma

What CNS tumor commonly produces tinnitus and hearing loss?

3039.

Sciatic nerve

What lower extremity nerve is described by the following motor loss?


Loss of flexion of the knee and all function below the knee, weakened
extension of the thigh

3040.

Scleroderma

What pathology involves excessive fibrosis throughout the body via


increased fibroblast activity, occurs in women more than men, and is
most commonly seen in the third to the fifth decade?

3041.

Scleroderma

What disease is diagnosed by findings of ANAs and anti-SCL-70


antibodies?

3042.

Scrofula

What is the term for TB with the cervical lymph node involved?

3043.

Scrotal cancer, due to the


high exposure to polycyclic
aromatic hydrocarbons

What cancer is particularly likely to affect English chimney sweeps?

3044.

Seasonal affective disorder


(treat with bright light
therapy)

What form of depression is due to abnormal metabolism of melatonin?

3045.

Secondary erectile disorder


(Male erectile disorder is the
same as impotence.)

What is the term to describe a man who has Used to be able to achieve
an erection but now cannot?

3046.

Secondary hypercortisolism
(pituitary)

What pathophysiologic disorder is characterized by the following


changes in cortisol and ACTH? Cortisol increased, ACTH increased

3047.

Secondary
hyperparathyroidism (vitamin
D deficiency, renal disease)

What endocrine abnormality is characterized by the following changes in


PTH, Ca2+, and inorganic phosphate (Pi)? PTH increased, Ca2+
decreased, Pi decreased

3048.

Secondary hyperthyroidism (Increased TSH results


in increased T4 production and increased negative
feedback on to hypothalamus and decreased
release of TRH.)

What thyroid abnormality has the following? TRH


decreased, TSH decreased, T4 increased

3049.

Secondary hypocortisolism (pituitary)

What pathophysiologic disorder is characterized


by the following changes in cortisol and ACTH?
Cortisol decreased, ACTH decreased

3050.

Secondary hypoparathyroidism (vitamin D toxicity)

What endocrine abnormality is characterized by


the following changes in PTH, Ca2+, and inorganic
phosphate (Pi)? PTH decreased, Ca2+
increased, Pi increased

3051.

Secondary hypothyroidism/pituitary (Low TSH


results in low T4 and increased TRH because of
lack of a negative feedback loop.)

What thyroid abnormality has the following? TRH


increased, TSH decreased, T4 decreased

3052.

Secondary lysosome (think of the primary as


inactive and secondary as active)

What type of lysosome is formed when lysosome


fuses with a substrate for breakdown?

3053.

Secondary spermatocyte

Match the chromosome and haploid number with


the stage of sperm development, spermatid,
spermatocyte (primary or secondary),
spermatogonia (type A or B): 23/2n

3054.

Second M PITS for pharyngeal pouch derivatives

From what pharyngeal pouch is the following


structure derived? Palatine tonsil

3055.

Second MS CARD is my mnemonic for the aortic


arch derivatives

From what aortic arch are the following structures


derived? Stapes artery

3056.

The second pharyngeal pouch and groove

What pharyngeal pouch and groove persist when


a pharyngeal fistula is formed?

3057.

Secretin stimulates the pancreas to secrete a


HCO3--rich solution to neutralize the acidity of the
chyme entering the duodenum.

What is secretin's pancreatic action?

3058.

The secretory phase is progesteronedependent and 14 days long, whereas the


length of the proliferative phase varies

What phase of the female reproductive cycle is 14 days


long?

3059.

Selection bias

What type of bias is it when the sample population is not


a true representative of the population?

3060.

Selective erectile disorder

What is the term to describe a man who has The ability


to have an erection sometimes and other times not?

3061.

Selegiline

What MAOI does not cause a hypertensive crisis?

3062.

Selegiline

What is the drug of choice for early Parkinson's disease?

3063.

Selegiline

What is the monoamine oxidase B (MAOB) inhibitor?

3064.

Selenium (Se)

What mineral is an important component of glutathione


peroxidase?

3065.

Semicircular canal

What component of the inner ear Contains perilymph


and responds to angular acceleration and deceleration of
the head?

3066.

Semicircular duct

What component of the inner ear Contains endolymph


and responds to head turning and movement?

3067.

Seminoma

What germ cell tumor is seen in the 15-to 35-year-old


age group, peaks when the person is 35 years of age,
and is a bulky mass that spreads via the lymphatic
system?

3068.

Seminoma

What is the most common one? Germ cell tumor in


men

3069.

Seminoma

Name the cancer associated with the following tumor


markers. (Some may have more than one answer.)
Placental alkaline phosphatase

3070.

Semipermeable membrane; a selectively


permeable membrane allows both water and
small solutes to pass through its membrane.

What type of membrane is characterized as being


permeable to water only?

3071.

Senile plaques, neurofibrillary


tangles, and granulovascular
changes in neurons

What are the three microscopic pathologic changes seen in


Alzheimer's disease?

3072.

Sensitivity (it deals with the sick)

The proportion of truly diseased persons in the screened


population who are identified as diseased refers to?

3073.

Sensory

What type of fiber or fibers are carried in (answer motor,


sensory, or both) Dorsal root ganglion?

3074.

Sensory

What type of fiber or fibers are carried in (answer motor,


sensory, or both) Dorsal root?

3075.

The sensory component is through


the superior laryngeal nerve, and the
motor limb is via the recurrent
laryngeal nerve.

What branches of CN X are the sensory and motor components


of the cough reflex? Be specific.

3076.

The sensory limb is via CN IX, and


the motor limb is from CN X.

Which CNs act as the sensory and motor components of the gag
reflex?

3077.

Separation anxiety

Name the reaction that appears in babies who are temporarily


deprived of their usual caretaker. (This reaction usually begins
around 6 months of age, peaks around 8 months, and
decreases at 12 months.)

3078.

SER

What cytoplasmic organelle carries the enzymes for elongation


and desaturation of fatty acyl CoA?

3079.

Series circuit

What type of circuit is described when the total resistance is


always greater than the sums of the individual resistors?

3080.

Serocystadenocarcinoma

What ovarian disease involves psammoma bodies?

3081.

Serocystadenocarcinoma

What is the most common one? Primary malignant tumor of the


ovary

3082.

Serocystadenoma

What is the most common one? Benign tumor of the ovary

3083.

Serotonergic crisis may be precipitated when an


SSRI is mixed with MAOIs, TCADs,
dextromethorphan, and meperidine. Treat it with
cyproheptadine.

A depressive patient who is taking paroxetine


goes to the ER with pain and is given meperidine.
Shortly afterward she develops diaphoresis,
myoclonus, muscle rigidity, hyperthermia, and
seizures. What is your diagnosis?

3084.

Serotonin

What are the primary neurotransmitters at the


following sites? Brainstem cells

3085.

Serotonin syndrome. It is also associated with high


doses and MAOI and synthetic narcotic
combinations (Ecstasy). Treatment consists of
decreasing SSRI dosage, removing the causative
agent, and giving cyproheptadine.

What syndrome is characterized by sweating,


insomnia, nausea, diarrhea, cramps, delirium,
and general restlessness secondary to MAOI and
SSRI in combination?

3086.

Serous exudates

Name the type of exudate, given the following


examples. Sunburn

3087.

Serratia

Which bacteria are associated with the following


pigment production? Red pigmentation

3088.

Serratus anterior

What muscle or muscles are innervated by the


following nerves? Long thoracic nerve

3089.

Sertoli cell

What cell is under control of FSH and


testosterone; secretes inhibin, MIF, and
androgen-binding protein; and phagocytizes the
excess cytoplasm of the spermatid?

3090.

Serum 25-hydroxy-vitamin D (25-OH-D)

What is the best measure of total body vitamin D


if you suspect a deficiency?

3091.

Seven-eighths of nephrons are cortical, with the


remainder juxtamedullary.

What percentage of nephrons is cortical?

3092.

Severe combined immunodeficiency disease

What disease involves a lack of both T cellmediated and humoral immune responses that
can be either X-linked or AR?

3093.

Severe (range 20-34)

What level of mental retardation is characterized by Having


the ability to communicate and learn basic habits but training
is usually not helpful?

3094.

Sexual aversion disorder

What is the term for a complete aversion to all sexual


contact?

3095.

Shaping (successive approximation)

What operant conditioning therapy or modification is


described as Reinforcing successive attempts that lead to
the desired goal (gradual improvement)?

3096.

Sheehan syndrome

Name the most common type or cause. Panhypopituitarism

3097.

Sheehan syndrome

What is the term for panhypopituitarism secondary to


ischemic necrosis and hypotension postpartum?

3098.

Shifts it to the right

Which way will the O2 dissociation curve shift with the


addition of 2, 3-bisphosphoglycerate (2, 3-BPG) to adult
hemoglobin (Hgb)?

3099.

Shiga toxin (enterohemorrhagic


Escherichia coli produces Vero toxin,
which is quite similar to shiga toxin)

What is the name of the exotoxin Shigella dysenteriae


produces, which interferes with the 60S ribosomal subunit
and results in eukaryotic protein synthesis inhibition?

3100.

Shigella

What nonmotile gram-negative, non-lactose-fermenting


facultative anaerobic rod uses the human colon as its only
reservoir and is transmitted by fecal-oral spread?

3101.

Shine-Dalgarno sequence

In prokaryotes, what is the name of the RNA sequence that


ribosomes bind to so translation can occur?

3102.

SHIP: Shifting of the aorta, Hypertrophy


of the right ventricle, Interventricular
septal defect, Pulmonary stenosis

What is the tetrad of tetralogy of Fallot?

3103.

The short arm of chromosome 6

What chromosome codes for HLA gene products?

3104.

Short-chain fatty acids

What must be supplemented in patients with medium-chain


acyl CoA dehydrogenase (MCAD) deficiency?

3105.

Short-chain fatty acids

What forms of fatty acids are absorbed from the small


intestine mucosa by simple diffusion?

3106.

Shortens

What happens to the following during skeletal muscle


contraction? Sarcomere

3107.

Shortens

What happens to the following during skeletal muscle


contraction? I band

3108.

Shortens

What happens to the following during skeletal muscle


contraction? H zone

3109.

Sickle cell disease

Name the most common type or cause. Hematologic


cause of papillary necrosis

3110.

Sideroblastic anemia

Name the hypochromic microcytic anemia based on the


following laboratory values. Increased iron, decreased
TIBC, increased percent saturation, increased ferritin

3111.

Sigma factor

What protein is required by prokaryotic RNA polymerases


to initiate transcription at the promoter region of DNA?

3112.

Sigmoid colon

What is the most common one? Site for colonic


diverticula

3113.

Silent

Name the type of mutation: New codon specifies for the


same AA

3114.

Silicosis Note: Coal worker's


pneumoconiosis is synonymous with black
lung disease, an upper lobe occupational
disorder

What pneumoconiosis is associated with exposure to the


following occupations or materials? Miners, metal
grinders, and sandblasters

3115.

Simon focus

What is the term for granuloma at the lung apex in TB?

3116.

Simple diffusion; it does not use proteinmediated transport

What is the main mechanism for exchange of nutrients


and gases across a capillary membrane?

3117.

Since CO2 is 24 times as soluble as O2, the rate at which


CO2 is brought to the membrane determines its rate of
exchange, making it perfusion-limited a gas. For O2 the more
time it is in contact with the membrane, the more likely it will
diffuse, making it diffusion-limited.

Is CO2 a perfusion-or diffusion-limited


O2 gas?

3118.

Since they are independent events, their probabilities would


be multiplied.

If the occurrence of one event had


nothing to do with the occurrence of
another event, how do you combine
their probabilities?

3119.

Single-strand DNA binding protein

What protein prevents ssDNA from


reannealing during DNA replication?

3120.

Sinus venosus

From what embryonic structure are the


following structures derived? The
sinus venarum, coronary sinus, and the
oblique vein of the left atrium

3121.

Site 1

Name the Hgb-O2 binding site based


on the following information: Remains
attached under most physiologic
conditions

3122.

Site 2

Name the Hgb-O2 binding site based


on the following information: Greatest
affinity of the three remaining sites for
attachment; requires PO2 levels of 26
mm Hg to remain attached

3123.

Site 3

Name the Hgb-O2 binding site based


on the following information: Requires
a PO2 level of 40 mm Hg to remain
attached

3124.

Site 4

Name the Hgb-O2 binding site based on the following information:


Least affinity for O2; requires the highest PO 2 levels for attachment
(approx. 100 mm Hg)

3125.

SITSSubscapularis,
Infraspinatus, Teres minor,
Supraspinatus

What muscles make up the rotator cuff?

3126.

Sixth MS CARD is my mnemonic


for the aortic arch derivatives

From what aortic arch are the following structures derived? Right
and left pulmonary arteries and the ductus arteriosus

3127.

Sjgren disease

For what disease are SS-A(Ro), SS-B(La), and R-ANA diagnostic


markers?

3128.

Sjogren's syndrome, active


hepatitis, systemic lupus
erythematosus (with HLA-DR2)
and type 1 diabetes (with HLADR4)

What diseases are associated with the HLA-DR3 allele

3129.

Sjgren's syndrome; SS-A (Ro)


and SS-B (La)

What autoimmune syndrome is characterized by keratoconjunctivitis,


corneal ulcers, xerostomia, and an increased risk of high-grade Bcell lymphomas? What two Ab tests are used in making the
diagnosis?

3130.

Skeletal muscle

Name the muscle type based on the histological features: Actin and
myosin in sarcomeres; striated; multinuclear; lacks gap junctions;
troponin:calcium binding; T tubules and SR forming triadic contacts;
highest ATPase activity; no calcium channels

3131.

Skeletal muscles

Name the muscle type based on these descriptions: Discontinuous


voluntary contraction, multinuclear striated unbranched fibers, actin
and myosin overlapping for banding pattern, triadic T tubules,
troponin and desmin as Z disc intermediate filament.

3132.

Skin, liver, and kidneys

What three organs are necessary for the production of vitamin


D3(cholecalciferol)?

3133.

SLE; ANA, anti-dsDNA


and anti-Sm (anti-Smith)

A 20-year-old black woman goes to you with nonspecific joint pain, fever, and
a malar rash over the bridge of her nose and on her cheeks. This is a classic
example of what autoimmune disease? What are three autoantibody tests
you could order to make the diagnosis?

3134.

Sleep latency

What is the period between going to bed and falling asleep called?

3135.

Sleepwalking is
associated with stage 4
and occurs most often
in the first third of sleep.

What stage of sleep is associated with somnambulism?

3136.

Small cell cancer of the


lung

Name the cancer associated with the following oncogenes. (Some may have
more than one answer) L-myc

3137.

Small cell carcinoma


(oat cell)

What very aggressive lung cancer metastasizes early and is associated with
smoking and paraneoplastic syndromes?

3138.

The small subunit (40S)


binds first.

What ribosomal subunit binds first to the mRNA strand?

3139.

Small to medium-sized
cerebral vessels

What cerebral vessel size is affected in patients with vascular dementia?

3140.

Smoking

What is the leading cause of preventable premature death and illness in the
United States?

3141.

Smooth muscle

Name the muscle type based on these descriptions: Involuntary contraction,


uninuclear nonstriated fibers, actin and myosin not forming banding pattern;
lack of T tubules, gap junctions, and calmodulin.

3142.

Smooth muscle

Name the muscle type based on the histological features: Actin and myosin
not in sarcomeres; nonstriated; uninuclear; gap junctions; calmodulin:calcium
binding; lacks T tubules; voltage-gated calcium channels

3143.

Snails

What are the first intermediate hosts for trematodes?

3144.

snRNA

Name the RNA subtype based on the following: Found only in the nucleus of
eukaryotes and functions to remove introns from mRNA

3145.

Solved as 30% T + 30% A = 60%;


therefore, C + G = 40%; then C = 20% and
G = 20% (example of Chargaff's rule)

If a sample of DNA has 30% T, what is the percent of C?

3146.

Somatization

Name these immature defense mechanisms: Psychic


feelings converted to physical symptoms

3147.

Somatization disorder

What somatoform disorder is described as Having a F:M


ratio of 20:1, onset before age 30, and having 4 pains (2
gastrointestinal, 1 sexual, 1 neurologic)?

3148.

Somatoform disorder

What disorder is described as having Unconscious


symptoms with unconscious motivation?

3149.

Somatoform pain disorder

What somatoform disorder is described as Severe,


prolonged pain that persists with no cause being found,
disrupts activities of daily living?

3150.

Somatomedins

What are the growth factors released from the liver


called?

3151.

Somatomedins (IGF-1)

What growth factors are chondrogenic, working on the


epiphyseal end plates of bone?

3152.

Somatotrophins are stimulated by IGF-1,


and they inhibit GH secretion. GHRH
stimulates GH secretion.

How do elevated blood glucose levels decrease GH


secretion? (Hint: what inhibitory hypothalamic hormone is
stimulated by IGF-1?)

3153.

Sorbitol (resulting in cataracts)

In a diabetic patient, to what does aldose reductase


convert glucose?

3154.

Southern blot

What blotting technique uses the following for analysis?


DNA

3155.

Spasms of the uterus, bladder, and the


biliary tree occur with all of the opioids
except meperidine.

A patient goes to the ER with signs and symptoms of


biliary colic. Which opioid analgesic do you choose?

3156.

Spearman correlation

What type of correlation compares two ordinal variables?

3157.

Specificity (it deals with the healthy)

What is the statistical term for the proportion of truly


nondiseased persons in the screened population who are
identified as nondiseased?

3158.

Specific rate (e.g.,


men aged 55-60)

What is the term for the rate measured for a subgroup of a population?

3159.

Spectinomycin

What is the drug of choice for penicillin-resistant gonococcal infections?

3160.

Spermatid

Match the chromosome and haploid number with the stage of sperm
development, spermatid, spermatocyte (primary or secondary), spermatogonia
(type A or B): 23/1n

3161.

Spermatogonia (type
A)

Match the chromosome and haploid number with the stage of sperm
development, spermatid, spermatocyte (primary or secondary), spermatogonia
(type A or B): 46/2n (divide meiotically)

3162.

Spermatogonia (type
B)

Match the chromosome and haploid number with the stage of sperm
development, spermatid, spermatocyte (primary or secondary), spermatogonia
(type A or B): 46/2n (divide mitotically)

3163.

S phase

What phase of the cell cycle are the following antineoplastic agents specific
for? Hydroxyurea

3164.

S phase

What phase of the cell cycle are the following antineoplastic agents specific
for? 6-Mercaptopurine

3165.

S phase

DNA replication occurs during what phase of the cell cycle?

3166.

S phase

Name the phase of the eukaryotic cell cycle: Period of DNA replication
(preparing for mitosis)

3167.

S phase

What phase of the cell cycle are the following antineoplastic agents specific
for? 6-Thioguanine

3168.

S phase

What phase of the cell cycle are the following antineoplastic agents specific
for? Cytarabine

3169.

Sphingomyelin

What substrate is built up in Niemann-Pick disease?

3170.

Sphingomyelinase

What lysosomal enzyme is deficient in Niemann-Pick disease?

3171.

Sphingomyelin
(lecithin also, but it is
not a sphingolipid)

What is the only sphingolipid that contains choline and PO4?

3172.

Sphingosine

What sphingolipid is formed by the union of serine and palmitoyl CoA?

3173.

Spinal trigeminal nucleus

What component of the trigeminal nuclei


Receives sensory input (all but pain and
temperature) from the face, scalp, dura, and the
oral and nasal cavities?

3174.

Spinothalamic tract (anterolateral system)

In what tract does pain, temperature, and crude


touch sensory information ascend to the
postcentral gyrus of the parietal lobe?

3175.

Spironolactone

Which diuretic is used as an adrenal steroid


antagonist?

3176.

Spironolactone

Which potassium-sparing drug is an aldosterone


receptor antagonist ?

3177.

Spironolactone (binds to aldosterone receptors)

What is the only diuretic that works on the blood


side of the nephron?

3178.

Spleen

What is the only organ supplied by the foregut


artery that is of mesodermal origin?

3179.

Splenic flexure

What is the most common one? Site of ischemia


in the GI tract

3180.

The splenic vein

The inferior mesenteric artery drains into it.

3181.

Splenomegaly and anemia (with a high index of


suspicion)

What are the two most important features in the


diagnosis of malaria?

3182.

Splitting

Name these narcissistic defense mechanisms:


Everything in the world is perceived as either good
or bad . No middle ground; it is all extremes.

3183.

Splitting of the first heart sound occurs during


inspiration because of the increased output of the
right ventricle, delaying the closure of the pulmonic
valve.

Does physiologic splitting of the first heart sound


occur during inspiration or expiration? Why?

3184.

Sporothrix schenckii

Which fungus is found worldwide on plants, is a


cigar-shaped yeast in tissue form, and results in
rose gardener's disease?

3185.

Sporozoites

What form of the Plasmodium species are injected into humans by mosquitoes?

3186.

Squamous cell
carcinoma

What is the most common one? Cancer of the esophagus in the world

3187.

Squamous cell
carcinoma

What is the most common one? Malignant tumor of the esophagus

3188.

Squamous cell
carcinoma

Keratin pearls

3189.

Squamous cell
carcinoma

What type of skin carcinoma occurring on sun-exposed sites has a low level of
metastasis?

3190.

Squamous cell
carcinoma

What bronchogenic carcinoma is associated with an elevated level of Ca2+,


involves keratin pearls, occurs in men more than women, is associated with
smoking, occurs in the major bronchi, and is seen in the central areas of the
lung?

3191.

Squamous cell
carcinoma

What malignant neoplasm of the skin is associated with keratin pearls?

3192.

Squamous cell
carcinoma

What is the most common one? Esophageal carcinoma

3193.

Squamous cell
carcinoma

What is the most common one? Cancer of the vulva

3194.

Squamous cell
carcinoma of the lung

Name the cancer associated with the following oncogenes. (Some may have
more than one answer) erb-1

3195.

Squamous cell
carcinoma (skin, lung)
and angiosarcoma of
the liver

Name the cancer associated with the following chemical agents. (Some may
have more than one answer.) Arsenic

3196.

SSRIs

What is the most widely used class of antidepressants?

3197.

Stable

Name the type of regeneration (i.e., labile, stable, or permanent) based on the
following examples. Pancreas

3198.

Stable

Name the type of regeneration (i.e., labile, stable, or permanent) based on the
following examples. Liver

3199.

Stable

Name the type of regeneration (i.e., labile, stable,


or permanent) based on the following examples.
Smooth muscle

3200.

Stable

Name the type of regeneration (i.e., labile, stable,


or permanent) based on the following examples.
Kidney

3201.

Stable

Name the type of regeneration (i.e., labile, stable,


or permanent) based on the following examples.
Fibroblasts

3202.

Stable angina

What form of angina is characterized by


Coronary artery luminal narrowing, symptom
occurrence during exertion, ST segment
depression on ECG?

3203.

Stable (Labile cells proliferate throughout life; stable


cells have a low level of proliferation; and
permanent cells as the name states, do not
proliferate.)

Name the type of regeneration (i.e., labile, stable,


or permanent) based on the following examples.
Osteoblasts

3204.

Stage 1

Name the stages of sleep with these EEG


patterns: Disappearance of alpha waves,
appearance of theta waves

3205.

Stage 2

Name the stages of sleep with these EEG


patterns: Sleep spindles, K-complexes

3206.

Stage 2, which accounts for approximately 45% of


total sleep time, with REM occupying 20%.

Most sleep time is spent in what stage of sleep?

3207.

Stage 3 and 4

Name the stages of sleep with these EEG


patterns: Delta waves

3208.

Stage 3 and 4 most commonly. It can occur at any


stage in the sleep cycle and is usually associated
with a major stressor being introduced into the
home.

With what stage of sleep is enuresis associated?

3209.

Stage 4

At what stage of sleep is GH output elevated?

3210.

Stage fright (discrete performance


anxiety)

Name the most common type. Phobia

3211.

Stages 3 and 4 (NREM)

In what stage of sleep is GH secreted?

3212.

Standardized rate

What rate removes any difference between two populations,


based on a variable, to makes groups equal?

3213.

Stanford-Binet Scale, developed in


1905, is useful in the very bright, the
impaired, and children less than 6
years old.

What is the first formal IQ test used today for children aged 2
to 18?

3214.

Stapedius muscle

What muscle keeps the stapes taut against the oval window?

3215.

The stapedius muscle

What muscle of the middle ear is innervated by CN VII?

3216.

Staphylococcus

What genus of bacteria is described by catalase-positive,


gram-positive cocci in clusters?

3217.

Staphylococcus aureus

What staphylococcal species is positive for Beta-hemolysis


and coagulase?

3218.

Staphylococcus aureus

Name the most common type or cause. Acute bacterial


endocarditis

3219.

Staphylococcus aureus

What is the most common one? Organism associated with


mastitis

3220.

Staphylococcus aureus

A patient goes to the ER with abdominal cramps, vomiting,


diarrhea, and sweating less than 24 hours after eating potato
salad at a picnic; what is the most likely responsible
organism?

3221.

Staphylococcus aureus

Name the most common type or cause. Pyogenic


osteomyelitis

3222.

Staphylococcus aureus

Which bacteria are associated with the following pigment


production? Yellow pigmentation

3223.

Staphylococcus aureus

What is the MCC of the following endocarditis scenarios?


Non-IV drug user

3224.

Staphylococcus aureus

What are the most common causes of osteomyelitis Overall?

3225.

Staphylococcus aureus (but they are


more prone to salmonella infections)

What are the most common causes of osteomyelitis In


patients with sickle cell disease?

3226.

Staphylococcus saprophyticus

Name the most common type or cause. Urethritis in a


young, newly sexually active individual

3227.

STARS 1. Upper Subscapularis 2.


Thoracodorsal 3. Axillary 4. Radial 5. Lower
Subscapularis

What are the five branches of the posterior cord of the


brachial plexus?

3228.

Stercobilin

What bile pigment is formed by the metabolism of


bilirubin by intestinal bacteria, giving stool its brown
color?

3229.

Stereocilia

What are the long microvilli in the inner ear and male
reproductive tract called?

3230.

Steroid synthesis, drug detoxification,


triglyceride resynthesis, and Ca2+handling

What are the four functions of SER?

3231.

Stibogluconate

What is the DOC for the following protozoal infections?


Leishmaniasis

3232.

Stimulus control

What operant conditioning therapy or modification is


described as Having a stimulus take over the control of
the behavior (unintentionally)?

3233.

Stimulus generalization must stop. (Pairing


of the unconditioned stimulus and the
conditioned stimulus must cease.)

In the classical conditioning model, when a behavior is


learned, what must occur to break the probability that a
response will happen?

3234.

The stomach

What is the only site in the body that uses M1 receptors?

3235.

Stranger anxiety

What form of anxiety, appearing at 6 months, peaking at


8 months, and disappearing by 1 year of age, is seen in
the presence of unfamiliar people?

3236.

Stratified squamous nonkeratinized


epithelium, which has cilia that beat toward
the oropharynx.

What is the epithelial cell lining the nasopharynx?

3237.

Stratum corneum (keratinized)

What is the most superficial layer of the epidermis?

3238.

Stratum granulosum

What epidermal layer's function is to release lipids to act


as a sealant?

3239.

Stratum lucidum

What is the first epidermal layer without


organelles and nuclei

3240.

Stratum spinosum

In what layer of the epidermis is melanin


transferred from melanocytes to
keratinocytes?

3241.

Streptococcus agalactiae

Describe the organism based on the


following information: Beta-Hemolytic
Streptococcus; positive cAMP test;
hydrolyzes hippurate

3242.

Streptococcus agalactiae

What are the most common causes of


osteomyelitis In neonates?

3243.

Streptococcus agalactiae and Escherichia coli

What is the MCC of the following


meningitides? In neonates to 3 months of
age

3244.

Streptococcus agalactiae (Group B Streptococcus, or


GBS)

Name the most common type or cause.


Neonatal septicemia and meningitis

3245.

Streptococcus pneumoniae

Name the most common type or cause.


Lobar pneumonia

3246.

Streptococcus pneumoniae

What is the MCC of the following


meningitides? In adults

3247.

Streptococcus pneumoniae

Name the MCC. Pneumonia in adults

3248.

Streptococcus pneumoniae, Klebsiella pneumoniae,


Haemophilus influenzae, Pseudomonas aeruginosa, and
Neisseria meningitidis; also Cryptococcus neoformans, a
fungus

Name at least three bacteria that use


capsules to prevent immediate destruction
from the host's defense system.

3249.

Streptococcus pyogenes

Name the most common type or cause.


Erysipelas

3250.

Streptococcus pyogenes

Describe the organism based on the


following information: Beta-hemolytic
Streptococcus sensitive to bacitracin

3251.

Streptococcus viridans

Which streptococcal species is associated with dental caries


and infective endocarditis in patients with poor oral hygiene?

3252.

Streptococcus viridans

What is the MCC of the following endocarditis scenarios?


Post dental work

3253.

Streptococcus viridans

Name the most common type or cause. Subacute bacterial


endocarditis

3254.

Streptococcus viridans

Describe the organism based on the following information:


Alpha-hemolytic Streptococcus; not lysed by bile; not sensitive
to Optochin

3255.

Streptokinase

What thrombolytic agent is derived from -hemolytic


streptococci, is antigenic, and produces depletion of circulating
fibrinogen, plasminogen, and factors V and VII?

3256.

Streptolysin O

Which Streptococcus pyogenes toxin is immunogenic?

3257.

Streptomycin

What aminoglycoside causes disruption of CN I?

3258.

Streptomycin

Which aminoglycoside is the DOC for tularemia and the


bubonic plague?

3259.

Stretch receptors prevent


overdistension of the lungs during
inspiration.

What receptor is in the smooth muscle cells of the small


bronchi, is stimulated during inflation, and inhibits inspiration?

3260.

Stroke volume

What is the term for the amount of blood expelled from the
ventricle per beat?

3261.

Strongyloides stercoralis is treated with


thiabendazole.

What nematode is known as threadworms? What is the


treatment?

3262.

Stylopharyngeus muscle is innervated


by CN IX; all other pharyngeal muscles
are innervated by CN X.

What is the only pharyngeal muscle not innervated by CN X?

3263.

Subacute bacterial endocarditis

What is the most common one? Complication of PDA

3264.

Subacute combined degeneration of the


spinal cord is treated with IM vitamin
B12 injections. If treatment is working,
you will see an increased reticulocyte
count on the peripheral smear in about 5
days.

A chronic alcohol abuser goes to the ER with weakness, a


sore, beefy red tongue, loss of vibration and position sense,
arm and leg dystaxia, elevated levels of methylmalonic acid
in the urine, and anemia with an MCV above 105 fL. What is
your diagnosis, and how will you monitor his response to
treatment?

3265.

Subacute combined degeneration,


which is bilateral below the level of the
lesion.

What is the name of demyelination of the corticospinal tract


and the dorsal column in the spinal cord due most commonly
to a vitamin B12 deficiency?

3266.

Subarachnoid hematoma

What type of cerebral bleed is due to a rupture of a berry


aneurysm in the circle of Willis?

3267.

Subarachnoid hemorrhage

Bloody tap on lumbar puncture

3268.

Subatmospheric pressure acts to


expand the lung; positive pressure acts
to collapse the lung.

Does subatmospheric pressure act to expand or collapse the


lung?

3269.

The subclavian artery

The vertebral artery is a branch of what artery?

3270.

Subclavian vein

What vessel can be found atop the scalene anterior?

3271.

Subdural hematoma is a rupture of the


cerebral veins where they enter the
superior sagittal sinus.

Is a subdural hematoma an arterial or venous bleed?

3272.

Sublimation

Name these mature defense mechanisms: Converting an


unacceptable impulse to a socially acceptable form (Hint: it is
the most mature of all defense mechanisms)

3273.

The submandibular ganglion.


(Submandibular ganglion innervates the
submandibular gland; easy enough.)

What is the name of the postganglionic parasympathetic


ganglion that innervates The submandibular and sublingual
glands?

3274.

Submandibular gland produces mainly


serous and the sublingual gland produces
mainly mucous secretions.

Both submandibular and sublingual glands are innervated


by CN VII (facial) and produce mucous and serous
secretions. Which one mainly produces serous secretions?

3275.

Subscapularis

What muscle or muscles are innervated by the following


nerves? Upper subscapularis nerve

3276.

Substance P. (Opioids relieve pain in part


by blocking substance P.)

What 11-amino acid peptide is the neurotransmitter of


sensory neurons that conveys pain from the periphery to the
spinal cord?

3277.

Substantia nigra

In Parkinson's disease, what area of the basal ganglia has


a decreased amount of dopamine?

3278.

The substantia nigra is the largest


nucleus in the midbrain. It contains
melanin and uses GABA and dopamine
as its neurotransmitters.

What is the largest nucleus in the midbrain?

3279.

Substituted judgment. It is made by the


person who best knows the patient, not
the closest relative.

What judgment states that the decision, by rights of


autonomy and privacy, belongs to the patient, but if the
patient is incompetent to decide, the medical decision is
based on subjective wishes?

3280.

Subthreshold potential

What type of potential is characterized as graded,


decremental, and exhibiting summation?

3281.

Succinate dehydrogenase

What enzyme of the TCA cycle catalyzes the production of


the following: FADH2

3282.

Succinate dehydrogenase

What enzyme of the TCA cycle also acts as complex II of


the ETC?

3283.

Succinylcholine and halothane (Treatment


is with dantrolene.)

What two drugs, when mixed, can lead to malignant


hyperthermia?

3284.

Succinyl CoA

What intermediate enables propionyl CoA to enter into the


TCA cycle?

3285.

Succinyl CoA synthetase

What enzyme of the TCA cycle catalyzes the production of


the following: GTP

3286.

Succinyl CoA
synthetase

What enzyme of the TCA cycle catalyzes the substrate level phosphorylation?

3287.

Sucralfate

Which medication requires an acidic gastric pH for activation to form a protective


gel-like coating over ulcers and GI epithelium?

3288.

Sulfonamides

What class of antimicrobial agents should be avoided in patients with a history of


GPDH deficiency?

3289.

Superego

According to Freud, what facet of the psyche represents the internalized ideals
and values of one's parents?

3290.

Superego

What Freudian psyche component is described as The conscience, morals,


beliefs (middle of the road)?

3291.

Superficial
spreading
melanoma

What is the most common one? Type of melanoma

3292.

Superior

Name the compartment of the mediastinum associated with the following thoracic
structures: Brachiocephalic vein

3293.

Superior

Name the compartment of the mediastinum associated with the following thoracic
structures: Left common carotid artery

3294.

Superior

Name the compartment of the mediastinum associated with the following thoracic
structures: Trachea

3295.

Superior

Name the compartment of the mediastinum associated with the following thoracic
structures: Left subclavian artery

3296.

Superior

Name the compartment of the mediastinum associated with the following thoracic
structures: Aortic arch

3297.

Superior

What is the label given to an individual whose IQ is 120 to 129

3298.

Superior and
anterior

Name the compartment of the mediastinum associated with the following thoracic
structures: Thymus

3299.

Superior and
middle

Name the compartment of the mediastinum associated with the following thoracic
structures: SVC

3300.

Superior and
middle

Name the compartment of the mediastinum associated with the following thoracic
structures: Phrenic nerve

3301.

Superior and
posterior

Name the compartment of the mediastinum associated with the following thoracic
structures: Thoracic duct

3302.

Superior and
posterior

Name the compartment of the mediastinum associated with the following thoracic
structures: Esophagus

3303.

Superior and posterior

Name the compartment of the


mediastinum associated with the
following thoracic structures: Azygos
vein

3304.

The superior border of the inferior intercostal rib is your


landmark for a pleural tap because along the inferior border
of each rib is the neurovascular bundle, and you would risk
injury if you went below the rib.

If inserting a needle to perform a pleural


tap or insertion of a chest tube, do you
use the inferior or the superior border of a
rib as your landmark? Why?

3305.

Superior cerebellar peduncle; the inferior and the middle


consist mainly of incoming (afferent) tracts and fibers.

Which one of the cerebellar peduncles is


mainly responsible for outgoing (efferent)
information?

3306.

Superior colliculus (Remember S for Superior and Sight).


The inferior colliculus processes auditory information from
both ears.

Which of the colliculi help direct the


movement of both eyes in a gaze?

3307.

Superior gluteal nerve

What lower extremity nerve is described


by the following motor loss? Loss of
abduction of the hip resulting in
Trendelenburg gait

3308.

The superior mesenteric vein and the splenic vein (after it


receives the inferior mesenteric vein) join to form the portal
vein.

What two veins form the portal vein?

3309.

Superior Oblique (CN IV) (LR6 SO4)3

What ocular muscle Depresses and


abducts the eyeball?

3310.

Superior olivary nucleus

What CN nucleus receives auditory


information from both ears via the
cochlear nuclei?

3311.

Superior rectus (CN III) (LR6 SO4)3

What ocular muscle Elevates and


adducts the eyeball?

3312.

Superior vena cava

What vein is formed by the union of the


right and left brachiocephalic veins?

3313.

Superoxide dismutase,
glutathione peroxidase,
catalase

Name the three enzymes that protect the cell from oxygen-derived
free radicals.

3314.

Suppression

Name these mature defense mechanisms: Forgetting on purpose (so


you can actually remember it)

3315.

Suprachiasmatic nucleus

What nucleus of the hypothalamus receives visual input from the


retina and helps set the circadian rhythm?

3316.

Supraoptic nuclei; lesions here


result in diabetes insipidus.

What hypothalamic nucleus is responsible for the production of ADH?

3317.

Supraspinatus and
infraspinatus

What muscle or muscles are innervated by the following nerves?


Suprascapular nerve

3318.

The surface of the medulla

What region of the brain houses the central chemoreceptors


responsible for control of ventilation?

3319.

Surface tension; in the alveoli,


it is a force that acts to collapse
the lung.

What is the greatest component of lung recoil?

3320.

Surface tension, the force to


collapse the lung, is greatest at
the end of inspiration.

When is the surface tension the greatest in the respiratory cycle?

3321.

Swan-neck deformities

What is the term for hyperextension of the PIP and flexion of the DIP
joints in rheumatoid arthritis?

3322.

Sympathetic chain ganglion

What ganglion receives preganglionic sympathetic fibers from T1 to


L1-2 and innervates smooth muscle, cardiac muscle, glands, head,
thoracic viscera, and blood vessels of the body wall and limbs?

3323.

Sympathetic nervous system

What component of the ANS is responsible for movement of semen


from the epididymis to the ejaculatory ducts?

3324.

Syncytiotrophoblast. (The
cytotrophoblast gets
incorporated into the
syncytiotrophoblast.)

Which trophoblast layer of the placenta remains until the end of


pregnancy?

3325.

Syphilitic

What form of vasculitis involves the ascending arch and causes


obliterative endarteritis of the vasa vasorum?

3326.

Syringomyelia

What disease is a cavitation of the spinal cord causing


bilateral loss of pain and temperature at the level of the
lesion?

3327.

Systematic desensitization

What classical conditioning therapy or modification is


described as Triage of a hierarchy of fears (from least
to most), then teaching muscle relaxation techniques in
the presence of those fears until the subject is not afraid
anymore?

3328.

Systemic lupus erythematosus

What disease is associated with the HLA-DR2 and


HLA-DR3 alleles

3329.

The systemic veins have the largest blood


volume, and the pulmonary veins have the
second largest blood volume in the
cardiovascular system. They represent the
reservoirs of circulation.

What component of the cardiovascular system has the


largest blood volume? Second largest blood volume?

3330.

Systolic interval decreases secondary to


increased contractility; diastolic interval
decreases secondary to an increase in heart
rate.

What happens to diastolic and systolic intervals with an


increase in sympathetic activity?

3331.

T3

What is the bioactive form of thyroid hormone?

3332.

T3 has a greater affinity for the nuclear


receptor and therefore is considered the
active form.

Does T3 or T4 have a greater affinity for its nuclear


receptor?

3333.

T3 increases both heart rate and cardiac


output by increasing the number of Betareceptors and their sensitivity to
catecholamines.

What is the effect of T3 on heart rate and cardiac


output?

3334.

T4, as long as T4 levels remain constant,


TSH will be minimally effected by T3.

Is T3 or T4 responsible for the negative feedback loop


on to the hypothalamus and anterior pituitary gland?

3335.

T4; because of the greater affinity for the binding


protein, T4 has a significantly (nearly fifty times)
longer half-life than T3.

Is there more circulating T3 or T4 in plasma?

3336.

T4 vertebral level posteriorly and anteriorly at


the sternal angle (angle of Louis).

At what vertebral level does the trachea bifurcate?

3337.

T9

What vertebral level is marked by the xiphoid


process?

3338.

Taenia saginata

Which cestode in raw or rare beef containing


cysticerci results in intestinal tapeworms?

3339.

Taenia solium

What cestode causes cysticercosis?

3340.

Takayasu arteritis (medium-size to large


vessels)

What vasculitis affects a 30-year-old Asian female


having visual field deficits, dizziness, decreased
blood pressure, and weakened pulses in the upper
extremities?

3341.

Tamoxifen

What competitive estrogen receptor antagonist is


used in the treatment of breast cancer?

3342.

T and B-cells belong to the adaptive branch,


whereas PMNs, NK cells, eosinophils,
macrophages, and monocytes belong to the
innate branch.

What two cell lines of the immune system do not


belong to the innate branch?

3343.

Tardive dyskinesia. It persists even after


treatment is discontinued and has no treatment.
Focus is on monitoring for side effects and
prevention.

What major side effect of neuroleptics is


characterized by pill rolling, shuffling gait, and
tremors that abate during sleep?

3344.

Target cell

What is the term for a RBC that has a peripheral rim


of Hgb with a dark central Hgb-containing area?

3345.

TAT (Thematic apperception test)

What projective test asks the patient to tell a story


about what is going on in the pictures, evaluating the
conflicts, drives, and emotions of the individual?

3346.

Tay-Sachs disease

What condition results from a deficiency in the enzyme


hexosaminidase A?

3347.

TB meningitis

What condition results in the following CSF results?


Opening pressure 250 mm H2O; WBCs 250 (90%
lymphocytes); glucose 35; protein 100

3348.

T cells, B cells, and macrophages

What three major cell lines participate in the acquired


immune system?

3349.

T cells, not B cells

What cells are atypical on a peripheral blood smear in


heterophil-positive mononucleosis?

3350.

TCR

What T-cell surface projection recognizes and reacts to


foreign Ags (presented by APCs)?

3351.

Teeth grinding is associated with stage 2


sleep.

With what stage of sleep are bruxisms associated?

3352.

Temporal arteritis

What is the most common one? Form of vasculitis

3353.

Temporal arteritis

What is the most common one? Vasculitis

3354.

Temporal arteritis (giant cell arteritis)

An 80-year-old woman presents to you with right-sided


temporal headache, facial pain and blurred vision on the
affected side, and an elevated ESR. Your diagnosis?

3355.

Temporal lobe

Name the area of the cerebral cortex with the function


described: Language, memory, and emotion (Hint:
herpesvirus infects here commonly)

3356.

Temporal lobes

What cerebral lobes are most commonly affected in


herpes encephalitis?

3357.

Tensor tympani

What muscle of the middle ear is innervated by the


mandibular division of CN V?

3358.

The tensor veli palatine is innervated by the


mandibular division of the trigeminal nerve;
all others are innervated by CN X.

What is the only muscle of the soft palate that is


innervated by CN V3?

3359.

Teratoma

What tumor constitutes 40% of testicular tumors in


children?

3360.

Teratoma (dermoid cyst)

What is the name of the ovarian cyst containing


ectodermal, endodermal, and mesodermal elements (i.e.,
skin, hair, teeth and neural tissue)?

3361.

Terbinafine

What antifungal agent is used to treat dermatophyte


infections by inhibiting squalene epoxidase?

3362.

Terminal bronchioles. (No gas exchange


occurs here.)

Where is the last conducting zone of the lungs?

3363.

Terminal collecting duct has the highest


concentration and Bowman's capsule has
the lowest concentration of inulin.

What segment of the nephron has the highest concentration


of inulin? Lowest concentration of inulin?

3364.

Terminal ileum

What is the most common one? Site of Crohn disease

3365.

Tertiary hypothyroidism/hypothalamic
(Low TRH causes all the rest to be
decreased because of decreased
stimulation.)

What thyroid abnormality has the following? TRH


decreased, TSH decreased, T4 decreased

3366.

Testicular feminization syndrome (Dude


looks like a lady!)

What syndrome occurs when a 46XY fetus develops testes


and female external genitalia?

3367.

Testicular lymphoma

What is the most common one? Testicular tumor in men


over 50 years old

3368.

Testicular tumors

What is the most common one? Tumor in men aged 15 to


35

3369.

Testosterone

Circulating levels of what hormone in men is responsible for


the negative feedback loop to the hypothalamus and the
anterior pituitary gland regulating the release of LH?

3370.

Test-retest reliability

What is the term for the same results achieved again on


testing a subject a second or third time?

3371.

Tetanospasmin (also called tetanus toxin)

What toxin, produced by Clostridium tetani, binds to


ganglioside receptors and blocks the release of
glycine and GABA at the spinal synapse?

3372.

Tetany

What is the term for the summation of mechanical


stimuli due to the skeletal muscle contractile unit
becoming saturated with calcium?

3373.

Tetracycline

Name the antimicrobial agent whose major side


effect is listed. Dental staining in children

3374.

Tetracycline

Which antimicrobial class may cause tooth enamel


dysplasia and decreased bone growth in children?

3375.

Tetralogy of Fallot

Name the most common type or cause. Cyanotic


heart disease

3376.

Th1; they are also responsible for delayed-type


hypersensitivity (type IV)

What subset of CD4 helper T-cell function is helping


the development of CD8 T cells?

3377.

Th2

What subset of CD4 helper T cells stimulate B-cell


division and differentiation?

3378.

Th2 cells

What subset of CD4 T cells is responsible for mast


cell and eosinophil precursor proliferation?

3379.

The thalamus (I like to think of the thalamus as


the executive secretary for the cerebral cortex. All
information destined for the cortex has to go
through the thalamus.)

What area of the brain serves as the major sensory


relay center for visual, auditory, gustatory, and tactile
information destined for the cerebral cortex,
cerebellum, or basal ganglia?

3380.

Thalassemia minor

Name the hypochromic microcytic anemia based on


the following laboratory values. Normal iron, TIBC,
percent saturation, and ferritin

3381.

Theca cell

What female follicular cell is under LH stimulation


and produces androgens from cholesterol?

3382.

Theca cells secrete estrogen. After fertilization the theca


cells form from the theca interna.

After fertilization, what cells of the corpus


luteum Secrete estrogen?

3383.

There are 10 bronchopulmonary segments on the right


and 8 on the left.

How many bronchopulmonary segments


are on the right lung? Left lung?

3384.

There are no valves and no smooth muscle in the walls of


the veins in the face.

What is the major difference between the


veins in the face and the veins in the rest of
the body?

3385.

There are two bacterial promoter regions upstream. The


TATA box is - 10 base pairs upstream, and the -35
promoter site is self-explanatory.

How many base pairs upstream is the


prokaryotic TATA box promoter?

3386.

There are two eukaryotic upstream promoters. The TATA


box is -25 base pairs upstream; the CAAT box is -75
bases upstream.

How many bases upstream is the


eukaryotic TATA box promoter?

3387.

There is a difference; it takes about 7 to 8 half-lives to


reach mathematical steady state and 4 to 5 half-lives to
reach clinical steady state.

Is there a difference between mathematical


and clinical steady states? If so, what are
their values (half-lives)?

3388.

There would be a Fab' region; thus, it would still be able to


participate in precipitation and agglutination.

What would be the result if an Ab were


cleaved with pepsin?

3389.

There would be two Fab and Fc regions.

What would be the result if an Ab were


cleaved with papain?

3390.

They are all formed by the fifth month of fetal life.

All primary oocytes in females are formed


by what age?

3391.

They are directly related; the greater the preload, the


greater the passive tension in the muscle and the greater
the prestretch of a sarcomere.

What is the relationship between preload


and the passive tension in a muscle?

3392.

They are in ganglia in the PNS.

Where are the postganglionic neuron cell


bodies, the CNS or the PNS?

3393.

They are in the grey matter of the CNS.

Where are the preganglionic neuron cell


bodies, the CNS or the PNS?

3394.

They are inversely proportional to each other; as pulse


pressure increases, compliance decreases.

How are pulse pressure and compliance


related?

3395.

They are inversely related. If ventilation increases, there will


be a decrease in PCO2 levels and vice versa.

What is the relationship between


ventilation and PCO2 levels?

3396.

They are preganglionic sympathetic axons. They are white


because they are myelinated.

What is the function of white rami


communicantes?

3397.

They block the midcycle surge of LH.

What is the physiologic basis for the


actions of birth control pills?

3398.

They chronically inhibit ADH secretion; when there is a


decrease in the blood volume, the stretch receptors send
fewer signals, and ADH is secreted.

What is the venous and arterial stretch


receptors' function regarding the
secretion of ADH?

3399.

They innervate LMNs.

What do UMNs innervate?

3400.

They innervate skeletal muscle.

What do LMNs innervate?

3401.

They prevent mast cell degranulation.

What cell type do cromolyn and


nedocromil affect for prophylactic
management of asthma via blockade?

3402.

Thiabendazole

What is the drug of choice for


threadworm, trichinosis, and larva
migrans?

3403.

Thiamine

What vitamin deficiency can result in


high-output cardiac failure?

3404.

Thiamine and folate

What two vitamins are inactivated when


they come in contact with acetaldehyde?

3405.

Thiazide diuretics

Which group of antihypertensive agents


best decreases left ventricular
hypertrophy?

3406.

Thick skin

What skin type on the palms and soles is characterized


by the absence of hair follicles and presence of stratum
lucidum?

3407.

Thiogalactoside transacetylase

Regarding the Lac operon, for what do the following


genes code? A gene

3408.

Thioridazine

What neuroleptic agent causes retinal deposits,


hypotension, and torsades de pointes?

3409.

Third aortic arch MS CARD is my mnemonic


for the aortic arch derivatives

From what aortic arch are the following structures


derived? Common and internal carotid arteries

3410.

Third M PITS for pharyngeal pouch


derivatives

From what pharyngeal pouch is the following structure


derived? Inferior parathyroid gland and thymus

3411.

Third week

During what embryonic week do somites begin to form?

3412.

Third week

What embryonic week sees the formation of the


notochord and the neural tube?

3413.

Third week

During what embryonic week does the intraembryonic


coelom form?

3414.

Third week; they are derived from the wall of


the yolk sac.

Blood and its vessels form during what embryonic


week?

3415.

This is the triad of renal cell carcinoma

What carcinoma produces hematuria, flank pain, and a


palpable mass?

3416.

Thoracic and sacral

What two regions of the vertebral column are


considered primary curvatures?

3417.

Thoracic splanchnic fibers

What preganglionic sympathetic fibers are responsible


for innervating the foregut and the midgut?

3418.

Thrombocytopenia, eczema, and


immunodeficiency is the triad of this X-linked
recessive disorder.

What is the triad of Wiskott-Aldrich syndrome?

3419.

Thrombus

What type of hemostasis in an intravascular space


consists of fibrin, platelets, RBCs, and WBCs?

3420.

Thrush

What is the term for Candida infection of the oral mucosa?

3421.

Thymic epithelial cells, dendritic cells,


and macrophages

What three cells are essential for T-cell maturation?

3422.

Thymidylate synthase

What enzyme of pyrimidine synthesis is inhibited by the


following? 5-FU

3423.

Thymidylate synthetase

What enzyme is blocked by 5-FU?

3424.

Thymine

Methylating uracil produces what pyrimidine base?

3425.

Thymine

What pyrimidine base is found Only in DNA?

3426.

Thymine (B1)

What vitamin is a component of the coenzyme thiamine


pyrophosphate (TPP)?

3427.

Thymus gland. (Thymus gland is


essential for T cell maturation.)

What encapsulated lymphoid organ is characterized by


presence of Hassall's corpuscles, and absence of germinal
centers and B cells?

3428.

Thymus-independent Ags

What is the term for Ags that activate B cells without T-cell
signaling?

3429.

Thyroarytenoid muscles

Name the laryngeal muscle described by the following:


Pulls the arytenoids cartilages closer to the thyroid, relaxing
the vocal ligaments and thereby decreasing the pitch

3430.

Thyroid gland

What gland is found in the muscular triangle of the neck?

3431.

Thyroid hormones are necessary for


conversion of carotene to vitamin A.

Why is hypothyroidism associated with night blindness?

3432.

Thyroid hormones increase serum


glucose levels by increasing the
absorption of glucose from the small
intestine.

What is the effect of T3 on the glucose absorption in the


small intestine?

3433.

Tibial nerve

What lower extremity nerve is described by the following


motor loss? Loss of flexion of the knees and toes,
plantarflexion, and weakened inversion

3434.

Ticlopidine or clopidogrel

Name one of the ADP receptor antagonists used in


patients who are post-MI, have had TIAs, have unstable
angina, and are allergic to ASA?

3435.

Tidal volume (VT)

Name the lung measurement based on the following


descriptions: The amount of air that enters or leaves
the lung system in a single breath

3436.

The time it takes ACh to diffuse to the


postjunctional membrane

What is the rate-limiting step in a conduction of a NMJ?

3437.

Tinea favosa (favus)

What is the most serious form of tinea capitis, which


results in permanent hair loss and is highly contagious?

3438.

Tip of the left ventricle

What part of the heart forms Apex?

3439.

Tip of the loop of Henle (1200 mOsm/L)

What region of the nephron has the highest osmolarity?

3440.

TLC, FRC, and RV have to be calculated.


(Remember, any volume that has RV as a
component has be calculated.)

What three lung measurements must be calculated


because they cannot be measured by simple
spirometry?

3441.

T lymphocyte

What is the main type of cell involved in cellular


immunity?

3442.

Tolcapone and entacapone

Inhibition of peripheral COMT, allowing increased CNS


availability of L-dopa, is accomplished by what two
agents?

3443.

Topoisomerase I (Relaxase)

What topoisomerase makes ssDNA cuts, requires no


ATP, relaxes supercoils, and acts as the swivel in front
of the replication fork?

3444.

To provide attachment between contiguous


cells and to maintain a semipermeable barrier

What are the functions of the zonula occludens and the


zonula adherens

3445.

Total and NREM sleep decrease


considerably as we age, but REM sleep
remains relatively constant (20%) up to age
80, then begins to decline.

In the elderly, what happens to total sleep time,


percentage of REM sleep, and percentage of NREM
sleep?

3446.

Total lung capacity (TLC)

Name the lung measurement based on the following


descriptions: Amount of air in the lungs after maximal
inspiration

3447.

Total ventilation (minute ventilation or minute


volume)

What is the term for the total volume of air moved in and
out of the respiratory system per minute?

3448.

Tourette's syndrome; it is usually first


reported by teachers as ADHD with
symptoms of obsessive-compulsive disorder
and learning disabilities.

What disease is described by the following


characteristics: multiple motor and vocal tics, average
age of onset 7, a M:F ratio of 3:1, and association with
increased levels of dopamine?

3449.

Toxic dose for 50% of drug takers (median


toxic dose)

What do the following values indicate? TD50

3450.

Toxic or viral hepatitis

Councilman bodies

3451.

Toxoplasma gondii

Which parasitic organism, when it crosses the placenta,


results in intracerebral calcifications, chorioretinitis,
microcephaly, hydrocephaly, and convulsions?

3452.

Toxoplasmosis

What is the most common opportunistic infection of the


CNS in HIV?

3453.

Toxoplasmosis (although you should rule out


cerebral abscess due to other organisms)

What infection is associated with ring-enhancing lesions


seen on computed tomography (CT) of the brain in an
HIV-positive individual?

3454.

Transcortical aphasia

Name the aphasia based on these characteristics:


Lesion in the prefrontal cortex; inability to speak
spontaneously; unimpaired ability to repeat

3455.

Transcortical apraxia; Wernicke's area of the


left hemisphere cannot communicate with the
right primary motor cortex because of the
lesion in the corpus callosum.

What disconnect syndrome results from a lesion in the


corpus callosum secondary to an infarct in the anterior
cerebral artery, so that the person can comprehend the
command but not execute it?

3456.

Transcription (C comes before L in the


alphabet, and transCription comes before
transLation)

What is the term for conversion of a dsDNA molecule


to the base sequence of an ssRNA molecule?

3457.

Transcription factor IID

In eukaryotes, what transcription factor binds to the


TATA box before RNA polymerase II can bind?

3458.

Transference. When it is from the physician to


the patient it is called countertransference.

What term is defined as a patient unconsciously


placing his or her thoughts and feelings on the
physician in a caregiver or parent role?

3459.

Transferrin

What transports iron in the blood?

3460.

Transient hypogammaglobinemia of infancy; it


usually resolves by age 16 to 30 months.

What is the term for a delay in the onset of normal IgG


synthesis seen in the fifth to sixth month of life?

3461.

Transient ischemic attack

What is the term for neurologic signs consistent with a


cerebrovascular accident but lasting 24 hours with full
recovery?

3462.

Transient protein C deficiency, because of its


relatively short half-life, may result if warfarin is
instituted alone.

What transient deficiency may result in a


hypercoagulable state if warfarin is given alone?

3463.

Transitional cell carcinoma

What is the most common one? Bladder tumor

3464.

Transitional epithelium. The distal portion of


the penile urethra is composed of stratified
epithelium.

What is the epithelial lining of the prostatic portion of


the urethra?

3465.

Transketolase and transaldolase. The


reactions they catalyze are reversible.

What are the nonoxidative enzymes of the HMP shunt?


Are the reactions they catalyze reversible or
irreversible?

3466.

Translation

What is the term for taking an mRNA molecule and arranging the
AA sequence forming a protein?

3467.

Transmembrane potential (an


absolute number)

What is the term for the potential difference across a cell


membrane?

3468.

Transport maximum (Tm) occurs


when all function carriers are
saturated and therefore is an index
of the number of functioning carriers.

What is used as an index of the number of functioning carriers for


a substance in active reabsorption in the kidney?

3469.

A transport maximum (Tm) system

What form of renal tubular reabsorption is characterized by low


back leaks, high affinity of a substance, and easy saturation? It is
surmised that the entire filtered load is reabsorbed until the
carriers are saturated, and then the rest is excreted.

3470.

Transposition of the great vessels


arises from a failure of the
aorticopulmonary septum to grow in
a spiral.

What right-to-left shunt occurs when the aorta opens into the
right ventricle and the pulmonary trunk opens into the left
ventricle?

3471.

Transposons

What are known as jumping genes?

3472.

Transtentorial (uncal)

Which type of cerebral herniation is associated with CN III palsy?

3473.

Transudative; exudative has the


opposite values and has an elevated
cellular content.

What is the term for edema that has LDH below 200, protein
level 2.5, and a specific gravity below 1.020?

3474.

Transverse abdominis

What abdominal muscle runs horizontally, contributes to the


posterior rectus sheath, and contributes to form the conjoint
tendon?

3475.

Transvestite fetishism

What is the term for having fantasies or dressing in female


clothes for sexual arousal by heterosexual men?

3476.

Trapezius and sternocleidomastoid

What two muscles do you test to see whether CN XI is intact?

3477.

Trazodone

What serotonin reuptake inhibitor's major sexual side effect is


priapism?

3478.

Treacher Collins syndrome

Mandibular hypoplasia, down-slanted palpebral fissures,


colobomas, malformed ears, and zygomatic hypoplasia
are commonly seen in what pharyngeal arch 1
abnormality?

3479.

The triad of respiratory depression, pinpoint


pupils, and coma in an IV drug abuser
suggests opioid overdose (e.g., heroin), and
you should give naloxone.

A known IV drug abuser goes to the ER with respiratory


depression and pinpoint pupils and in a semicomatose
state. What overdose do you suspect, and what agent will
you give to reverse its effects?

3480.

Triazolam

Which benzodiazepine has the shortest half-life?

3481.

Trichomonas vaginalis (treat with


metronidazole)

Which protozoal parasitic vaginal infection produces a


positive whiff test with KOH staining?

3482.

Trichophyton

Which genus of dermatophytes is associated with the


following sites of infection? Skin, hair, and nails

3483.

Trichuris trichiura is treated with


albendazole.

What nematode is known as whipworms? What is the


treatment?

3484.

Trimethoprim

Name the antimicrobial agent whose major side effect is


listed. Altered folate metabolism

3485.

Trimethoprim/sulfamethoxazole (TMP-SMX)

What is the DOC for the following protozoal infections?


Pneumocystis carinii pneumonia

3486.

Trisomy 12 (nearly 50% of patients with CLL


have abnormal karyotypes.)

What is the most common chromosomal abnormality


associated with CLL?

3487.

tRNA

Name the RNA subtype based on the following: Carries


AA to the ribosome for protein synthesis

3488.

Tropomyosin

What is the name of the regulatory protein that covers the attachment site on actin in
resting skeletal muscle?

3489.

TRUE

True or false? Monocytosis is seen in TB.

3490.

TRUE

True or false? Dopamine antagonists at the D2A receptor have the capacity to induce
pseudoparkinsonism.

3491.

TRUE

True or false? Klinefelter syndrome cannot be diagnosed until puberty.

3492.

TRUE

True or false? Ethyl alcohol induces the cytochrome P-450 enzymes.

3493.

TRUE

True or false? Somatic motor neurons innervate the striated muscle of the
bulbospongiosus and ischiocavernous muscles and result in ejaculation of semen.

3494.

TRUE

True or false? The epithelial lining of the urinary bladder and the urethra are embryologic
hindgut derivatives.

3495.

TRUE

True or false? The parasympathetic nervous system has very little effect on arteriolar
dilation or constriction.

3496.

TRUE

True or false? Live vaccines are contraindicated in patients with SCID.

3497.

TRUE

True or false? All streptococci are catalase-negative

3498.

TRUE

True or false? Below the arcuate line, all the aponeurotic fibers run anterior to the rectus
abdominis.

3499.

TRUE

True or false? All spore formers are gram positive.

3500.

TRUE

True or false? All of the hormones in the hypothalamus and anterior pituitary gland are
water soluble.

3501.

TRUE

True or false? Succinylcholine is a nicotinic receptor agonist.

3502.

TRUE

True or false? There are no central O2 receptors.

3503.

TRUE

True or false? NSAIDs have analgesic, antipyretic, anti-inflammatory, and antiplatelet


activities.

3504.

TRUE

True or false? Progestins are used in combination with estrogens to decrease the risk of
endometrial cancer.

3505.

TRUE

True or false? All Proteus species are urease positive.

3506.

TRUE

True or false? Acetaminophen has analgesic and antipyretic activities but lacks antiinflammatory effects.

3507.

True; 80% have visited a doctor in the previous 6 months. And 50%
within the last month!

True or false? Four-fifths of


those who attempt suicide first
give a warning.

3508.

True. Adipose depends on glucose uptake for dihydroxyacetone


phosphate (DHAP) production for triglyceride synthesis.

True or false? Adipose tissue


lacks glycerol kinase.

3509.

True; also, stressful marriage, early sexual experiences in the back of


a car, and sex with a prostitute all increase the risk of premature
ejaculation.

True or false? Being college


educated increases a man's risk
of having premature ejaculation.

3510.

True. Although the CSF normally contains 0 to 4 lymphocytes or


monocytes, the presence of PMNs is always considered abnormal.

True or false? The presence of


PMNs in the CSF is always
abnormal.

3511.

True. Aluminum reacts with PO4, resulting in AlPO4, an insoluble


compound that cannot be absorbed.

True or false? All aluminumcontaining antacids can cause


hypophosphatemia.

3512.

True. Aminoglycosides use O 2-dependent uptake and therefore are


ineffective for treatment of anaerobic infections.

True or false? Anaerobes are


resistant to the effects of
aminoglycosides.

3513.

True. At high doses ASA decreases uric acid reabsorption in the renal
tubules, resulting in uricosuric actions. At low doses ASA decreases
tubular secretion of uric acid, leading to hyperuricemia.

True or false? At high doses


aspirin has uricosuric properties.

3514.

True. A thrombus has platelets, but clots do not.

True or false? Blood clots lack


platelets.

3515.

True. B cell antigen receptors are Abs.

True or false? B-cell Ag


receptors can be secreted.

3516.

True. Because of intrapulmonary shunting, there is a slight decrease


in PO2 and increase in PCO2 between the pulmonary end capillary
blood and the systemic arterial blood.

True or false? The alveolar PO2


and PCO2 levels match the
pulmonary end capillary blood
levels.

3517.

True. Being female, having positive symptoms, quick onset, and


family history of mood disorders are all good prognostic predictors of
schizophrenia.

True or false? Paranoid and


catatonic schizophrenia are
good prognostic predictors.

3518.

True. Blood flows from the portal tracts (distal) to


the central vein (proximal), so it is the first area
affected during hypoxia.

True or false? The central vein of the liver lobule is


the first area affected during hypoxia.

3519.

True. Cimetidine can decrease androgen


production and lead to gynecomastia.

True or false? Gynecomastia is a side effect of


cimetidine.

3520.

True. Common variable hypogammaglobinemia


first appears by the time patients reach their 20s
and is associated with a gradual decrease in Ig
levels over time.

True or false? Patients with common variable


hypogammaglobinemia have B cells in the
peripheral blood.

3521.

True; cortisol inhibits glucose uptake in most


tissue, making it available for neural tissue use.

True or false? Cortisol inhibits glucose uptake in


skeletal muscle.

3522.

True. Delta-Cell tumors produce excess


somatostatin, which inhibits CCK, gastrin
(hypochlorhydria), and insulin secretion
(diabetes).

True or false? Pancreatic delta-cell tumors inhibit


CCK secretion, leading to gallstones and
steatorrhea.

3523.

True. DNA polymerases have 3'-5' exonuclease


activity for proofreading.

True or false? DNA polymerases can correct


mistakes, whereas RNA polymerases lack this
ability.

3524.

True. Don't forget this in your differential


diagnosis of an asthmatic.

True or false? GERD is a cause of asthma.

3525.

True. Don't forget this list; you will be asked.

True or false? All of the following are risk factors for


cervical cancer: multiple pregnancies, early age of
intercourse, multiple sexual partners, OCP use,
smoking, HIV, and STDs.

3526.

True. Elevated plasma levels of progesterone can


raise the body temperature 0.5 to 1.0F.

True or false? Progesterone has thermogenic


activities.

3527.

True. Fractures and trauma, however, are the


most common causes.

True or false? Sickle cell anemia, Caisson disease,


chronic steroid use, and Gaucher disease are
causes of avascular necrosis of bone.

3528.

True hermaphrodism

What condition is defined by


both testicular and ovarian
tissues in one individual?

3529.

True. High-frequency sound waves stimulate the hair cells at the base of
the cochlea, whereas low-frequency sound waves stimulate hair cells at
the apex of the cochlea.

True or false? Highfrequency sound waves


stimulate hair cells at the
base of the cochlea.

3530.

True. HPV serotypes 16 and 18 are risk factors for squamous cell
carcinoma.

True or false? HPV infection


increases the risk of
developing squamous cell
carcinoma of the penis.

3531.

True. Hyperpolarization inhibits the postsynaptic membrane.

True or false? Depolarization


of the postsynaptic
membrane excites the
neuron.

3532.

True if she is Rho-negative; false if she is Rho-positive

True or false? If a mother


delivers a Rho-positive baby,
she should receive Rhoimmunoglobulin.

3533.

True. Increased alkaline phosphatase levels also are associated with


increased bone formation.

True or false? An elevated


serum osteocalcin level is a
marker for increased bone
formation.

3534.

True. Increased renin and AT II levels occur as a result of the decreased


production of aldosterone.

True or false? Renin


secretion is increased in 21-hydroxylase deficiency.

3535.

True. In positively skewed distributions the mode is less than the median
is less than the mean.(Remember to name a skewed distribution: the tail
points in the direction of its name. positive skew tails point to the positive
end of a scale.)

True or false? In a positively


skewed curve the mean is
greater than the mode.

3536.

True. In the distal colon, sweat glands, and salivary ducts, aldosterone
has sodium-conserving effects.

True or false? Aldosterone


has a sodium-conserving
action in the distal colon.

3537.

True. IQ correlates well with education and academic achievement but is


not a predictor of success.

True or false? There is a


strong positive correlation
between IQ and academic
achievement.

3538.

True. It deposits at the gingivodental line, known as the lead line.

True or false? Excess lead


deposits in the oral cavity.

3539.

True. It is considered medical treatment, so it can be withdrawn or


refused. (Remember the Cruzan case.)

True or false? A patient can


refuse a feeding tube.

3540.

True; it requires both a concentration gradient and ATP to work.

True or false? Active protein


transport requires a
concentration gradient.

3541.

True; military service and independent self-care by a child over 13 years


old also emancipate.

True or false? Marriage


emancipates a child less
than 17 years old.

3542.

True. Muscle spindles are modified skeletal muscle fibers. They are the
sensory component of the stretch reflexes.

True or false? Intrafusal


fibers form muscle spindles.

3543.

True. No loss of drug and no absorption if a drug is given IV.

True or false? There is no


absorption of an IVadministered drug.

3544.

True. on the right lung the oblique fissure divides the middle from the
inferior lobe and the horizontal fissure further divides the middle from the
upper lobe. On the left the oblique divides the superior from the inferior
lobe.

True or false? both the left


and right lungs have an
oblique fissure?

3545.

True. Pancreatic enzymes begin the breakdown of


vitamin B12-R complex in the duodenum.

True or false? Pancreatic insufficiency results


in vitamin B12 malabsorption.

3546.

True. PIF is dopamine in the tuberoinfundibular system.

True or false? Prolactin levels can serve as a


rough indicator of overall dopamine activity.

3547.

True. Probenecid can precipitate uric acid crystals in


the kidney if the patient is an overexcretor of urate.

True or false? You would not prescribe


probenecid for a patient with gout who is an
overexcretor of uric acid.

3548.

True. Remember, all nucleated cells (and platelets)


have MHC class I Ags, and RBCs are not nucleated.

True or false? RBCs do not have MHC class I


Ags on their surface.

3549.

True. (Remember, blood flows from the portal tracts to


the central vein, so it is the first area to receive blood
and therefore oxygen.)

True or false? The portal tract of the liver


lobule is the first area to be oxygenated in the
liver.

3550.

True. Remember, carbamoyl phosphate synthetase and


ornithine transcarbamoylase are mitochondrial
enzymes.

True or false? The urea cycle takes place in


both the cytoplasm and the mitochondria.

3551.

True. Remember, it degenerates 4 to 5 days post


fertilization, and implantation occurs 7 days post
fertilization!

True or false? For implantation to occur the


zona pellucida must degenerate.

3552.

True. Remember, RBCs do not contain mitochondria,


so they cannot metabolize aerobically.

True or false? RBCs anaerobically use


glucose in both the well-fed and fasting states.

3553.

True. Remember, the second X chromosome is


inactivated, and so is the Barr body. Turner syndrome
has only one X chromosome.

True or false? Patients with Turner syndrome


have no Barr bodies.

3554.

True. Restriction endonucleases cut only unmethylated


DNA.

True or false? Methylation of bacterial DNA


prevents restriction endonuclease from cutting
its own chromosomes.

3555.

True. Sarcoplasmic calcium-dependent ATPase


supplies the energy to terminate contraction, and
therefore it is an active process.

True or false? In skeletal muscle relaxation is


an active event.

3556.

True. Scl-70 Abs are used in diagnosing diffuse


scleroderma.

True or false? Anticentromere Abs are used in


diagnosing CREST syndrome.

3557.

True. So are rubeola, smallpox, yellow fever, and the


Sabin polio vaccine.

True or false? All of the following are live


attenuated vaccines available in the United
States: measles, mumps, varicella zoster, and
Francisella tularensis.

3558.

True. Staphylococcus aureus and group D


enterococci also grow in high-salt media.

True or false? Vibrio parahaemolyticus require


NaCl in its growth medium.

3559.

True. The elimination of the unfertilized egg is


menses.

True or false? In females, meiosis II is


incomplete unless fertilization takes place.

3560.

True. The energy status of the cell dictates if the


cycle is running or relaxing.

True or false? There is no hormonal control to


the TCA cycle.

3561.

True; the greater the fat, the less the total body
water.

True or false? There is an inverse relationship


between fat content and total body water.

3562.

True. The ileum is the site where vitamin B12 is


absorbed.

True or false? Removal of the ileum results in


vitamin B12 deficiencies.

3563.

True. The lack of dopamine leads to an excess of


ACh, hence extrapyramidal dysfunction.

True or false? The lack of dopamine production


in the substantia nigra leads to the
extrapyramidal dysfunction, characteristically
seen in patients with Parkinson's disease.

3564.

True. The laryngotracheal (respiratory) diverticulum


is divided from the foregut by the tracheoesophageal
septum. -------------------------------------------------------------------------------

True or false? The respiratory system is derived


from the ventral wall of the foregut.

3565.

True. The odontoid process of C2 acts as the


vertebral body of C1 allowing lateral rotation of the
head.

True or false? The first cervical vertebra has no


vertebral body.

3566.

True. The thyroid gland, the lungs, and the


pharyngeal pouches are foregut derivatives that are
not a component of the gastrointestinal system.

True or false? The thyroid gland is an


embryologic foregut derivative.

3567.

True. They absorb nutrients from the


host's GI tract.

True or false? Cestodes have no GI tract.

3568.

True. They all have helical nucleocapsids


and virion-associated polymerases too.

True or false? All negative sense RNA viruses are


enveloped.

3569.

True. They are also necessary for normal


brain maturation.

True or false? Thyroid hormones are necessary for normal


menstrual cycles.

3570.

True. They are evaginations of the


surface membranes and therefore
extracellular.

True or false? In a skeletal muscle fiber, the interior of the Ttubule is extracellular.

3571.

True. (They are quite a busy bunch of


cells!)

True or false? The following are functions of hepatocytes:


protein production, bile secretion, detoxification, conjugation,
and lipid storage.

3572.

True. They are the sensory component of


a spinal reflex.

True or false? Neurons in the dorsal horn participate in


reflexes.

3573.

True. They are useful in the early-phase


response to an asthmatic attack.

True or false? Beta2-Adrenergic agents are used in the


management of an acute asthmatic attack.

3574.

True. They block the dopamine receptors


in the chemoreceptor trigger zone.

True or false? Dopamine antagonists are antiemetic.

3575.

True. They have no gates, so by


definition they are always open.

True or false? Ungated channels are always open.

3576.

True. They inhibit DNA gyrase (topo II).

True or false? Fluoroquinolones are bactericidal.

3577.

True. They lyse the host cell

True or false? There are no persistent infections with naked


viruses?

3578.

True. Transcription (conversion of DNA to


RNA), as well as replication, occurs in
the nucleus.

True or false? The nucleus is the site of transcription.

3579.

True. Without ADH hypotonic urine would


be formed.

True or false? Without ADH the collecting duct would be


impermeable to water.

3580.

True. Yohimbine, sodium lactate, and epinephrine can also induce


panic attacks; they are considered panicogens.

True or false? Panic attacks


can be induced by
hyperventilation or carbon
dioxide.

3581.

True. -Blockade would result in unopposed-adrenergic stimulation


and worsen the patient's symptoms. Calcium channel blockers are the
way to go.

True or false? Cocaineinduced coronary ischemia


should not be treated with blockers.

3582.

True. -Blockers are a component of CHF regimen for stable patients,


not those who are decompensated. (This means you discharge a
patient from the hospital with a prescription for a -blocker; you don't
start -blockers when the patient is admitted.)

True or false? -Blockers are


contraindicated in patients
who present to the hospital
with CHF.

3583.

Truncus (1) arteriosus Transposition of the (2) great vessels


Tri(3)cuspid atresia Tetra(4)logy of Fallot They all begin with T

Name the four right-to-left


congenital cardiac shunts.

3584.

Truncus arteriosus

From what embryonic


structure are the following
structures derived? The
ascending aorta and the
pulmonary trunk

3585.

Trypanosoma cruzi

What hemoflagellate species


is the cause of Chagas
disease?

3586.

Trypsin

What enzyme is needed to


activate the following
reactions?
Chymotrypsinogen to
chymotrypsin

3587.

Trypsin

What enzyme is needed to


activate the following
reactions?
Procarboxypeptidase to
carboxypeptidase

3588.

Tryptophan

What AA is a precursor of the


following substances?
Serotonin

3589.

Tryptophan

What AA is a precursor of the following substances? NAD

3590.

The tsetse fly

What is the vector of African sleeping sickness?

3591.

TSH. 5-HT and prolactin increase during


sleep, and dopamine levels decrease
during sleep.

What pituitary hormone is inhibited during sleep?

3592.

The T-test (used when comparing two


groups)

What statistical test checks to see whether the groups are


different by comparing the means of two groups from a
single nominal variable?

3593.

TTP; thrombocytopenia with


megathrombocytes is more characteristic
of ITP.

If a peripheral blood smear shows schistocytes,


reticulocytes, and thrombocytopenia, is it more commonly
seen in patients with ITP or TTP?

3594.

Turcot syndrome

What syndrome has multiple adenomatous colonic polyps


and CNS gliomas?

3595.

Turner syndrome

Streaky ovaries

3596.

T wave

What component of an ECG is associated with the


following? Ventricular repolarization

3597.

Twelve pairs through twelve thoracic


vertebrae. Totaling 31 pairs of spinal
nerves.

How many pairs of spinal nerves are associated with


Thoracic vertebrae?

3598.

Two Lateral foramina of Luschka and 1


Medial foramen of Monroe (L for Lateral
and M for Medial)

What are the three sites where CSF can leave the
ventricles and enter the subarachnoid space? (Name the
lateral and the medial foramina.)

3599.

Two major or one major and two minor

How many major and/or minor Jones criteria are required


for the diagnosis of rheumatic fever?

3600.

Two-way ANOVA. It allows the test to


check several variables at the same time.

What statistical test compares the means of groups


generated by two nominal variables by using an interval
variable?

3601.

TXA2

Name the product or products of arachidonic acid:


Vasoconstriction and platelet aggregation produced by
platelets

3602.

TXA2

What potent platelet aggregator and vasoconstrictor is


synthesized by platelets?

3603.

Tylosis

What AD disease involves hyperkeratosis of the palms and soles in


association with esophageal carcinoma?

3604.

Type 2

What type of neurofibromatosis is associated with bilateral


acoustic schwannomas?

3605.

Type I error (alpha error).


(Remember it as saying something
works when it doesn't.) The
chance of a type I error occurring
is the P value.

What type of error is made if you reject the null hypothesis when it
is true?

3606.

Type I hypersensitivity
(anaphylactic)

Name the type of hypersensitivity reaction based on the following


properties. IgE-mediated release of chemical mediators from
basophils and mast cells; need prior exposure to Ag in the past;
eosinophils amplify and continue reaction; can be system or
localized.

3607.

Type II

Which phenotype of osteogenesis imperfecta is incompatible with


life?

3608.

Type II error (beta error).


(Remember it as saying something
doesn't work when it does.)

What type of error is made if you accept the null hypothesis when it
is false?

3609.

Type II hypersensitivity (cytotoxic)

Name the type of hypersensitivity reaction based on the following


properties. IgG or IgM Abs against a specific target cell or tissue;
complement-dependent or ADCC.

3610.

Type II hypersensitivity reaction

What type of hypersensitivity is an Ab-mediated response against


our own cells, receptors, or membranes via IgG or IgM?

3611.

Type III

What type of collagen is associated with keloid formation?

3612.

Type III hypersensitivity (immune


complex)

Name the type of hypersensitivity reaction based on the following


properties. Circulating Ab-Ag immune complexes deposited in the
tissue result in neutrophil attraction and the release of lysosomal
enzymes.

3613.

Type III hypersensitivity reaction

What type of hypersensitivity is a result of high


circulating levels of soluble immune complexes
made up of IgG or IgM Abs?

3614.

Type II pneumocytes

In the alveoli, what cell type is responsible for


producing surfactant?

3615.

Type II symptoms (negative); otherwise healthy


persons have them, but schizophrenics do not.

What type of symptoms in schizophrenia are


associated with Muscarinic receptors (ACh)?

3616.

Type I (makes sense, since they have a


predisposition for fractures and type I collagen is
associated with bones and tendons)

What type of collagen is abnormal in patients with


osteogenesis imperfecta?

3617.

Type I pneumocytes

In the alveoli, what cell type is for gas exchange?

3618.

Type I symptoms (positive); schizophrenics have


them, but otherwise healthy persons do not.

What type of symptoms in schizophrenia are


associated with Dopamine receptors?

3619.

Type IV collagen

What type of collagen is associated with the


basement membrane?

3620.

Type IV collagen

Goodpasture Ag is a component of what type of


collagen?

3621.

Type IV hypersensitivity (cell-mediated)

Name the type of hypersensitivity reaction based


on the following properties. Reaction-mediated by
sensitized T-cells

3622.

Type IV hypersensitivity reaction (delayed type


because of the 48-96 hour latency)

What type of hypersensitivity is a T cell-mediated


response to Ags that are not activated by Ab or
complement?

3623.

Tyramine

What agent, in combination with a MAOI inhibitor,


can cause hypertensive crisis?

3624.

Tyrosine

What AA is a phenol?

3625.

Ulcerative colitis

What inflammatory bowel disorder is continuous,


with extensive ulcerations and pseudopolyps, and
is associated with HLA-B27?

3626.

Ulcerative colitis

What disease is seen in the 20-to 40-year-old age group, is


more prevalent in women than men, involves diarrhea with or
without bloody stools, starts in the rectum and ascends without
skipping areas, includes pseudopolyps, and has a thickness of
the bowel that does not change?

3627.

Ulcerative colitis

Is ulcerative colitis or Crohn disease more commonly associated


with primary sclerosing cholangitis?

3628.

Ulnar and radial arteries (ulnar is the


main supplier)

What two arteries join to form the superficial and deep palmar
arches of the hand?

3629.

Ulnar nerve

What nerve is associated with the following functions?


Innervation of the flexor carpi ulnaris, flexor digiti profundus
(pinky and ring fingers), and the intrinsic muscles of the hand

3630.

Ulnar to radial progression

In regard to motor development during infancy, choose the


motor response that happens first. Radial or ulnar progression

3631.

Ultralente

Name the insulin preparation based on the peak effect and


duration of action. Peak, 8 to 16 hours; duration, 24 to 36 hours

3632.

Ultraviolet B (UVB) sunlight

What form of sunlight is the most carcinogenic?

3633.

Umbilical vein and ductus venosus


(80%)

What two vessels in fetal circulation have the highest PO2


levels?

3634.

UMP

What is the primary end product of pyrimidine synthesis?

3635.

Unconjugated free bilirubin

What form of bilirubin can cross the blood-brain barrier?

3636.

Unconjugated (indirect)

What form of bilirubin is carried on albumin?

3637.

Under normal resting conditions no,


but they are strongly stimulated when
PO2 arterial levels decrease to 50 to
60 mm Hg, resulting in increased
ventilatory drive.

Do the PO2 peripheral chemoreceptors of the carotid body


contribute to the normal drive for ventilation?

3638.

Under resting conditions expiration is considered


a passive process; therefore, no muscles are
used. In the active state the abdominal muscles
can be considered the major muscle of
expiration.

What is the major muscle used in the relaxed state


of expiration?

3639.

Undoing

Fixing impulses by acting out the opposite of an


unacceptable behavior

3640.

Ungated potassium channel

Name the ventricular muscle membrane channel:


Always open

3641.

Unipolar disorder (major depression)

Anhedonia, lack of motivation, feelings of


worthlessness, decreased sex drive, insomnia, and
recurrent thoughts for at least 2 weeks, representing
a change from previous level of function, describes
what disorder?

3642.

Unstable (crescendo) angina

What form of angina is characterized by Coronary


artery nonocclusive thrombus; symptom occurrence
with increasing frequency, duration, intensity, and
decreasing activity, frequently at rest?

3643.

Upper outer quadrant (left more than right)

In which quadrant is breast cancer most commonly


found?

3644.

Upper third skeletal muscle, middle third both


skeletal and smooth muscle, and lower third
smooth muscle

What is the rule of one-third regarding muscle type of


the esophagus?

3645.

Up to 72 hours; the ovum losses its ability to be


fertilized 8 to 25 hours after release.

Up to how many hours post ejaculation do sperm


retain their ability to fertilize the ovum?

3646.

Urachus

What embryonic structure forms the following adult


structures? Median umbilical ligament

3647.

Uracil

What pyrimidine base is found Only in RNA?

3648.

Uracil

What pyrimidine base is produced by deaminating


cytosine?

3649.

Uracil and orotic acid levels increase with ornithine


transcarbamoylase deficiency and are normal in
carbamoyl phosphate synthetase deficiency.

How can a genetic deficiency of carbamoyl


phosphate synthetase be differentiated from an
ornithine transcarbamoylase deficiency?

3650.

Ureaplasma urealyticum

Which mycoplasma species is associated with


urethritis, prostatitis, and renal calculi?

3651.

Uric acid

What is the end product of purine catabolism?

3652.

Urinary 17-ketosteroids

What is used as an index for both adrenal and


testicular androgens?

3653.

Urinary 17-OH steroids

What is used as an index of cortisol secretion?

3654.

Urinary trigone

What is the name of the urinary bladder where


the ureters enter and the urethra exits?

3655.

The urine becomes hypertonic because of water


reabsorption in the collecting duct.

What happens to the tonicity of the urine with


increased ADH secretion?

3656.

Urogenital folds

What embryonic structure forms the adult male


structure? Ventral part of the penis

3657.

Urogenital folds

What embryonic structure forms the adult


female structures? Labia minora

3658.

Urogenital sinus

What embryonic structure forms the adult


female structures? Urinary bladder, urethra,
greater vestibular glands, vagina

3659.

Urogenital sinus

What embryonic structure forms the adult male


structure? Urinary bladder, urethra, prostate
gland, bulbourethral gland

3660.

Urogenital sinus

What embryonic structure forms the following


adult structures? Urinary bladder and urethra

3661.

Urophilia

What paraphilia is defined as Combining sex


with urination?

3662.

Uroporphyrinogen III cosynthase

What enzyme is deficient in congenital erythropoietic porphyria?

3663.

Uroporphyrinogen-I synthase

What enzyme of heme synthesis is deficient in the autosomal


dominant disorder acute intermittent porphyria?

3664.

Uroporphyrinogen I synthetase

What enzyme is deficient in acute intermittent porphyria?

3665.

Utricle and saccule

What component of the inner ear Contains endolymph and gravity


receptors monitoring linear acceleration and deceleration of the
head, noting changes in head position?

3666.

The vaccine contains 23


capsular polysaccharides.

How many strains of Pneumococcus capsular polysaccharides are


present in the vaccine?

3667.

The vagina

What area of the female reproductive tract is lined by stratified


squamous epithelium rich in |glycogen?

3668.

Vaginismus; it is the female


counterpart of premature
ejaculation.

What is the term for involuntary constriction of the outer third of the
vagina to prevent penile penetration?

3669.

Vagus nerve

What CN is associated with the sensory innervation of


Laryngopharynx?

3670.

Valence

What is the term for the number of Ag-binding sites on an Ig?

3671.

Validity (remember, reliability is


necessary but not the only thing
needed for validity)

What is the term for the degree to which a test measures what it is
intended to measure?

3672.

Valine

What AA is substituted for glutamic acid at position 6 on the Betachain in patients with sickle cell anemia?

3673.

Valproic acid

Which anticonvulsant is also used in the treatment of bipolar disorder


and for migraine headaches?

3674.

Vancomycin

What bactericidal agent is the DOC for MRSA, enterococci, and


Clostridium difficile?

3675.

Vancomycin

Name the antimicrobial agent whose major side effect is listed.


Auditory toxicity

3676.

Vanillylmandelic acid (VMA)

What urinary metabolite is elevated in pheochromocytoma?

3677.

Variable interval

What type of scheduled reinforcement states that after


a desired response, the reinforcement is given
Varying in time?

3678.

Variable ratio If it is based on time, it is an


interval, and if it is based on the number of
responses, it is a ratio.

What type of scheduled reinforcement states that after


a desired response, the reinforcement is given
Varying in the number of responses?

3679.

Varicella

What virus lies dormant in the Dorsal root ganglia?

3680.

Varicella virus and influenza virus

What two viruses are associated with Reye's


syndrome?

3681.

Varicocele

What is the term for dilated veins within the spermatic


cord?

3682.

Variola (smallpox)

With which DNA virus are Guarnieri bodies


associated?

3683.

Variola virus (Smallpox)

What virus is associated with Guarnieri bodies?

3684.

Vascular dementia. It is characterized as a


stepwise deterioration in cognitive function.

What is the term for the dementia characterized by


decremental or patchy deterioration in cognitive
function due to a cerebrovascular accident?

3685.

Velocity is inversely related to cross-sectional


area.

How is velocity related to the total cross-sectional area


of a blood vessel?

3686.

The venae recta maintain the gradient via


countercurrent flow.

What maintains the osmotic gradient that is critical to


the concentrating ability of the kidney?

3687.

Venlafaxine. (It also has a mild dopaminergic


effect.)

What antidepressant, which recently was approved for


general anxiety disorder, inhibits the reuptake of NE
and 5-HT?

3688.

Venous pressure above the heart is


subatmospheric, so a puncture there has the
potential to introduce air into the system.

Why would a puncture to a vein above the heart have


the potential to introduce air into the vascular system?

3689.

Ventilation of an unperfused lung segment is


highly suspicious for pulmonary embolism.

If you order a V/Q scan for suspected pulmonary


emboli, is the filling defect seen on the ventilation or
perfusion side?

3690.

Ventral anterior nucleus

Name the thalamic nucleus based on its input and output. Input from
globus pallidus and substantia nigra; output to primary motor cortex

3691.

Ventral lateral nucleus

Name the thalamic nucleus based on its input and output. Input from
globus pallidus and the cerebellum; output to the primary motor cortex

3692.

Ventral posterolateral
nucleus

Name the thalamic nucleus based on its input and output. Input from
medial lemniscus and the spinocerebellar tracts; output to the primary
somatosensory cortex

3693.

Ventral posteromedial
nucleus

Name the thalamic nucleus based on its input and output. Input from
the trigeminal pathways; output to primary somatosensory cortex of the
parietal lobe

3694.

Ventricular contraction Atrial,


Contraction, Venous

On the venous pressure curve, what do the following waves represent?


C wave?

3695.

Ventricular fibrillation (V fib)

Name the most common type or cause. Sudden cardiac death

3696.

Ventromedial nucleus;
lesions here result in
obesity.

What nucleus of the hypothalamus is the satiety center, regulating food


intake?

3697.

Verbigeration

What term describes senseless repetition of words or phrases?

3698.

The vermis

In a topographical arrangement of the cerebellar homunculus map, what


area or lobe Controls the axial and proximal musculature of the limbs?

3699.

Vertebra prominens (C7 in


70% of cases, C6 in 20%,
T1 in 10%)

What is the name for the most prominent spinous process?

3700.

Very superior (<2.5% of the


population)

What is the label given to an individual whose IQ is 130

3701.

Vesicle (e.g., poison ivy)

What is an elevated, fluid-filled cavity between skin layers up to 0.5 cm?

3702.

Vesicular stomatitis virus

What virus causes hoof-and-mouth disease?

3703.

The vestibulo-ocular reflex

What reflex enables the eyes to remain focused on a target while the
head is turning?

3704.

Vibrio cholera

What organism is associated with the following types of


diarrhea? Rice water stools

3705.

Vibrio parahaemolyticus

What organism is associated with the following types of


diarrhea? Diarrhea associated with raw or undercooked
shellfish

3706.

Vibrio vulnificus

What is the most likely organism causing cellulitis in a


patient who was cut by an oyster shell?

3707.

Vimentin

What is the name of the protein coat that surrounds the


nuclear envelope?

3708.

VIP is an inhibitory parasympathetic


neurotransmitter that results in relaxation
of the lower esophageal sphincter.

What hormone controls relaxation of the lower esophageal


sphincter during swallowing?

3709.

VIPoma

What pancreatic islet cell tumor is associated with watery


diarrhea, hypokalemia, and achlorhydria?

3710.

Viral meningitis

What condition results in the following CSF results?


Opening pressure 100 mm H2O; 120 WBCs (90%
lymphocytes); normal glucose levels; protein 17

3711.

Virchow triad, associated with the


formation of a thrombus.

What triad consists of endothelial injury, changes in laminar


flow, and hypercoagulation?

3712.

Viruses

Which of the following characteristics accurately describe


fungi, bacteria, viruses, or parasites? Acellular; some are
enveloped; replicate within the host cell; no cell walls

3713.

Visceral pleura

What type of pleura is adherent to the surface of the organ?

3714.

Viscosity and resistance are proportionally


related. The greater the viscosity, the
greater the resistance is on the vessel.

How are resistance and viscosity related regarding flow?

3715.

Vital capacity (VC)

Name the lung measurement based on the following


descriptions: Maximal volume that can be expired after
maximal inspiration

3716.

Vitamin A

What vitamin is an important component of


rhodopsin?

3717.

Vitamin A is responsible for vision and epithelial


health.

What vitamin is necessary for epithelial health?

3718.

Vitamin B6

What vitamin deficiency may result in sideroblastic


anemia?

3719.

Vitamin B 6 (pyridoxine)

What vitamin is a cofactor of aromatic amino acid


decarboxylase and decreases the efficacy of L-dopa
if used concomitantly?

3720.

Vitamin B12 deficiency and folate deficiency

What are the two reasons for megaloblastic anemia


with elevated MCV?

3721.

Vitamin C

What water-soluble-vitamin deficiency is associated


with poor wound healing, easy bruising, bleeding
gums, anemia, and painful glossitis?

3722.

Vitamin D

What causes an increase in bone mineralization and


Ca2+ along with PO4- absorption from the GI tract
and kidney tubules?

3723.

Vitamin D3

What hormone is characterized by the following


renal effects? Calcium reabsorption, phosphate
reabsorption

3724.

Vitamin E

What fat-soluble vitamin is connected to selenium


metabolism?

3725.

Vitamin K

Name the antidote. Warfarin

3726.

Vitamin K deficiency has normal bleeding time


and increased PT, and vitamin C deficiency has
increased bleeding time and normal PT.

How can you differentiate vitamin K from vitamin C


deficiency by bleeding time and PT levels?

3727.

Vitelline fistula

What is the term for a direct connection between the


intestine and the external environment through the
umbilicus because the vitelline duct persists?

3728.

Vitreous humor

What is the fluid of the posterior compartment of the


eye?

3729.

VLDL

Name the lipoprotein based on the following characteristics. apo E,


apo B-100, apo C-II

3730.

VLDLs

In what form are triglycerides sent to adipose tissue from the liver?

3731.

Vmax

What is the maximum rate possible with a given amount of enzyme?

3732.

Volkmann's contracture

What is the term when the brachial artery is compressed, resulting in


ischemic contracture of the hand?

3733.

Voltage-gated calcium channel

Name the ventricular muscle membrane channel: Closed at rest;


depolarization causes channels to open slowly

3734.

Voltage-gated channel

What type of membrane channel opens in response to


depolarization?

3735.

Voltage-gated potassium
channels

Name the ventricular muscle membrane channel: Open at rest;


depolarization is stimulus to close; begin to reopen during the plateau
phase and during repolarization

3736.

Voltage-gated sodium channel

Name the ventricular muscle membrane channel: Closed at rest;


depolarization causes channels to open quickly; will not respond to a
second stimulus until cell is repolarized.

3737.

Volume of distribution is
decreased when a large
percentage of drug is protein
bound.

What is the volume of distribution (increased or decreased) of a drug


when a large percentage is protein bound?

3738.

Volvulus

What is the term for a twisting of the bowel around its vascular axis
resulting in intestinal obstruction?

3739.

von Gierke's disease

What glycogen storage disease is due to the following enzyme


deficiencies? Glucose-6-phosphatase

3740.

von Hippel-Lindau syndrome

In what syndrome does the patient have angiomatosis; renal cell


carcinomas; pheochromocytomas; retinal, cerebellar, medulla, or
spinal cord hemangioblastomas; and epidermal cysts?

3741.

Voyeurism

What paraphilia is defined as Deriving sexual pleasure from


watching others having sex, grooming, or undressing?

3742.

VPL nucleus sends its fibers to


synapse in the postcentral gyrus
of the parietal lobe.

The fibers of nucleus gracilis and nucleus cuneatus cross at the


medullary decussation and ascend contralateral to what thalamic
relay nucleus?

3743.

The V/Q ratio increases, since


the area is ventilated but
hypoperfused as a result of the
occlusion.

What happens to V/Q ratio if a thrombus is lodged in the pulmonary


artery?

3744.

VSD

Name the most common type or cause. Pulmonary HTN in children

3745.

VSD

What is the most common one? Congenital cardiac anomaly

3746.

VSD, RVH, overriding aorta, and


pulmonary stenosis

What is the tetrad of tetralogy of Fallot?

3747.

V wave

What wave is the cause of the following venous pulse deflections?


The rise in right atrial pressure secondary to blood filling and
terminating when the tricuspid valves opens

3748.

Waldenstrm's
macroglobulinemia

What lymphoma is associated with bleeding and cryoglobulin


precipitation at low temperatures, headache and confusion due to
hyperviscosity, IgM M-protein spike on serum electrophoresis, and
Russell bodies?

3749.

Waldeyer's ring

Where are the tonsillar tissues?

3750.

The wall tension is greater


because the aneurysm has a
greater radius than the
surrounding vessel.

Using Laplace's relationship regarding wall tension, why is the wall


tension in an aneurysm greater than in the surrounding normal blood
vessel's wall?

3751.

Water

What is absorbed in the gallbladder to concentrate bile?

3752.

Water flows from a low-solute to


high-solute concentrations.

In regards to solute concentration, how does water flow?

3753.

Waterhouse-Friderichsen
syndrome

What syndrome is due to a Neisseria sp. infection in a child resulting


in bilateral hemorrhagic infarcts of the adrenal glands?

3754.

Water-soluble
hormones are
considered fast-acting
hormones.

What form of hormone is described as having membrane-bound receptors that


are stored in vesicles, using second messengers, and having its activity
determined by free hormone levels.

3755.

A wave

What wave is the cause of the following venous pulse deflections? The
contraction of the right atrium

3756.

Week four, and they


remain dormant there
until puberty.

In men, at what embryonic week do the primordial germ cells migrate to the
indifferent gonad?

3757.

Wegener
granulomatosis

What rare vasculitis has the following characteristics: males aged 40 to 60;
affecting small arteries and veins; involving nose, sinuses, lungs, and kidneys;
C-ANCA and autoantibodies against proteinase 3?

3758.

Wernicke's aphasia

Name the aphasia based on these characteristics: Lesion in Brodmann area


22; impaired comprehension; incoherent rapid, fluent speech; verbal
paraphrasias; trouble repeating statements

3759.

Wernicke's
encephalopathy

What encephalopathy causes ocular palsies, confusion, and gait abnormalities


related to a lesion in the mammillary bodies and/or the dorsomedial nuclei of
the thalamus?

3760.

Western blot

What blotting technique uses the following for analysis? Protein

3761.

Western blot

What is the confirmatory test for HIV?

3762.

When a patient closes the eyes while standing with feet together, the visual
and cerebellar components of proprioception are removed, so you are testing
the dorsal columns. Swaying with eyes closed is a positive Romberg's sign
indicating a lesion in the dorsal columns. The cold water caloric test mimics a
brainstem lesion by inhibiting the normal reflex response. (COWS: Cold
Opposite Warm Same)

By asking a patient to
close the eyes while
standing with feet
together, what two
pathways are you
eliminating from
proprioception?

3763.

When calcium is removed from troponin and pumped back into the SR, skeletal
muscle contraction stops.

What causes a
skeletal muscle
contraction to
terminate?

3764.

When it crosses the teres major

At what point does the


axillary artery become
the brachial artery?

3765.

When showing two pictures in rapid succession, you split the pictures half a
second apart, resulting in the second picture masking the first (indicating poor
short-term memory). This is seen in nearly 33% of schizophrenic patients.

What is backward
masking, and is there
a positive correlation
with schizophrenic
patients?

3766.

When the intra-alveolar pressure equals zero, there is no airflow.

Which direction is air


flowing when the intraalveolar pressure is
zero?

3767.

When the pH is more acidic than the pI, it has a net positive charge, and when
the pH is more basic than the pI, it has a net negative charge.

If the pH is more
acidic than the pI,
does the protein carry
a net positive or net
negative charge?

3768.

White muscle; short term too

What type of muscle is


characterized by no
myoglobin, anaerobic
glycolysis, high
ATPase activity, and
large muscle mass?

3769.

White rami are preganglionic


fibers, whereas grey rami are
postganglionic fibers.

Are white rami preganglionic or postganglionic fibers?

3770.

Wilms tumor

What tumor is seen in the 2-to 4-year-old age group; does not cross
the midline; has immature glomeruli, tubules, and stroma; and
metastasizes late to the lungs?

3771.

Wilson disease

Kayser-Fleischer rings

3772.

Wilson disease. (Remember,


patients commonly present with
psychiatric manifestations and
movement disorders but may be
asymptomatic.)

What AR disorder of copper metabolism can be characterized by


Kayser- Fleischer rings, decreased ceruloplasmin levels, and
increased urinary copper excretion and tissue copper levels?

3773.

Wilson's disease (Remember


chromosome 13 and
hepatolenticular degeneration)

Kaiser-Fleischer rings, abnormal copper metabolism, and


ceruloplasmin deficiency characterize what disease, which may
include symptoms of dementia when severe?

3774.

Wiskott-Aldrich syndrome

What X-linked recessive immune disorder is characterized by


recurrent infections, severe thrombocytopenia, and eczema?

3775.

Wiskott-Aldrich syndrome

What disease is X-linked recessive, is associated with eczema


thrombocytopenia and an increased chance of developing recurrent
infections, involves a decrease in serum IgM and in the T celldependent paracortical areas of the lymph nodes, and means that the
patient is likely to develop malignant lymphoma?

3776.

With an increased depth of


breathing both the total and
alveolar ventilation increase.
This concept is always tested
on the boards, so remember it.

What happens to total and alveolar ventilation with Increased depth


of breathing?

3777.

With an increased rate of breathing the total


ventilation is greater than the alveolar
ventilation. Rapid, shallow breathing
increases dead space ventilation with little
change in alveolar ventilation. (This is
hypoventilation).

What happens to total and alveolar ventilation with


Increased rate of breathing?

3778.

With arteriolar constriction both the flow and


pressure downstream decrease.

What happens to blood flow and pressure downstream


with local arteriolar constriction?

3779.

With opioid use the pupils become pinpoint


(miosis) because of increased cholinergic
activity.

How is the eye affected by use of opioids?

3780.

Without children (but don't think about this


one for too long)

Is marital satisfaction higher for couples with or without


children?

3781.

Xanthine, hypoxanthine, theophylline,


theobromine, caffeine, and uric acid are all
purines.

Name three purine bases that are not found in nucleic


acids.

3782.

Xanthine oxidase

What enzyme requires molybdenum as a cofactor?

3783.

Xenograft

Name the type of graft described by these transplants:


From one species to another

3784.

X-linked dominant

Name the pattern of genetic transmission characterized


thus: both M and F affected; no M-to-M transmission;
affected M passes trait to all daughters, every
generation; affected F passes trait to both sons and
daughters; a single mutant allele can produce the
disease.

3785.

X-linked recessive

What pattern of genetic transmission affects only M and


has no M-to-M transmission, and mother is usually an
unaffected carrier?

3786.

X-linked recessive

What pattern of inheritance is G-6-PD deficiency?

3787.

X-linked recessive

What pattern of inheritance does G-6-PD deficiency


follow?

3788.

Yellow fever (flavivirus)

What viral infection is associated with


black vomit?

3789.

Yersinia pestis

What small coagulase-positive, gramnegative rod with bipolar staining is a


facultative intracellular parasite resulting
in buboes?

3790.

Yes

Is the Salk polio vaccine inactivated?

3791.

Yes

Can advance directives be oral?

3792.

Yes, as long the illness does not threaten limb or life. If


illness is critical or an emergency, treat the child.

Can parents withhold treatment from their


children?

3793.

Yes, because they both activate the BZ1-receptors, which


can be reversed by flumazenil.

Could an overdose with either zolpidem


or zaleplon be reversed with flumazenil?

3794.

Yes, by three to four times. Having multiple sexual partners,


being overweight, and pelvic pain and/or inflammatory
disorders are also likely to be seen in sexually abused
females.

Are sexually abused females more likely


to have learning disabilities than the
general population?

3795.

Yes. In a negatively skewed distribution the mean is greater


than the median is greater than the mode.

In a negatively skewed curve is the mean


greater than the mode?

3796.

Yes, it decreases fat and muscle uptake of glucose, thereby


increasing blood glucose levels.

Is GH considered a gluconeogenic
hormone?

3797.

Yes. It increases the rate of suicide to nearly 50 times that of


the general population.

Does alcoholism increase the rate of


suicide?

3798.

Yes. It is also a cause of cancers of the lung, esophagus,


ureter, and kidney, just to name a few.

Is cigarette smoking associated with


transitional cell carcinoma of the bladder?

3799.

Yes. (Remember, Jehovah's witnesses refuse blood


transfusions.)

Can a patient refuse life-saving treatment


for religious reasons?

3800.

Yes. The only civil liberty they lose is the freedom to come
and go as they please.

Can committed mentally ill patients


refuse medical treatment?

3801.

Yes, they stimulate the growth of mammary tissue but block


milk synthesis. At parturition, the decrease in estrogen lifts the
block on milk production.

Do high levels of estrogen and


progesterone block milk synthesis?

3802.

Yes. This is done to help prevent underreporting out of fear of


lawsuit. Remember that it is your duty to protect the child first,
not worry about legal responsibility.

If you report a suspected case of child


abuse and are wrong, are you
protected from legal liability?

3803.

Yohimbine

What 2-antagonist is used to treat


impotence and postural hypotension?

3804.

Yolk sac tumor

What is the most common one?


Testicular tumor in infants and children

3805.

Yolk sac tumor

Schiller-Duval bodies

3806.

Yolk sac tumor

What testicular tumor of infancy is


characterized by elevated-fetoprotein
levels and Schiller-Duval bodies?

3807.

Yolk sac tumor

What is the most common one? Germ


cell tumor in boys

3808.

Your duty is to tell the patient, not the family. The patient
decides who gets to know and who doesn't, not you.

Should information flow from the patient


to the family or vice versa?

3809.

Y, W-135, and C and A capsular polysaccharides

What are the four capsular


polysaccharides used in the Neisseria
meningitides vaccine?

3810.

Zafirlukast

Which prophylactic asthmatic agent is


an antagonist of LTD4?

3811.

Zanamivir and oseltamivir

What antiviral agents inhibit


neuraminidases of influenza A and B?

3812.

Zero correlation

What type of correlation is defined as


Two variables with no linear relation to
one another?

3813.

Zero-order elimination

What type of drug elimination is characterized by the following?


Elimination independent of plasma concentration; constant amount
eliminated per unit time; no fixed half-life

3814.

Zileuton

Which agent used in the treatment of asthma is a selective inhibitor of the


lipoxygenase pathway?

3815.

Zinc (Zn) deficiency

What mineral deficiency in children is associated with poor growth and


impaired sexual development?

3816.

Zollinger-Ellison syndrome

What syndrome is associated with gastrin-producing islet cell tumor


resulting in multiple intractable peptic ulcers?

3817.

Zona fasciculata and zona


reticularis

What region or regions of the adrenal cortex are stimulated by ACTH?

3818.

Zonula occludens (tight


junctions)

What type of cell surface projection lies on the lateral surface of cells
closest to the apex and acts to seal off the outside environment from the
rest of the body?

3819.

Zoophilia

What paraphilia is defined as Sexual fantasies or practices with animals?

3820.

The zygote divides


mitotically; only germ cells
divide meiotically.

Does the zygote divide mitotically or meiotically?

3821.

-1, 6 transferase

Deficiencies in what enzyme result in insoluble glycogen synthesis


formation?

3822.

1-Receptors

What adrenergic receptors use inositol triphosphate (IP3) and


diacylglycerol (DAG) for their second messenger system?

3823.

-and -chains are on


most T cells.

What are the two most common chains of the TCR?

3824.

-Blockers (-zosins)

Which class of antihypertensives is used in the treatment of BPH?

3825.

-Methyldopa (Guanabenz
or clonidine is also used.)

What is the drug of choice for hypertensive patients with a decreased renal
function?

3826.

-Endorphin

What is the neurotransmitter at the -receptor?

3827.

-Galactosidase

Regarding the Lac operon, for what do the following genes code? Z gene

3828.

-Hydroxybutyrate

What is the major ketone body produced during alcoholic


ketoacidosis?

3829.

-Oxidation of fatty acids

From where is the energy for gluconeogenesis derived?

3830.

The -subunit; remember, the -subunit is


nonspecific.

What subunit of hCG is used to detect whether a patient


is pregnant?

3831.

-ALA synthase

What enzyme catalyzes the rate-limiting step in heme


synthesis?

You might also like